Nothing Special   »   [go: up one dir, main page]

Gastroenterology and Hepatology Complete PDF

Download as pdf or txt
Download as pdf or txt
You are on page 1of 257

~

ACP IM KSAP® Medical Knowledge Self-Assessment Program®

Gastroenterology and
Hepatology

22AMAPRA
Category 1 Credits™
available until
·AcP·
American College of Physicians
leading Internal Medicine, Improving Lives
July31,2021 .
Attention MKSAP 18 Complete subscribers:
If you are receiving the MKSAP 18 print books as part of your MKSAP 18
Complete subscription, remember that you will receive regular content
updates in your online account through MKSAP 18 Digital's New Info
Updates. New Info Updates will focus on practice-changing updates, includ-
ing new guidelines. You will also receive four sets of MKSAP 18 Extension
Questions (formerly MKSAP Update Questions) that will allow you to earn
CME credits and MOC points through January 2024.

Not a MKSAP 18 Complete subscriber?


If you wish to add MKSAP 18 Digital to your print subscription at any time,
upgrade to MKSAP 18 Complete for the most value-packed, cost-efficient
combination of MKSAP products. Please call our ACP Member and Customer
Service Department, and they will help you arrange easy access to MKSAP 18
Complete. As part ofMKSAP 18 Complete, you will receive MKSAP 18 Digital.
including regular content updates through New Info Updates, and four sets
ofMKSAP 18 Extension Questions (formerly MKSAP Update Questions) that
will allow you to earn CME credits and MOC points through 2024. You will
also receive MKSAP 18 Flashcards, Virtual Dx, and Board Basics (print and
e-book) at no extra charge.
ACP Member and Customer Service
800-ACP-1915
Outside the U.S. call 215-351-2600
(Monday to Friday, 9 am - S pm ET)

Disclaimer Regarding Direct Purchases from Online Retailers


CME and/or MOC for MKSAP 18 is available only if you purchase the pro-
gram directly from ACP. CME credits and MOC points cannot be awarded to
those purchasers who have purchased the program from non-authorized
sellers, such as Amazon , eBay, or any other such on line retailer.
Table of Contents

Disorders of the Esophagus Gastric Adenocarcinoma ......................... 18


Symptoms of Esophageal Disease . . . . . . . . . . . . . . . . . . . 1 Epidemiology and Risk Factors .... .... . . ...... 18
Dysphagia ................................... 1 Screening and Surveillance ................... 18
Reflux and Chest Pain ......................... 2 Clinical Manifestations and Diagnosis .......... 18
Odynophagia . ........ .. ..... ........... .. ... 2 Gastric Surgery Complications ............ ..... .. . 19
Globus Sensation ............................. 2
Nonmalignant Disorders of the Esophagus .. .... ..... 3 Disorders of the Pancreas
Gastroesophageal Reflux Disease ............... 3 Acute Pancreatitis . . . . . . . . . . . . . . . . . . . . . . . . . . . . . . . 19
Eosinophilic Esophagitis ........... .. ......... 5 Clinical Presentation and Diagnosis ............ 20
Infectious Esophagitis .... ......... .......... .. 6 Prognostic Criteria .......................... 20
PilHnduced Esophagitis ....................... 6 Management ............................... 20
Esophageal Motility Disorders .................. 6 Complications .............................. 21
Metaplastic and Neoplastic Disorders of the Esophagus ... 8 Chronic Pancreatitis ............................. 22
Barrett Esophagus ............................ 8 Clinical Presentation and Diagnosis ............ 22
Esophageal Carcinoma ............. ..... ... ... 9 Management ............................... 23
Autoimmune Pancreatitis and IgG4 Disease ...... ... 23
Disorders of the Stomach and Duodenum Clinical Presentation and Diagnosis .......... . . 23
Dyspepsia .................................... .. 1O Treatment ...... .. ....... ............. ..... . 24
Clinical Features ............................ 10 Pancreatic Adenocarcinoma ...................... 24
Evaluation and Management .................. 10 Epidemiology and Risk Factors ................ 24
Peptic Ulcer Disease . . . . . . . . . . . . . . . . . . . . . . . . . . . . . 11 Clinical Presentation ......................... 24
Clinical Features, Diagnosis, and Complications ... 11 Diagnosis ............... . .................. 24
Management ............................... 12 Ampullary Adenocarcinoma ...................... 24
Helicobacter pylori Infection ..................... 13 Cystic Lesions of the Pancreas ..................... 25
Indications for Helicobacter pylori Testing ...... 13 Pancreatic Neuroendocrine Tumors ..... .. ...... . .. 25
Diagnosis . . . . . . . . . . . . . . . . . . . . . . . . . . . . . . . . . . 13
Treatment. . . . . . . . . . . . . . . . . . . . . . . . . . . . . . . . . . 13 Disorders of the Small and Large Bowel
Eradication Testing . . . . . . . . . . . . . . . . . . . . . . . . . . 13 Diarrhea ....................................... 26
Miscellaneous Gastropathy . . . . . . . . . . . . . . . . . . . . . . . 13 Classification ............................... 26
Atrophic Gastritis .... ... . .. .... . ............ 13 Acute Diarrhea . . .. . .... .. . .. ... . ...... . ..... 26
Intestinal Metaplasia . . . . . . . . . . . . . . . . . . . . . . . . . 15 Chronic Diarrhea ............................ 26
Eosinophilic Gastritis .... .... ................ 15 Celiac Disease and Nonceliac Gluten Sensitivity ...... 28
Lymphocytic Gastritis. . . . . . . . . . . . . . . . . . . . . . . . 15 Testing .................................... 28
Gastrointestinal Complications of NSAIDs ... .. ...... 15 Diagnosis .... . ............................. 29
Epidemiology and Risk Factors ................ 15 Management and Monitoring ................. 29
Prevention of NSAID-Induced Injury .... .. ..... 15 Nonresponsive Celiac Disease ................. 29
Gastroparesis ................................... 16 Nonceliac Gluten Sensitivity ...... . ........... 29
Presentation ................................ 16 Malabsorption ............................... . .. 29
Diagnostic Testing .. .. ............... . ....... 16 Small Intestinal Bacterial Overgrowth .......... 29
Management ............................... 16 Short Bowel Syndrome ....................... 29
Gastric Polyps and Subepithellal Lesions ............ 17 Carbohydrate Malabsorption ....... . .......... 30
Gastric Polyps .............................. 17 Inflammatory Bowel Disease ...................... 30
Gastric Subepithelial Lesions. . . . . . . . . . . . . . . . . . 18 Risk Factors ...................... ... ..... .. 30

ix
Clinical Manifestations . . .. .. ....... . .. .. . .... 30 Autoimmune Hepatitis ..... ..... ......... .. . . . .. . SS
Extraintestinal Manifestations . . . . . . . . . . . . . . . . . 31 Alcohol-Induced Liver Disease ................ . . .. 55
Diagnosis . . . .. .. .. . . . . ......... .. ..... . .... 32 Drug-Induced Liver Injury .. . ... ...... .. ... ... .... 56
Treatment . .. . . . . . ... . . . .. . . . . . .. .. . . ..... . . 32 Acute Liver Failure . ... . . .... . .. . ... . .. .. . . . .. . .. 57
Health Care Considerations . .... . ... . . . ... .... 34 Metabolic Liver Diseases ... ..... . . . . . .. . .. . . . .. . .. 57
Microscopic Colitis . .. . . . . ... .... .. . ....... .. 34 Nonalcoholic Fatty Liver Disease . . ...... .. . . .. . 57
Constipation . . .. ..... ... . .. . ... . . . . .... .. . . . . .. 3S n 1-Antitrypsin Deficiency ... . . . ..... .. . .. . . .. 58
Evaluation ..... ... . ..... . . . .. . . .. . .. . .. . . .. 36 Hereditary Hemochromatosis .. .. . .. .... . .. . .. 58
Management .... . . .. ... ........ . . .... . . . ... 36 WI.Ison Disease.. ..... ..... . ..... . . .. ... ..... 58
Irritable Bowel Syndrome.. . ..... . . .. ... .. . . .. . . .. 37 Cholestatic Liver Disease ... .. . ... .. . .... . . .. . . ... 58
Evaluation . . ... . . .. . ........ ... .. . ..... . . .. 37 Primary Biliary Cholangitis .. .. . . .. .. . . ... ... . 58
Management ... .. .. .... . . . ..... ... ..... . .. . 37 Primary Sclerosing Cholangltis . . . ... .... . . . . . . 59
Management of Patients with Indeterminate Determining Prognosis .. ... ...... . ... .. ......... . 59
Abdominal Pain ...... . . . . . . . . .. .. . ... .. ..... .... 38 Complications of Advanced Liver Disease . ....... . . . 60
Diverticular Disease . ... .... .... . .. . .... ...... . . . 38 Portal Hypertension .. .. .. ... . . . ....... . ... . . 60
Ischernic Bowel Disease ..... ... ... ... . . . . ....... . 39 Esophageal Varices ... .. .... . . . .... . . . .. .. ... 60
Acute Mesenteric Ischernia.. . ... . . . .. . .... .. . . 39 Gastric Varices and Portal Hypertensive
Chronic Mesenteric Ischemia. .. .. . . . ... . . . ... . 40 Gastropathy . .. . . .. . ..... . .. . . .. . ... . . . . . .. . 60
Colonic Ischemia ... .... .. .. ... .... . .... .. .. .40 Hepatic Encephalopathy .. .. . .. .... . ...... . . .. 61
Anorectal Disorders . .... .. .... .... . . .. . .. . . ..... 41 Ascites .. ...... . . . .. . .. . . ..... ... .. .. . ... . .. 61
Perianal Disorders . . .. ... . . . .. ... ... . .. .... .. 41 Spontaneous Bacterial Peritonitis .... .. .. . . . ... 61
Fecal Incontinence .. .... .... . .. ...... ....... 42 Hepatorenal Syndrome . .. . . ... .. . .... .. ... . .. 62
Anal Cancer ... . .. .. .. .. .... . .. .... .. ... . ... 42 Hepatopulmonary Syndrome .... . ....... . .. . . 62
Portopulmonary Hypertension .. . ......... . .. . 63
Coloredal Neoplasia Health Care Maintenance in Patients with
Chronic Liver Disease . . .. .. . . . ....... . . .. . . .. .. .. 63
Epidemiology . ... .. ...... . .. .. . .. . . .. ... . . ... .. . 43
Hepatic Tumors, Cysts, and Abscesses . ..... .. .. ... . 63
Pathogenesis ... . .. . ... .... ... . .... . ...... .... . . 43
Hepatic Cysts .... . . . . . . . . .. ....... . .. . .. .. . . 64
Risk Factors ....... . ........................ .. . . 44
Hepatic Adenomas . ... . ... . . . ..... . ... ... .. . 64
Chemoprevention . .. . . . . . . .. . ... . . .. . ... . ...... . 44
Focal Nodular Hyperplasia .. . . . ... . . ... ....... 64
Screening . ....... ... ..... .. . . ... . . .. . .. ... . . . .. 44
Hepatic Hemangiomas ...... .... . .. ....... . .. 64
Clinical Presentation ... .. .. . .... . ..... . ... . . . ... . 4S
Hepatic Abscesses . ... . . . . .... . .. ... .... . ... . 64
Diagnosis and Staging.. . .. ... . .. . ... . . .. . ... .. .. . 4S
Hepatocellular Carcinoma ............. . ... .. . 65
Surveillance .. .... . .... . .. .. ... . ..... . .. .. . . .. . . 46
Liver Transplantation ..... . . . .. .... . .. . .. . ... .... 65
Hereditary Colorectal Cancer Syndromes .... ..... .. 46
Pregnancy-Related Liver Diseases . .... . . . . .. .. . . ... 66
Lynch Syndrome .. . . ... . .. .. . . . ... .. . ..... . . 46
Vascular Diseases of the Liver .... ... .. . . .. . ...... . 66
Adenomatous Polyposis Syndromes . . ... . .. .. .. 47
Portal Vein Thrombosis..... ..... . .. . ......... 66
Hamartomatous Polyposis Syndromes . . .. . ..... 48
Budd-Chiari Syndrome.. . .... .. . .. . . . . ... . . .. 67
Serrated Polyposis Syndrome . .... . ... . .. ..... 48

Disorders of the Liver Disorders of the Gallbladder and Biie Duds


Approach to the Patient with Abnormal Liver Asymptomatic Gallstones . .. . .. . .... . ... ... . .. . ... 67
Chemistry Studies . . . . . . .. ... .. ...... .. .......... 49 Biliary Colic . ... . .. ....... . .. . .. . . . . . .. ..... .. .. 67
Vlral Hepatitis ... ... .. .. ... .. ... . ...... . ... . . ... so Acute Cholecystitis . . ... . .. . .. .. ... . ... .. . ... . . .. 67
Hepatitis A ... .. .. . . . . .. .. . . .. ... .... ....... 50 Acalculous Cholecystitis .... . .. . .. . ... ...... .. . ... 68
Hepatitis B . . . . . . . . . . . . . . . . . . . . . . . . . . . . . . . . . 51 Functional Gallbladder Disorder ... . .. .. .. . .. .. . .. . 68
Hepatitis C .. .. . . . . . . .. . .. ....... . ..... . . . .. 53 Common Bile Duct Stones and Cholangitis ... . ... ... 68
Hepatitis D ... . .. . . .. . ... . ... .. . ... .... .. ... 54 Gallbladder Polyps .... .. .. . . .. ... .... ..... . ... .. 69
Hepatitis E . ..... . . . ... . ... . ... . ..... . . . .... 54 Gallbladder Cancer . .... . ... . ... .. .. .. ... ... .. ... 69
Other Viruses .. . ...... .. .... .. .. .. . ... .. . . .. SS Cholangiocarctnoma ....... ...... . .. ... ...... 69

x
Gastrointestinal Bleeding Small-Bowel Bleeding ............................ 75
Overview ...................................... 70 Causes ..................................... 75
Upper Gastrointestinal Bleeding ..... ......... ... .. 70 Evaluation ................................. 75
Causes .................................... . 70 Management ............................... 77
Evaluation ................................. 70
Management ............................... 70 Bibliography ....... .... ... .. .... . ....... ...... 77

Lower Gastrointestinal Bleeding ................... 74


Causes ..................................... 74 Self·AssessmentTest. . .......................... 81
Evaluation . . . . . . . . . . . . . . . . . . . . . . . . . . . . . . . . . 74
Management ............................... 74 Index ........................................ 153

xi
Gastroenterology and Hepatology
High Value Care Recommendations

The American College of Physicians, in collaboration with • Patients 60 years and older with dyspepsia should
multiple other organizations, is engaged in a worldwide undergo upper endoscopy
initiative to promote the practice of High Value Care • Patients younger than age 60 years with dyspepsia should
(HVC). The goals of the HVC .initiative are to improve be tested and treated for Helicobucter pylori infection.
health care outcomes by providing care of proven benefit • Enteral nutrition is preferred in patients with acute
and reducing costs by avoiding unnecessary and even pancreatitis (see Item 26).
harmful interventions. The initiative comprises several • Same-admission cholecystectomy reduces rates of
programs that integrate the important concept of health gallstone-related complications for patients with mild
care value (balancing clinical benefit with costs and gallstone pancreatitis (see Item 20).
harms) for a given intervention into a broad range of • Asymptomatic patients with walled-off necrosis of the
educational materials to address the needs of trainees, pancreas require no intervention (see Item 95).
practicing physicians, and patients. • Biopsy and endoscopic retrograde cholangiopancreatogra-
phy are not indicated in the diagnosis of chronic pancreatitis.
HVC content has been integrated into MKSAP 18 in several
• Evaluation with imaging, endoscopy, and other testing is
important ways. MKSAP 18 includes HVC-identified key
not indicated for irritable bowel syndrome or functional
points in the text, HVC-focused multiple choice questions,
diarrhea.
and, for subscribers to MKSAP Digital, an HVC custom
• Irritable bowel syndrome can often be managed with
quiz. From the text and questions, we have generated
reassurance, lifestyle modifications, and dietary
the following list ofHVC recommendations that meet the
modifications.
definition below of high value care and bring us closer to
• The evaluation for centrally mediated pain syndrome
our goal of improving patient outcomes while conserving
does not require extensive laboratory testing or imaging.
finite resources.
• Acute diverticulitis usually does not require abdominal
High Value Care Recommendation: A recommendation to imaging.
choose diagnostic and management strategies for patients • Patients with small hyperplastic polyps should undergo
in specific clinical situations that balance clinical benefit repeat colonoscopy no sooner than 10 years (see Item 11).
with cost and harms with the goal of improving patient • Patients with uncomplicated diverticulitis should be
outcomes. treated with oral antibiotics (see Item 74).
• Ultrasonography is the most effective means of
Below are the High Value Care Recommendations for the
diagnosing ascites.
Gastroenterology and Hepatology section ofMKSAP 18.
• Asymptomatic hepatic cysts require no follow-up.
• In patients with gastroesophageal reflux symptoms • Focal nodular hyperplasia does not require follow-up.
without alarm features, empiric proton pump • Incidentally diagnosed gallstones do not require
inhibitor therapy is the first therapeutic and cholecystectomy.
diagnostic step. • The diagnosis of acute cholecystitis can be made by
• Attempts to stop or reduce long-term proton pump ultrasonography
inhibitor therapy for uncomplicated gastroesophageal • In patients with upper gastrointestinal bleeding, a trans-
reflux disease should be considered at least annually fusion threshold of less than 7 g/dL (70 g/L) with a target
(see Item 59). hemoglobin level of7-9 g/dL (70-90 g/L) is associated
• Women with gastroesophageal reflux disease do not with decreased mortality and length of hospital stay
require screening for Barrett esophagus. (see Item 83).

xiii
Gastroenterology and Hepatology

include food "hanging up" or feeling "lodged" or "stuck" dur-


Disorders of the ing a meal. Determining whether the underlying cause is
Esophagus oropharyngeal or esophageal is important in developing a dif-
ferential diagnosis and management plan. Table 1 highlights
Symptoms of Esophageal Disease the common causes of dysphagia.
Dysphagia
Dysphagia represents a disruption in the swallowing mecha- Oropharyngeal Dysphagia
nism, resulting in food not passing from the mouth to the Oropharyngeal dysphagia, also known as transfer dysphagia,
stomach. Common descriptions of the sensations of dysphagia occurs when the patient is unable to transfer the food bolus

TABLE 1. Causes of Dysphagia


Condition Diagnostic Clues
Oropharyngeal Dysphagia

Structural disorders
Cervical osteophytes High dysphagia, degenerative joint disease
Cricoid webs High dysphagia, iron deficiency
Pharyngoesophageal (Zenker) diverticulum Aspiration, neck mass, and regurgitation of foul-smelling food
Goiter Neck mass
Neurologidmyogenic disorders
Amyotrophic lateral sclerosis Upper and lower motoneuron signs, fasciculations
Central nervous system tumor Headache, vision changes, nausea, seizures, balance problem
Stroke Focal neurologic deficits
Muscular dystrophy Slow progression of muscular weakness over years
Myasthenia gravis Weakness with repetitive activity
Multiple sclerosis Episodes of neurologic dysfunction with variable degrees of recovery
Parkinson disease Bradykinesia, rigidity, tremor
Dementia Altered cognition
Sjogren syndrome Dry mouth, dry eyes
Esophageal Dysphagia

Structural disorders
Dysphagia lusoria (vascular dysphagia) Vascular extrinsic compression on the esophagus on imaging
Epiphrenidtraction diverticulum Outpouching of the esophagus at any level on imaging
Esophageal strictures Intermittent dysphagia, especially for solid food; history of reflux
Eosinophilic esophagitis Food impactions, atopic history, rings or strictures on endoscopy
Esophageal webs or rings Upper esophageal webs may be associated with iron deficiency anemia
Neoplasms Rapidly progressive dysphagia for solids, then liquids; anorexia; weight loss
Motility disorders
Achalasia Concomitant liquid and solid dysphagia
Diffuse esophageal spasm Chest pain i'
Systemic sclerosis Tight skin, telangiectasias, sclerodactyly, gastroesophageal reflux disease, Raynaud j
phenomenon ------- ___ i
1
Disorders of the Esophagus

from the mouth into the upper esophagus by swallowing.


Symptoms commonly reported include choking, coughing,
and nasal regurgitation of food. Patients are at risk for aspira-
tion pneumonia. Other presenting symptoms include hoarse-
ness (resulting from laryngeal nerve damage) and dysarthria
(from weakness of the soft palate or pharyngeal constrictors),
both representing an underlying neurologic disorder. A phar-
yngoesophageal (Zenker) diverticulum should be considered
when undigested food is brought up several hours after a meal
or if a patient reports hearing a gurgling noise in the chest.
The initial study for suspected oropharyngeal dysphagia is
a modified barium swallow, with both a liquid and a solid
phase to help identify the underlying cause. Management
strategies include dietary changes and a swallowing exercise
program implemented with a speech pathologist.

Esophageal Dysphagia
Patients with esophageal dysphagia are able to initiate the
swallowing process, but often feel discomfort in the mid to
lower sternum as the food bolus passes through the esophagus.
Esophageal dysphagia is the result of one of two underlying
causes: a mechanical obstruction or a motility disorder.
Dysphagia occurring with solids alone suggests a mechanical
obstruction, whereas dysphagia with either liquids alone or
the combination ofliquids and solids favors a motility disorder.
Dysphagia that progresses from occurring with solids only to
occurring with both solids and liquids suggests mallgnancy.
Achalasia often presents with nonacidic regurgitation of
undigested food Chest pain while taking liquids that are very
hot or very cold may indicate esophageal spasm. Mechanical
esophageal obstruction may be benign or malignant and may Fl GU RE 1. Barium esophagram showing a Schatzki ring, a subtype of esophageal
be caused by strictures, masses, esophageal ring (for example, ring located at the squamocolumnar junction and a common cause of dysphagia.
a Schatzki ring [Figure I]), or webs. Upper endoscopy allows
for diagnostic (biopsy and inspection) and therapeutic inter-
vention (dilation). Clinical management is based on the under- ulceration in the esophagus. Odynophagia is commonly asso-
lying cause. ciated with pill-induced damage, infection, or caustic inges-
tion, and is less commonly caused by GERD or esophageal
Reflux and Chest Pain cancer. Upper endoscopy with biopsies is the most appropri-
The development of chest pain from an esophageal cause can ate diagnostic test to determine the degree of inflammation
mimic chest pain from cardiac disease. Reports of heartburn and underlying cause.
with history ofRaynaud phenomenon could signify a systemic
condition, such as scleroderma. Globus Sensation
Once a cardiac cause is ruled out, the most common cause Patients commonly report globus sensation as a "lump in the
of chest pain is gastroesophageal reflux disease (GERD). Starting throat" or "throat tightness," usually not linked to meals.
a course of an acid-reducing agent, such as an H2 blocker or Causes of globus include GERD (with or without heartburn),
proton pump inhibitor (PPO, can be both diagnostic and thera- stress, and psychiatric conditions (anxiety, panic disorders,
peutic. Patients whose symptoms do not respond require further somatization). A diagnosis of globus should not be made if the
evaluation, including upper endoscopy and possibly ambulatory patient reports other esophageal symptoms, such as dysphagia
pH testing with or without esophageal manometry. or odynophagia. Evaluation to detern1ine the underlying cause
See Gastroesophageal Reflux Disease for information should include evaluation for thyroid goiter and an underlying
about diagnosis and management. pharyngeal lesion, which can be diagnosed by transnasal
endoscopy or barium swallow.
Odynophagia Treatment with acid suppression or cognitive behavioral
A presentation of pain while swallowing defines odynopha- therapy should be initiated once a structural cause has been
gia, which suggests active mucosa! inflammation and ruled out.

2
Disorders of the Esophagus

KEY POINTS TABLE 2. Factors Associated with Reflux


• Oropharyngeal dysphagia occurs when the patient is Category Factor
unable to transfer the food bolus from the mouth into
Lifestyle Cigarette smoking
the upper esophagus by swallowing and should be eval-
Obesity
uated with a modified barium swallow.
Eating habits Eating large meals
• Esophageal dysphagia occurring with solids alone sug-
gests a mechanical obstruction, whereas dysphagia with Eating late at night

either liquids alone or the combination of liquids and Lying supine shortly after eating
solids favors a motility disorder. Foods and Alcohol
beverages
• Upper endoscopy is diagnostic and may be therapeutic Chocolate
for esophageal dysphagia. Citrus fruits and juices
• Chest pain is common in patients with gastroesopha- Coffee
geal reflux disease, but a cardiac cause of chest pain Fatty and fried foods
must be ruled out first. Onions
Peppermint

Nonmalignant Disorders Medications Anticholinergic agents


Aspirin and NSAIDs
of the Esophagus
Calcium channel blockers
Gastroesophageal Reflux Disease
Nitrates
GERD is characterized by food and acid refluxing from the
Progesterone
stomach into the esophagus and throat. Its prevalence is 10% to
Opioids (due to delayed gastric emptying)
20% in the Western world, and there is a strong relationship
between GERD and obesity. The most common symptoms Body position Bending over, exercising (both result in
increased intra-abdominal pressure)
reported are heartburn, regurgitation, and chest pain, for
which a cardiac cause must be excluded. Reflux can be trig- Other Pregnancy
gered by a number of factors (Table 2). Protective mechanisms Tight-fitting clothing
to minimize the esophagus' exposure to acid consist of peri- Hiatal hernia
stalsis, a competent lower esophageal sphincter (LES). and
gastric emptying; reflux occurs when these physiologic protec-
tors become ineffective. Uncontrolled GERD can negatively esophageal cancer (Figure 2). Pregnant women may experi-
affect quality of life due to poor sleep, low productivity, and ence GERD during any trimester of pregnancy, but symptoms
work absences. Longstanding GERD can lead to complications, may worsen as the pregnancy progresses. Heartburn symp-
including erosive esophagitis, stricture, Barrett esophagus, and toms resolve after delivery.

Esophageal syndromes Extraesophageal syndromes

Symptomatic Syndromes with Established Proposed


syndromes esophageal injury associations associations

1. Typical reflux 1. Reflux esophagitis 1. Reflux cough 1. Pharyngitis


syndrome 2. Reflux stricture syndrome 2. Sinusitis
2. Reflux chest 2. Reflux laryngitis 3. Idiopathic
3. Barrett esophagus
pain syndrome syndrome pulmonary
4. Esophageal fibrosis
adenocarcinoma 3. Reflux asthma 4. Recurrent otitis
syndrome
media
4. Reflux dental
erosion syndrome

FIGURE 2. Classification of gastroesophageal reflux disease and its subsets.


Reprinted with permiision from Vakil N, van Zan ten '!N. Kahrilas P. OenlJ,Jonei R; Global Consensus Group. The Montreal definition and daSliRcalion of gastroesophageal renux disease: a global evidence-based consensus.
AsnJ Gastroenterol. 2006;101:190().20; quiz 1943.[PMIO: 169282S4iCopyright2006, Springer Nature.

3
Disorders of the Esophagus

Diagnosis Lifestyle Changes


Strategies for diagnosing GERO include use of clinical history, Patients with recent weight gain or who are overweight should
response to medical therapy, and testing, including endoscopy develop a weight-loss plan. Patients with nocturnal GERD
and ambulatory pH monitoring; there is no single gold-stand- should eat at least 2 to 3 hours before going to sleep and should
ard diagnostic test. Clinical symptoms of heartburn and regur- consider raising the head of the bed.
gitation strongly suggest GERD. Patients with GERD symp- Dietary modification should focus on eliminating foods
toms, but without alarm features (such as dysphagia, that trigger an individual patient's GERO symptoms, rather
unintentional weight loss, hematemesis, or melena) may than globally eliminating all common trigger foods (caffeine,
undergo an empiric trial of a PP!. Symptom relief from medical chocolate, spicy foods, acidic foods such as citrus fruits, and
therapy can confirm the diagnosis. fatty foods). Cessation of alcohol and tobacco use is universally
Upper endoscopy is warranted In patients reporting dys- supported.
phagia to rule out an underlying ring, web, or malignancy, and
in patients with suspected erosive esophagitis, stricture, or Medical Therapy
Barrett esophagus. Most patients with GERD have a normal Pharmacologic therapy includes antacids, H 2 blockers, and PPI
upper endoscopy examination. therapy. A PPI once daily for 8 weeks is the therapy of choice
Ambulatory pH monitoring can assess the degree of acid for symptom relief, as well as for treatment of erosive esophagi-
exposure in the esophagus, especially in patients unresponsive tis. PPI therapy is superior to H2 blocker therapy for GERD
to acid-reducing therapy. Impedance-pH testing can help dif- with or without erosive esophagitis. The PPI should be taken
ferentiate between acid and nonacid renux. Testing to detect once daily, 30 to 60 minutes before the first meal of the day.
active acid reflux can be done with a 24- hour transnasal cath- Patients with partial response to PPI therapy should increase
eter or a 48-hour wireless capsule, which patients generally the dosage to twice daily. Patients requiring long-term main-
tolerate better. Esophageal manometry has limited value in tenance therapy should be placed on the lowest effective PPI
diagnosing GERD, but should be considered as part of the dose, including on-demand or intermittent usage, and for
evaluation for antireflux surgery to rule out motility disorders patients with uncomplicated GERD, an attempt to stop or
such as achalasia. reduce chronic PPI therapy should be made once a year.
Adverse effects of PPis are shown in Table 3. Switching to a
Treatment different PPI may be warranted for adverse reactions or for
An algorithm outlining the management of GERD is presented unresponsive symptoms. Initial studies suggested an interac-
inflgure3. tion with clopidogrel; however, subsequent data suggest that

GERD symptoms

i
Esophageal symptoms Extraesophageal symptoms
(heartburn, acid regurgitation) (cough, laryngitis, asthma)

Alarm symptoms' Alarm symptoms• Esophageal features Esophageal symptoms/


absent present present complications absent

Therapeutic trial Refer to a Therapeutic trial Explore alternative


of a PPI gastroenterologist of a PPI causes
for endoscopy and
other evaluation

If clinical response, If clinical response,


No or Noor
continue with PPI continue with PPI
incomplete incomplete
regimen at lowest regimen at lowest
response response
effective dose effective dose

FIGURE 3. Management of gastroesophageal reflux disease.


GERO• gastroesophageol reflux disease: PPt : proton pump inhibitor.
•Alarm symptoms indude dysphagia, unintentional weight iO<S, hematemesis, and melena.

4
Disorders of the Esophagus

~O='!*MMMl#iil"#.1MN4""'·1M'*'·1t~
symptoms. If a patient has atypical symptoms only, ambula-
tory esophageal pH monitoring should be considered before a
Common Unusual Proposed
i Associations PPl trial. Surgery is less effective in this group and should be
1
considered only in patients whose symptoms respond to PP!
Headache Vitam in 8 12 deficiency Kidney injury therapy.
I Diarrhea Hypomagnesemia Dementia
Refractory GERD
I Dyspepsia Community-acquired
pneumonia The first step in addressing refractory GERD is to optimize PPT

l - - - ----
Clostridium difficile infection
Hip fracture
--- --- --------
therapy by emphasizing the importance of taking medication
30 to 60 minutes before eating, increasing the dosage to twice
daily, or switching to another PPL If symptoms are still unre-
sponsive, alternative causes must be considered. For typical
concurrent use does not increase risk for a cardiac event. PPis symptoms, use endoscopy to rule out eosinophilic esophagitis
are safe in pregnant patients. or erosive esophagitis. Esophageal impedance-pH testing may
Sucralfate has no role in the treatment ofGERD. Pro kinetic also be useful and should be performed while the patient is
agents such as metoclopramide should not be used to treat receiving optimized PPI therapy. For atypical symptoms, refer
GERD unless gastroparesis is present. the patient to otorhinolaryngology, pulmonary, or allergy spe-
cialists to identify and treat the underlying cause. In these
Antirefiux Surgery patients, medical therapy should be stopped before imped-
Surgical treatments for GERO are laparoscopic fundoplication ance-pH testing. A negative impedance-pH test likely means
or bariatric surgery for obese patients, as well as magnetic that the patient does not have GERD and PPI therapy should
sphincter augmentation, a newer technique in which a mag- be discontinued. For patients with both types of symptoms, if
netic ring is placed around the LES without surgical alteration further evaluation is unremarkable, impedance-pH testing
of the stomach. Surgery is infrequently required; indications should be performed.
include failure of optimal therapy, wanting to stop medication,
and intolerable medication side effects. Patients should Eosinophilic Esophagitis
undergo objective testing, such as impedance-pH monitoring, Eosinophi!ic esophagitis (EE) is a condition commonly associ-
to confirm true acid reflux and correlation with symptoms ated with dysphagia and food bolus obstruction. Most patients
before surgery. Surgery is most effective in patients with typi- are diagnosed between the second and fifth decades of life,
cal symptoms of heartburn and regurgitation that are respon- and EE is more commonly seen in men. Patients often have
sive to therapy. However, about one third of patients require other atopic conditions such as asthma, rhinitis, dermatitis,
resumption of a PPI s to 10 years after surgery. Postoperative and seasonal or food allergies. The reported prevalence is as
complications include dysphagia, diarrhea, and inability to high as 40 to 90 per 100, 000 in the United States. The diagnos-
belch from a tight fundoplication. tic criteria for EE are esophageal symptoms (dysphagia),
esophageal biopsies showing 15 eosinophlls/hpf or greater,
Endoscopic Therapy persistent mucosa] eosinophilia despite a trial of PPI therapy.
Endoscopy-based therapies for GERD include thermal radio- and exclusion of other causes of eosinophilia. EE is a diagnosis
frequency to augment the LES, silicone injection to the LES, made in the absence of peripheral eosinopb.ilia.
and suturing of the LES. Early relief of reflux symptoms has Endoscopic findings include rings, longitudinal furrows,
been seen with these therapies, but long-term benefits have luminal narrowing, and white exudates and plaques. Patients
not been proven. None of these therapies has been associated with GERD may have elevated numbers of eoslnophils in
with normalized esophageal pH levels. Newer approaches mucosa! biopsies. Therefore, patients with prominent esopha-
include transoral incislonless fundoplication, which is a full- geal mucosa! eosinophils should first undergo an 8-week trial
thickness suture to create an endoscopic fundoplication, but of a PP! to rule out GERD. If there Is no clinical improvement,
there are no long-term data on its efficacy. repeat endoscopy with biopsy is recommended. If biopsies
continue to show 15 eosinophils/hpf or greater, EE can be
Extraesophageal Manifestations diagnosed and appropriate therapy initiated, with swallowed
Asthma, chronic cough, and laryngitis have been linked to aerosolized topical glucocorticoids (fluticasone or budeson-
GERO (see Figure 3). It is important to eliminate other non- ide). Diet modification has been suggested as a treatment
GERD causes when these symptoms are present. Laryngoscopy option to prevent EE flares. An empiric elimination diet-
often shows edema and erythema as signs of retlux-induced removing the foods most commonly associated with food
laryngitis. However, more than 80% of healthy persons also allergies, such as egg, soy, wheat, peanuts, cow's milk, and
have these findings; therefore, Jaryngoscopy should not be fish/shellfish-has been used. Endoscopic dilation should be
used to diagnose GERD-related laryngitis. A PPT trial is rec- considered in patients with continued dysphagia caused by
ommended in patients who also have typical GERO esophageal stricture not responding to medical therapy.

5
Disorders of the Esophagus

Infectious Esophagitis Esophageal Motility Disorders


Infectious esophagitis can be caused by fungal, viral, bacterial The esophagus is a muscle that passes a food bolus from the
(uncommon), and parasitic pathogens. Patients most commonly hypopharynx to the stomach through peristalsis. The upper
present with odynophagia or dysphagia. Candida esophagitis third of the esophagus is composed of skeletal muscle inner-
most commonly causes dysphagia, while viral esophagitis vated by axons of lower motoneurons. The lower two thirds is
produces odynophagia. Other organisms associated with smooth muscle innervated by the vagus nerve. The upper and
esophagitis include Lactobacillus, B-hemolytic streptococci, lower esophageal sphincters relax during swallowing.
Cryptosporidlum, Pneumocystis jirouecii, Mycobacterium Additional peristaltic activity occurs when the esophagus
auium complex, and Mycobactertum tuberculosis. is distended. High-resolution esophageal manometry is used
Candida infection can occur in immunocompetent or to evaluate suspected esophageal motility disorders.
immunocompromised hosts. Diagnosis is usually made clini- GERD is the most common cause of noncardiac chest
cally based on the presence of compatible symptoms and oral pain. However, hypercontractile disorders of the esophagus
candidiasis. Endoscopy and biopsy can be considered for should also be considered in patients with noncardiac chest
patients who do not respond to empiric therapy or have atypi- pain. Esophageal manometry is used to differentiate these
cal symptoms. Endoscopy shows small, white, raised plaques, disorders.
and esophageal brushings confirm the diagnosis (Figure 4).
The most common species is Candida albtcans, treated with Hypertonic Motility Disorders
oral fluconazole. Hypertonic motility disorders are characterized by dysphagia
Herpes simplex virus and cytomegalovlrus are seen in with both liquids and solids. Other symptoms can include
immunodeficient or immunosuppressed individuals, but regurgitation of undigested food, in particular when in a
rarely in immunocompetent patients. Endoscopy with biopsy recumbent position. Treatment of hypertonic disorders of the
is needed to confirm the diagnosis. Herpes simplex virus esophagus is aimed at relieving symptoms.
infection is treated with acyclovir, and cytomegalovirus infec-
tion with ganciclovir. Achalasia and Pseudoachalasia
Achalasia is a hypertonic motility condition defined by inade-
Pill-Induced Esophagitis quate relaxation of the LES and aperistalsis. Achalasia can be
idiopathic or associated with viral, autoimmune, and neurode-
Medications can cause esophageal injury resulting in esophagi-
generative disorders and infection (Chagas disease). Damage to
tis. Risk factors associated with pill-induced esophagitis
the ganglion cells and myenteric plexus in the esophageal body
include decreased salivary output, esophageal dysmotility,
and LES leads to uncontested cholinergic nerve activation,
large pills, medications that increase the LES tone (opioids),
which prevents LES relaxation. Achalasia affects men and
and ingestion of medications in the supine position. Patients
women equally, with an annual incidence ofl in 100,000 indi-
commonly report chest pain, dysphagia, and odynophagia
viduals. It commonly occurs between 30 and 60 years of age.
occurring several hours to days after taking medication. Pill-
Patients present with dysphagia occurring with both solids
induced esophagitis has been observed with alendronate, qui-
and liquids along with nonacidic regurgitation of undigested
nidine, tetracycline, doxycycline, potassium chloride, ferrous
food. Additional symptoms include heartburn, weight Joss,
sulfate, and mexiletine. Medications associated with stricture
and chest pain unresponsive to acid-reducing agents. Extrinsic
formation include alendronate, ferrous sulfate, NSAIDs, and
compression from surgical procedures, such as fundoplication
potassium chloride. Preventive strategies include drinking
or bariatric surgery (gastric band), can also cause secondary
sufficient water with medication and remaining in an upright
achalasia. Barium esophagography is the initial diagnostic test,
position for 30 minutes after pill ingestion.
which shows dilation of the esophagus with narrowing at the
gastroesophageal junction (GEJ), known as a "bird's beak~
(Figure 5). Upper endoscopy reveals retained food and saliva,
no signs of mechanical obstruction or mass, and "tightness" at
the GEJ while advancing the scope into the stomach.
Esophageal manometry showing incomplete LES relaxation
and aperistalsis confirms the diagnosis of achalasia.
Pseudoachalasia is a result of malignant tumor infiltra-
tion or other secondary causes, and can present similarly to
achalasia causing myenteric plexus damage. Unlike achalasia,
this condition has been associated with sudden weight loss
later in life, usually after age SO years. Suspected pseudoacha-
lasia should be evaluated with CT or endoscopic ultrasound.
FIGURE 4. Upper endoscopy showing white adherent plaques suggestive of Approaches to treatment of achalasia include endoscopic
Candida esophagitis. or surgical intervention with the goal of lowering LES

6
Disorders of the Esophagus

Diffuse Esophageal Spasm and


Nutcracker Esophagus
Diffuse esophageal spasm is a hypercontractile state present-
ing with chest pain or dysphagia. Symptoms often respond to
nitroglycerin, suggesting a flaw in esophageal nitric-oxide pro-
duction. The esophagus has a "corkscrew" (Figure 6) or "rosary
bead" appearance on esophagography. Esophageal manometry
shows simultaneous high-amplitude (>30 mm Hg) esophageal
contractions with intermittent aperista!Uc contractions.
Medical therapy with antidepressants (trazadone and imipra-
mine) or a phosphodiesterase inhibitor (sildenafil) can relieve
chest pain. Dysphagia may respond to calcium channel block-
ers. Botulinum toxin injection has been reported to alleviate
dysphagia symptoms.
"Nutcracker" or "jackhammer" esophagus is found in
patients with high-amplitude peristaltic contractions of
greater than 220 mm Hg.

Hypotonic Motility Disorders


Hypotonic disorders of the esophagus are marked by lack of
FIGURE 5. Barium esophagram showing the typical appearance of a dilated
contractility and incomplete peristalsis. Patients may report
esophagus and "bird's beak" narrowing at the gastroesophageal junction in a
patient with achalasia. symptoms of GERD, which result from decreased LES pres-
sure or dysphagia from incomplete peristalsis. In most cases,
the cause of hypotonic esophageal disease is unknown.
pressure, which relieves symptoms. Medical therapy is uncom- However, secondary causes include smooth-muscle relax-
monly used. ants, anticholinergic agents, estrogen, progesterone, connec-
Endoscopic therapy involves injection of botulinum tive tissue disorders (scleroderma), and pregnancy.
toxin, pneumatic dilation, or peroral endoscopic myotomy. Esophageal manometry shows hypotonic, weak nonperistal-
Botulinum toxin injection into the LES inhibits acetylcholine tic contractions in the distal esophagus. Findings can mimic
release, causing LES relaxation. However, after 1 year, only achalasia.
40% of patients have continued relief of symptoms.
Botulinum toxin injection can be repeated, but successive
treatments are often less effective for symptom management.
Pneumatic dilation is an effective nonsurgical therapy.
Dilators ranging from 30 mm to 40 mm in size are used to
disrupt the circular muscle. Clinical symptom improvement
ranges from 50% to 90%, and the most common complica-
tion is perforation. Peroral endoscopic myotomy is a newer
endoscopic procedure in which the physician creates an
esophageal submucosal tunnel extending to the level of the
LES and then performs a myotomy. Studies have shown reso-
lution of symptoms in over 80% of patients. Surgical treat-
ment consists of laparoscopic myotomy of the circular
muscle fibers. Fundoplicatlon Is also recommended to avoid
reflux symptoms after myotomy. Determining whether a
patient should receive endoscopic or surgical therapy often
depends on local expertise.
Medical therapy Is reserved for patients who are poor
candidates for endoscopic or surgical therapy. LES pressure
can be reduced with medical therapy, including calcium chan-
nel blockers (nifedipine) or long-acting nitrates.
Patients with achalasia for more than 10 years have
increased risk for squamous cell carcinoma and may benefit
from surveillance endoscopy, but there are no established FIGURE 6. Findings of a 'corkscrew esophagus• (caused by multiple simultaneous
guidelines for frequency of surveillance. contractions) on esophagography are typical of diffuse esophageal spasm.

7
Disorders of the Esophagus

Treatment includes lifestyle changes, such as eating risk for cancer is o. 7% per year, and for Barrett esophagus with
upright and eating liquid or semisolid rather than solid food. high-grade dysplasia, 7% per year.
Medical therapy includes acid-reducing agents for GERD and About 10% of patients with GERO are found to have
low-dose antidepressants to reduce chest discomfort. Barrett esophagus on endoscopy. Studies have suggested indi-
Prokinetic agents, such as metoclopramide, are not recom- viduals with multiple risk factors for esophageal carcinoma
mended for treatment. and chronic GERD might benefit from screening. Men older
than age 50 years with GERO symptoms for more than 5 years
KEY POINTS and additional risk factors (nocturnal reflux symptoms, hiatal
HVC • ln patients with symptoms of gastroesophageal reflux hernia, elevated BMI, intra-abdominal distribution of body fat,
disease but without alarm features, an empiric trial of a tobacco use) may benefit from screening endoscopy. Women
proton pump inhibitor can reli.eve symptoms and con- do not require routine endoscopic screening for Barrett esoph-
firm the diagnosis. agus. Evidence does not support routine screening for Barrett
esophagus based on GERD symptoms for the general
• Most patients with gastroesophageal reflux disease do
population.
not require surgery; indications include failure of opti-
mal proton pump inhibitor therapy, wanting to stop Diagnosis and Management
medication, and intolerable medication side effects. The diagnosis of Barrett esophagus is established based on
• The diagnostic criteria for eosinophilic esophagitis are endoscopic findings (Figure 7) with biopsy, which are then
esophageal symptoms (most commonly dysphagia), confirmed by pathology showing specialized intestinal meta-
esophageal biopsies showing persistent counts oflS plasia with acid-mucln- containing goblet cells. Endoscopy
eosinophils/hpf or greater despite a trial of proton measurements have categorized Barrett esophagus into short-
pump inhibitor therapy, and exclusion of other causes segment (:5:3 cm) or long-segment (>3 cm). The Prague classi-
of eosinophilia. fication more accurately assesses a segment of Barrett esopha-
• Odynophagia and dysphagia are the most common pre- gus using the degree of circumferential disease (C followed by
senting symptoms of infectious esophagi tis. a number indicating the length in cm) along with maximal
Barrett esophagus segment length (M followed by a number
• To prevent pill-induced esophagitis, patients should
indicating the length in cm). For example, a patient with
drink water with medication and remain upright for at
Barrett esophagus extending circumferentially for 2 cm above
least 30 minutes after ingestion.
the squamocolumnar junction but with tongues of Barrett

Metaplastic and Neoplastic


Disorders of the Esophagus
Barrett Esophagus
Epidemiology and Screening
Barrett esophagus is defined as the extension of the metaplastic
columnar epithelium above the GEJ into the esophagus. Barrett
esophagus is a consequence of GERD, even in patients who
experience no clinical symptoms, and is classified as a prema-
lignant condition because it has the potential to progress to
esophageal cancer. Risk factors associated with Barrett esopha-
gus include chronic GERD (for more than 5 years), age older
than 50 years, male sex, white race, tobacco use, and obesity.
Orlnking alcohol is not associated with increased risk for Barrett
esophagus, and wine consumption might be protective.
Risk factors associated with Barrett esophagus progres-
sion to dysplasia or esophageal cancer include older age, Jong
Barrett esophagus length, obesity, and tobacco use. Some stud-
ies have suggested that the use of PPJs, NSAIDs, and statins
may be protective; however, this has not been established, and
the use of these agents for prevention of progression to dyspla-
sia ls not recommended. The annual cancer risk associated Fl GU RE 7. Upper endoscopic view of Barrett mucosa, with salmon-colored
with Barrett esophagus without dysplasia is 0.2% to O.S'Yo per mucosa representing Barrett mucosa compared with the normal pearl-colored
year. For Barrett esophagus with low-grade dysplasia, annual squamous mucosa.

8
Disorders of the Esophagus

esophagus extending S cm above the squamocolumnar junc- Risk Factors


tion would have a Prague classification ofC2MS. Risk factors for the development of adenocarcinoma include
Barrett esophagus progresses along a pathway from intesti- GERO, Barrett esophagus, obesity, tobacco use, past thoracic
nal metaplasia, to indefinite for dysplasia, to low-grade dyspla- radiation, a diet low in fruits and vegetables, older age, male
sia, to high-grade dysplasia, to invasive adenocarcinoma. sex, and the use of medications that relax the LES. Risk factors
Surveillance and treatment recommendations are based on for squamous cell carcinoma include tobacco and alcohol use,
grade ofBarrett esophagus and the presence ofclysplasia (Table 4). caustic injury, achalasia, past thoracic radiation, nutritional
Chemoprevention is recommended in the form of PP! deficiencies (zinc, selenium), poor socioeconomic status, poor
therapy, but dosing should be based on symptom control and oral hygiene, nonepidermolytic palmoplantar keratoderma
healing of erosive esophagitis. initial dosing is once daily. (an autosomal dominant disorder associated with yellow, wax-
There is no strong evidence that antireflux surgery can prevent like hyperkeratosls on the palms and soles, also known as
the progression of Barrett esophagus to adenocarcinoma when tylosis), human papillomavirus infection, and nitrosaroine
compared to medical therapy with PP!s. NSAIDs and aspirin exposure.
should not be prescribed as an antineoplastic strategy.
Patients with a diagnosis of Barrett esophagus indefinite for Diagnosis and Staging
dysplasia should start or optimize PPI therapy, followed by repeat The most common initial presentation of esophageal carci-
upper endoscopy. Treatment to remove Barrett esophagus is rec- noma is dysphagia with solid foods, but asymptomatic indi-
ommended for patients with high-grade dysplasia and confirmed viduals have been diagnosed based on surveillance endoscopy.
low-grade dysplasia. Endoscopy-based therapies include radiof- Other symptoms include weight loss, anorexia, anemia (sec-
requency ablation and endoscopic mucosa! resection, possibly ondary to gastrointestinal bleeding) , and chest pain. Upper
used in combination. Endoscopic therapies have had similar endoscopy with biopsy is the preferred diagnostic test
outcomes to surgery (esophagectomy) in patients with high- Squamous cell carcinoma is most commonly located in
grade dysplasia, but local expertise and patient preference will the proximal esophagus, and adenocarcinoma is usually found
determine the best course of therapy. The Prague classification is in the distal esophagus.
useful to more accurately assess response to endoscopic therapy. TNM staging is often done with endoscopic ultrasound for
locoregional disease and with CT and PET to identify meta-
Esophageal Carcinoma static disease. For treatment of esophageal carcinoma, see
MKSAP 18 Hematology and Oncology.
Epidemiology
The incidence of esophageal cancer varies widely between KEY POINTS
regions of the world; high-prevalence areas include Asia and
• Barrett esophagus is a premalignant condition caused
southern and eastern Afiica. Worldwide, squamous cell carci-
by longstanding gastroesophageal reflux disease.
noma comprises about 90% of all esophage-al cancers, but inci-
dence has been decreasing in Western countries as the incidence • Women with gastroesophageal reflux disease do not HVC
of adenocarcinoma is rising. Esophageal cancer occurs in the fifth require routine screening for Barrett esophagus.
to seventh decade ofllfe and ls three to four times more common • Patients with Barrett esophagus should receive proton
in men. In the United States, about 16,000 new cases are reported pump inhibitor therapy with dosing based on symptom
annually, with lS,000 deaths occurring within the same year. The relief and healing of erosive esophagitis.
overall 5-year survival rate ranges from 15% to 25%, depending on • Dysphagia with solid foods is the most common pre-
the stage of the cancer at the time of initial presentation. The rate senting symptom of esophageal cancer.
has remained relatively unchanged since 2000.

TABLE 4. Practice Guidelines for Endoscopic Surveillance of Barrett Esophagus


Dysplasia Grade Recommendation

None If no dysplasia is present, repeat upper endoscopy every 3 to 5 years


Indefinite Start or adjust proton pump inhibitor therapy, then repeat endoscopy in 3 to 6 months
If still present, then repeat endoscopy in 1 year
Low-grade Confirmation by expert pathologist. then proceed to endoscopic eradication therapy (surveillance
endoscopy in 12 months is an alternative)
High-grade Confirmation by expert pathologist
Endoscopic eradication therapy is preferred

Adapted w~h permission from Shaheen NJ. Falk GW, Iyer PG. Gerson LB; American College. of Gastroenterology. ACG clinical guideline: diagnosis and management of Barrett's
esophagus. Am J Gastroenterol. 2016;111 :30-50; quiz S 1. (PMIO: 265260791doi: 10. I 038/aig.2015.322.

9
Disorders of the Stomach and Duodenum

the fourth Rome working group for functional gastrointestinal


Disorders of the Stomach disorders are shown in Table 6. Functional dyspepsia can be
and Duodenum further categorized into postprandial distress syndrome if the
patient's predominant symptoms are meal-induced (Table 7) ,
Dyspepsia or epigastric pain syndrome if the predominant symptoms of
Clinical Features epigastric pain or burning are unrelated to meals (Table 8).
Dyspepsia is a term used to describe epigastric pain. Associated
symptoms can include early satiety, epigastric burning, nau- Evaluation and Management
sea, and postprandial symptoms, including pain, bloating, or The approach to testing for dyspepsia should be based on
belching. Coexistent heartburn may be present, but it is not a patient age, severity of symptoms, and risk for gastric cancer.
primary symptom of dyspepsia. Recurrent or bothersome Upper endoscopy is commonly ordered to exclude upper gas-
vomiting should raise concern for other conditions. trointestinal neoplasia; however, it is invasive and costly. Given
The list of possible organic causes of dyspepsia is exten- the low incidence of malignancy and consideration for cost
sive (Table 5); the most corrunon conditions are peptic ulcer effectiveness in the evaluation of dyspepsia in younger adults,
disease and gastroesophageal reflux disease (20% of patients the American College of Gastroenterology (ACG) and the
with erosive esophagitis present with dyspepsia). If no organic Canadian Association of Gastroenterology (CAG) have jointly
cause of dyspepsia symptoms is identified after testing, the recommended routine use of upper endoscopy only in patients
term functional dyspepsia applies. aged 60 years and older to exclude malignancy. The ACG/CAG
functional dyspepsia represents a heterogeneous group of guidelines recommend against the routine use of upper endos-
functional gastrointestinal disorders predominated by one or copy in patients younger than age 60 years, even in the pres-
more upper gastrointestinal symptoms. Diagnostic criteria from ence of alarm features including weight loss, anemia, dysphagia,

TABLE 6. Rome 4 Diagnostic Criteria for


Functional Dyspepsia
Gastroesophageal reflux disease, peptic
ulcer disease, Helicobacter pylori One or more ofthe following:•
gastritis, gastric cancer/lymphoma,
gastric amyloidosis, Menetrier disease, Bothersome postprandial fullness
gastroparesis Bothersome early satiation
Small bowel Celiac disease, Crohn disease Bothersome epigastric pain
Pancreatic Pancreatitis, pancreatic cancer Bothersome epigastric burning

d
Infectious Giardia lamblia, Strongyloides No evidence of structural disease (including upper endoscopy)
stercoralis, tuberculosis, syphilis j that is likely to explain the symptoms
Common NSAIDs, aspirin, iron, antibiotics I 'Criteria fulfilled for the last 3 months with symptom onset at least 6 months before
medications (erythromycin, ampicillin), narcotics, I diagnosis.
estrogens and oral contraceptives, Reprinted with permission from Stanghellini V, Chan FK, H•sler WL, Malagelada JR,
theophylline, levodopa, digitalis Suzuki H, Tack J, et al. Gastroduodenal disorders. Gastroenterology.
2016;150:1380·92. [PMID: 27147122] doi:10.1053/j.gastro.2016.02.011. Copyright
Other systemic Coronary artery disease, kidney disease, 2016, The AGA Institute.
conditions thyroid dysfunction, adrenal insufficiency,
hyperparathyroidism, pregnancy

TABLE 7. Rome 4 Diagnostic Criteria for Postprandial Distress Syndrome


Criteria• Supportive Remarks

Must include one or both ofthe following at least 3 days Postprandial epigastric pain or burning, epigastric bloating, excessive
per week: belching, and nausea can also be present
Bothersome postprandial fullness (i.e., severe enough Vomiting warrants consideration of another disorder
to affect usual activities)
Heartburn is not a dyspeptic symptom but may often coexist
Bothersome early satiation (i.e., severe enough to
Symptoms that are relieved by evacuation of feces or gas should generally
prevent finishing a regular-sized meal)
not be considered as part of dyspepsia
No evidence of organic, systemic, or metabolic disease
that is likely to explain the symptoms on routine
Other individual digestive symptoms or groups of symptoms, such as from I
investigations (including upper endoscopy)
gastroesophageal reflux disease and irritable bowel syndrome, may coexist I
1

I
-Criteria fulfilled for the last 3 months with symptom onset at least 6 months before diagnosis.

Reprinted with pennission !Tom Stanghellini Y, Chan FK, HaslerWL, Malagefada JR, Suzuki H. Tack J, et al. Gastroduodenal disorders. Gastroenterology. 2016;150:1380-92.
(PMID: 27147122) doi: 10.1053/j.gastro.2016.02.D11. Copyright 2016, The AGA Institute.
'--~~~~~--~~~~~~~--~~~~~~~~~~~~~~~~~~~~~~~~~~~~~~~~---------

10
Disorders of the Stomach and Duodenum

TABLE 8. Rome 4 Diagnostic Criteria for Epigastric Pain Syndrome


Criteria• Supportive Remarks

Must include the following symptoms at least Pain may be induced by ingestion of a meal, relieved by ingestion of a meal, or may
1 day per week: occur while fasting
Bothersome epigastric pain and/or burning Postprandial epigastric bloating, belching, and nausea can also be present
(i.e., severe enough to affect usual activities)
Persistent vomiting suggests another disorder
No evidence of organic, systemic, or metabolic
disease that is likely to explain the symptoms on Heartburn is not a dyspeptic symptom but may often coexist
routine investigations (including upper The pain does not fulfill biliary pain criteria
endoscopy)
Symptoms that are relieved by evacuation of feces or gas should generally not be
considered as part of dyspepsia
Other individual digestive symptoms or groups of sym ptoms, such as from
gastroesophageal reflux disease and irritable bowel syndrome, may coexist
•Criteria fulfilled for the last 3 months with symptom onset at least 6 months before diagnosis.

Reprinted with permission from Stanghellini V, Chan FK, Hasler WL, Malagelada J R, Suz.uki H, Tack J, et al. Gastroduodenal disorders. Gastroenterology. 2016; 150: 1380-92.
[PMID: 271471221doi:10.1053/j.gastro.2016.02.011. Copyright 2016, The AGA Institute .

persistent vomiting, a family history of gastric cancer, immi- Peptic Ulcer Disease
gration from a region with increased risk for gastric cancer
(Asia, Russia, and South America), and severe symptoms, Clinical Features, Diagnosis, and Complications
because these features are poor predictors of organic pathol- The most frequently reported symptom of peptic ulcer dis-
ogy, such as malignancy, peptic ulcer disease, or esophagi.tis. ease (PUD) is epigastric pain, frequently described as worse
The ACG/CAG guidelines also state that upper endoscopy can during fasting and improved with eating or use of antacid or
be considered regardless of age In patients with a combination antisecretory therapy. Pain may be accompanied by other
of alarm features. dyspeptic symptoms, such as early satiety, abdominal bloat-
For patients younger than age 60 years with dyspepsia ing, nausea, belching, or heartburn (from coexistent
symptoms, the ACG/CAG guidelines recommend noninvasive esophagi.tis). Epigastric pain may be minimal or absent in the
testing for Helicobacter pylori. Other testing, such as routine elderly, immunosuppressed patients, and chronic NSAlD
laboratory testing, screening for celiac disease, abdominal users.
imaging, or gastric emptying, should be considered on an indi- NSAIDs and H. pylori infection are the two most common
vidual basis. causes of PUD, although idiopathic (NSAlD-negative, H. pylori-
Management of dyspepsia is directed at treatment of the negative) PUD is becoming more common in the United States.
underlying cause. Patients who test positive for H. pylori The causes of idiopathic PUD remain unclear; however,
should receive eradication therapy. If H. pylori testing is nega- cocaine, methamphetamine, bisphosphonates, selective sero-
tive or H. pylori eradication fails to relieve the dyspepsia, an tonin reuptake inhibitors, smoking, excessive alcohol con-
empiric trial of a proton pump inhibitor (PPI) should be pur- sumption, and stress have been implicated. Rare causes of
sued for 4 weeks. Patients whose symptoms are not alleviated PUD include gastrinoma (Zollinger-Ellison syndrome), sys-
with PPI therapy should undergo further evaluation with an temic mastocytosis, ~ -antitrypsin deficiency, COPD, chronic
upper endoscopy. Patients diagnosed with functional dyspep- kidney disease, gastric cancer, gastric lymphoma, Crohn dis-
sia after the exclusion of organic disease can be treated with a ease, eosinophilic gastroenteritis, or cytomegalovirus (in
variety of interventions (Figure 8). i.mmunocompromised patients).
The diagnosis of PUD is most often made by upper endos-
KEY POINTS
copy. Abdominal CT is the diagnostic study of choice for sus-
HVC • For patients with dyspepsia, routine upper endoscopy pected perforating PUD, with a sensitivity of 98%.
to exclude malignancy is reserved for patients older Complications of PUD include bleeding, perforation, and
than age 60 years. obstruction. PUD is the most common cause of upper gastro-
• Upper endoscopy should be considered for patients intestinal bleeding. Overt bleeding presents with melena,
with alarm features such as a family history of gastric hematemesis, and/or hematochezia, whereas obscure bleed-
cancer, immigration from a region with increased risk ing presents with iron deficiency anemia and/or stool positive
for gastric cancer, or severe symptoms, regardless of age. for occult blood. Perforation is likely to present with sudden,
severe epigastric pain that can become more generalized,
HVC • Patients younger than age 60 years with dyspepsia
symptoms should be tested and treated for He/icobacter along with peritoneal findings. Symptoms associated with
obstruction may include vomiting, early satiety, abdominal
pylori infection.
distension, and weight loss.

11
Disorders of the Stomach and Duodenum

Dietary/Lifestyle Interventions
Avoidance of food triggers
Smaller, more frequent meals
low-fat diet
Elimination diets (dairy, gluten, FODMAP)
Avoidance of NSAIDs, coffee, alcohol,

l
and smoking

Eplgastric Pain Syndrome

Antisecretory Agents
l No relief of
symptoms Postprandial Distress Syndrome

Prokinetic Agents
(if not already tried) Metoclopramide (use for short term and with
Proton pump inhibitor caution)
Histamine receptor antagonist Domperidone (where available)
Buspirone

No relief of
symptoms

Antidepressants
Tricyclic antidepressants
Mirtazapine

No relief of
,1r symptoms

Referral for additional functional testing and


alternative therapies

FIGURE 8. Treatment offunctional dyspepsia.


FODMAP - FermenlableOligosmharides, Disaa:harides, Monosaccharides,And Polyols.

Management should be discontinued in patients with NSAID-induced


Following confirmation of uncomplicated PUD by upper bleeding PUD. If there is no effective alternative to NSAIDs, a
endoscopy, H. pylori infection should be treated if identified selective cyclooxygenase-2 inhibitor plus a once-daily PPI
and eradication confirmed after treatment. If H. pylori testing should be used. Patients with idiopathic ulcers that have bled
is negative, a once-daily PPI should be initiated and any agent should receive once-daily PPI therapy indefinitely because of
causing PUD (such as an NSAID) should be discontinued. the significant risk for rebleeding.
In patients with bleeding PUD, risk factors that increase Repeat upper endoscopy is not performed routinely.
the risk for recurrent bleeding and death include tachycardia, Reasons for a follow-up upper endoscopy may include persis-
hypotension, age older than 60 years, hemoglobin level less tent symptoms after 8 to 12 weeks of therapy, ulcers of
than 7 g/dL (70 g/L) (8 g/dL [80 g/L] in patients with cardio- unknown cause, and if gastric ulcer biopsy was not performed
vascular disease), and comorbid illness. Interventions to during the initial upper endoscopy.
address the modifiable risks should be pursued before upper Perforated PUD is a surgical emergency with a mortality
endoscopy. rate of up to 30%; older age, comorbidity, and delayed surgery
Upper endoscopy should be performed within 24 confer increased risk. Prompt identification, urgent surgical
hours. The use of PPis before upper endoscopy reduces the intervention. and proper pre- and postoperative management
need for endoscopic therapy only in patients with high-risk of sepsis are essential. Postoperative upper endoscopy to rule
ulcer findings. An intravenous dose of erythromycin 30 out gastric cancer should be considered for patients with per-
minutes before upper endoscopy reduces the need for repeat forating gastric ulcers.
endoscopy by improving gastroduodenal visualization. Gastric outlet obstruction resulting from inflammation or
Nasogastric tube placement is not required before upper scarring at the pylorus or proximal duodenum should be biop-
endoscopy. sied at the time of diagnosis to exclude malignancy. For mild
In patients with bleeding PUD related to low-dose aspirin symptoms, PPI therapy, treatment of H. pylori infection if
use for secondary cardiovascular prevention, aspirin should be Identified, and/or cessation of NSAIDs may be effective. For
restarted I to 7 days after cessation of bleeding. PPl therapy can severe or persistent symptoms, endoscopic dilation should be
be discontinued after confirmation of H. pylori eradication in pursued, reserving surgery for obstruction that persists after
patients with H. pylori-associated bleeding PUD. NSAlDs more than two attempts at endoscopic dilation.

12
Disorders of the Stomach and Duodenum

KEY POINTS gastric biopsies for H. pylori testing by histology or a rapid


urea test.
• NSA!Ds and Helicobacter pylori infection are the two
most common causes of peptic ulcer disease.
Treatment
• Upper endoscopy is the diagnostic test of choice for Any patient with a test result that is positive for active infec-
peptic ulcer disease.
tion requires treatment with the goal of H. pylori eradication.
• Repeat upper endoscopy is reserved for patients with The fl rst course of treatment offers the best chance of eradica-
persistent symptoms after 8 to 12 weeks of therapy, tion and should be carefully chosen. Several first-line treat-
ulcers of unknown cause, or if gastric ulcer biopsy was ments are available, with the choice based on resistance pat-
not performed during the initial upper endoscopy. terns of H. pylori, previous antibiotic use by the patient, and
antibiotic allergies (Table 9). In most cases, duration of therapy
is 14 days. Second-line therapy in the event of treatment failure
Helicobacter pylori Infection should be for a minimum of 14 days, ideally using antibiotics
different from those used for initial treatment (Table 10).
Indications for Helicobacter pylori Testing
Indications for H. pylori testing include active PUD, history Eradication Testing
of PUD without documented cure of H. pylori infection, gas-
Testing for eradication of H. pylori should be done after com-
tric mucosa-associated lymphoid tissue lymphoma, or his-
pletion of eradication therapy, using a 13 C-urea breath test, fecal
tory of endoscopic resection of early gastric cancer. Other
antigen test, or biopsies obtained during upper endoscopy at
conditions in which H. pylori testing should be considered
least 4 weeks after completion of eradication therapy. Serologic
include dyspepsia in patients younger than age 60 years
testing is not used to confirm H. pylori eradication because it
without alarm features warranting early upper endoscopy,
can remain positive in the absence of active infection.
before long-term low-dose aspirin or NSAID use, unex-
plained iron deficiency anemia, and idiopathic thrombocyto- KEY POINTS
penic purpura in adult patients with gastrointestinal symp- • Patients with active peptic ulcer disease, history of
toms. Routine testing for H. pylori is not indicated in patients peptic ulcer disease without documented cure of
with typical gastroesophageal reflux disease symptoms, lym- Helicobacter pylori infection, gastric mucosa-associated
phocytlc gastritis, hyperplastic gastric polyps, or hyperemesis lymphoid tissue lymphoma, or history of endoscopic
gravidarum, or in asymptomatic individuals with a family resection of early gastric cancer must be tested for
history of gastric cancer. H. pylori and treated if positive.
• Gastric biopsies during upper endoscopy or noninvasive
Diagnosis testing methods, including 13C-urea breath and stool
The diagnosis of H. pylori infection can be made with gastric antigen testing, can confirm the presence of H. pylori
biopsies during upper endoscopy, or noninvasively using infection; negative serologic testing can exclude infec-
13 C-urea breath, stool antigen, or serologic testing. The
tion, but a positive serologic result requires confirmation.
13 C-urea breath and the monoclonal stool antigen are the pre-
• Treatment regimens for H. pylori consist of a minimum
ferred noninvasive tests because both have sensitivity and
of three agents, including two antimicrobial agents and
specificity greater than 95% for evidence of active infection.
one antisecretory agent; treatment duration is 14 days in
Serologic testing for lgG antibodies against H. pylori is the
most cases.
cheapest and most convenient method; however, it has the
significant disadvantage of being unable to distinguish • Testing for eradication of H. pylori is performed using
between active and previous infection. Although the negative a 13 C-urea breath test, fecal antigen test. or biopsies
predictive value of serologic testing is reasonably high, a posi- obtained during upper endoscopy at least 4 weeks after
tive serologic result in a population with low prevalence of H. completion of eradication therapy.
pylori infection, such as in the United States, requires confir-
mation with the 13C-urea breath test, stool antigen test, or
gastric biopsy. Also, conditions that lower the H. pylori den-
Miscellaneous Gastropathy
sity, such as PPI use, recent antibiotic use, atrophic gastritis,
intestinal metaplasia, or mucosa-associated lymphoid tissue Atrophic Gastritis
lymphoma, can result in false-negative results for any of the Chronic atrophic gastritis can present as either an environ-
noninvasive studies. Serologic testing is most useful as an mental metaplastic atrophic gastritis (EMAG), also called mul-
adjunct test in patients with bleeding PUD because of the ti focal atrophic gastritis, or an autoimmune atrophic gastritis
decreased sensitivity of biopsy in the setting of acute bleed- (AMAG}. EMAG is the result of H. pylori infection, and typi-
ing. Any upper endoscopy for the evaluation of dyspepsia, or cally improves after H. pylori eradication. AMAG is caused
with the finding of gastric erosions or ulcers, should include by autoantibody formation to parietal cell antigens, and is

13
Disorders of the Stomach and Duodenum

TABLE 9. First·Line Treatment Options for Helicobacter pylori Infection


Treatment Regimen Duration of Therapy Clinical Indicators
PPI, standard• or double dose twice daily 14 days H. pylon clarithromycin resistance is known to be <15%
(esomeprazole, once daily only)
No previous history of macrolide exposure for any reason
Clarithromycin, 500 mg twice daily
Amoxicillin, 1 g twice daily
PPI, standard or double dose twice daily 14 days H. pylori clarithromycin resistance is known to be <15%
Clarithromycin, 500 mg twice daily No previous history of macrolide exposure for any reason
Metronidazole, 500 mg three times daily Penicillin allergy
PPI, standard dose twice daily 10-14 days Previous macrolide exposure
Bismuth subcitrate, 120-300 mg or Penicillin allergy
subsalicylate, 300 mg four times daily
Tetracycline, 500 mg three times daily
Metronidazole, 250 mg four times daily or
500 mg three times daily
PPI, standard dose twice daily 10-14 days May be an alternative to standard clarithromycin triple therapy
Clarithromycin, 500 mg twice daily Not validated in North America
Amoxicillin, 1 g twice daily
Nitroimidazole, 500 mg twice daily
PPI, standard dose twice daily 10-14days May be an alternative to standard clarithromycin triple therapy
Levofloxacin, 500 mg twice daily Not validated in North America
Amoxicillin, 1 g twice daily

PPI •proton pump inhibitor.


•St•nd•rd dose PPls: esomeprazole, 40 mg; lansoprazole, 30 mg; omepr•zole, 20 mg; pantopr•zole, 40 mg; rabeprazole, 20 mg.
Adapted with permission from Chey WD, Leonti•dis Gt, Howden CW, Moss SF. ACG clinical guideline : treatment of Helicobacter pylori infection. Am J Gastroenterol.
2017;112:212-239. [PMID: 280716591doi:10.1038/ajg.2016.563.

TABLE 10. Second-Line Treatment for Helicobacter pylori Infection after Failure of Initial Treatment
Treatment Regimen Duration of Therapy Clinical Indicators

PPI, standard dose twice daily 14 days Failure of clarithromycin-based or


levofloxacin-based therapy
Bismuth subcitrate, 120-300 mg, or subsalicylate, 300 mg four times daily
Tetracycline, 500 mg three times daily
Metronidazole, 250 mg four times daily, or 500 mg three times daily
PPJ, standard dose twice daily 14 days Failure of bismuth-based or
clarithromycin-based therapy
Levofloxacin, 500 mg twice daily
Amoxicillin, 1 g twice daily
PPI, standard or double dose twice daily 14 days Failure of bismuth-based therapy
Clarithromycin, 500 mg twice daily
Amoxicillin, 1 g twice daily
PPI, standard or double dose twice daily 14 days Failure of bismuth-based therapy
Clarithromycin, 500 mg twice daily Penicillin allergy
Metronidazole, 500 mg three times daily
PPI • proton pump inhibitor.
Adapted with permission from Chey WD, Leontiadis GI, Howden CW, Moss SF. ACG clinical guideline: treatment of Helicobacter pylori infection. Am J Gastroenterol.
2017; 112:212-239. IPMID: 28071659) doi:l 0.1038/ajg.2016.563.

14
Disorders of the Stomach and Duodenum

commonly associated with pernicious anemia. The achlorhy- KEY POINTS


dria from chronic atrophic gastritis can lead to iron defi-
• Autoimmune atrophic gastritis is associated with perni-
ciency, small intestinal bacterial overgrowth, and enteric
cious anemia, iron deficiency, small intestinal bacterial
infections. The hypergastrinemia associated with AMAG can
overgrowth, and gastric cancer.
promote the development of gastric carclnoid. Goals of ther-
apy for patients with AMAG include the prevention of perni- • Helicobacter pylori is associated with upper gastroin-
cious anemia and iron deficiency with vitamin B12 supple- testinal conditions other than peptic ulcer disease
mentation and iron replacement, and surveillance for gastric including environmental metaplastic atrophic gastritis,
neoplasm. There are no universally accepted surveillance gastric intestinal metaplasia. and, rarely, lymphocytic
protocols for gastric neoplasm in the United States. A screen- gastritis.
ing upper endoscopy with gastric biopsy is recommended in
the setting of pernicious anemia.
Gastrointestinal Complications
Intestinal Metaplasia of NSAIDs
Gastric intestinal metaplasia is a preneoplastic gastropathy Epidemiology and Risk Factors
arising from chronic inflammation associated with H. pylori Upper gastrointestinal complications associated with NSAID
Infection. Other causes of chronic inflammation, including use can occur with short- or long- term NSAID use and are
other gastric infections, chemical agents, and autoimmune dose dependent, with a linear increase in incidence over time
disease, may also promote progression to gastric intestinal with chronic NSAlD use. Nearly 1% to 2% of chronic NSAJDs
metaplasia. There is no conclusive evidence that long- term PPI users suffer a clinically significant upper gastrointestinal event
use promotes the development of intestinal metaplasia. Gastric (such as a bleeding ulcer, perforation, or obstruction) annually.
intestinal rnetaplasia is believed to be an intermediary stage in The rate of upper gastrointestinal adverse events rises to 14%
the multistage progression from chronic atrophic gastritis to for elderly NSATD users. The greatest risk factor for an NSAID-
gastric adenocarcinoma. Given the rare occurrence of gastric related upper gastrointestinal bleed is a history of gastrointes-
adenocarcinoma in the United States, there are no universally tinal bleeding. Other risk factors are listed in Table ll.
accepted surveillance strategies. Surveillance programs do The use of low-dose aspirin for cardioprophylaxis is asso-
exist In other parts of the world, and endoscopic surveillance ciated with a two- to fourfold increase in risk for upper gastro-
should be considered in patients at increased risk, such as intestinal complications including bleeding ulcer, perforation,
those with a family histozy of gastric cancer or who have and obstruction. Use of enteric-coated aspirin does not lower
immigrated from East Asia, Russia, or South America. this risk, and an increase In aspirin dosage is associated with
an increased risk for upper gastrointestinal complications.
Eosinophilic Gastritis
Eosinophilic gastritis is a rare gastropathy characterized by Prevention of NSAID-lnduced Injury
infiltration of eosinophils in the stomach. Secondary causes of Treatment strategies for the prevention of NSAlD-related upper
eosinophtlla should first be excluded, including parasitic and gastrointestinal adverse events include the avoidance ofNSAJDs,
bacterial infections of the stomach, inflammatory bowel dis- addressing modifiable risk factors, use of a selective cyclooxy-
ease, hypereosinophilic syndrome, myeloproliferative disor- genase (COX)-2 inhibitor, or coadministration of gastroprotec-
ders, polyarteritls, allergic vasculitis, scleroderrna, drug injuzy, tive agents, such as PPis, misoprostol, or H2 blockers. PPis are
and drug hypersensitivity. The cause of primary eosinophilic the preferred gastroprotective agent for the treatment and
gastritis is unknown, but believed to be an allergic process. prophylaxis ofNSAID-related (including aspirin-related) upper
Symptoms vary widely based on depth of eosinophilia and
organ involvement, as the small bowel may also be involved.
Treatment includes avoidance of food allergens and use of TABLE 11 . Risk Factors for NSAID-Related Upper
elemental diets and/or glucocorticoids. Gastro intest inal Complications

History of upper gastrointestinal bleeding


Lymphocytic Gastritis History of peptic ulcer disease
Lymphocytic gastritis Is a rare gastropathy typically presenting
Helicobacter pylori infection
with mild, nonspecific dyspeptic symptoms and a normal-
appearing stomach on endoscopy. On occasion, the stomach
I Age older than 65 years

may have thickened folds covered by small nodules and aph- Hemodialysis o r p eritoneal dialysis

thous ulceration. Celiac disease is the most common cause of Use of high-dose o r multiple NSAIDs
lymphocytic gastritis. Other causes include HIV infection, Concomitant use of aspirin (even low-dose), nonaspirin
Crohn disease, common variable immunodeficiency, and, ' anti platelet agents, anticoagulants, oral glucocorticoids,
selective serotonin reuptake inhibitors
rarely, H. pylori infection.

15
Disorders of the Stomach and Duodenum

gastrointestinal complications, and are specifically superior to


Gastroparesis
H2 blockers in the prevention of PUD and bleeding related to
low-dose aspirin use. Misoprostol may be used instead of PP!s Presentation
in patients with intolerance or unwillingness to take PP!s. Gastroparesis is a heterogeneous clinical syndrome with
However, side effects of misoprostol, such as diarrhea, abdomi- three components to the diagnosis: the presence of specific
nal discomfort, and nausea, can be limiting, particularly at symptoms, absence of mechanical outlet obstruction, and
therapeutic doses, and it is contraindicated in pregnancy. objective evidence of delay in gastric emptying into the duo-
When coadministered with aspirin, a selective COX-2 denum. The most commonly reported symptoms in order of
inhibitor provides no advantage over an NSAlD in the preven- prevalence are nausea (90%), vomiting (84%), upper abdomi-
tion of an upper gastrointestinal adverse event.. Given the nal pain (72%), and early satiety (60%). Other symptoms may
increased risk for PUD with the concomitant use of either an Include abdominal fullness and bloating. Symptoms may be
NSAID or selective COX-2 inhibitor with low-dose aspirin, at- chronic or persistent, or may occur intermittently. More
risk individuals should receive gastroprotective therapy. severe cases may involve weight loss and evidence of malnu-
Compared to an NSAID plus a PPI in patients at high risk, a trition and/or dehydration. A viral prodrome, such as gastro-
selective COX-2 inhibitor plus a PPJ offers a lower risk for per- enteritis or respiratory infection before symptom onset, may
foration, obstruction, and bleeding, as well as for NSAID and suggest postviral gastroparesis, a condition that frequently
COX-2 withdrawal due to gastrointestinal adverse events. resolves over time. Gastroparesis can result from a variety of
After an NSAID-induced bleeding peptic ulcer, the safest causes (Table 13).
strategy is avoidance of future NSAJD use. If an NSAID must be
used, the combination of a selective COX- 2 inhibitor plus a PPJ Diagnostic Testing
provides the best gastrointestinal protection, as this combination The first study performed to evaluate suspected gastroparesis
is more likely to prevent a rebleed than a selective COX-2 inhibi- is an upper endoscopy to exclude a gastric outlet obstruction.
tor alone or an NSAID plus a PPI. A treatment approach based on Once a structural cause for symptoms has been excluded, an
risk for NSAID-related ulcer complications and the need for low- objective test to assess gastric emptying is performed. There
dose aspirin for cardioprophylaxis is provided in Table 12. are three testing modalities available to assess gastric empty-
KEY POINTS ing: gastric scintigraphy, wireless motility capsule, and the
• Upper gastrointestinal complications, such as bleeding, rad!olabeled carbon breath test using 1 ~C-labeled Splrulina
are common with the use of NSAIDs (both short- and platensis (Table 14). Gastric scintigraphy is the most com-
long-term) and low-dose aspirin. monly used modality. Narcotic and anticholinergic agents
must be stopped at least 72 hours before a gastric emptying
• Proton pump inhibitors are the preferred agent for the
study. Once delayed emptying is objectively confirmed, addi-
prevention and treatment of NSAID-related (including
tional testing maybe required to determine the underlying
aspirin-related) upper gastrointestinal complications.
cause of the gastroparesis.
• fn high-risk individuals without cardiovascular disease,
including those with previous NSAID- induced gastroin-
Management
testinal bleeding, the combination of a selective
The severity of gastroparesis-related symptoms does not cor-
cyclooxygenase-2 inhibitor plus a proton pump inhibi-
relate with the severity of delayed gastric emptying, particu-
tor provides the best gastrointestinal protection if avoid-
larly with regard to the symptoms of abdominal bloating and
ance of NSAIDs is not possible.
epigastric pain. This suggests that gastroparesis is not simply

TABLE 12. Strategies for Prevention of NSAID-Related Ulcer Complications


No GI Risk Factors• One to lWo GI Risk Facto~·b More than lWo GI Risk Factors•

No aspirin NSAID NSAID plus PPI Ideally, avoidance of NSAIDs


If no alternative to NSAIDs, then a
selective COX-2 inhibitor plus a PPI
Low-dose aspirin (81 mg/d) for Naproxen plus PPI Naproxen plus PPI Avoid selective COX-2 inhibitors and
cardiovascular prophylaxis use alternative analgesic

COX-2 - cyclooxygenase-2; GI • gastrointestinal; NSAID • nonsteroidal anti-inflammatory d rug; PPI • proton pump inhibitor.

•GI risk factors: history of GI bleeding; history of peptic ulcer disease; concurrent use of NSAIOs and aspirin (including low-dose aspirin), nonaspirin antiplatelets, anticoagulants,
or gluccxorticoids; age older than 65 years; any chronic, debilitating illness.

"With no prior NSAID· related upper GI bleeding.


Adapted with pennission from Lanza FL, Chan Fl<, Quigley EM; Practice Parameters Committee of the American College of Gastroenterology. Guidelines for prevention of NSAID-
related ulcer complications. Am J Gastroenterol. 2009;104:728-38. [PMID: 192406981 doi: 10.1038/ajg.2009.11 S.

16
Disorders of the Stomach and Duodenum

TABLE 13. Causes of Gastroparesis Therapy must be stopped immediately if neurologic side
effects develop_
Common causes Diabetes mellitus (40% in long-standing
type 1 diabetes mellitus, 20% in type 2
Erythromycin improves gastric emptying, but its use
d iabetes mellitus) should be limited to the treatment of flares or short-term use
Postsurgical (e.g., Nissen fundoplication, (2 to 3 weeks) due to the risk for tachyphylaxis.
bariatric surgery, pancreatic surgery) Antiemetlcs for treatment of nausea and vomiting as well
Idiopathic (e.g ., postviral) as centrally acting modulators, including tricyclic antidepres-
Infrequent causes Connective tissue disease (e.g., systemic
sants and rnirtazapine, can also provide relief of symptoms but
sclerosis) have no beneficial effect on gastric emptying.
Neurologic disease {e.g., Parkinson lnterventional therapy, such enteral feeding via jejunos-
disease) tomy, gastric stimulator placement, pyloroplasty, and subtotal
Eating disorders or total gastrectomy, can be considered in patients who do not
Hypothyroidism
respond to dietary and pharmacologic therapy.
Amyloidosis KEY POINTS
Paraneoplastic syndromes (e.g., small • Gastroparesis is a heterogeneous clinical syndrome with
cell lung cancer) three components to the diagnosis: the presence of spe-
Mesenteric ischemia cific symptoms, absence of mechanical outlet obstruc-
Medications (e.g ., opiates, tion , and objective evidence of delay in gastric emptying
, anticholinergic agents) into the duodenum.
1________ - - - - - -

• Initial management of gastroparesis includes correction


a motility disorder but one of altered sensation as well. of dehydration and electrolyte abnormalities; nutri-
Because poor glycemic control (blood glucose level >200 mg/ tional support; small, frequent meals that are low in fat
dL [11.1 mmol/L]} can worsen symptoms as well as gastric and soluble fiber; and improved glycernic control in
emptying, tight glycemic control is the most important ele- patients with diabetes mellitus.
ment of treatment of diabetic gastroparesis. Initial manage- • Metoclopramide is a prokinetic drug that improves gas
ment also includes correction of dehydration and electrolyte tric emptying but is associated with dystonia, tardive
abnormalities, and nutritional support if needed. Initial die- dyskinesia, and Parkinsonism.
tary intervention should consist of small, frequent meals that
are low in fat and soluble fiber. Referral to a dietician may be
beneficial. Gastric Polyps and
Pharmacologtc therapy is used to improve gastric empty-
Subepithelial Lesions
ing and to treat symptoms. Metoclopramide is the only proki-
netic approved in the United States for the treatment of Gastric Polyps
gastroparesis. To minimize the significant risk for neurologic Polyps in the stomach include fundic gland polyps, hyperplas-
side effects, including dystonia, Parkinsonian movements, and tic polyps, and aden01nas_ All gastric polyps should be biop-
tard.ive dyskinesia, the lowest effective dose should be used. sied to determine polyp histology. Fundic gland polyps and

TABLE 14. Diagnostic Tests Assessing Gastric Emptying


Test Modality Advantages Disadvantages Clinical Pearfs
Gastric scintigraphy Considered the gold Radiation exposure (technetium 4-Hour study is most accurate
standard radiolabeled mea l)
Assesses solid emptying (liquid emptying
Requires specially trained personnel is less accurate)
Cost Blood sugar should be less than 275 mg/
dl(15.3 mmol/L)
Wireless motility Can also assess small Cost Consider in a patient with suspected
capsule bowel, colon, and global motility problem
Can't be used with pacemaker or
whole gut transit
defibrillator Stop antisecretory agents as study relies
No radiation on measurement of pH
I Ambulatory study
Risk for capsule retention

I Gastric emptying Low cost Only recently commercially available

I breath test

l________ _
Efficacy and accuracy limited to
clinical trials
--- --·-- ------- ------- - _J

17
Disorders of the Stomach and Duodenum

hyperplastic polyps account for 70% to 90% of stomach polyps. KEY POINTS
Fundic gland polyps have no potential for malignancy and are • Polyps in the stomach include fundic gland polyps,
commonly seen in the setting of PPI use. Multiple fundic gland which have no malignant potential; hyperplastic pol-
polyps are also found in patients with familial adenomatous yps; and, less commonly, adenomas, which should be
polyposis.
resected.
Hyperplastic polyps of the stomach are thought to have
malignant potential, with 5% to 19% harboring dysplasia or • Gastrointestinal stromal tumors should be evaluated
malignancy. Risk factors for malignant potential of hyperplas- with endoscopic ultrasonography and excised if symp-
tic polyps include siz.e greater than 1 cm and pedunculated toms or high-risk features are present.
morphology. All hyperplastic polyps greater than 0.5to1.0 cm • Carcinoid tumors are well-differentiated neuroendo-
in size should be resected. crine tumors that can occur throughout the gastrointes-
Adenomas found in the stomach can be sporadic or asso- tinal tract, including the stomach.
ciated with hereditary syndromes, including familial adeno-
matous polyposis and Lynch syndrome. Adenomas in the
stomach should be resected and endoscopic surveillance Gastric Adenocarcinoma
should be performed 1 year after resection and then every 3 to
Epidemiology and Risk Factors
s years thereafter.
Stomach adenocarcinorna has an incidence rate of 6.7 in
100,000 persons and a mortality rate of 3.4 per 100,000 per-
Gastric Subepithelial Lesions sons in the Uruted States. Rates have steadily decreased since
Gastrointestinal Stromal Tumors the 1990s. There are two types of gastric cancer: intestinal-
Gastrointestinal stromal tumors (GISTs) are the most common type, which is more common, and diffuse-type. H. pylori is a
mesenchymal tumors of the stomach. GISTs may present with recognized risk factor for both types of cancer. Other risk fac-
symptoms, such as bleeding or abdominal pain, but are also tors primarily associated with intestinal-type gastric adeno-
found incidentally. Endoscopic ultrasonography is the best carcinoma include male sex; ethnicity (incidence is highest Jn
diagnostic modality for evaluation of a GISI High-risk features persons of Asian and Pacific Island descent, and mortality is
on endoscopic ultrasonography include size greater than 2 cm, highest In non-Hispanic white persons); geography (the high-
lobu1ated or irregular borders, invasion into adjacent struc- est rates worldwide occur in Asia, Eastern Europe, and Central
tures, and heterogeneity. Biopsies of a GIST can be done, but and South America); diet high in smoked, salted, and pickled
the high- risk endoscopic features are better predictors of foods as well as nitrates and nitrites; smoking; and obesity.
malignant potential. Treatment consists of surgical excision if Additional risk factors include previous stomach surgery; per-
the GIST is symptomatic or high-risk features are present. For nicious anemia, and hereditary syndromes such as hereditary
GISTs without high-risk features, yearly endoscopic surveil- diffuse gastric cancer (associated with the diffuse type), Lynch
lance is indicated. See MKSAP 18 Hematology and Oncology syndrome, and familial adenomatous polyposis. Gastric intes-
for staging and treatment of GISTs. tinal metaplasla and dysplasia are also risk factors for gastric
adenocarcinoma.
Carcinoid Tumors
A carcinoid tumor is a well-differentiated neuroendocrine Screening and Surveillance
tumor originating in the digestive tract, lungs, or, rarely, the There is no recommendation for population-based screen-
kidneys or ovaries. Carcinoid tumors can be encountered ing for gastric adenocarcinorna in countries with a low inci-
throughout the gastrointestinal tract, including the stom- dence of gastric cancer, such as the United States. If intesti-
ach. They may present symptomatically or may be found nal metaplasia with high-grade dysplasia is identified, it
incidentally. Carcinoid tumors are usually sporadic but can should be resected because 25% of cases progress to adeno-
be seen In the setting of Zollinger-Ellison syndrome, carctnoma. Screening and surveillance ls indicated for
atrophic gastritis, and rare syndromes, such as multiple patients with familial adenomatous polyposis and Lynch
endocrine neoplasia type 1 and neurofibromatosis type 1. syndrome.
Carcinoid tumors are classified by their size, number, and
anatomic distribution. Management includes endoscopic Clinical Manifestations and Diagnosis
surveillance for lesions smaller than 1 cm in size, especially Symptoms of gastric adenocarcinorna may be vague. They
when multiple lesions are present. Endoscopic or surgical include poor appetite, weight loss, abdominal pain, early sati-
excision is indicated for carclnoid tumors with high-risk ety, nausea, and vomiting. Signs of gastric adenocarcinoma
features, such as solitary lesions not found in the setting of include iron deficiency anemia. Diagnosis is typically made by
atrophic gastritis or Zollinger-Ellison syndrome. See MKSAP upper endoscopy with biopsies.
18 Hematology and Oncology for treatment of gastric neu- For treatment of gastric cancer, see MKSAP 18 Hematology
roendocrine tumors. and Oncology.

18
Disorders of the Pancreas

KEY POINTS KEY POINTS


• In countries with a low incidence of gastric cancer. • Patients who undergo partial gastrectomy for malig-
screening for gastric cancer should be reserved for nancy require lifelong surveillance for recurrence of
patients with genetic cancer syndromes. cancer.
• The primary nongenetic tisk factor for gastric cancer is • Dumping syndrome results from rapid gastric emptying
Helicobacter pylori infection. after gast1ic surgery; first-line treatment is smaller. more
• Upper endoscopy with biopsy is the diagnostic test of frequent meals with liquids taken following meals.
choice for gastric cancer.

Disorders of the Pancreas


Gastric Surgery Complications
For complications of bariatric surgery, see MKSAP 18 General
Acute Pancreatitis
Internal Medicine. Acute pancreatitis is an inflammatory process involving the
Partial or complete gastric resections are performed for pancreas and extrapancreatic organs. it is the most common
benign and malignant disease. The extent ofresection, type of gastrointestinal cause of hospitalization in the United States.
reconstruction, and nature of the disease affect postoperative Passage of gallstones, sludge, or biliary crystals (microlithiasis)
morbidity and mortality. Partial gastric resection allows for is the most common cause of acute pancreatitis (Table 15).
preservation of some function of the stomach, but in the set-
ting of malignancy, it requires lifelong surveillance of the
remaining stomach for recurrence. Patients who undergo par-
~'H!i•Ml'tJH.!l.Mi41J!MiB!t~
Common
tial gastrectomy for benign disease have an increased risk for
Biliary disease
cancer in the gastric remnant 15 to 20 years after surgery, with
reported frequency ranging from 0.8% to 8.9%.
Dumping syndrome, which results from rapid gastric emp-
I Gallstones
Microlithiasis (1-to 2-mm stones that are not detected by
imaging studies)
tying after gastric surgery, can cause significant postprandial
gastrointestinal and vasomotor symptoms. Clinical features of
I Alcohol use

early dumping syndrome occur within 30 minutes of eating due Post- endoscopic retrograde cholangiopancreatography
to gastrointestinal hormone hypersecretion, autonomic dys- Occasional
regulation, and bowel distention. Symptoms include palpita-
Medications
tions, flushing or pallor, diaphoresis, lightheadedness,
hypotension, and fatigue, followed by diarrhea, nausea, abdom- Furosemide
inal bloating, cramJling, and borborygmus. Late symptoms Didanosine
occur 1 to 3 hours after meals because of reactive hypoglycemia Asparaginase
and can include decreased concentration, faintness, and altered Mesalamine
consciousness. Jn severe cases, protein-wasting malnutrition
Hydrochlorothiazide
can occur. It is estimated that 25% to 50% of all patients who
have undergone gastric surgery experience some symptoms of 6-Mercaptopurine/azathioprine

dumping syndrome, but severe, persistent symptoms occur in Simvastatin


only about 10%. Oral glucose challenge testing is useful to make : Hypertriglyceridemia
the diagnosis, with a sensitivity as high as 100% and a specificity Hypercalcemia
of94%. Choledochocele
First-line treatment for dumping syndrome is dietary,
with smaller, more frequent meals and ingestion of liquids Rare
after meals. Decreasing carbohydrate intake, especially simple Autoimmune
carbohydrates, and increasing protein and fiber intake may Infectious
2lso alleviate symptoms. I
Viral (mumps, coxsackie B virus, cytomegalovirus)
Acarbose, an <I-glycosidase hydrolase inhibitor that inter-
feres with the digestion of polysaccharides to monosaccha- Parasitic (Toxoplasma species, Ascaris lumbricoides)

rides, can be used for late symptoms of dumping syndrome. lschemia


Other pharmacologic therapies include anticholinergics to Trauma
slow gastric emptying, and antispasmodics. Severe cases of Neoplasia
dumping rarely require octreotide. if a trial of subcutaneous Celiac disease
injections is effective, monthly intramuscular injections of
Genetic (only if attacks are recurrent)
long-acting octreotlde can be used.

19
Disorders of the Pancreas

Premature activation of digestive enzymes and release of Because acute pancreatitis is most commonly caused by
cytokines cause "autodigestion" of the pancreas and intlamma- biliary disorders, patients with acute pancreatitis should
tion, which may involve surrounding tissues and distant organs. undergo transabdominal ultrasonography. Transabdominal
Acute pancreatitis is classified as mild, moderately severe, ultrasonography is preferred over CT because it has a higher
or severe. Mild acute pancreatitis does not involve organ failure sensitivity for detection of gallstones, avoids the risks associ-
or local or systemic complications, usually resolves within 1 ated with intravenous contrast, and is more cost effective.
week, and has a low mortality rate. Twenty percent of patients However, abdominal air can limit the visualization of the pan-
with acute pancreatitis develop moderately severe or severe creas in patients with acute pancreatitis. Magnetic resonance
disease. Moderately severe acute pancreatitis involves local or cholangiopancreatography (MRCP) may be considered in
systemic complications such as necrosis or transient organ patients who do not have abnormal findings on ultrasonogra-
failure (for less than 48 hours). Severe acute pancreatitis phy. CT may be indicated if the diagnosis is in question or if
involves systemic inflammatory response syndrome (SIRS), clinical symptoms are not alleviated within the first 48 hours.
persistent organ failure (usually kidney or respiratory failure), Acute liver-enzyme elevation at presentation suggests bil-
duration longer than 48 hours, and one or more local compli- iary obstruction. Serum amylase and lipase levels may be ele-
cations; it has a mortality rate as high as 50%. vated in conditions other than acute pancreatitis, such as
kidney disease, acute appendicitis, cholecystitis, intestinal
Clinical Presentation and Diagnosis obstruction or ischemia, peptic ulcer, or gynecologic disorders.
The diagnosis of acute pancreatitis requires two of the follow- Enzyme levels may be falsely low or normal in patients with
ing three criteria: (1) acute-onset abdominal pain characteris- hypertriglyceride-induced pancreatitis because of llpemic-
tic of pancreatitis (severe, persistent for hours to days, and serum interference with laboratory assays. Triglyceride levels
epigastric in location, often radiating to the back); (2) serum should be measured in patients without a biliary cause of acute
lipase or amylase levels elevated to three to five times the pancreatitis, and if the triglyceride level is greater than 1000
upper limit of normal; and (3) characteristic radiographic mg/dL (11.3 mmol/L) , it can be considered the cause of the
findings on contrast-enhanced CT (Figure 9), MRI, or transab- acute pancreatitis.
dominal ultrasonography. The presence of high fever and
leukocytosis is part of the cytokine cascade and does not nec- Prognostic Criteria
essarily indicate infection. Risk factors for severe disease include age older than 55 years,
medical comorbidities, BMI greater than 30, the presence of
SIRS, signs of hypovolemia on presentation (for example, a
serum blood urea nitrogen level greater than 20 mg/dL
[7 .1 rnmol/L) and rising, a hematocrit greater than 44 %, or an
elevated serum creatinine), the presence of pleural effusions
and/or infiltrates, and altered mental status. A systematic
review of 18 multiple-factor scoring systems, including the
Ranson criteria and the Acute Physiologic Assessment and
Chronic Health Evaluation (APACHE) Ir score, for predicting
outcome in acute pancreatitis found these systems to have
limited clinical value and accuracy. Scoring systems only
identify severe disease as it develops, without enough lead
time for intervention, and they are too cumbersome for rou-
tine use. Data suggest serum hematocrit, elevated blood urea
nitrogen levels, and the presence of SIRS to be as accurate as
complex scoring systems in predicting outcome, and they are
easier to use.

Management l:J
Mainstays of management include fluid resuscitation, pain
management, and antinausea medication.
Aggressive hydration (250-500 mUh of intravenous lac-
tated Ringer solution) should be given to patients with acute
FIGURE 9. CT scan showing acute pancreatitis with peripancreaticfat stranding
pancreatitis on presentation and is most beneficial in the first
and inflammation. The hazy appearance of the mesenteric lat surrounding the
pancreas in this image is called fat stranding, and the blurring of the margins of 12 to 24 hours. More rapid tluid resuscitation {boluses) may be
the pancreas is consistent with peripancreatic edema, features seen with needed fa patients with severe volume depletion. Patients
inflammatory changes of acute pancreatitis. with organ failure or SIRS should be admitted to an ICU or

20
Disorders of the Pancreas

, . , intermediary care setting, with reassessment of fluid require- developed a well-defined wall, and contain no solid
Llil ments every 6 hours for the first 24 to 48 hours. debris (necrosis).
CONT.
Routine use of antibiotics is not warranted in acute pan- 3. Acute necrotic collections (Figure 10) are areas of necro-
creatitis, unless there is evidence of extra pancreatic infection, sis in the pancreatic parenchyma and/or peripancreatic
such as ascending cholangitis, bacteremia, urinary tract infec- tissues within the first 4 weeks of acute pancreatitis.
tion, or pneumonia. Use of prophylactic antibiotics in patients
4. Walled-off necrosis (Figure 11) occurs after 4 weeks,
with sterile pancreatic necrosis to prevent infected necrosis is
when the body liquifies the necrosis and contains it
not recommended.
within a well-defined wall.
In mild acute pancreatitis, oral feedings can be started as
soon as nausea and vomiting are controlled and clinical symp- Contrast-enhanced CT may not be able to distinguish
toms begin to subside. Enteral feeding should begin within solid from liquid content in fluid collections; therefore,
72 hours if oral feeding is not tolerated; it is usually required in necrotic collections are frequently misdiagnosed as pancreatic
patients with moderately severe or severe acute pancreatitis. pseudocysts. Pancreatic pseudocysts do not require drainage
Feeding with a nasojejunal tube has traditionally been pre-
ferred, but data suggest that nasogastric feedings are likely
equally effective and easier to administer. Enteral feeding
promotes a healthy gut-mucosa! barrier to prevent transloca-
tion of bacteria into inflamed tissues.
If a biliary cause of acute pancreatitis is suspected, serial
liver chemistry tests and clinical symptoms can show whether
the biliary obstruction is ongoing or resolving. Endoscopic
retrograde cholangiopancreatography (ERCP) is not indicated
in patients with gallstone pancreatitis unless there is persis-
tent elevation of liver chemistries or if choledocholithiasis is
seen on imaging studies. Patients with cholangitis should
undergo ERCP within 24 hours of admission. Patients with
uncomplicated gallstone pancreatitis should be considered for
cholecystectomy before discharge.
There is no value in rechecking serum amylase and lipase
levels after the diagnosis is established.

Complications FIGURE 10. CT scan showing acute pancreatitis with hypoperfusion of the
There are two overlapping phases of acute pancreatitis with body of the pancreas as indicated by lack of enhancement following intravenous
contrast infusion (necrosis) and normal perfusion of the pancreatic tail.
two peaks in mortality. The early phase is the first week of the
disease, when the body is responding to local pancreatic
injury and the cytokine cascade, and SIRS and organ failure
are possible. The late phase occurs after the first week and
may persist for weeks to months in patients with moderately
severe or severe acute pancreatitis. Significant risk for infec-
tion in peripancreatic fluid collections and necrosis occurs in
the late phase.
Proper classification of fluid collections in acute pancrea-
titis is Important to guide management. An international con-
sensus group updated the Atlanta classification and defini-
tions of acute pancreatitis and its complications in 2012 to try
to promote consistency in diagnosis and management. Four
types offluid collections were defined:
1. Acute peripancreatic fluid collections are collections that
occur in edematous interstitial pancreatltis (no necrosis)
within the first 4 weeks, are thought to occur because of
rupture of main or side branch ducts as a result of inflam-
mation, are sterile, and usually resolve spontaneously.
FIGURE 11 . CT scan showing maturation and liquefaction of pancreas necrosis
2. Pancreatic pseudocysts are acute peripancreatic fluid of nearly the entire pancreas over 4 weeks in duration with a well-defined rim or
collections that have persisted for longer than 4 weeks, wall (arrows), known as walled-off necrosis.

21
Disorders of the Pancreas

Cl unless they cause significant symptoms. regardless of size.


Because they contain only fluid, pseudocysts are easily drained
Clinical Presentation and Diagnosis
Abdominal pain ls the most common presenting symptom of
CONT. under endoscopic or radiographic guidance. Walled-off necro- chronic pancreatitis (seen in 85% of patients). but some
sis is not as amenable to percutaneous or endoscopic drainage patients have no pain. Pain patterns can vary from constant,
due to solid necrotic debris within the cavity and may require daily pain to intermittent attacks of severe pain. Pancreatic
endoscopic. radiologic, or surgical debridement. enzyme levels may not increase during attacks of pain because
The management of suspected infected necrosis includes of fibrosis and atrophy of acinar cells and decreased enzyme
initiation of antibiotics (for example, imipenem -cilastatin, production. Constant daily pain In chronic pancreatitis causes
meropenem, or ciprofloxacin plus metronidawle) with con- significant reduction in quality of life, with increased use of
sideration of fine-needle aspiration with Gram stain and cul- health care resources, disability benefits, and time away from
ture under CT guidance. Drainage procedures or debridement employment. Exocrine or endocrine Insufficiency occurs In
should be delayed for at least 4 weeks if possible to allow some patients as a result of significant tissue destruction.
encapsulation of the necrosis with a fibrous wall. CJ Diagnosis of chronic pancreatitis remains challenging
KEY POINTS because hallmark anatomic features, such as pancreatic calci-
fications (Figure 12), only occur in 25'~. of patients, and other
• Biliary disease, such as gallstone or microlithiasis, is the
features of atrophy or duct dilation can occur normally with
most common cause of acute pancreatitis.
aging or other disease processes. If pancreatic calcifications
• Diagnosis of acute pancreatitis requires two of three cri- are not present on CT imaging of the abdomen, diagnosis usu-
teria: (1) acute-onset upper abdominal pain, (2) serum ally requires a combined approach in patients with symptoms,
lipase or amylase levels elevated at least three times
greater than the upper limit of normal, and (3) charac-
TABLE 16. Causes of Chronic Pancreatitis
teristic findings on imaging.
• Patients with acute pancreatitis should undergo Toxic or metabolic

transabdominal ultrasonography to evaluate for biliary Alcohol, tobacco, hypercalcemia, hypertriglyceridemia,


chronic kidney d isease
disease.
Genetic
• Patients with acute pancreatitis should be hospitalized
for fluid administration and close monitoring of cardio- Mutations or polymorphisms of the CFTR, PRSSI, SPINK/,
CTRC, CASR, CLDN2 genes
vascular, respiratory, and kidney status.
Recurrent and severe acute pancreatitis
Vascular disease/ischemia

Chronic Pancreatitis Obstructive

Chronic pancreatitis is thought to develop when the inflam- Pancreatic tumor, intraductal papillary mucinous neoplasm
matory response to acute pancreatitis persists and ongoing Posttraumatic (pancreatic duct stricture)
inflammation activates stellate cells, resulting in a fibro- Autoimmune (type 1 and type 2)
inflammatory response. This causes distorted tissue archi- Idiopathic
tecture, loss of normal parenchyma, activation of pancreatic
nociceptors, and loss of acinar and islet cell function.
Genetic variants affecting inflammatory response, enzyme
activation, and tissue repair are thought to play important
roles in the pathogenesis of chronic pancreatitis. Many
genes have been identified as disease modifiers in chronic
pancreatltls, but the gene-environment interaction is not
fully understood.
Alcohol use has long been described as a risk factor for
chronic pancreatitis, but less than 3% of heavy alcohol users
develop pancreatic disease. Patients who drink more than five
drinks per day (or 35 drinks per week) seem to be more sus-
ceptible to chronic pancreatitis. In the largest study of patients
with chronic pancreatitis in North America, only 46% had a
history of significant alcohol use. The higher prevalence of
alcohol-associated pancreatitis among men could be partially
explained by an X chromosome- linked genetic variant, the
CWN2 gene. Tobacco is considered an independent risk factor. FIGURE 1 2. CT scan showing chronic calcific pancreatitis with multiple stones
Table 16 lists common causes of chronic pancreatitis. in the main duct and side branches of the pancreas.

22
Disorders of the Pancreas

with advanced imaging using MRI, MRCP, or endoscopic ultra-


Autoimmune Pancreatitis
sonography and evidence ofpancreatic dysfunction. Pancreatic
biopsy ls not indicated. ERCP is no longer used as a diagnostic and lgG4 Disease
tool for chronic pancreatitis. MRI and MRCP are safer and Autoimmune pancreatitis (AIP) is a frequent manifestation of
more readily available; however, sensitivity and specificity are IgG4-related disease. Other organs that can be affected include
variable. Endoscopic ultrasonography for the diagnosis of the lacrimal and salivary glands, central nervous system, kid-
chronic pancreatitis has relatively high sensitivity but low neys, thyroid gland, lungs, biliary tract and liver, prostate
specificity, and is not reliable as a single diagnostic test. gland, retroperitoneum, and lymph nodes. Storiforn1 fibrosis
and obliterative phlebitis are characteristics seen in the pan-
Management creas and biliary tract. The most common pancreatic manifes-
The management of chronic pancreatitis focuses on treating tation of IgG4-related disease is type l AIP, with abundant
symptoms. No effective treatment exists to reverse or halt dis- infiltration of IgG4-positive plasma cells and lymphocytes.
ease progression. Patients should be counseled to avoid alco- Type 2 AIP has no or few lgG4-positive cells but is character-
hol and tobacco, as this may lessen attacks of inflammation ized by idiopathic duct-centric neutrophil infiltration, known
and pain. Intermittent attacks of severe acute pain are treated as a granulocytic epithelial lesion. Type 2 AIP is a pancreas-
as in acute pancreatitis. Complications of chronic pancreatitis, specific disease with occasional association with ulcerative
including pseudocyst, pancreatic duct stones causing upstream colitis. The 2011 International Consensus of Diagnostic Criteria
obstruction, and malignancy, should be evaluated with imag- for Autoimmune Pancreatitis endorsed the concept of type I
ing when a patient's symptom pattern changes. Constant, and type 2 disease, but there is some debate over whether type
daily pain is more challenging to manage and frequently 2 should be considered an lgG4-related disease.
involves the use of medications such as tramadol, gabapenti- For more information on lgG4-related disease, see MKSAP
noids, and possibly narcotics, although long-term use of nar- 18 Rheumatology.
cotics should be avoided due to hyperesthesia and develop-
ment of tolerance or addiction. Pancreatic enzymes are used to Clinical Presentation and Diagnosis
treat steatorrhea but have not been shown to effectively treat Diagnostic criteria for AIP require image findings of a nar-
pain or prevent attacks ofpancreatitis. Two large randomized rowed main pancreatic duct and parenchymal swelling (the
trials using antioxidants to treat chronic pancreatitis pain "sausage-shaped" pancreas [Figure 13)) and response to gluco-
showed conflicting results, and guidelines do not support their corticoids. Patients with type 1 AIP have elevated levels of
use. Nerve blocks and neurolysis procedures are not recom- IgG4-positive cells in pancreatic tissue (>10 IgG4-positive
mended because the response rate is low, and pain relief, if cells/hpt), and 60% to 80% of patients have associated scleros-
achieved, only lasts a few weeks. ing cholangitis, sclerosing sialoadenitis, or retroperitoneal
Surgery offers the best long-term results for chronic, fibrosis. A significant elevation of serum IgG4 level is helpful in
refractory pain management and, depending on anatomy and diagnosing type 1 AIP. Patients with type 2 AIP will have no
cause, may include lateral pancreaticojejunostomy, duodenal- elevation of IgG4-positive cells in pancreatic tissue. Patients
preserving pancreatic head resection, pancreaticoduodenec- with both types may present with abdominal pain or
tomy, distal pancreatectomy, or even total pancreatectomy
with or without auto-islet cell transplantation. These proce-
dures have been shown to improve outcomes in patients at
high-volume pancreas referral centers. Pancreatic endocrine
insufficiency may require specialty referral for labile diabetes
management and nutritional consultation.
KEY POINTS
• Abdominal pain, which may occur as intennittent
attacks or as ongoing, daily pain, is the most common
symptom of chronic pancreatitis.
HVC • Pancreatic biopsy and endoscopic retrograde cholangio-
pancreatography are not indicated in the diagnosis of
chronic pancreatitis.
• Symptomatic management is the cornerstone of treat-
ment for chronic pancreatitis, including patients with
refractory pain.
• Patients with chronic pancreatitis should be counseled
to avoid alcohol and tobacco use. FI GU RE 13 . CT scan showing the homogeneous, hypodense •sausage-shaped•
swelling (arrows) seen in autoimmune pancreatitis.

23
Disorders of the Pancreas

obstructive jaundice with or without a mass. Jn patients present- Clinical Presentation


ing with obstructive jaundice or with a mass, pancreatic malig- Symptoms in patients with pancreatic adenocarcinoma
nancy must be considered. Many patients with malignancy have include abdominal pain, back pain, weight loss, and jaundice
a dilated upstream main pancreatic duct, whereas patients with if the lesion is located in the head of the pancreas obstructing
AlP may have a narrow main duct. Endoscopic ultrasonography the common bile duct. Pancreatic cancer ls highly associated
and biopsy may be required to differentiate AIP and pancreas with diabetes mellitus, and two thirds of patients develop
neoplasm. Ten percent of patients with type 1 AIP may develop new-onset diabetes mellitus in the 36 months surrounding the
chronic pancreatitis or pancreatic stone formation. diagnosis. Venous thromboembolic events and depression
have been observed to occur at higher rates in patients with
Treatment pancreatic adenocarcinoma than in patients with other
Glucocorticoids are effective treatment for types 1 and 2 AIP. mallgnancies.
starting with oral prednisolone, 0.6 to 1.0 mg/kg/d, and
tapered over 2 to 3 months. Response is determined by symp- Diagnosis
tom relief and imaging features. Failure of clinical symptoms to Pancreas protocol CT uses multiphasic arterial and venous
respond to glucocorticoids suggests an incorrect diagnosis, and phases with thin cuts (3 mm) through the abdomen to view the
other causes should be investigated. Up to 60% of patients may prtmary tumor's relationship to mesenteric vasculature and to
relapse. ReadministJ:ation of glucocorticoids or immunomod- detect metastatic lesions. Some studies suggest that pancreas
ulators, such as 6-mercaptopurine, azathioprine, mycopheno- protocol MRI may be superior to CT for pancreatic disease.
late, or rltuximab, may be used to treat recurrent AIP. Histopathologic confirmation of pancreatic adenocarcinoma ls
KEY POINTS increasingly recommended before initiation of therapy.
• Diagnosis of autoimmune pancreatitis requires the Endoscopic ultrasound-guided biopsy is preferable to CT-guided
presence of a narrowed main pancreatic duct and biopsy in patients with potentially resectable disease because of
parenchymal swelllng ("sausage-shaped" pancreas) on ultrasonography' s higher diagnostic yield, safety, and lower risk
imaging and disease response to glucocorticoids. for peritoneal seeding. Patients are considered for primary sur-
gical resection if they have no involvement ofmesentertc vascu-
• Type 1 autoimmune pancreatitis is characterized by ele-
lature or metastatic disease and are healthy enough for major
vated numbers oflgG4 positive cells in pancreatic tis-
intra-abdominal surgery. Preoperative chemotherapy and/or
sue; most patients also have a significant elevation of
radiotherapy may be recommended for patients with locally
lgG4 in serum.
advanced disease.
• Patients with type 2 autoimmune pancreatitis have nor- For staging and treatment of pancreatic adenocarcinoma,
mal IgG4-positive cell counts. see MKSAP 18 Hematology and Oncology.
• Types 1 and 2 autoimmune pancreatitis are treated with KEY POINTS
glucocorticoids, with a relapse rate of up to 60%.
• Diagnosis of pancreatic cancer is suggested by weight
loss, abdominal pain, jaundice, and new-onset diabetes
mellitus; pancreas protocol CT or MRI help support the
Pancreatic Adenocarcinoma
diagnosis and delineate the extent of disease.
Pancreatic ductal adenocarcinom.a has a poor prognosis and
• Endoscopic ultrasound-guided fine-needle aspiration of
increasing incidence. In 201S, there were approximately
the pancreas is recommended to provide histological
49,000 cases diagnosed in the United States and 41,000
confirmation of cancer.
deaths. It is predicted to become the second leading cause of
cancer-related death in the United States over the next decade,
with an overall S-year survival rate of less than 6%.
Ampullary Adenocarcinoma
Epidemiology and Risk Factors Adenocarcinoma of the ampulla of Vater accounts for 0.2%
Risk factors for pancreatic cancer include age older than SO years, of gastrointestinal malignancies and usually presents early
smoking history, obesity, chronic pancreatitis, and mucinous because of obstruction of the biliary system. Endoscopic
cystic lesions of the pancreas. Inherited conditions associated ultrasonography with biopsy is the preferred method for
with pancreatic cancer include Peutz-Jeghers syndrome, BRCA2 staging and tissue diagnosis, with 90% accuracy. At least
germline mutations, hereditary pancreatitis, familial atypical 50% of patients with familial adenomatous polyposis syn-
multiple mole melanoma, Lynch syndrome, and familial pancre- drome develop adenomatous changes of the periampul-
atic cancer with at least two affected first-degree relatives. lary region; upper endoscopy screening In patients with
There are no standard guidelines for screening high-risk or this syndrome should begin at age 2S to 30 years. See
average-tisk patients because no means of accurately detecting Colorectal Neoplasia for more information on adenoma-
tumors at a curable stage has been identified. tous polyposis syndromes.

24
Disorders of the Pancreas

Cystic Lesions of the Pancreas for surveillance. Survelllance can be discontinued if no worri-
some features develop over S years or if the patient is no longer
Pancreatic cysts are found incidentally in 15% of patients
a surgical candidate.
u ndergoing abdominal imaging. The detection of a cystic
lesion in the pancreas causes anxiety in patients and clini- KEY POINTS
cians and is a growing driver of health care use in the • Most pancreatic cysts never become malignant, with the
United States. exception of intraductal papillary mucinous neoplasms
Cystic neoplasms of the pancreas are subcategorized as involving the main pancreatic duct.
mucin-producing and non- mucin-producing cysts. Mucin- • Cystic lesions are managed with surveillance and surgi-
producing cysts are thought to have malignant potential, but cal resection of high-risk cysts.
most never become malignant. Intraductal papillary muci-
nous neoplasms (IPMNs) are the most commonly found
cystic lesions of the pancreas. Most IPMNs arise from a Pancreatic Neuroendocrine
branched duct and have a very low rate of malignant trans-
formation. In contrast, the rate of malignant transformation Tumors
is greater than 65% in IPMNs involving the main pancreatic Pancreatic neuroendocrine tumors are rare, representing
duct (Figure 14). Other high-risk features are the presence of 3% of primary pancreatic neoplasms. Ten to twenty-five
symptoms, cyst size greater than 3 cm, and a solid compo- percent of pancreatic neuroendocrine tumors are func-
nent to the cyst. tional and hypersecrete hormones. Tumors may be sporadic
Mucinous cystic neoplasms are mucin-producing cysts or part of an inherited disorder. Initial evaluation includes
seen almost exclusively in women (more than 98%) and found blood and urine tests for chromogranin A, 5-hydroxyin-
in the pancreas body or tail in 90% of cases. They have a thick doleacetic acid, gastrin, glucagon, insulin and proinsulin (if
fibrous capsule with epithelioid cells similar to ovarian stroma clinically indicated; requires fasting with concurrent glu-
surrounding the tumor. Differentiating mucinous cysts from cose), pancreatic polypeptide, and vasoactive intestinal
serous cysts, pseudocysts, and cystic neuroendocrine tumors polypeptide to determine functional status. Most functional
of the pancreas can be challenging, as there is no definitive pancreatic neuroendocrine tumors secrete gastrin (gas-
test with high sensitivity and specificity. Endoscopic ultra- trinoma) or insulin (insulinoma). Genetic testing for multi-
sonography with fine-needle aspiration is used for cyst fluid ple endocrine neoplasia type 1 ls recommended in all young
analysis In worrisome lesions; carcinoembryonic antigen lev- patients with gastrinomas or insulinomas, and in any
els are frequently elevated in patients with mucinous cysts, patient with a family or personal history of other endo-
whereas serum amylase levels are often elevated Jn patients crinopathies or multiple pancreatic neuroendocrine
with pseudocysts. Cytology of cyst fluid has a low sensitivity tumors. Genetic testing for other associated inherited disor-
(<60%) and is an unreliable predictor of malignant ders, such as von Hippel-Lindau syndrome, tuberous scle-
transformation. rosis, or neurofibromatosis-1, should be considered based
Surgical resection of high-risk cysts is currently the only on clinical and family history.
option for treatment. Tn 2018, the American Gastroenterological Imaging is recommended with multlphaslc CT or MRI of
Association published guidelines based on expert opinion for the abdomen. Endoscopic ultrasonography (90% sensitive)
the management of asymptomatic cystic neoplasms of the or octreotide scintigraphy may be needed to detect small
pancreas. Worrisome features include symptoms related to the lesions. For functional pancreatic neuroendocrine tumors,
cyst (jaundice, pancreatitis), size of 3 cm or greater, dilated surgery is recommended if more than 90% of the tumor can
main pancreatic duct, and a solid component. MRI is preferred be resected . For nonfunctional tumors associated with mul-
tiple endocrine neoplasia type 1 and von Hippel-Lindau
syndrome, surgery is usually recommended if the tumors are
greater than 2 to 3 cm in size. Medical management of gas-
trinomas includes oral proton pump inhibitors two or three
times daily or somatostatin analogs. High-volume or pro-
gressive disease is best managed in conjunction with a
gastroenterologist.
KEY POINTS
• Ten to twenty-five percent of pancreatic neuroendo-
crine tumors are functional and hypersecrete hor-
mones, most commonly gastrin or insulin.
Fl GU RE 14. Magnetic resonance cholangiopancreatogram of a main·duct • Genetic testing for inherited tumor syndromes should
intraductal papillary mucinous neoplasm of the tail of the pancreas. Cystic dilation
be considered based on patient age and history.
of the main pancreatic duct in the tail is seen (arrows).

25
Disorders of the Small and Large Bowel

loperamide to relieve symptoms and consideration of empiric


Disorders of the Small antibiotics if symptoms are moderate or severe.
and Large Bowel
Chronic Diarrhea
Diarrhea Causes
Classification Chronic diarrhea can be grouped into six categories according
Diarrhea is a condition marked by passage of a greater number to type of cause: (1) osmotic, (2) secretory, (3) steatorrhea, (4)
of less-formed stools than normal. The classification of diar- inflammatory, (5) motility, and (6) miscellaneous (Table 17).
rhea as acute or chronic is determined by duration of symp- These categories may overlap because some diseases have
toms: acute diarrhea is defined by duration of less than multiple mechanisms that cause diarrhea. The most common
2 weeks, whereas chronic diarrhea is defined by duration of at
least 4 weeks. Acute diarrhea is usually caused by infectious TABLE 17. Types of Chronic Diarrhea
agents; chronic diarrhea is most commonly noninfectious.
Dianhea Type Causes (examples)

Acute Diarrhea Osmotic Medications (laxatives)


Undigested sugars (lactose, fructose,
Causes sorbitol, mannitol)
Causes of acute diarrhea are primarily infectious, including
Secretory Medications (nonosmotic laxatives,
bacterta, viruses, and parasites. These can be divided into non- antibiotics)
invasive and Invasive causes. Noninvasive causes lead to watery
Endocrine (carcinoid, gastrinoma, VI Poma,
diarrhea; they include Clostr/dtum difficile, viruses (such as adrenal insufficiency, hyperthyroidism)
norovirus and rotavirus), Escherichia coli, cholera, crypto-
Bile salt malabsorption (ileal resection,
sporidia, and Glardia lamblia. Invasive causes lead to dysen- cholecystectomy)
tery or bloody diarrhea and include Campy!obacter, hemor- Noninvasive infections (giardiasis,
rhagic E.coli, Entamoeba htstolytica, Shigella, and Salmonella. cryptosporidiosis)
Persistent diarrhea (lasting between 14 and 30 days) is usually Small intestinal bacterial overgrowth
caused by a parasitic infection. For example, Glardia lamblia
Steatorrhea Maldigestion (decreased bile salts,
can begin as an acute infection but then become persistent or pancreatic dysfunction)
even chronic. See MKSAP 18 Infectious Disease for further Malabsorption (celiac disease, tropical
discussion ofinfectious diarrhea. sprue. giardiasis, Whipple disease, chronic
mesenteric ischemia, short bowel syndrome,
bacterial overgrowth, lymphatic obstruction)
Evaluation
Acute diarrhea may be accompanied by abdominal cramping, Inflammatory Inflammatory bowel disease (ulcerative
colitis, Crohn disease, microscopic colitis)
tenesmus, flatulence, fever, nausea, and vomiting. For all
patients with acute diarrhea, evaluation of hydration status is Malignancy (colorectal cancer, lymphoma)
important. Stool testing for infectious causes depends on Invasive infections (Clostridium difficile,
cytomegalovirus, Entamoeba histolytica,
many factors, including suspicion of disease outbreak, sever- tuberculosis)
ity ofsymptoms, duration ofsymptoms (>7 days), and whether
lschemia
an individual is considered at increased risk for infectious
Radiation colitis/enteritis
diarrhea (such as a worker in day care, health care, or food
preparation). For example, individuals with Giarrlia present Motility Postsurgical (vagotomy, dumping}
with watery diarrhea that can be explosive and is associated Endocrine (diabetes mellitus,
with bloating due to malabsorption. A stool enzyme-linked hyperthyroidism)

immunosorbent assay Is the best diagnostic test for Gfardia Scleroderma


infection. Miscellaneous Irritable bowel syndrome
Functional diarrhea•
Management Factitious
Fluid therapy is the mainstay of treatment for acute diarrhea.
Overflow
In patients with severe diarrhea or travelers with cholera-like
diarrhea, balanced electrolyte rehydration solutions should be Fecal incontinence
used. Confirmed infection with C. difficile should be treated •Frequent or loose or watery stools without abdominal discomfort in the absence
of other identifiable causes.
with metronidazole, vancomycln, or other agents, depending
Adapted with permission from Schiller LR. Pardi DS, Spiller R, Semrad CE. Surawicz CM,
on patlent comorbidities and clinical severity (see MKSAP 18 G1ann.ella. RA, et al. Gastro 2013 APDW/WCOG Shanghai working party report:
Infectious Disease for discussion of C. difficile infection). chronic diarrhea: definition, classification, diagnosis. J Gastroenterol Hepatol.
2014;29:6·25. JPM ID: 24117999] doi:l0.1 111/jgh.12392.
Treatment of diarrhea in travelers includes bismuth and

26
Disorders of the Small and Large Bowel

causes of chronic diarrhea are irritable bowel syndrome with evaluating chronic diarrhea, special attention should be given
predominant diarrhea (IBS-D) and functional causes. to features suggestive of malignancy, including rectal bleeding,
Medications are also an important cause of chronic diarrhea weight Joss, and age older than so years. Conditions such as
(Table IS). IBS and microscopic colitis are more common in women than
in men. IBS is most common in the third and fourth decades
Evaluation of life, whereas microscopic colitis is most common in the
The evaluation of chronic diarrhea includes a careful history seventh and eighth decades.
and physical examination. Because IBS-D and functional Physical examination should assess for hydration and
causes are the most common causes, further diagnostic stud- nutrition status, perineal disease suggestive of Crohn disease,
ies should be reserved for patients whose symptoms suggest and sphincter defects suggestive of fecal incontinence.
another cause.
Additional Testing
History and Physical Examination The decision to do additional testing, including serologies,
The pattern of diarrhea can help determine the most likely stool studies, imaging, and endoscopy, depends on the clinical
diagnosis. Steatorrhea is often described as malodorous, scenario and likelihood of disease.
greasy, or oily stools that float. The relation of symptoms to Evaluation of the small intestine may include imaging
eating can also provide clues to the cause of diarrhea. Osmotic with small-bowel radiography, CT enterography, or MR enter-
diarrhea occurs with eating and subsides with fasting, whereas ography to identify conditions that could lead to bacterial
secretory diarrhea does not subside with fasting. Information overgrowth, such as small intestinal diverticular disease,
about specific dietary factors that commonly cause osmotic inflammation, or strictures. Upper endoscopy Is indicated
diarrhea, such as lactose, fructose, or artificial sweeteners when small-bowel mucosa! disease (such as celiac disease or
(sorbitol) should be obtained. The patient should be asked chronic infection) is suspected. Capsule endoscopy can be
about new medications (both prescription and nonprescrip- used to visualize the small intestine but does not allow for
tion) and the timing of the initiation of medication in relation sampling. Device-assisted enteroscopy (balloon or spiral over-
to the onset of diarrhea. Nocturnal symptoms are often related tubes) can be used in selected patients who require sampling
to inflammatory conditions, such as inflammatory bowel dis- of small-intestinal tissue based on abnormalities found on
ease (IBD). Abdominal pain accompanying diarrhea suggests imaging or capsule endoscopy. The diagnostic yields for cap-
IBS-D, especially when a bowel movement relieves the pain. In sule endoscopy and device-assisted enteroscopy are low in
patients with chronic diarrhea and normal laboratory or imag-
ing studies.
TABLE 18. Medications that Cause Diarrhea Colonoscopy is the primary diagnostic tool for evaluating
Common causes of diarrhea related to the colon, such as IBD, micro-
scopic colitis, chronic colonic infections, and malignancy. By
Antacids, proton pump inhibitors
definition, colon biopsies are required to exclude microscopic
Chemotherapy colitis. Colonoscopy is especially important in patients with
Antibiotics rectal bleeding and/or age older than SO years.
Colchicine In cases of chronic diarrhea without an underlying diag-
Metformin nosis, stool studies can help clarify the cause. Analysis of stool
NSAIDs, mesalamine
includes fecal weight, stool electrolytes, fecal pH, fat content,
fecal calprotectin, and presence of blood and leukocytes. In
Cholesterol-lowering drugs
watery stools, fecal electrolytes can be used to calculate the
Rarer fecal osmotic gap:
ACE inhibitors 290 - (2 x [stool sodium +stool potassium])
Angiotensin receptor blockers
An osmotic gap of less than SO mOsm/kg (SO mmol/kg)
~-adrenergic receptor antagonists
suggests secretory diarrhea, and a gap greater than 100 mOsm/
Carbamazepine kg (100 mmol/kg) suggests osmotic diarrhea.
Lipase inhibitors The presence of blood or leukocytes in the stool suggests
Lithium an inflammatory cause. A positive 72-hour stool collection for
Prostaglandin fecal fat confirms steatorrhea; a random fecal fat assessment
may be helpful lf a timed collection is not possible. When laxa-
Vitamin/mineral supplements
tive abuse Is suspected, a stool or urine laxative screen can aid
Adapted with permission from Schiller LR, Pardi OS, Spiller R, Semrad CE, Surawicz
CM, Giannella RA, et al. Gastro 2013 APDW/WCOG Shanghai working party
in the diagnosis.
report chronic diarrhea: definition, classification, diagnosis. J Gastroenterol Additional testing may be warranted in immunocompro-
Hepatol. 2014;29:6-25. (PMID: 24117999) doi:10.1111/jgh.12392.
-------- mised patients, patients with secretory diarrhea, and those in

27
Disorders of the Small and Large Bowel

whom celiac disease is suspected. Patients found to have a small- Celiac Disease and Nonceliac
bowel tumor in the setting of diarrhea should be considered for
additional testing including radioimmunoassays for peptides Gluten Sensitivity
and/or 24-hour urine 5-hydroxyindoleacetic acid measurement Celiac disease is an immune- mediated disease that primarily
for carcinoid tumors. Testing for carcinoid tumors should be affects the small intestine in response to dietary gluten. It is
limited to patients with chronic diarrhea and flushing. one of the most common causes of malabsorption, and only
affects individuals who are genetically predisposed. The
Management immune reaction leads to destruction of the small intestinal
The management of chronic diarrhea is determined by its villi starting in the proximal duodenum. Although antibodies
underlying cause. IBS-D and chronic diarrhea with a _func- are produced as part of the immune reaction, they are not
tional cause are treated with various medications aimed at thought to be involved in the pathogenesis of celiac disease.
slowing motility (loperamide), binding bile acids (cholesty-
ramine), altering the gut microbiome (rifaximin), decreasing Testing
bowel contractions (eluxadoline), and modulating neuro- Testing should be pursued in patients with typical gastrointes-
transmitters (antidepressants). tinal symptoms or extraintestinal manifestations, and in those
Treatment of osmotic diarrhea requires avoiding offending with increased risk (patients with type 1 diabetes mellitus,
agents, such as following a low-Fermentable Oligosaccharides, autoimmune thyroid disease, or a first-degree family member
Disaccharides, Monosaccharides, And Polyols (FODMAP) with celiac disease). Gastrointestinal symptoms of celiac dis-
(Table 19) and lactose-free diet. ease typically include chronic diarrhea, bloating, and weight
Treatment of secretory diarrhea is aimed at identification loss, but may also include atypical symptoms, such as consti-
of the underlying cause as well as careful management of pation and dyspepsia. Other manifestations include Iron defi-
hydration. Repletion of fat-soluble vitamins is important in ciency anemia, bone loss, abnormal liver aminotransferase
cases of fat malabsorption. Probiotics are not recommended levels, neurologic symptoms, and dermatitis herpetiformis
for treatment of dJarrhea. (Figure 15). Patients may also be asymptomatic.
Diarrhea induced by medications necessitates changing Ideally, testing for celiac disease should be done while the
medications unless the benefits exceed the potential harms patient is on a gluten-containing diet. The best initial step is to
and the diarrhea is manageable from the standpoint of the test for JgA tissue transglutaminase antibodies. Because lgA
patient. deficiency is more common In patients with celiac disease,
total lgA levels may also need to be measured. Testing for anti-
KEY POINTS
deamidated gliadin peptide IgG antibodies may also be used.
• Acute diarrhea (of less than 2 weeks' duration) is usually Anti-endomysial antibodies are highly specific and can be
caused by infectious agents; chronic diarrhea (of more
than 4 weeks' duration) is most commonly noninfectious.
• Maintaining hydration is the primary goal of treatment
for acute diarrhea.
• The primary causes of chronic diarrhea are irritable
bowel syndrome with predominant diarrhea, functional
causes, and medications.
HVC • Evaluation with imaging, endoscopy, and additional
testing should be reserved for patients whose symptoms
do not suggest irritable bowel syndrome or a functional
cause of chronic diarrhea.

Fructose: honey, apples, pears, peaches, mangos, fruit juice,


dried fruit
Lactose: milk, custard, ice cream, yogurt, soft cheeses
Fructans: wheat, rye, onions, leeks, zucchini
• Galactans: legumes
Sugar alcohols: xylit ol, sorbitol, maltitol, mannitol
FIGURE 15. Dermatitis herpetiformis, a manifestation of celiac disease, is
FODMAP - Fermentable Oligosaccharides, Oisaccharides, Monosaccharides, And characterized by pruritic papules and transient, almost immediately excoriated
Polyols.
blisters on the elbows, knees, and buttocks.

28
Disorders of the Small and Large Bowel

helpful in making the diagnosis of celiac disease, but measure- with refractory celiac disease should be managed in a special-
ment with immunofluorescent microscopy is not reliable. ized celiac disease center.
Testing for anti-gliad!n antibodies (IgA and IgG) should not be
used because of low sensitivity and specificity Jf clinical sus- Nonceliac Gluten Sensitivity
picion is high and initial testing is negative, additional testing Nonceliac gluten sensitivity is defined by gastrointestinal and
should be pursued. Antibody testing is less reliable if the extraintestinal symptoms that occur with gluten ingestion and
patient is on a gluten-free diet. In such patients, genetic testing subside with avoidance of gluten. By definition, celiac disease
for HLA-DQ2 or HLA- DQB should be considered. must be excluded. Because gluten is a nonabsorbable carbohy-
The vast majority of patients with celiac disease carry drate, it can cause gastrointestinal symptoms due to osmotic
HLA-DQ2 or HLA-DQB genetic susceptibility; however, these mechanisms, as well as fermentation by colonic bacteria. Other
genes can be foimd in up to 40% of the general population. than assessing symptom response after withdrawal of gluten,
Therefore, genetic testing can rule out disease but not confirm it. there is no diagnostic test for nonceliac gluten sensitivity.
KEY POINTS
Diagnosis • The best initial test for celiac disease is testing for lgA
A positive serologic test for celiac disease requires upper tissue transglutaminase antibodies.
endoscopy with biopsies from the duodenum for confirmation
• Up to 40% of the general population canies the HLA-DQ2 HVC
of the disease. There are specific recommendations for the
or HLA-DQB mutation, so genetic testing alone cannot
biopsy procedure to minimize false-negative results. Celiac
be used to diagnose celiac disease.
disease Is confirmed by the presence of increased intraepithe-
lial lymphocytosis and villous blunting or atrophy of the duo- • The treatment of celiac disease is lifelong avoidance
denal villi. of gluten.
• The most common cause ofrefractory celiac disease
Management and Monitoring symptoms is gluten exposure.
Patients with a diagnosis of celiac disease should be educated
about the gluten-free diet by a knowledgeable registered dieti-
cian. The treatment of celiac disease is lifelong avoidance of Malabsorption
wheat, rye, and barley. Patients should be aware of gluten !n Small Intestinal Bacterial Overgrowth
medications and over-the-counter products. Dietary oats are Small intestinal bacterial overgrowth (SIBO) is caused by vari-
generally safe in patients with celiac disease. Monitoring ous conditions, including impaired motility, strictures (for
includes clinical assessment of symptoms and signs of celiac example, in Crohn disease), or blind loops (for example. small-
disease, assessment of dietary adherence, and confirmation of bowel diverticula). There is some evidence that SIBO could
normalization of antibody levels. Repeat upper endoscopy contribute to symptoms in IBS-D.
with biopsies should be considered for individuals with ongo- Classically, S!BO is defined by the presence of more than
ing symptoms while adhering to a gluten-free diet. Assessment 105 colony-forming units per mL ofjejunal aspirate. Based on
of bone density at the time of diagnosis should be considered its use in research studies, jejunal aspirate is considered the
to ensure adequate bone health. gold standard but is often not used in clinical practice due to
the requirement for endoscopy to obtain cultures, the patchy
Nonresponsive Celiac Disease nature of small intestinal overgrowth, and oropharyngeal
The most common cause of refractory cellac disease symp- contamination of the specimen. Glucose and lactulose breath
toms is gluten exposure. When a patient's symptoms do not tests have acceptable specificity (around 80'1o) but poor sen-
resolve completely with adherence to a gluten-free diet, the sitivity (30%-40%) for diagnosing SIBO. Diagnosis requires
accuracy of the original diagnosis of cel!ac disease should be typical symptoms and a confirmatory test, usually breath
reassessed. Other conditions that may account for ongoing testing. Treatment consists of antibiotic therapy and often
symptoms include !BS, lactose or fructose intolerance, bacte- requires repeated courses if the underlying condition cannot
rial overgrowth, microscopic colitis, or IBD. Additional evalu- be resolved.
ation to exclude these as causes is critical before considering
the patient to have refractory celiac disease. Short Bowel Syndrome
A very small number of patients develop refractory celiac Short bowel syndrome is defined by a small-intestine length of
disease, in which small-intestinal inflammation persists less than 200 cm, with loss of absorptive area leading to mal-
despite adherence to a strict gluten- free diet. Individuals with digestion, malabsorption, and malnutrition. Surgery for stran-
refractory sprue are typically older than age 65 years and pre- gulated bowel, Crohn disease, ischemia, trauma, and weight
sent with diarrhea, weight loss, dehydration, and nutritional loss can all lead to short bowel syndrome.
deficiencies. Possible medication-induced sprue should be Treatment depends on whether the patient has an intact
managed by discontinuing the offending medication. Patients ileocecal valve and colon or an ostomy. Patients with short

29
Disorders of the Small and Large Bowel

bowel syndrome and an ostomy usually require parenteral IBD has a bimodal age presentation, with an initial peak
nutrition and hydration. Those with extensive ilea! resection incidence in the second to fourth decades of life followed by a
should be tested and treated for vitamin B12 deficiency. less prominent second peak in the seventh and eighth decades.
Adjunctive therapies include antimotility and antisecretory
drugs. Glucagon-like peptide 2 and its analog, teduglutide, Clinical Manifestations
are new pharmacologic agents for treatment of short bowel
Ulcerative CoJitis
syndrome. Both have been evaluated in randomized trials
The major symptoms of ulcerative colitis include diarrhea,
and found to increase intestinal wet weight absorption and
abdominal discomfort, rectal bleeding, and tenesmus, with
decrease parenteral fluid support in patients with short
symptoms varying depending on the extent and severity of
bowel syndrome.
disease. Symptoms typically have a slow, insidious onset and
often have been present for weeks or months by the time the
Carbohydrate Malabsorption
patient seeks care, although ulcerative colitis may present
Carbohydrates can be classified as monosaccharides (glucose,
acutely, mimicking infectious colitis.
fructose), disaccharides (lactose, sucrose), oligosaccharides
Rectal inflammation (proctitis) causes frequent defeca-
(maltodextrose), or polyols (sorbitol, mannitol). These short-
tory urges and passage of small liquid stools containing
chain carbohydrates are osmotically active and can lead to
mucus and blood. Although bloody diarrhea is considered
increased luminal water retention and gas production through
the hallmark presentation of ulcerative colitis, diarrhea is not
colonic fermentation. These two actions can cause gastrointes-
always present. Patients with proctitis or proctosigmoiditis
tinal symptoms, including gas, bloating, and diarrhea. may have constipation. Abdominal pain is usually not a
Lactose malabsorption is commonly due to loss of the
prominent symptom of ulcerative colitis; however, most
brush border lactase enzyme in adulthood. Fructose malab-
patients with active disease experience vague lower-abdomi-
sorption can also lead to gastrointestinal symptoms such as
nal discomfort relieved with defecation. Physical examina-
bloating and diarrhea. Although both lactose and fructose
tion in patients with mild or moderate ulcerative colitis is
breath tests are available, testing is often not required, as
usually normal but may reveal mild lower-abdominal dis-
symptoms subside with exclusion of the sugar from the diet
comfort over the affected colonic segment. The presence of
and recur with ingestion.
fever, nausea, vomiting, or severe abdominal pain indicates a
KEY POINTS severe attack or complication such as superimposed infection
• Small intestinal bacterial overgrowth is caused by vari- or toxic megacolon.
ous conditions, including impaired motility, strictures,
or blind loops. and is treated with antibiotic therapy. Crohn Disease
• Short bowel syndrome is defined by a small-intestine The clinical presentation of Crohn disease may be subtle and
length of less than 200 cm, resulting in maldigestion, varies depending on the location and severity of inflammation
malabsorption, and malnutrition. along the gut axis as well as the presence of intestinal compli-
cations such as abscess, stricture, or fistula. Compared with
• Lactose malabsorption is common, with symptoms
ulcerative colitis, abdominal pain is a more common symptom
occurring when lactose is ingested and subsiding with
of Crohn disease. The ileocecal area is the most common bowel
exclusion of lactose from the diet.
segment affected by Crohn disease, and it often presents insid-
iously with mild diarrhea and abdominal cramping.
Abdominal examination may reveal fu1lness or a tender mass
Inflammatory Bowel Disease in the right hypogastrium. Some patients present initially with
IBO is an idiopathic chronic inflammatory condition of the gut a small-bowel obstruction caused by impaction of indigestible
that includes ulcerative colitis and Crohn disease. In addition, vegetables or fruit. Occasionally, the main presenting symp-
microscopic colitis Is considered a type of IBO with distinct tom is acute pain in the tight lower quadrant, mimicking
clinical and pathologic features. The pathogenesis of IBD lil<ely appendicitis. In patients with Crohn colitis, tenesmus is less
involves host genetic predisposition and abnormal immuno- common than in patients with ulcerative colitis because the
logic responses to endogenous gut bacteria. rectum is often less inflamed than other colonic segments.
Perianal disease is a common presentation of Crohn disease
Risk Factors with anal fissures, ulcers, and stenosis.
The primary risk factor for development of IBD is family his- Fistulae are a frequent manifestation of the transmural
tory, with a risk of approximately 10% for first-degree rela- nature of Crohn disease and consist of abnormal connections
tives of affected patients. Individuals of Ashkenazt Jewish between two epithelial surfaces (perianal, enteroenteric,
descent have increased risk for IBO. Tobacco smoking enterocutaneous, rectovaginal, enterovesical). Drainage of fecal
increases the risk for Crohn disease and is protective for material from fistulae leads to symptoms such as passage of
ulcerative colitis. feces through the vagina (rectovaginal fistula). Intra-abdominal

30
Disorders of the Small and Large Bowel

abscesses may form; the classic presentation is spiking fevers experience an extraintestinal manifestation at some time dur-
and focal abdominal tenderness, which may be masked by the ing the course of their disease; peripheral arthritis is the most
use of glucocorticoids. Strictures represent long-standing common. See MKSAP 18 Rheumatology for discussion of TB D-
inflammation and may occur in any segment of the gastroin- re lated arthritis.
testinal tract, although the terminal ileum is the most common The two most common dermatologic extraintestinal man-
site. Strictures may be secondary to fibrosis or severe inflam- ifestations are erythema nodosum and pyoderma gangreno-
matoiy luminal narrowing. Patients with intestinal strictures sum. Erythema nodosum most commonly presents as single
often initially present with colicky postprandial abdominal or multiple tender nodules on extensor surfaces of the lower
pain and bloating that may progress to complete intestinal extremities (Figure 16). Pyoderma gangrenosum typically pre-
obstruction. sents as a papule that rapidly develops into an ulcer with
Table 20 summarizes the features of u lcerative colitis and undermined and violaceous borders (Figure 17). Both mani-
Crohn disease. festations usually correspond to underlying IBD activity. See
MKSAP 18 Dermatology for discussion of cutaneous manifes-
Extraintestinal Manifestations tations ofIBD.
Inflammatoiy conditions involving extraintestinal structures, Ocular extraintestinal manifestations ofIBD include epis-
including the joints, eyes, liver, and skin, may occur in patients cleritis and uveitis. Episcleritis is more common and consists
with IBD. These extraintestinal manifestations are categorized of injection of the sclera and conjunctiva. It does not affect
as either associated with active bowel disease or independent visual acuity and occurs in association with active bowel dis-
of bowel inflammation. Up to 30% of patients with IBD ease. Uveitis presents with headache, blurred vision, and

TABLE 20 . Features of Ulcerative Colitis and Crohn Disease


Feature Ulcerative Colitis Crohn Disease

Depth of inflammation Mucosal Transmural


I
I
Pattern of disease Contiguous and symmetric Skips areas and asymmetric
Location Colorectum Mouth to anus
I Rectal invo lvement Nearly 100% Less common
I
/ 11eal disease Backwash ileitis (15%) Common
Fistulas, abscess, and strictures Rare Common i
I Perianal disease
Granulomas
Rare
Unlikely
Common
In approximately 30%
II
I
I Overt rectal bleeding Common Less common I
l_~_o_b_a_c_co_us_e_ _ _ _ _ _ _ _ _ _ _ _ _P_r_o_te_ct_iv_e_ _ _ _ _______ Exacerbates I
------------~__)

FIG UR f. 16. Erythema nodosum, a manifestation of inflammatory bowel FIGURE 1 7. Pyoderma gangrenosum, a manifestation of inflammatory bowel
disease, typically appears as ill-defined erythema overlying subcutaneous, tender disease, typically begins as a small pustule or red nodule that rapidly expands with
nodules most commonly symmetrically located on the anterior shins. an edematous, infiltrated, actively inflamed border and a painful, exudative wet ulcer.
The border is characteristically violaceous with an edge that overhangs the ulcer.

31
Disorders of the Small and Large Bowel

photophobia. Uveitis is an ocular emergency requiring prompt Radiographic studies establish the location, extent, and
referral to an ophthalmologist. See MKSAP 18 General Internal severity of IBD. Patients with a severe attack of IBD require a
Medicine for discussion of episclerltis and uveitis. plain abdominal radiograph to assess for a dilated colon
Primary sclerosing cholangitis is a major liver manifesta- (indicative of evolving toxic megacolon) or small-bowel
tion of IBD, occurring in 5% of patients. Patients most often obstruction (Figure 18). CT or MR enterography provides
present with isolated elevations in the serum alkaline phos- information about the location and severity of small bowel
phatase level. The liver disease is typically progressive and disease and the presence of complicating fistula, abscess, or
independent of the outcome of the IBD. See Disorders of the stricture. Video capsule endoscopy is a highly sensitive modal-
Liver for discussion of primary sclerosing cholangitis. ity for detection of small inflammatory lesions of the intestine,
although it is not commonly required.

Diagnosis Treatment
The diagnosis of IBD relies on the Integration of the clinical The goals of therapy for IBD are to induce and maintain
presentation, exclusion of infectious enteropathogens, endo- remission, and to prevent disease- and treatment-related
scopic appearance, histologic assessment ofmucosal biopsies, complications. Four categories of drugs are used to treat IBD:
and radiologic features. IBD should be considered in any 5-aminosalicylates, glucocorticoids, irnmunomodulators,
patient with chronic or bloody diarrhea. It is paramount to and biologics. Stratification based on clinical severity is
exclude infection, particularly with C. difficile and Shiga important in guiding IBD management. Currently, there are
toxin-producing E. coll, by stool tests, especially ln patients no validated or consensus definitions of mild, moderate, or
with acute onset of symptoms. Fecal calprotectin should be severe IBD. However, three domains are relevant to the evalu-
considered to help differentiate IBD from irritable bowel syn- ation of disease severity in IBD: impact of the disease on the
drome. Laboratory testing helps to assess disease activity. patient (clinical symptoms, quality of life, and disability);
Common findings include anemia and hypoalbuminemia. inflammatory burden (extent, location, and severity of bowel
Many patients develop iron deficiency anemia from chronic involvement); and disease course, including structural
blood loss. Hematologic changes, such as thrombocytosis and
leukocytosis, reflect active inflammatory disease. Persistently
abnormal serum alkaline phosphatase levels should prompt
further investigation for primary sclerosing cholangitis.
Endoscopy (either sigmoidoscopy or colonoscopy) with
biopsy is needed to help make the diagnosis of IBD.
Colonoscopy is most commonly used to assess the extent and
severity of disease. At presentation, 50"/o of patients with ulcer-
ative colitis have disease limited to the rectum and sigmoid
(proctosigmoiditis), 20% have left-sided disease (to the splenic
flexure), and 30'1o present with pancolitls (to the cecum).
Endoscopic findings range from decreased vascular pattern
with erythema and edema in mild disease to large and deep
ulcerations in severe disease. Histopathology characteristically
shows features of chronic colitis with distorted and branching
colonic crypts along with crypt abscesses.
Crohn disease has a different pattern of distribution from
ulcerative colitis: SO'Yo of patients have ileocolonic disease, 30%
have isolated small bowel disease, and 20% have colonic disease.
A minority of patients have isolated upper gastrointestinal tract
or perianal disease in the absence of 1ntlammation in the small
bowel or colon. The earliest endoscopic findings of Crohn disease
include aphthous ulcers, which can coalesce to form stellate
ulcers, and a "cobblestone" mucosa! appearance. A characteristic
mucosa! feature of Crohn disease is the so-called "skip lesion,"
consisting of affected areas separated by normal mucosa.
Granulomatous inflammation is characteristic of Crohn disease
but uncommonly found on mucosa! biopsies. Hlstopathology in
small-intestinal Crohn disease will show chronic jejunitis or ile- FIGURE 1 8. CT scan of the abdomen and pelvis in a patient with Crohn
itis, and Crohn colitis will have histology similar to that in ulcera- disease, showing small-bowel obstruction with dilated loops of small intestine
tive colitis, with exception of granulomas. (arrow) and matted loops of bowel (arrowhead) in the pelvis.

32
Disorders of the Small and Large Bowel
-------
damage. Surgery is reserved for refractory symptoms and maintenance therapy and have significant adverse effects. One
complications. formulation of oral budesonide is a controlled-release gluco-
Patients with IBD are at markedly increased risk for corticoid with high first-pass metabolism in the liver and mini-
venous thromboembolism. The cause of thromboembolism is mal systemic adverse effects. It is effective in inducing remis-
multifactorial and related to severity of disease, immobiliza- sion in mild to moderate ileocolonic Crohn disease. Another
tion, and hospitalization. It is important that all hospitalized oral formulation is multirnatrix (MMX) budesonlde, designed
patients with IBO be given venous thromboembolism prophy- to release the drug throughout the colon. It is effective in
laxis with subcutaneous heparin. Only in cases of massive inducing remission in mild to moderate ulcerative colitis.
gastrointestinal bleeding with severe anemia, tachycardia, and
hypotension should nonpharmacologlc prophylaxis with Immunomodulators
intermittent pneumatic compression of the lower extremities Thiopurines (azathioprine and rnercaptopurlne [6-MP]) are
be used. lmmunomodulators used as glucocorticoid-sparing agents.
They have a slow onset of action (2-3 months) and patients
Pharmacotherapy require a tapering glucocorticoid regimen to bridge the time
5-Aminosalicylates interval until the thiopurines take effect. Thiopurine methyl-
5-Aminosalicylates (5-ASAs) are believed to have an anti- transferase, a key enzyme involved in the metabolism of aza-
inflammatory mechanism of action. Various formulations and thioprine and 6-MP, exhibits a population polymorphism that
controlled-release systems are available, with some prepara- greatly increases the risk for bone marrow toxicity with use of
tions purported to deliver 5-ASAs to the small bowel. these agents. Therefore, before initiation ofthiopurine therapy,
5-ASAs are the mainstay of treatment of mild to moderate testing for the TPMT genotype or phenotype (enzyme activity)
ulcerative colitis, with a dose-dependent response when used is recommended to help prevent toxicity by identifying indi-
to induce disease remission. Three major factors need to be viduals with low or absent TPMT enzyme activity. However,
considered when choosing therapy for ulcerative colitis regardless of TPMTstatus, all patients require monitoring with
(Figure 19). Patients with proctitis or left-sided disease should complete blood counts and liver chemistry testing because
receive topical therapy with 5-ASA suppositories and enemas. 70% of patients who develop leukopenia while using these
In mild to moderate ulcerative colitis, combined 5-ASA ther- agents do not have TPMT mutations. Azathloprine and 6-MP
apy (oral and topical) Is superior for induction of remission are reported to cause the rare hepatosplenlc T-cell lymphoma.
compared with oral or topical therapies alone. Once remission Azathioprine and 6-MP are effective in maintaining remission
is achieved, 5-ASAs are effective in maintaining it. Of the avail- in patients with ulcerative colitis and should be considered in
able agents, sulfasalazine has the most adverse effects, includ- glucocorticoid-dependent patients. This includes patients who
ing fever, rash, nausea, vomiting, and headache. In addition, require two courses of glucocorticoids for induction of remis-
sulfasalazine may cause reversible sperm abnormalities and sion within 1 year, or patients who require intravenous gluco-
impair folate absorption. corticoids for acute disease flare.
Despite the availability of several formulations designed Methotrexate is an imrnunomodulator that is beneficial in
to deliver the drug to the small bowel, 5-ASAs have not proved inducing and maintaining remission in Crohn disease but not
to be efficacious in small-bowel Crohn disease. in ulcerative colitis. Side effects of methotrexate include hepa-
totoxicity and interstitial pneumonitis, which can manifest
Glucocorticoids with cough and dyspnea of insidious onset.
Oral and intravenous glucocorticoids are corrunonly used to
treat moderate to severe flares of IBD and are effective in induc-
Biologic Agents
ing remission. However, glucocorticoids are not effective for
Tumor necrosis factor (TNF)-a is a proinflarrunatory cytokine
that plays a critical role in the pathogenesis of both Crohn
Disease Severity: Prior Therapy: disease and ulcerative colitis. The anti-TNF agents intliximab,
Mild Response adalimumab, and certolizumab are used to treat moderate to
Moderate Side effects
Severe Compliance severe Crohn disease. Infliximab is administered by intrave-
nous infusion; adalimumab and certolizurnab are given sub-
cutaneously. Combination therapy with intliximab and aza-
thioprine is more efficacious than rnonotherapy with either
Distribution of
Disease:
agent alone in achieving glucocorticoid-free remission and
Proctitis mucosa! healing. There is increasing evidence for the use of
Left-sided biologic agents early in the course of disease. Before Initiation
Extensive
of anti-TNF agents, patients should undergo testing for latent
Fl GU RE 19. Factors to consider when choosing medical therapy for ulcerative tuberculosis because of an increased risk for reactivation of
colitis. tuberculosis during therapy. If latent tuberculosis is present,

33
Disorders of the Small and Large Bowel

treatment with isoniazid should occur for at least 2 months extraintestinal manifestations. Patients with IBD are at
before initiation of anti-TNF therapy. Patients should also be increased risk for metabolic bone disease due to use of gluco-
assessed for chronic hepatitis B viral infection before starting corticoids and diminished vitamin D and calcium absorption.
anti-TNF therapy and receive treatment if needed. Patients with Crohn disease are at greater risk than those with
The anti-TNF agents infliximab, adalimurnab, and goli- ulcerative colitis. Screening for osteoporosis should be consid-
mumab are used to treat moderate to severe ulcerative colitis. ered In patients at increased risk.
Combination therapy with intliximab and azathioprine is Patients with IBO are at increased risk for developing
more effective than monotherapy with either agent in achiev- cancers of the colorecturn, cervix, and skin. Longstanding
ing glucocorticoid-free remission and mucosal healing. inflammation of the colorecturn Increases the risk for cancer.
The anti-adhesion agents natalizumab and vedolizumab In patients with IBO (ulcerative colitis with disease proximal
are effective in inducing and maintaining remission for mod- to the sigmoid colon and Crohn disease with more than one
erate to severe Crohn disease. Ustekinumab, a monoclonal third of the colon involved), surveillance colonoscopy should
antibody that blocks the biologic activity of interleukin-12 and begin 8 years after diagnosis and recur every 1 to 2 years there-
-23 by inhibiting receptors for these cytoklnes on T-cells, is after. Primary scleroslng cholangitis increases the risk for
efficacious in severe Crohn disease. These agents are typically colorectal cancer, and surveillance colonoscopy should begin
used after anti-TNF therapies prove ineffective. at the time of diagnosis and recur yearly thereafter.
Patients with IBO are at increased risk for both melanoma
Surgery and nonmelanoma skin cancers. Most of the risk has been
Indications for surgery In patients with Crohn disease include associated with specific treatments; however, there is some
medically refractory fistula, fibrotic stricture with obstructive evidence that IBD is associated with an increased risk for
symptoms, symptoms refractory to medical therapy, and can- melanoma, independent of treatment. All patients with lBD
cer. The guiding principle of surgery in Crohn disease is the should be advised to use sunscreen, wear protective clothing,
preservation of bowel length and function, as the rate of dis- avoid tanning beds, and have a yearly skin examination.
ease recurrence after segmental resection is high. Patients
with Crohn disease who undergo surgery require aggressive Microscopic Colitis
pharmacologic treatment with anti-TNF agents and/or immu- Microscopic colitis is a distinct type of IBO characterized by
nomodulators to decrease the rate of postoperative recurrence macroscopically normal mucosa with inflammatory changes
of Crohn disease. seen only on histopathology of colon biopsies. It is subclassi-
In patients with ulcerative colitis, total proctocolectomy fied Into lymphocytic colitis and collagenous colitis (Figure 20)
with end-ileostomy or Heal pouch-anal anastomosis is per- based on predominating histologic features. It predominantly
formed for medically refractory disease, toxic megacolon, or affects middle-aged women and is associated with other auto-
carcinoma. immune conditions, particularly celiac disease. It presents
with abrupt or gradual onset of watery diarrhea that has a
Health Care Considerations relapsing and remitting course over months to years, some-
Patients with IBO are at Increased risk for vaccine-preventable times accompanied by mild weight loss.
illnesses, and vaccines are underutilized in this patient popu- Several classes of medications, including NSAlDs, selec-
lation. Inactivated vaccines can be safely administered to all tive serotonin reuptake inhibitors, and proton pump Inhibitors
patients with IBD, regardiess of irnmunosuppression. Patients
with IBD should receive a seasonal influenza vaccine as well as
the 13-valent pneumococcal conjugate vaccine and the
23-valent pneumococcal polysaccharide vaccine. Ideally,
pneumococcal vaccination should occur before beginning
immunosuppressive therapy. Live vaccines such as measles,
mumps, rubella; varicella; and herpes wster are contraindi-
cated In immunosuppressed patients with IBD. See MKSAP 18
General Internal Medicine for discussion of vaccination
strategies.
Women with IBD are at Increased risk for developing
cervical dysplasia; this risk is greater in women with Crohn
disease than in those with ulcerative colitis, and is also greater
in women using irnmunosuppressive therapy. Women with
IBD should undergo Pap testing annually, and human papil-
lomavirus vaccination is recommended. FIGURE 2 0. Collagenous colitis: colon mucosa! biopsy showing a pink,
All patients with IBD should be encouraged to stop smok- abnormal subepithelial collagen band (arrowhead) and lamina propria expanded
ing. Smoking increases Crohn disease activity and the risk for by inflammatory cells (arrow).

34
Disorders of the Small and Large Bowel

have been associated with development of microscopic colitis. TABLE 21. Secondary Causes of Constipation
The first step in management is to discontinue any potentially
Medications
causative medication. First-line treatments include supportive
treatment with antidiarrheal agents such loperamide or bis- Opioids
muth subsalicylate. The next step is oral budesonide, which is Antidiarrheals
efficacious but has a high rate of recurrent symptoms when
Anticholinergics (antispasmodics, antiparkinsonian drugs,
discontinued. Unlike Crohn disease and ulcerative colitis, tricyclic antidepressants, antipsychotics)
there is no long-term increased risk for colorectal cancer in
Antihistamines
patients with mi.croscopic colitis.
NSAIDs
KEY POINTS
Iron supplements
• The major symptoms of ulcerative colitis include diar-
rhea, abdominal discomfort, rectal bleeding, and tenes- Calcium supplements
mus, with a slow onset of symptoms. Bismuth
• Fistula, abscess, and stricture are characteristic compli- Antihypertensives (calcium channel blockers, diuretics,
cations of Crohn disease. clonidine)

• Endoscopy with biopsy is needed to help make the


diagnosis of inflammatory bowel disease.
I Serotonergic antagonists (ondansetron)

' Mechanlc:al Causes


• The goals of therapy are to induce and maintain remis-
Colorectal cancer
sion of inflammatory bowel disease, and to prevent dis-
ease- and treatment-related complications. Rectocele

• Unlike patients with Crohn disease or ulcerative colitis, I Rectal intussusception


patients with microscopic colitis are not at increased Rectal prolapse
risk for colorectal cancer. Sigmoidocele

Enterocele
I
Constipation Anastomotic stricture
Constipation is one of the most common gastrointestinal symp- Anal stenosis/ stricture
toms, affecting 20% of the general population. Constipation can
Extrinsic compression from pelvic/abdominal process
present with symptoms including infrequent, difficult, or
incomplete defecation. lt can be acute or chronic, and either Systemic Illnesses
secondary or idiopathic in nature. Medications are the most Endocrinologic
common cause of secondary constipation; other causes include
Diabetes mellitus
mechanical obstruction, systemic illnesses, altered physiologic
states, and psychosocial conditions (Table 21). Hypothyroidism
Once secondary causes have been excluded, chronic con- Panhypopituitarism
stipation is considered to be functional (idiopathic). The defi-
Pheochromocytoma
nition of functional constipation has been refined by the
fourth Rome working group for functional gastrointestinal Glucagonoma

disorders. Functional constipation is subtyped into categories Neuropathy/myopathy


of slow transit, normal transit, or dyssynergic defecation. Altered Physiologic State
Slow transit constipation is defined as the delayed passage of
fecal contents through the colon based on objective transit Hypercalcemia
testing (radiopaque marker study, scintigraphy, or the wi.reless Hypokalemia
motility capsule). Normal transit constipation is idiopathic Pregnancy
constipation in which colonic transit times are adequa~e
Porphyria
based on objective transit testing. Dyssynergic defecation
(also termed pelvic floor dyssynergia, obstructed defecation, Heavy-metal poisoning (arsenic, lead, mercury)
or outlet obstruction) refers to difficulty with or inability to Psychosocial
expel stool as a result of some combination of abnormalities
in contraction and/or relaxation of the muscles of the pelvic Depression

floor during defecation. In some cases, functional constipa- Cognitive impairment


tion can be the result of slow transit constipation and coexist- Immobility
ent dyssynergic defecation.

35
Disorders of the Small and Large Bowel

Evaluation change in bowel habits, or unexplained anemia. It is also used


A careful medical, surgical, and medication history identifies for colon cancer screening in patients at average and increased
most causes of secondary constipation. The medication his- risk due to family history of colorectal cancer. Additional
tory should focus on the temporal relationship between medi- imaging or laboratory studies should only be considered if
cations and the development of constipation symptoms. The clinically indicated. Physiologic testing, including colon transit
history should include an assessment for the presence of testing, anorectal manometry, balloon expulsion testing, or
alarm features: hematochezia, acute constipation in elderly defecography, is reserved for patients with constipation symp-
patients, unintentional weight loss, family history of colorectal toms that do not respond to initial trials oflaxative therapy.
cancer, unexplained anemia, and age older than 50 years with Functional (idiopathic) constipation is diagnosed once
no previous colonoscopy. Anorectal examination, including a secondary causes have been excluded.
digital examination during Valsalva maneuver, Is an important
part of the physical examination because it may identify ana- Management
tomic or functional causes of an evacuation disorder. Various treatment strategies may benefit patients with consti-
A flat plate radiograph of the abdomen is the most useful pation when lifestyle and dietary interventions have been
initial study because it can assess for the presence and distri- ineffective. Treatment options include bulking agents, stimu-
bution of excessive stool in the colon and/or rectum. lant laxatives, osmotic laxatives, stool softeners, secretagogues,
Colonoscopy is used to assess blood in the stool, a sudden and/or biofeedback (Table 22). The evidence for treatment

TABLE 22 . Treatments for Constipation


Intervention Mechanism of Action Considerations

Bulk Laxative Increases ability of stool to retain water Start with a low dose and increase slowly
Soluble fiber (psyllium, methylcel lulose, Bulks stool Soluble fiber works better than insoluble fiber
calcium polycarbophil, wheat dextrin) Speeds movement of stool through colon Bloating, distension, flatulence, and cramping
Insoluble fiber (bran, rye, flax seed) may be limiting
Stool Softener A detergent that allows water to penetrate Minimal effectiveness in clinical trials
the stool Only role is in mild constipation symptoms
Docusate sodium, docusate calcium
Well tolerated
Few side effects
Osmotic Laxative Poorly absorbed compounds Polyethylene glycol 3350 and lactulose
improve stool frequency and consistency;
Polyethylene glycol 3350 Creates an osmotic gradient
others have not been tested in cl inical trials
Lactulose Water moves into the bowel lumen
Bloating and gas can be limiting
Magnesium hydroxide/magnesium citrate
Caution with use of magnesium in renal
Sorbitol insufficiency
Stimulant Laxative Irritates the colon wall, increasing Quickest acting (works within 8-12 hours of
contractions ingestion)
Anthraquinones (senna, cascara)
Stimulates sensory nerves lining the colon Senna can cause melanosis coli (benign
Diphenylmethanes (bisacodyl, sodium
pigmentation ofthe colon)
pi cosulfate) May inhibit water absorption in the colon
Diarrhea, cramping, bloating, and nausea can
be limiting
Secretagogue Lubiprostone activates type-2 chloride Lubiprostone improves stool form and
channels on enterocytes lining the g ut frequency, straining, and abdominal pain;
Lubiprostone
lumen, causing chloride ions to move into nausea is a common side effect
Linaclotide the colonic lumen with sodium and water
Linaclotide and plecanatide improve stool
Plecanatide following the ionic gradient
form and frequency, straining, bloating, and
Linaclotide and plecanatide activate abdominal pain; diarrhea is a common side
guanylate cyclase C receptors on enterocytes, effect
leading to chloride channel activation through
a series of processes within the cell
Serotonergic Agent Increases intestinal contractions Not available in the United States
Prucalopride Increases intestinal secretion Available in Canada and Europe
Neuromuscular Reeducation Retrains the skeletal muscle involved in Effective for dyssynergic defecation
(biofeedback) defecation (abdominal wall, pelvic floor,
Requires a specially trained physical therapist
anorectal)
Corrects altered rectal sensation

36
Disorders of the Small and large Bowel

efficacy is strongest for polyethylene glycol, lubiprostone, and control of symptoms. There is limited evidence to suggest that
linaclotlde. Fiber supplements, polyethylene glycol, magne- exercise, stress management, and correction of impaired sleep
sium, senna, docusate, and bisacodyl have the advantage of can alleviate the symptoms of IBS.
being available without a prescription. For refractory cases of The most common dietary intervention is an increase in
constipation, combination therapy should include agents with fiber, either through diet or use of fiber supplements. Water-
different mechanisms of action, such as an osmotic plus a soluble fiber supplements such as psyllium are more effective
stimulant laxative. than insoluble dietary fiber such as bran. Dietary restrictions
Although lubiprostone has an FDA indication forthe treat- can include avoidance of trigger foods, gluten (in the absence
ment of opioid-induced constipation, initial treatment should of celiac disease), dairy products, and FODMAPs (see TabIe 19).
be the same as for other forms of constipation. Three peripher- Randomized trials have shown that a low-FODMAP diet allevi-
ally acting µ-opioid receptor antagonists (methylnaJtrexone, ates symptoms in patients with JBS. When these initial meas-
naloxegol, and alvimopan) have also been approved for the ures fail to relieve symptoms, medications, typically directed
treatment of opioid-induced constipation in selected patients. at the primary symptoms of JBS such constipation, diarrhea,
or abdominal pain, are employed. The evidence supporting the
KEY POINTS
use of the various agents is variable, with few rigorous studies
• The most common cause of secondary constipation is showing long-term effectiveness.
medication.
• Treatment strategies include dietary and lifestyle modi-
Therapy for Irritable Bowel Syndrome
fication. addressing causes of secondary constipation,
with Predominant Constipation
and various pharmacologic agents.
Several peripherally acting medications have demonstrated
efficacy and safety in the treatment oflBS-C (see Table 22). The
American Gastroenterological Association (AGA) has given a
Irritable Bowel Syndrome strong recommendation based on high-quality evidence for
IBS represents a heterogeneous group of functional bowel dis- the use of linaclotide, followed by a conditional recommenda-
orders defined by the presence of abdominal pain in associa- tion for the use of lubiprostone based on moderate-quality
tion with defecation and/or a change In bowel habits. evidence and a conditional recommendation for the use of
Abdominal pain may worsen or subside with defecation. The polyethylene glycol based on low-quality evidence in the treat-
altered bowel habits may include constipation, diarrhea, or a ment of IBS-C. Probiotics may offer benefit particularly for
mix of both types. Other commonly reported symptoms symptoms of pain, bloating, and flatulence, although specifics
include abdominal bloating and abdominal distention. The regarding the most beneficial species and strains is unknown.
exact cause of IBS remains unknown, and there are no all-
encompassing pathophysiologic mechanisms to explain the Therapy for Irritable Bowel Syndrome
symptoms. IBS is more common in women and adults younger with Predominant Diarrhea
than age 50 years, and is frequently seen in association with Prescription medications with FDA approval for the treatment
psychosocial disturbance. There are no specific anatomic or of IBS- D include rifaximin, eluxadoline, and alosetron. A
physiologic abnormalities, nor are there any reliable biomark- 14-day course ofrifaximin has shown superiority to placebo in
ers to define IBS. The diagnosis of IBS requires symptoms of relieving the global symptoms, bloating, abdominal pain, and
recurrent abdominal pain at least 1 day a week for a period of loose stools associated with IBS-D for up to 10 weeks after
3 months, along with at least two of the following three addi- treatment. A retreatment study of patients with recurring
tional criteria: pain related to defecation, change in stool fre- IBS-D symptoms after an initial course of rifaximin showed
quency, or change in stool consistency. IBS can then be further that a second 14-day treatment of rifaximin was superior to
subtyped into JBS with predominant constipation (IBS- C), JBS placebo in relieving abdominal pain and improving stool fre-
with predominant diarrhea (IBS-D), IBS with mixed bowel quency for 4 weeks after treatment.
habits, or IBS unclassified. Eluxadoline (combination of a µ-opioid receptor agonist
and a a-opioid receptor antagonist) was superior to placebo in
Evaluation the treatment of men and women with IBS-D for up to 26
The diagnosis of IBS is no longer one of exclusion; instead, the weeks based on a composite response of decreased abdominal
diagnosis can reliably be made by clinical criteria in the pain and improved stool consistency in two randomized trials.
absence of alarm features. Therefore, evaluation for IBS relies Use of eluxadoline is contraindicated in patients without a
on a comprehensive history and physical examination. gallbladder and in those with known or suspected biliary
obstruction, Sphincter of Oddi disease or dysfunction, inges-
Management tion of three or more alcoholic beverages a day, history of
Management of JBS includes lifestyle and dietary modifica- pancreatitis or structural disease of the pancreas, severe
tions, and reassurance that the disease is benign. The focus is hepatic impairment, or history of severe constipation.

37
Disorders of the Small and Large Bowel

Alosetron (a selective 5-HT3 antagonist) has alleviated laboratory studies to exclude gastrointestinal bleeding and
abdominal pain and the global symptoms of women with inflammation.
IBS-D in pooled data from several randomized trials. The AGA Successful treatment depends on the patient-physician
has given it a conditional recommendation for the treatment relationship, and should focus on setting appropriate goals
of women with IBS-D based on moderate-quality evidence. and expectations. A combination of pharmacologic and/or
Due to the risk for severe constipation and ischemic colitis, psychological therapies may be needed, including tricyclic
prescribers must be enrolled in an FDA-mandated risk evalu- antidepressants, selective serotonin reuptake inhibitors, or
ation and mitigation strategy program in order to prescribe serotonin-norepinephrine reuptake inhibitors. Four classes of
alosetron. psychotherapy have shown benefits in CAPS when combined
Other medications with clinical evidence for the treat- with medical therapy: cognitive-behavioral therapy, psycho-
ment of IBS-D include loperamide, antispasmodics, and tricy- dynarnlc-interpersonal therapy, mindfulness- and accept-
clic antidepressants. The AGA has given a conditional ance-based therapy, and hypnotherapy.
recommendation for loperamide use in patients with IBS-D. Narcotic bowel syndrome, also known as opiate-induced
Although loperamide has not demonstrated global relief of gastrointestinal hyperalgesia, ls characterized by the paradoxi-
IBS-D symptoms, this recommendation is based on its ability cal increase in abdominal pain with increasing doses of nar-
to reduce stool frequency in other diarrheal conditions, as well cotics despite clinical evidence showing improvement or
as its low cost, favorable safety profile, and wide availability. stability of the underlying condition. Often these patients fear
Antispasmodics (for example, cimetropium-dicyclomine, tapering off narcotics and believe the narcotics are "the only
peppermint oil, pinaverium, and trimebutine) decreased thing that helps." The only treatment Is complete detoxifica-
abdominal pain and global symptoms ofIBS in a meta-analysis tion and cessation of narcotic use, which requires a trusting
of 22 randomized trials. Although the overall quality of studies patient-physician relationship and understanding of the
was low, the AGA gave a conditional recommendation for the pathophysiology of the pain. Enrollment in a supervised
use of antispasmodics in IBS-D. Based on data from several detoxification program is recommended.
randomized trials, tricyclic antidepressants offer modest relief
KEY POINTS
of the abdominal pain and global symptoms oflBS-D. The AGA
has given a conditional recommendation based on low-quality • The evaluation for centrally mediated pain syndrome in HVC
evidence for the use of tricyclic antidepressants in the treat- the setting of chronic abdominal pain requires a
ment of IBS-D. detailed medical and psychosocial history and physical
examination with limited laboratory testing.
KEY POINTS
• Narcotic bowel syndrome, also known as opiate-
• Irritable bowel syndrome is diagnosed based on clinical induced gastrointestinal hyperalgesia, is characterized
criteria and is no longer a diagnosis of exclusion. by the paradoxical increase in abdominal pain with
HVC • Many cases of irritable bowel syndrome can be effec- increasing doses of narcotics.
tively managed with reassurance, lifestyle modifica-
tions, and dietary modifications.
• Pharmacotherapy should target predominant symp-
toms and be used if conservative treatment is not
Diverticular Disease
effective. A diverticulum is the herniation of the mucosa and submucosa
through a weakness in the muscle wall. Diverticulosis (the
presence of diverticula) is the most common finding identified
during colonoscopy. Most patients with diverticulosis are
Management of Patients with asymptomatic, but 15% will develop complications. The causa-
Indeterminate Abdominal Pain tive mechanism is thought to be multifactorial, involving diet,
When abdominal pain is the primary symptom and is unre- rnicrobiota, genetics, and colonic motility. The incidence of
lated to food intake and defecation, centrally mediated abdom- diverticulosis continues to rise in the Western world and
inal pain syndrome (CAPS) should be considered. Abdominal increases with age, with a reported prevalence of 80% in per-
pain is described as constant, nearly constant, or frequently sons aged 85 years and older. Diverticulosis may also cause
recurring. The pain is not localized and may include extraintes- bleeding, more commonly in the right colon, which often
tinal symptoms such as musculoskeletal pain. It can be associ- resolves spontaneously.
ated with impairment in activities of daily living and psycho- Diverticulitis is the consequence of a diverticulum
social issues. CAPS is a result of central sensitization with becoming blocked, trapping bacteria, and subsequently devel-
dlslnhlbltlon of pain signals. Umited evaluation is needed In oping inflammation. Diverttculttis can be classified as uncom-
the setting of chronic pain meeting the diagnostic criteria for plicated or complicated. In some patients, a focal area of colitis
CAPS. Initial evaluation should include a detailed medical and can develop in a segment of diverticulosis, called segmental
psychosocial history, physical examination, and limited colitis associated With diverticulosis.

38
Disorders of the Small and Large Bowel

The clinical presentation of uncomplicated diverticulitis be obstructive or nonobstructive in nature. Obstructive causes
is colicky abdominal pain that is relieved with flatus or a bowel of AMT Include emboli from a cardiac source (such as atrial or
movement. On physical examination, left-lower-quadrant ventricular thrombus) or thrombosis related to underlying
abdominal tenderness is often present. Patients with compli- atherosclerotic disease. Nonobstructlve AMI is most com-
cated diverticulitis may present with dysuria, urinary fre- monly caused by vasoconstriction of the mesenteric vascula-
quency, pneumaturia, fecaluria, and recurrent urinary ture in the setting of severe sepsis or marked reduction in
infection concerning for colovesical fistula. It is uncommon for effective circulating volume (low-flow states). Mesenteric vein
patients with diverticulitis to present with rectal bleeding. thrombosis may cause AMI and often is associated with malig-
Acute diverticulitis is often a clinical diagnosis, usually nancy, hypercoagulable states, or intra-abdominal inflamma-
not requiring imaging. However, a CT scan will help to differ- tory conditions. If the reduction of blood flow is prolonged,
entiate uncomplicated from complicated diverticulltis. bowel infarction occurs, which is the most important prog-
The medical management of diverticulitis is based on the nostic factor for adverse outcome.
degree of the patient's symptoms and severity of diverticular AMI most commonly presents with abrupt onset of
disease. Uncomplicated diverticulitis is treated with oral anti- severe, periumbilical abdominal pain followed by the urge to
biotics (ciprofloxacin or metronidazole) and a liquid diet. defecate. The abdominal pain is constant and the patient
Intravenous antibiotics and hospitalization are required in may subsequently have loose, nonbloody stools.
patients unable to tolerate an oral diet; patients with severe Hematochezia occurs in a minority of cases of AMI (15%) and
comorbid!ties, advanced age, or immunosuppression: and its presence signifies concomitant right colon ischemia. In
patients for whom oral antibiotics have been ineffective. the early course of AMI. the abdominal examination may be
Treatment of complicated diverticulitis depends on the falsely reassuring: despite the patient's reporting severe
severity of illness and CT findings. If the CT scan shows a large abdominal pain, the physical examination reveals a soft,
(>5 cm) abscess, CT-guided drainage may be needed. Surgery nontender, nondistended abdomen without peritoneal signs.
is indicated for patients presenting with, or who develop, peri- This is called "pain out of proportion to physical examina-
tonitis or persistent sepsis. tion findings" and should immediately raise suspicion for
A high-fiber diet is recommended to prevent recurrence early AMI. With delayed or late presentation, intestinal
of diverticulitis. There is no evidence supporting restriction of ischemia progresses to infarction and peritoneal signs
foods such as nuts, berries, and seeds to prevent recurrent develop. This represents late AMI, which carries a high mor-
diverticulitis. tality rate.
KEY POINTS
Diagnosis
• Diverticulitis is the consequence of a diverticulum Laboratory studies and abdominal imaging findings in the
becoming blocked, trapping bacteria, and subsequently early course of AMI, before the development of intestinal
developing inflammation. infarction, are nonspecific. Most patients with AMI will
• The main presenting symptom of diverticulitis is col- have elevated leukocyte counts. A serum lactate concentra-
icky abdominal pain that is relieved with flatus or a tion may be normal, and this should not exclude the
bowel movement. diagnosis.
HVC • Acute diverticulitis is often a clinical diagnosis, usually CT angiography is the recommended method of imaging
not requiring abdominal imaging. for the diagnosis of AMI. CT angiography depicts the vessel
origins and length of vessels, and characterizes the occlusion.
While MR angiography avoids the risks of radiation and con-
lschemic Bowel Disease trast exposure, it takes longer and is less sensitive for distal and
nonocclusive disease. Duplex ultrasonography is an effective,
Ischemic bowel disease is a broad term describing a decrease
low-cost method to assess the proximal mesenteric vessels,
in blood flow to the small or large Intestine, which is insuffi-
but an adequate ultrasound examination often cannot be per-
cient to meet intestinal cellular metabolic function. It repre-
formed in patients with AMI. The classic angiographic finding
sents a spectrum of conditions that can be related to acute or
in obstructive AMI is either an embolus or thrombus in the
chronic alterations of arterial or venous intestinal blood flow.
superior mesenteric artery.
Ischemic bowel disease can be broadly subcategortzed into
three major clinical syndromes: acute mesenteric ischemia, Treatment
chronic mesenterlc ischemia, and colonic ischemia. After aggressive resuscitation and administration of broad-
spectrum antibiotics, angiography with selective catheteriza-
Acute Mesenteric lschemia tion of the mesenteric vessels is indicated. Findings signifying
Clinical Features mesenteric infarction, such as pneumatosis intestinalis (pres-
Acute mesenteric ischemia (AMI) is a rare condition caused by ence of gas within the wall of the intestine) or portal venous
an abrupt decrease in blood flow to the small intestine that can gas, require emergent surgery.

39
Disorders of the Small and Large Bowel

Chronic Mesenteric lschemia Diagnosis


Clinical Features Physical examination most commonly reveals mild lower
Chronic mesenteric ischemia (CMI) is an uncommon condi- abdominal tenderness over the involved bowel segment with-
tion that involves a gradual decrease in blood flow to the small out peritoneal signs. Laboratory studies may show a mild ele-
intestine over months to years. It most commonly results from vation In the leukocyte count and blood urea nitrogen level.
atherosclerotic narrowing of the mesenteric arteries. CMI Abdominal CT is indicated to assess the severity, phase,
occurs when two of the three adjacent mesenteric vessels and distribution of colonic ischernia. CT findings in colonic
(celiac, superior mesenteric, and inferior mesenteric} are ischemia are nonspecific and include bowel-wall thickening
severely narrowed. (Figure 21) and pericolonic fat stranding (increase in density
The classic symptom triad of CMI is postprandial abdom- within the pericolonic fat secondary to Inflammation).
inal pain, sitophobia (fear of eating), and weight loss. Infections that can mimic colonic lschemia, such as cytomeg-
However, this triad is seen Jn only 30% of patients with CMI, alovirus, C. difficile, and enterohemorrhagic E. coli, must be
and CMI should be suspected in the setting ofrecurrent post- excluded. Colonoscopy with biopsy is the test of choice to
prandial abdominal pain. The pain begins approximately 30 confirm the diagnosis of colonic ischemia.
minutes after food ingestion and results from "shunting of Because colonic ischemia is most commonly caused by a
blood" away from the small intestine to meet the increased nonocclusive low-flow state, dedicated imaging of the mesen-
functional demand of the stomach. Due to fixed narrowing of teric vasculature is of low yield and generally not indicated.
the mesenteric arteries, blood flow cannot increase suffi- The exception to this rule is right-sided colonic ischemia,
ciently to meet the intestinal metabolic demand and ischemJa which can be the harbinger of AMI caused by a focal thrombus
results. or embolus of the superior mesenteric artery. Therefore,
patients with right-sided colonic ischernia require noninvasive
imaging of the mesenteric vasculature to exclude occlusive
Diagnosis
process of the superior mesenteric artery.
The diagnosis of CMI requires symptoms (the CMI triad),
exclusion of alternative causes of postprandial abdominal pain
Treatment
and weight loss, and compatible radiographic findings.
Most cases of colonic ischernia are mild and transient with
Imaging modality should be chosen based on patient charac-
rapid spontaneous resolution. Patients with more severe
teristics and availability. CT or MR angiogram findings sugges-
tive of CMI Include severe stenosls (>70%) of two of the three
mesenteric arteries. If the patient has compatible symptoms
and suggestive angiographlc findings, and alternative explana-
tions for postprandial abdominal pain have been excluded,
then the diagnosis of CMI is secure.

Treatment
CMI is treated with percutaneous endovascular stenting or
surgical revascularization. Choice of therapy depends on oper-
ative risk and occlusive lesion characteristics. Periprocedural
morbidity and mortality are lower with endovascular stentlng;
however, it is less durable than surgical revascularization.

Colonic lschemia
Colonic ischemia is the most common form of ischemic bowel
disease. It most commonly results from a nonocclusive low-
flow state in rnicrovessels, which occurs in the setting ofhypo-
volemia or hypotension. Risk factors include age (older than
60 years), female sex, vasoconstrictive medications, constipa-
tion, thrombophilia, and COPD.

Clinical Features
Colonic ischemia presents with the abrupt onset of lower
abdominal discomfort that is mild to moderate and cramping,
and is followed within 24 hours by hematochezia. The onset
of bleeding often prompts the patient to seek medical FIGURE 21 • Colon isdtemia causing segmental thickening (arrow) of the
attention. transverse colon wall on CT.

40
Disorders of the Small and Large Bowel

disease require hospitalization for supportive care with bowel Other conditions to consider in the differential diagnosis are
rest, restoration of lntravascular volume, antimicrobial ther- perianal dermatitis, lichen sclerosis, condyloma, solitary rec-
apy in select cases, and close observation. Only a small per- tal ulcer syndrome, colorectal or anal polyps or cancer, and
centage of patients require operative intervention for necrotic TBD. Hemorrhoids are categorized as internal if proximal to
bowel or irreversible complications such as stricture. the dentate line, external if distal to the dentate line, or mixed
KEY POINTS if crossing the dentate line.
The evaluation of hemorrhoids should always include a
• Acute abdominal pain out of proportion to physical careful perianal and digital rectal examination. Symptomatic
examination findings should immediately raise suspi- hemorrhoids can be confused with rectal mucosal prolapse,
cion for early acute mesenteric ischemia. full thickness rectal prolapse, or a prolapsed rectal polyp. The
• CT angiography is the recommended imaging method presence of prolapsed concentric folds indicates rectal pro-
for the diagnosis of acute mesenteric ischemia. lapse as opposed to a prolapsed hemorrhoid or anal polyp
• The classic symptom triad of chronic mesenteric (Figure 22) . Further direct evaluation of the anorectum can be
ischemia is postprandial abdominal pain, sitophobia considered with anoscopy or flexible sigmoidoscopy.
(fear of eating), and weight loss. First-line therapy for hemorrhoids includes increased
fiber intake, adequate liquid intake, and avoidance of strain-
• Colonic ischemia is the most common form of ischemic
ing. Although various topical agents may reduce hemorrhoid
bowel disease, and most cases resolve spontaneously
and rapidly. symptoms, they are not necessary for curative therapy. Patients
with internal hemorrhoids unresponsive to medical therapy
should be considered for banding. Other options, including
Anorectal Disorders sclerotherapy and infrared coagulation, are less effective.

Perianal Disorders
Perianal disorders can range from relatively common and
benign disorders, such as hemorrhoids, to potentially disabling
conditions, such as anal fissure or fecal incontinence, to the
life-threatening condition of anal cancer. Perianal symptoms,
including bright red blood per rectum, anal pain, anal itching,
or a reported anal mass, should prompt a detailed evaluation
of the anus, anal canal, and rectum that includes visual inspec-
tion of the anus and digital rectal examination. Further evalu-
ation with anoscopy or proctoscopy may be required to estab-
lish the diagnosis. Colonoscopy is indicated when additional
alarm features are present; these include age older than SO
years, altered bowel habits, anemia, IBD, unexplained weight
loss, and/or family history of colorectal cancer.

Hemorrhoids
Hemorrhoids are submucosal, arteriovenous sinusoids that
are part of normal anorectal anatomy and are believed to play
an important role in anal canal function. Risk factors for
symptomatic hemorrhoids include ascites, pregnancy, exces-
sive sitting or squatting, systemic rheumatic disorders, low
dietary-fiber intake, obesity, and sedentary lifestyle.
Recurrent straining and persistent bowel alterations (either
constipation or diarrhea) can lead to engorgement of hemor-
rhoid plexuses, causing symptoms of bleeding, prolapse, and
swelling. Age-related changes can cause the hemorrhoid
beds to slide back and forth during defecation, resulting in
mucoid anal discharge, as well as perianal wetness, soiling, FIGURE 2 2. Hemorrhoids are vascular cushions that support the muscles of the
anal sphincter by swelling up as needed to maintain stool continence. Prolapsed
irritation, and/or pruritus. Pain is not a common symptom of
hemorrhoids {A) are internal hemorrhoids that protrude out of the rectum. The
uncomplicated hemorrhoids and should raise suspicion for hallmark of rectal prolapse (8) is the identification of concentric rings of the rectum
hemorrhoid complications, including thrombosis, lschemia, protruding through the anus. Prolapsed polyps (C, D) present as mucosa-covered
or incarceration from prolapse, or for an alternative diagnosis globular masses protruding from the anus.
such as anal fissure, perirectal infection, or perianal abscess. Images courtesy of Richard E. Burney, MO, Professo r of Surgery, Michigan Medicine at the Unive.,ity of Micnlgan.

41
Disorders of the Small and Large Bowel

Thrombosed external hemorrhoids are best treated with of at least 3 months' duration. Fecal incontinence does not
surgical excision within 72 hours of symptom onset. Surgical include the passage of clear mucus or flatus incontinence.
hemorrhoidectomy should be reserved for refractory hemor- Fecal incontinence is grossly underreported due to associated
rhoids, large external hemorrhoids, or combined internal and embarrassment, and health care providers should inquire
external hemorrhoids with rectal prolapse. about it during clinic visits, particularly in patients at risk.
Fecal incontinence is more common in multiparous women
Anal Fissure and in patients with advanced age, obesity, diarrhea or urinary
Anal fissures are longitudinal mucosa] tears in the anal canal incontinence, or histmy of obstetric complications, anorectal
characterized by anorectal pain worsened by bowel move- surgery or anorectal disease. Various factors can contribute to
ments and sitting (Figure 23). Rectal bleeding with bowel fecal incontinence, including loose stool, impairment of rectal
movements or wiping is frequently reported. Anal fissures are storage capacity, altered rectal sensation, and reduced anal
either idiopathic or the result of trauma due to the passage of sphincter tone. Constipation with resultant fecal loading and
hard stool, receptive anal intercourse, or the insertion of a overflow diarrhea can also cause fecal incontinence.
foreign body such as an enema or endoscope. Anal fissures A digital rectal examination is essential in the evaluation.
occurring laterally should raise concern for other entities This can identify reduced anal sphincter tone, prolapse of hem-
including Crohn disease, ulcerative colitis, tuberculosis, syphi- orrhoids or rectum, or rectal stool impaction. Additional diag-
lis, HIV, psoriasis, or anal cancer. Anal fissures lasting more nostic testing is determined by the history and rectal
than 8 to 12 weeks are considered chronic and may be charac- examination. If fecal loading is suspected, an abdominal radio-
terized by edema and fibrosis as well as the presence of a graph can be diagnostic. If diarrhea is present, stool testing and/
sentinel pile (appearing as a skin tag) or hypertrophied anal or colonoscopy should be considered. If weak sphincter tone is
papilla within the anal canal. appreciated or rectal urgency is reported, evaluation may include
Acute anal fissures generally resolve within a few weeks anorectal manometry to assess anal sphincter pressure and rec-
with the use of sitz baths, psyllium, and bulking agents. tal sensation, anal endosonography for anal sphincter defects, or
Topical anesthetics and anti-inflammatory agents can be con- defecography for structural evaluation of the pelvic floor.
sidered to address pain or bleeding, but their use is not Initial treatment should be directed at the treatment of
required to promote fissure healing. Rectal spasm is the major reversible factors such as diarrhea, constipation, offending
reason for anal fissures becoming chronic and should be medications, smoking, obesity, and physical inactivity.
treated with either topical calcium channel blockers or Effective dietary modifications include fiber supplementation
nitrates. Patients with chronic fissures unresponsive to these and avoidance of trigger foods. lf there is lack of response to
measures should be referred for internal sphincter botulinum conservative measures, the next step is pelvic floor muscle
toxin injection or surgical sphincterotomy. training with specially trained physical therapists. In the event
of lack of response to biofeedback therapy, other treatment
Fecal Incontinence options include the use of anal plugs, anal wicking, various
Fecal incontinence, also called accidental bowel leakage, is minimally invasive procedures (for example, the use of inject-
defined as the recurrent, uncontrolled passage offecal material able bulking agents or sacral nerve stimulation), or surgery
such as sphincteroplasty.

Anal Cancer
Anal cancer (Figure 24) is rare, with a yearly incidence of 8000
cases in the United States. Squamous cell cancer is the most
common type, representing BO'ro of all anal cancers. The inci-
dence has been rising by 2.2% each year for the last decade.
Delayed diagnosis by up to 2 years in more than half of cases
has resulted in increased need for aggressive surgical
intervention.
Human papillomavirus infection and presence of intraep-
ithelial neoplasia are strongly linked with anal cancer. Risk is
increased in men who have sex with men, in patients with HIV
infection, history of condylomata, and kidney or liver trans-
plantation. Presenting symptoms of anal cancer may include
bleeding, pain, or pruritus; however, 25% of cases present
without any anorectal symptoms. Anal squamous cell cancer
occurs in patients with IBD or ulcerative colitis at a rate ofl in
FIGURE 23. Analfissures are painful longitudinal mucosal tears in the anal canal. 100,000, and at a rate of 2 in 100,000 in patients with Crohn
Image courtesy of Richard l Burney, MD, Professor of Surgery, Michigan Medicine at the University of Michigart disease, largely in those with perianal disease, warranting

42
Colorectal Neoplasia

Colorectal Neoplasia
Epidemiology
Colorectal cancer is the third most common cancer and sec-
ond most common cause of cancer-related mortality among
men and women in the United States. Its annual incidence
rate among adults in the United States is 42 cases per 100,000,
and its mortality rate is 15.5 deaths per 100,000. The inci-
dence of colorectal cancer is approximately 25% higher in
men than in women.
The incidence and mortality rates for colorectal cancer
have decreased over the past 30 years, largely because of early
detection through screening and changes in risk factors (such
as decreased smoking). However, there has been a steady
FIGURE 24. Anal cancer presenting as an ulcerated mass protruding from the
increase in the number of colorectal cancer cases in patients
anal canal. The presentation of anal cancer can vary from an obstructing anal mass
to a non protruding polyp or flat growth in the anal canal. younger than age SO years, for unknown reasons. Survival
Image courtesy of Riclrard E. Sumey, MD, Professor of Surgery, Michigan Medicine at the University of Mlclrigan.
rates are greater than 90% when the disease is localized,
underscoring the Importance of early detection. Between 20%
and 53% of adults older than age 50 years have premalignant
adenomas of the colon detected on screening, and 3% to 8% of
heightened vigilance in patients with longstanding perianal
the adenomas have advanced histological features. Despite
disease. Cancer occurs at a young age in patients with Crohn
strong evidence supporting colorectal cancer screening, only
disease (mean age of 42 years) with a typical presentation of
about 59% of Americans older than age SO years are up-to-date
anal pain. Anal cancer is aggressive in patients with Crohn
for recommended testing.
disease; most patients require radical surgery, and cumulative
overall and disease-free survival is 37% at 5 years. KEY POINT
Although no formal recommendations address screening • Colorectal cancer is the third most common cancer and
for anal dysplasia in at-risk populations, screening modalities second most common cause of cancer-related mortality
to consider include digital rectal examination, anal Pap test, among men and women in the United States.
human papillomavirus testing, and high-resolution anoscopy.
Biopsy is required to make the diagnosis of anal cancer.
Unlike colorectal cancer, staging of anal cancer is based on
tumor size and does not involve depth of invasion. Only anal Pathogenesis
cancers involving the vagina, urethra, or bladder are classified Colorectal cancer develops through one of three mechanisms:
as T4 lesions, regardless of size. Invasion of the rectum, skin, chromosomal instability, microsatellite instability. and ser-
or sphincter muscles does not classify lesions as T4. Lymph rated neoplasia.
node metastasis is divided into perirectal, iliac, and/or ingui- Chromosomal instability is the most common mecha-
nal lymph nodal involvement. nism, accounting for approximately 85% of colorectal cancers.
For treatment and follow-up of anal cancer, see MKSAP 18 Chromosomal instability is characterized by cancer cells that
Hematology and Oncology. gain or lose whole chromosomes or large fractions of chromo-
somes (aneuploidy) at an increased rate compared with nor-
KEY POINTS
mal cells. Stepwise accumulation of mutations leads to
• First-line therapy for hemorrhoids includes increased progression from normal colon to adenoma to cancer. The
fiber intake, adequate liquid intake, and avoidance of most commonly mutated gene in this pathway is the adeno-
straining. matous polyposis coli (APC) gene, a multifunctional tumor
• Fecal incontinence is grossly underreported due to suppressor gene. Deregulation caused by mutations in APC is
embarrassment, and health care providers should implicated in the development and spread of colon cancer.
inquire about it during clinic visits, particularly in Germline mutations in the APC gene lead to hereditary famil-
patients at risk. ial adenomatous polyposis.
• Presenting symptoms of anal cancer may include bleed- Microsatellites are dozens to hundreds of repetitive nucle-
ing, pain, or pruritus; however, 25% of cases present otide sequences seen throughout the human genome. The
without any anorectal symptoms. term microsatellite instability refers to the presence of mis-
matched bases at repeated DNA micmsatellites. Microsatellite
• Patients with Crohn disease with longstanding perianal
instability accounts for about 15% of colorectal cancers and is
disease are particularly at risk for anal cancer.
characterized by defective DNA mismatch repair, leading to

43
Colorectal Neoplasia

multiple mutations, adenomas, and cancer. Defective mis- severity and extent of bowel involvement. Patients with uret-
match repair can occur if there is a germline mutation in one erocolic anastomoses after extensive bladder surgeries and
of the mismatch repair genes in Lynch syndrome (including adult survivors of childhood malignancy who received abdom-
MLHl, MSH2, MSH6, PMS2) or in the epithelial cell adhesion inal radiation are also at increased risk for colorectal cancer.
molecule gene (EPCAM), or as a result of sporadic methylation Modifiable risk factors associated with increased risk
of the MLHl promoter. include diets high in red and processed meat; low intake of
A more recently described mechanism involves hyper- fruits, vegetables, fiber, and dairy; use of alcohol and tobacco;
methylation of tumor suppressor genes and the development type 2 diabetes mellitus; sedentary lifestyle; and obesity.
of serrated polyps and cancer. Serrated polyps are a heteroge-
KEY POINTS
neous group of lesions classified by their saw-toothed appear-
ance on histology. Serrated polyps are often found in the • Nonmodifiable risk factors for colorectal cancer include
proximal colon and may be flat and difficult to distinguish age (SO years and older), male sex, black race, and a
from normal mucosa (Figure 25) . Hyperplastic polyps that are personal or family history of colorectal cancer.
benign also develop through this mechanism. Interval colon • Modifiable risk factors for colorectal cancer include diets
cancers, which are defined as cancers that develop after a high in red and processed meat; low intake of fruits.
negative colonoscopy and before the next recommended vegetables, fiber, and dairy; use of alcohol and tobacco;
screening colonoscopy, most commonly develop through the type 2 diabetes mellitus; sedentary lifestyle; and obesity.
serrated neoplasia mechanism. A hereditary condition called • Longstanding inflammatory bowel disease (both ulcera-
serrated polyposis syndrome has been described, but its tive colitis and Crohn disease) is associated with
genetic basis has not been elucidated. increased risk for colorectal cancer.
KEY POINT
• Chromosomal instability is the most common mecha- Chemoprevention
nism responsible for the development of colorectal can-
Substantial epidemiological and experimental data show that
cer, accounting for 85% of cases.
aspirin use prevents colorectal cancer. The U.S. Preventive
Services Task Force updated their guidelines in 2015 to include
Risk Factors the use oflow-dose aspirin (81 mg/d) for preventing colorectal
Several risk factors have been established for colorectal cancer. cancer and cardiovascular disease in individuals aged 50 and
Nonmodifiable risk factors include age (50 years and older), 59 years who are at increased risk for cardiovascular disease.
· male sex, black race, and a personal or family history of colon Aspirin use is associated with a 30% decreased risk for colo-
adenomas or colorectal cancer. The risk doubles with a family rectal cancer based on cohort and case-control studies.
history of colorectal cancer in a first-degree relative. However, randomized controlled trials have not shown a ben-
Long-standing (;?:8 years) colonic inflammatory bowel efit of aspirin in prevention of colorectal cancer. Randomized
disease (both Crohn disease and ulcerative colitis) is associated controlled trials have shown that aspirin decreases the risk for
with about a 2.7-fold increased risk and is dependent on the recurrent adenomas.
Studies of other NSAIDs for prevention of colon cancer
and adenomas have also yielded positive results; however,
benefits of aspirin and other NSA1Ds must be weighed against
harms, most notably gastrointestinal bleeding.
Some evidence suggests that selective cyclooxygenase-2
inhibitors prevent recurrent adenomas and decrease the inci-
dence of advanced lesions; however, the associated increased
cardiovascular risk confounds a recommendation of their use
for chemoprevention.
KEY POINT
• The U.S. Preventive Services Task Force recommends the
use of low-dose aspirin for preventing colorectal cancer
and cardiovascular disease in individuals aged 50 and 59
years who are at increased risk for cardiovascular disease.

Screening
FIGURE 2 5. Aserrated polyp in the ascending colon, showing flat morphology Screening individuals at average risk is discussed in MKSAP 18
that can make these polyps difficult to identify on colonoscopy. General Internal Medicine.

44
Colorectal Neoplasia

Screening strategies for individuals at increased risk for bowel habits, abdominal pain, colonic obstruction, and weight
colorectal cancer are described in Table 23. loss in more advanced cases.
Key quality indicators for screening colonoscopy are ade-
quate bowel preparation, preparation sufficient to identify
polyps 6 mm in siz.e or larger, visualization of the entire colon Diagnosis and Staging
to the cecum, and longer duration of colonoscopy. The diagnosis of colorectal cancer is usually made by colonos-
Because adenomas are an intermediary step in the pro- copy with biopsy. Staging is based on tumor size and extent of
gression from normal colonic mucosa to colon cancer, the invasion into local tissues, lymph node involvement, and
identification and management of polyps is critical. Adenomas evidence of metastasis (TNM system). Cross-sectional imag-
can be found throughout the colon. They are classified based ing of the chest, abdomen, and pelvis is used for staging in the
on morphology, histology, and degree of dysplasia (Table 24). initial evaluation after diagnosis . Management of colon can-
Adenomas with any degree of villous histology or high-grade cer depends on stage, evidence of microsatellite instability,
dysplasia have greater malignant potential. Risk is also sub- and presence of specific mutations, including KRASINRAS
stantially increased in polyps larger than 1 cm in size. The andBRAF.
progression of adenoma to carcinoma takes approximately 8 to For treatment of colorectal cancer, see MKSAP 18
10 years. Hematology and Oncology.
Favorable characteristics include stage 1 to 2, lack of angi-
K EY POINTS
olymphatic involvement, and negative margins after polyp
• Screening for colon cancer should be individualized in removal. Endoscopic resection of polyps is curative in some
patients at increased risk. cases. Malignant sessile or flat polyps are associated with
• The risk for colon cancer depends on the size of the higher risk for recurrence and local or distant spread.
adenoma, degree ofvillous histology, and presence of
KEY POINTS
dysplasia.
• The diagnosis of colorectal cancer is usually made by
colonoscopy with biopsy.

Clinical Presentation • Staging of colon cancer is based on tumor size and


extent of invasion into local tissues, lymph node
Colorectal cancer may be asymptomatic, or it may present
involvement. and evidence of metastasis.
with iron deficiency anemia, gastrointestinal bleeding, altered

TABLE 23. Screening for Colorectal Cancer in Individuals at Increased Risk


, Risk Category Crit eria Screening Recommendations (age; modality; interval)

Increased Family history of CRC:


CRC diagnosed in FDR <60 years old or Begin at age 40 years or 10 years earlier than age of youngest
two or more FDRs at any age FDR at diagnosis, whichever comes first; colonoscopy; repeat
every 5 years•
CRC diagnosed in FDR >60 years old Begin at age 50 years; any modality; repeat every 10 years•
Personal history of CRC Perform at time of diagnosis; colonoscopy; repeat at 1 year, 3
years, and, if normal, every 5 years thereafter until the benefit of
continued screening is outweighed by risks
High Familial adenomatous polyposis Begin at age 10-12 years; flexible sigmoidoscopy or
colonoscopy; repeat every 1-2 years until colectomy
Lynch syndrome Begin at age 20-25 years or 10 years earlier than youngest
cancer in family; colonoscopy; repeat every 1-2 years
Inflammatory bo.wel disease (Crohn disease Begin after 8 years of chronic colitis; colonoscopy with
or ulcerative colitis) biopsies; repeat every 1-2 years

CRC - colorectal cancer; FDR • first-degree relative (parent, sibling. or child).

•1f baseline examination is normal .


Data from Rex DK. Johnson DA. Anderson JC, Schoenfeld PS, Burke CA. lnadomi JM; American College of Gastroenterology. American College of Gastroenterology guidelines
for colorectal cancer screening 2009 [corrected]. Am J Gastroenterol. 2009;104:739·50. [PMID: 19240699] doi:l 0.1038/ajg.2009.104; Rubenstein JH, Enns R, Heidelbaugh J,
Bari<un A; Clinical Guidelines Committee. American Gastroenterological Association Institute Guideline on the Diagnosis and Management of Lynch Syndrome. Gastroenterology.
2015;149:777-82; quiz e16-7. [PMID: 26226577] doi:10.10S3/j.gastro.2015.07.036; Syngal S, Brand RE, Church JM, Giardiello FM, Hampel HL. Burt RW; American College of
Ga.troenterology. ACG clinical guideline: Genetic testing and management of hereditary gastrointestinal cancer syndromes. Am J Gastroenterol. 201S;110:223-62; quiz 263.
[PMID: 2S64S574] doi:l0.1038/ajg.2014.435.

45
Colorectal Neo pla sia

TABLE 24. Classification of Colorectal Polyps hepatobiliary and urinary tracts, small bowel, brain, and pan-
Adenomatous Polyps• creas, and sebaceous skin adenoma or cancer.
In persons with Lynch syndrome, the lifetime risk for
Tubular adenoma colorectal cancer depends on the location of the gene muta-
I Tubulovillous adenoma tion, and the risk can be as high as 50% to 80%. Colorectal
Villous adenoma cancer in Lynch syndrome is more likely to occur in the proxi-
mal colon and can display characteristic pathological features,
Serrated Polyps
such as tumor-infiltrating lymphocytes and medullary growth
I
Hyperplastic polyp pattern. Colorectal tumors in Lynch syndrome result from
I Sessile serrated polyp with or without cytologic dysplasia microsatellite instability that can be assessed using polymerase
I
t Traditional serrated adenoma
chain reaction or immunohistochemistry (see Pathogenesis).
I All colorectal cancers should be screened for Lynch-syndrome
I Other genetic mutations or microsatellite instability.
I
i
Hamartomatous polyp Clinical criteria to evaluate for Lynch syndrome include the
Amsterdam and Bethesda criteria, as well as newer models such
I
I
Inflammatory polyp
as the PRF.diction Model for gene Mutations 5 (PREMM5) . The
~ith or without high-g rade dysplosia. _ _J Amsterdam II criteria follow the "3-2-1-1-0 rule," which states
that a diagnosis is warranted ifall of the following criteria are met:

Surveillance • 1hree family members are affected with a Lynch syn-


drome- associated cancer
Surveillance for colorectal cancer after screening or polypectomy
is based on findings on the baseline examination (Table 25). • Two successive generations are affected
• One affected family member is a first-degree relative of
Hereditary Colorectal Cancer the other two affected family members
Syndromes • One of the cancers was diagnosed before age SO years
• A familial polyposis syndrome has been ruled out
Lynch Syndrome
Lynch syndrome is characterized by germline mutations in the • Tumors have been verified histologically
mismatch repair genes (MLHl, MSH2, MSH6, PMS2) or the These criteria are specific but not sensitive for the diagno-
epithelial cell adhesion molecule gene (EPCAM), leading to an sis of Lynch syndrome. The Bethesda criteria have good sensi-
increased risk for neoplasia of the colon and other organs. tivity but poor specificity. Genetic counseling and testing
Another name for this syndrome, hereditary nonpolyposis should be offered to patients with a personal and/or family
colorectal cancer, is no longer used because the syndrome is history consistent with Lynch syndrome, as well as patients
associated with colorectal polyps as well as extracolonic can- whose tumor shows evidence of microsatellite instablllty or
cers. The syndrome follows an autosomal dominant inherit- loss of mismatch repair protein expression. When a mutation
ance pattern and new mutations are rare. is identified in a family, testing of first-degree relatives should
Individuals with Lynch syndrome are at increased risk for be performed and surveillance instituted.
colorectal and endometrial cancers (most common), as well as Screening for colorectal cancer in patients with Lynch
other cancers. including tumors of the stomach, ovary, syndrome should begin at age 20 to 25 years (or 2-5 years

TABLE 25. Surveillance for Colorectal Cancer After Screening or Polypectomy


Adenomatous Polyps Interval to Next Colonoscopy
1-2 tubular adenomas <10 mm in size 5-10 years
3-10 adenomas ~10 mm in size, villous histology, o r high grade dysplasia 3 years
~10 adenomas on single examination <3 years; a genetic cause of disease should be
investigated

Serrated Polyps Interval to Next Colonoscopy


Rectosigmoid hyperplastic polyps <10 mm in size 10 years
SSP <10 mm in size 5 years
SSP ~ 10 mm in size or SSP with dysplas1a or TSA 3 years
Serrated polyposis syndrome 1 year

SSP • sessile serrated polyps; TSA - traditional serrated adenomas.


~------· ----------------·--------------------------------·-'

46
Colorectal Neoplasia

before the age of diagnosis of the earliest cancer in the family). require surveillance because the lifetime risk for duodenal
Screening should be done by colonoscopy and repeated every adenocarcinoma is estimated at 4%. Fundic gland polyps of the
1 to 2 years. Risk-reducing total hysterectomy and bilateral stomach do not have malignant potential but can mask gastric
salpingo-oophorectomy should be considered starting at age adenomas and cancer. Risk for gastric cancer is estimated at
40 to 45 years after childbearing is completed in women who less than 1%. Risk for desmoid tumors is increased, especially
carry Lynch syndrome mutations. after surgery to remove the colon. Risk for papillary thyroid
Screening with upper endoscopy for stomach and small- cancer is also increased, especially in women. Nonmalignant
bowel cancers can be considered starting at age 30 to 35 years findings include extra teeth, cysts, osteomas, and congenital
and repeated every 2 to 5 years. Testing for Helicobacter pylori hypertrophy of the retinal pigmented epithelium.
is also recommended in patients at risk for or with Lynch Screening in classic FAP mutation carriers should begin at
syndrome. age 10 to 12 years with sigmoidoscopy or colonoscopy and
repeated every 1 to 2 years. Screening in patients with AFAP
Adenomatous Polyposis Syndromes can be delayed until age 20 to 25 years and should be per-
Syndromes that predispose to multiple adenomatous polyps in formed with colonoscopy. For individuals with FAP and AFAP,
the colon include familial adenomatous polyposis (FAP), screening with upper endoscopy should begin at age 25 to 30
MutYH-associated polyposis (MAP), and polymerase proof- years and include visualization of the papilla with a duodeno-
reading-associated polyposis (PPAP) . scope. Fundic gland polyps in the stomach should be ran-
domly sampled. Surveillance of the upper gastrointestinal
Fami1ial Adenomatous Polyposis tract is determined by findings in the duodenum. Annual
FAP is an inherited disorder characterized by multiple (usu- thyroid ultrasound is also recommended.
ally more than 100) adenomatous colon polyps. FAP is caused Colectomy is the treatment of choice for classic FAP and
by germline mutations in the APC gene that are inherited in may be pursued in patients with AFAP. Absolute indications
an autosomal dominant pattern, although de novo mutations for surgery include cancer and significant symptoms such as
occur in about 25% of cases. Adenomas in classic FAP are rectal bleeding. Relative indications include multiple adeno-
more numerous in the distal colon than in the proximal mas greater than 6 mm size, increase in number of polyps, an
colon. Adenoma and cancer develop in 100% of classic FAP adenoma with high-grade dysplasia, and multiple diminutive
cases if surgery is not performed. The average age of colorec- polyps, which can prevent identification of adenomas. The
tal cancer onset is 39 years, with a riskof93% by age 50 years. type of surgery depends on the polyp burden in the rectum;
An attenuated form ofFAP (AFAP) causes fewer polyps (<100 total colectomy with ileorectal anastomosis is indicated for
synchronous polyps) with more proximal colonic distribu- patients with a small rectal polyp burden, and total procto-
tion. The risk for colon cancer in patients with AFAP is about colectomy with ilea-pouch anastomosis is indicated for
70% by age 80 years, with an average age of onset of 58 years. patients with a large rectal polyp burden. Postsurgical surveil-
Both FAP and AFAP are associated with extracolonic lance is yearly sigmoidoscopy for those with an intact rectum
intestinal manifestations (Figure 26). Duodenal adenomas and ileoscopy every 2 years for patients with an ileostomy.

Fl GU RE 26. Intestinal features of familial adenomatous polyposis (FAP). Patients with FAP can develop polyps throughout the gastrointestinal tract. Adenomatous polyps
always develop in the colon. Duodenal adenomas can also develop in patients with FAP, especially involving the ampulla ofVater. Regular upper endoscopic surveillance is
indicated to remove polyps larger than 10 mm (left). If a patient with FAP has had surgery and has an ileorectal anastomosis, that patient must continue to be surveyed
regularly because rectal polyps can develop in the remaining rectum (center). Numerous fundic gland polyps of the stomach can develop in FAP (right).

47
Colorectal Neoplasia

MutYH-Associated Polyposis
MAP is an inherited syndrome characterized by fewer adeno-
mas than classic FAP. MAP is caused by mutations in the
MutYH gene, a component of base excision repair. MAP is a
recessive condition in which an affected patient has inherited
two mutated copies from their parents. Most patients have
between 20 and 99 adenomatous colon polyps. Some indi-
viduals can develop cancer with only a few or no synchronous
adenomas. Mean age of cancer onset is about 52 years. In
addition, other polyp types can be encountered in patients
with MAP. including serrated and hyperplastic polyps.
Duodenal cancer risk in MAP is estimated at 4% and surveil-
lance with upper endoscopy is similar to that indicated in
patients with FAP. Surgical management of the colon is simi-
larto that ofFAP.

Polymerase Proofreading-Associated Polyposis


This more recently described syndrome is inherited in an FIGURE 2 7. Juvenile polyps in the colon of a patient with juvenile polyposis
autosomal dominant manner. PPAP appears to have features of syndrome, on endoscopy.Juvenile polyps can have an erythematous and waxy
appearance.
both FAP and Lynch syndrome and is caused by mutations in
polymerase proofreading genes POLE and POLDl. Endometrial
cancer has been described in this syndrome, and tumor testing Small bowel harnartomas develop at a young age in
has been reported to show microsatellite instability. patients with PJS, and may present with intussusception,
Management guidelines have not been established for PPAP. obstruction and bleeding. Patients with PJS have increased
but management of colonic polyps follows similar principles risk for cancer in the colon, stomach, small bowel, breast,
as for other adenomatous polyp syndromes. ovary, cervix, uterus, testicles, lung, and pancreas.

Hamartomatous Polyposis Syndromes PTENHamartoma Syndrome


Hamartomas refer to polyps caused by overgrowth of normal Also known as Cowden syndrome, this syndrome is caused by
tissue. There are three primary syndromes associated with germline mutations in the tumor suppressor gene PTEN. It is
hamartomas in the gastrointestinal tract: juvenile polyposis characterized by gastrointestinal hamartomas associated with
syndrome (JPS) , Peutz-Jeghers syndrome (PJS), and PTEN a number of cancers, such as breast, thyroid, kidney, and colon
hamartoma syndrome, also known as Cowden syndrome. cancer, as well as macrocephaly, esophageal glycogenic acan-
thosis, and dermatological manifestations. Colorectal cancer
Juvenile Polyposis Syndrome prevalence is estimated at 9% to 18%.
JPS is caused by mutations in the BMPRlA and SMAD4 genes.
It has an incidence of approximately 1in130,000 live births. Serrated Polyposis Syndrome
Multiple juvenile polyps (>5 polyps) are found in the colon Serrated polyposis syndrome is characterized by numerous
(98%), stomach (14%), and small bowel (14%) of patients with serrated polyps in the colon. The definition is based on having
JPS (Figure 27). Sporadic juvenile polyps can be found in up any of the following:
to 1% of children and are not considered syndromic. The • Five or more serrated polyps proximal to the sigmoid co-
average age of patients at the time JPS is diagnosed is 18.S lon, with two or more greater than 10 mm in size
years, and rectal bleeding is the most common symptom.
• Any number of serrated polyps in patients with first-de-
Patients with JPS are at increased risk for colon, stomach,
gree relatives with serrated polyposis syndrome
and small-bowel cancer. Individuals with mutations in the
SMAD4 gene can also have hereditary hemorrhagic • More than 20 serrated polyps throughout the colon
telangiectasia. Estimates suggest that the prevalence of serrated polypo-
sis syndrome is between 0 .3'7'. and 0.6%. Risk for colon cancer
Peutz-Jeghers Syndrome in patients with this syndrome is increased, and smoking
PJS is caused by mutations in the SfKll gene. Its incidence is appears to be a risk factor. Colonoscopy is recommended every
approximately 1 in 200,000 live births. PJS hamartomas are 1 to 3 years with removal of all polyps greater than 5 mm in
found primarily in the small bowel but can also develop in the slze. Surgery may be considered if the polyps cannot be man-
stomach and colon (Figure 28). PJS is also characterized by aged endoscopically. No extracolonic manifestations have been
hyperplgmented mucocutaneous macules on the lips and buc- noted in serrated polyposis syndrome. The genetic basis of the
cal mucosa (Figure 29). syndrome is unknown.

48
Disorders of the Liver
- ~ -- - ---- - - - - - -

FIGURE 2 8. Peutz-Jeghers syndrome (PJS) polyps. Capsule endoscopy showing a PJS hamartomatous polyp in the small intestine (left); polyps in the small intestine can
cause bleeding and/or obstructions. Colonoscopy showing a hamartomatous polyp in the colon of a patient with PJS (right).

Disorders of the Liver


Approach to the Patient with
Abnormal Liver Chemistry Studies
Basic metabolic panels commonly include liver chemistry
tests. Uver chemistry tests are often abnormal (10% to 20% of
the time); therefore, it is important to take a systematic
approach to their evaluation. The patterns of the elevations of
liver tests can be used to group causes into categories; how-
ever, these patterns are not specific. Elevations of aspartate
aminotransferase (AST) and alanine aminotransferase (ALT)
levels represent hepatic parenchymal inflammation. Elevated
ALT levels are more specific for hepatic inflammation because
FIGURE 2 9. Peutz-Jeghers syndrome is associated with multiple AST is also found in other tissues such as heart and muscle.
hamartomatous polyps in the gastrointestinal tract and distinctive mucocutaneous Elevations in alkaline phosphatase (ALP) and bilirubin
pigmentations. The pigmented lesions occur most commonly on the lips and levels result from inflammation of the biliary tree or bile flow
perioral region but can also occur on the nose, perianal area, and genitals.
abnormalities. ALP is also produced in bone and placenta and
high levels are seen in pregnancy. Elevation of ALP and other
liver chemistries typically reflects liver injury. Fractionation of
KEY POINTS
ALP levels can be performed to determine the source of eleva-
• Lynch syndrome is caused by germline mutations in the tion. Bilirubin can be divided into conjugated and unconju-
mismatch repair genes and carries a lifetime risk for gated forms. Elevation of conjugated bilirubln reflects a liver
colorectal cancer of 50% to 80%. disorder, whereas elevation of unconjugated bilirubin is seen
• Syndromes that predispose persons to multiple adeno- in hematological disease or in benign alterations of bilirubin
matous polyps in the colon include familial adenoma- conjugation, such as Gilbert syndrome.
tous polyposis, MutYH-associated polyposis, and poly- The prothrombin time and serum albumin levels reflect
merase proofreading-associated polyposis. the synthetic function of the liver. Albumin levels are decreased
• Hamartomatous polyposis syndromes are associated In the setting of malnourishment, the nephrotlc syndrome,
with increased risk for multiple cancers. including acute inflammation, and protein-losing enteropathies.
Prothrombin time can be prolonged in vitamin K deficiency,
colon cancer.
warfarln therapy, coagu]opathy of liver disease, inherited or

49
Disorders of the Liver

acquired factor deficiency, and the antiphospholipid antibody occur with hepatic congestion, infiltrative disorders, and bile
syndrome. In patients with abnormal liver enzyme levels, duct obstruction.
thrombocytopenia may be a clue to the presence of portal
KEY POINTS
hypertension.
The relative levels and severity of elevations of AST, ALT, • Elevations of aspartate aminotransferase and alanine
and ALP provide clues about the cause of the liver inflamma- aminotransferase levels represent hepatic parenchymal
tion (Table 26). The duration of abnormal liver tests is also inflammation; alanine aminotransferase levels are more
important in evaluating the causes of liver injury. Acute liver specific for hepatic inflammation.
inflammation is defined as less than 6 months in duration, • Elevations in alkaline phosphatase and bilirubin levels
whereas chronic hepatitis is defined as elevated liver chemis- result from inflammation of the biliary tree or bile flow
tries of greater than 6 months' duration. abnormalities.
Abdominal imaging can be helpful in the assessment of a • The prothrombin time and serum albumin levels reflect
patient with abnormal liver tests. Abdominal ultrasonography the synthetic function of the liver.
may show increased echogenicity, consistent with fatty infil-
tration of the liver, as well as nodularity of the liver seen in the
setting of cirrhosis. Cross-sectional imaging may show find- Viral Hepatitis
ings of fatty infiltration of the liver as well as changes of cir-
rhosis. Noninvasive assessments of hepatic fibrosis are Hepatitis A
becoming more frequently used. Ultrasound-based transient Hepatitis A virus (HAV) is an RNA virus that causes acute
elastography and magnetic resonance (MR) elastography hepatitis mediated by the host immune response. HAV infec-
measure tissue stiffness. Tissue stiffness correlates with stages tion is a self-limited illness, but atypical forms exist, including
of hepatic fibrosis, although high stiffness values may also a relapsing, remitting infection with cholestatic features and,

Disease AST ALT

Acute viral hepatitis iii iii Normal to i Normal to iii Exposure history, fatigue,
nausea
Chronic viral hepatitis i It Normal to i iifadvanced History of exposure to infected
blood orbodyfluids
Nonalcoholic steatohepatitis i i Normal to i Normal Metabolic syndrome
Alcoholic hepatitis ii Normal ori i Normal to iii Excess alcohol intake
Acute autoimmune hepatitis iii iii Normal to i Normal to ii Autoantibodies
Chronic autoimmune hepatitis i ii Normal to i Normal Autoantibodies
Wilson disease i i Low i and often Hemolysis if acute, neurologic
unconjugated symptoms if chronic
cx1-Antitrypsin deficiency i i Normal i if advanced May have pulmonary disease
Hemochromatosis Normal Normal Normal Normal Joint symptoms, family history,
other organ involvement
Primary biliary cholangitis i i iii i if advanced Female, sicca symptoms,
antimitochondrial antibody
Primary sclerosing cholangitis i i iii i if advanced or Ulcerative colitis, abnormal
dominant stricture cholangiogram
is present
Large duct obstruction i{ii if i(iiif ii it Pain if acute, dilated ducts on
acute) acute) imaging
Infiltrative liver disease i i iii Normal Features of malignancy, sarcoid,
amyloid, or mycobacterial or
fungal infection
Hepatic ischemia iii iii Normal Normal AST >5000 U/L, history of
hypotension
Celiac disease Normal ori Normal ori Normal or i Normal Usually other features of celiac
disease
ALP• alkaline phosphatase; ALT • alanine aminotransferase; AST- aspartate aminotransferase.

so
Disorders of the Liver

rarely, acute liver failure. HAV is transmitted by the fecal-oral as a result of acute infection with aminotransferase levels as
route. Risk factors include international travel, contacts with high as 3000 U/L and nonspecific symptoms including
household members with HAV infection, men who have sex malaise, nausea, and right-upper-quadrant pain. Acute liver
with men, and exposure to day care or institutionalized set- failure (ALF) develops in approximately 0.5% of patients.
tings. The incidence of HAV infection declined dramatically Typically, adult patients recover within 1 to 4 months. Chronic
after introduction of the HAY vaccination (see MKSAP 18 HBV infection is diagnosed after 6 months in patients with
Infectious Diseases). Mortality is rare but may be increased in persistent hepatitis B surface antigen (HBsAg) detected in
patients with preexisting chronic liver disease. The incubation serum.
period for HAV is 15 to SO days. A prodrome of malaise, nausea, Interpretation of HBV serologies is shown in Table 28.
vomiting, fever, and right-upper-quadrant pain is followed by There are four phases of chronic HBV infection: immune tol-
development of jaundice, with physical examination findings erant, immune active, immune control, and reactivation
of jaundice and hepatomegaly. HAV can be transmitted during (Figure 30). Patients who have acquired HBV through vertical
the prodrome stage and up to 1 week after development of transmission remain in the immune-tolerant phase for the
jaundice. Laboratory studies may show aminotransferase lev- first two to three decades. This stage does not require treat-
els greater than 1000 U/L and total bilirubin level oflO mg/dL ment except in specific cases (patients older than 40 years
(171 µmol/L) or higher, mostly direct (conjugated) . A positive with an HBV DNA level of at least 1 million lU/mL and signifi-
test for lgM antibodies to HAV is suggestive of acute illness, cant inflammation or fibrosis).
although false positives may occur in the setting of other viral Patients transition to hepatitis B e antigen (HBeAg)-
infections. The presence of lgG antibodies to HAV indicates positive, immune-active hepatitis later in lite. Hallmarks of
previous infection or vaccination and provides immunity from the immune-active phase include elevated ALT levels, an HBV
reinfection. Treatment is supportive, and 90% of patients or DNA level ofat least 20,000 lU/mL, and a positive HBeAg test.
more recover fully within 3 to 6 months of infection. Moderate to severe inflammation can occur, fibrosis can pro-
Postexposure vaccination is sufficient for immunocompetent gress, and treatment is warranted in this phase.
patients, and HAY immunoglobulin can be administered to Spontaneous seroconversion to the immune-control
immunocompromised patients. (inactive) phase with a loss of HBeAg and development of anti-
HBe occurs at a rate oflO'Yo per year. To be considered inactive,
Hepatitis B the ALT level must be normal and the HBV DNA level must be
Hepatitis B virus (HBV) is a DNA virus affecting 240 million 2000 lU/mL or lower when measured every 3 to 4 months for
persons worldwide and 2.2 million in the United States. See 1 year.
MKSAP 18 General Internal Medicine for HBV vaccination Approximately 60% to 80% of cases remain in the inac-
strategies. HBV can be transmitted vertically, through sexual tive phase, but up to 20% can revert to the HBeAg-positive,
exposure, percutaneously, or by dose person- to-person con- immune-active phase. Jn addition, the HBeAg-negative reac-
tact. The risk for developing chronic HBV infection differs by tivation phase can develop in 10% to 30% of cases; this phase
age. Newborns acquiring HBV have the highest risk (90%), is marked by fluctuating elevations in ALT levels and an HBV
whereas adults have an approximately 5% risk. Testing for HBV DNA level that is low but at least 2000 JU/mL, accompanied
is recommended in individuals with risk factors (Table 27). by ongoing inflammation and fibrosis that require treatment.
HBV infection presents as acute hepatitis in a minority of Not all patients progress through each one of these phases or
patients. Approximately 30% of adults may develop jaundice in sequence.

TABLE 27. Risk Factors Requiring Testing for Hepatitis B Virus

Individuals born or raised in regions with high rates of hepatitis B virus infection, including Asia, Africa, the South Pacific, European
Mediterranean countries, Eastern Europe, most of South America, Honduras, Guatemala, and the Middle East (except Israel and Cyprus)
U.S.-born persons not vaccinated as infants whose parents were born in endemic areas
Household or sexual contact with hepatitis B surface antigen-positive persons
Intravenous drug use
Multiple sex partners or history of sexually transmitted infection
Men who have sex with men
History of incarceration
History of hepatitis C virus or HIV infection
Hemodialysis
Pregnancy
Elevated aminotransferase levels of unknown cause

51
Disorders of the Liver

Clinical Scenario

Acute hepatitis B; occasionally + + + >20,000


reactivation of chronic hepatitis B
Resolved previous infection + + +/- Undetected
Immunity due to previous vaccination + Undetected
False positive anti-HBc or resolved + Undetected
previous infection
Immune-tolerant chron ic hepatitis B + + + >1 million
(perinatally acquired, age <30 years)
Inactive chronic hepatitis B + + + <10,000
HBeAg-positive immune-active + + + >10,000
chronic hepatitis B
HBeAg-negative immune-reactive + + + >10,000
chronic hepatitis B

I Anti HBc =hepatitis B core antibody; anti-HBe - hepatitis Be antibody; anti-HBs =hepatitis B surface antibody; HBeAg - hepatitis Be antigen; HBsAg =hepatitis B surface
n; HBV • hepatitis 8 virus.
---------------------------- -·----- _____,
Vertically Horizontally
acquired HBV acquired HBV

Immune tolerant Immune active


(HBeAg-positive chronic HBV) (HBeAg-positive chronic HBV)
Age <30 years Abnormal ALT
Normal ALT HBeAg positive, anti-HBe negative
HBeAg positive, anti-HBe negative ALT abnormal
HBV DNA >1 million HBV DNA> 10,000 IU/mL
No inflammation or fibrosis Inflammation and fibrosis

Immune control
(chronic HBV inactive carrier)
Normal ALT
HBeAg negative, anti-HBe positive ILiver injury I
HBV DNA <10,000 IU/mL
No inflammation, variable fibrosis

Reactivation
(HBeAg-negative chronic HBV)
Abnormal ALT
HBeAg negative, anti-HBe positive
HBV DNA >10,000 IU/mL
Inflammation and fibrosis

Fl GU RE 3 0. Phases of chronic hepatitis Binfection. It is assumed that patients progress through the phases in sequence, although not all patients develop HBeAg-
negative chronic hepatitis B, and only patients with vertical transmission of hepatitis Bhave a clinically recognized immune-tolerant phase. All phases have positive HBsAg,
negative anti-HBs, and positive lgG anti-HBc.
AU- alanine aminotransferase; anti·HBc • hepatitis Bcore antibody; anti-HBe •hepatitis Be antibody; anti-HBs •hepatitis Bsurface antibody; HBeAg - hepatitis Be antigen; HllsAg - hepatitis Bsurface antigen;
HBV- hepatitis Bvirus; HBVONA- hepatitis Bvirus ONA.

Risk factors for the development of cirrhosis and hepato- infection are an ALT level at least twice the upper limit of nor-
cellular carcinoma in patients with chronic HBV infection are mal and an HBV DNA level of at least 20 ,000 IU/mL (HBeAg-
listed in Table 29. positive, immune-active phase), or an HBV DNA level of at
Treatment is advised for patients with acute liver failure, least 2000 IU/mL (HBeAg-negative, reactivation phase). First-
infection in the immune-active phase or reactivation phase, line treatment is entecavir or tenofovir. Lamivudine, adefovir,
and cirrhosis, and in irnmunosuppressed patients. Treatment and telbivudine are less commonly used due to resistance.
thresholds in chronic immune-active or reactivation HBV Pegylated interferon can be used for 48 weeks in patients with

52
Disorders of the Liver

male sex plus age older than 40 years; (3) Asian descent plus
female sex plus age older than so years; (4) sub-Saharan
African descent plus age older than 20 years; (5) persistent
Age older than 40 years inflammatory activity (defined as an elevated ALT level and
HBV DNA levels greater than 10,000 IU/mL for at least a few
Hepatitis B virus DNA level >2000 IU/ml
years); and (6) a family history of hepatocellular carcinoma.
Elevated alanine aminotransferase level
Genotype C infection Hepatitis C
Heavy alcohol use Worldwide, 130 to 150 million individuals are infected with
Development of hepatitis B e antigen-negative reactivation hepatitis C virus (HCV), with 2.7 to 3.9 million individuals
phase of chronic hepatitis B virus infection with HCV infection in the United States. HCV Is most com-
HIV infection monly transmitted through intravenous or intranasal drug
Hepatitis C virus or hepatitis D virus infection use, blood transfusions before 1992, or sexual intercourse. The
efficiency of the virus' spread through vaginal intercourse is
low. Individuals born between 1945and1965 require one-time
high ALT levels, low HBV DNA levels, and without cirrhosis. HCV testing, as the prevalence is nearly 3% in this group and
Candidates for interferon are those who have a desire for finite these individuals account for 75% of cases of HCV infection.
therapy, are not pregnant, and do not have significant psychi- Patients with risk factors should be tested (Table 30).
atric disease, cardiac disease, seizure disorder, cytopenia, or Acute HCV infection is asymptomatic in most patients.
autoimmune disease. Jaundice, nausea, right-upper-quadrant pain, dark urine, and
Treatment goals for patients in the HBeAg-positive, acholic stools can occur in the symptomatic cases. Evaluation
immune-active phase are HBeAg loss and anti-HBe seroconver- of suspected acute infection includes HCV antibody and RNA
sion, which should be followed by an additional 12 months of tests. The HCV RNA test becomes positive first, and the HCV
treatment. Goals of treatment in the HBeAg-negative, reactiva- antibody test becomes positive within 1 to 3 months. HCV
tion phase are HBV DNA suppression and ALT normalization; antibody seroconversion within 12 weeks in the presence of an
oral antiviral agents are generally continued indefinitely. initial positive HCV RNA test confirms an acute HCV infection.
Patients with cirrhosis should continue oral antiviral medica- fnfection that clears spontaneously, usually within 6 months,
tions indefinitely. HBsAg seroconversion rarely occurs with oral is more common in patients with symptoms, high ALT levels,
antiviral treatment and, therefore, is not a goal of treatment. female sex, younger age, and the TL-28 CC genotype. Monitoring
Regression of fibrosis and even of cirrhosis can occur with HCV RNA quantification for clearance for 6 months is recom-
treatment. mended in patients with acute infection.
Prophylactic oral antiviral therapy should be given to
patients who are HBsAg-positive or isolated core antibody-
positive and receiving B-cell depleting therapy (for example, TABLE 30. Conditions Requiring Testing for Hepatitis C
rituximab, or ofatumumab), prednisone (::::10 mg/d for at least Virus
4 weeks), or anthracydine derivatives. Patients undergoing Birth year 1945-1965
therapy with tumor necrosis factor-a or tyrosine kinase inhib-
lnjection·drug use or intranasal illicit-drug use (ever)
itors should be considered for prophylaxis.
Long-term hemodialysis (ever)
Rarely, patients with HBV infection develop polyarteritis
nodosa or cryoglobulinemia, which should prompt treatment Percutaneous/parenteral exposures in an unregulated setting
(nonsterile technique)
with oral antiviral therapy. Membranous glomerulonephritis is
a rare extrahepatic association. Needlesticks, sharps, or mucosa! exposure to hepatitis C virus-
infected blood
The survival rate after liver transplantation for end-stage
Children born to women infected with hepatitis C virus
liver disease from HBV infection is greater than 90% at 1 year.
Recurrence ofHBVinfection in transplant recipients is prevented Receipt of blood or blood·components transfusion or organ
transplantation before 1992
with HBV immunoglobulin and/or oral antiviral therapy.
The prognosis for untreated individuals with HBV infec- Receipt of clotting-factor concentrates produced before 1987
tion worsens with age, particularly with age older than History of incarceration
40 years. Approximately 40% of deaths in HBV-infected per- HIV infection
sons older than age 40 years are related to hepatocellular Sexually active persons about to start preexposure prophylaxis
carcinoma or decompensated cirrhosis. The foJJowing charac- for HIV
teristics are associated with an increased risk for hepatocel- Undiagnosed chronic liver disease
lular carcinoma in patients with HBV infection and are Elevated alanine aminotransferase level
indications for surveillance with ultrasound or cross-sectional
Living organ donors, before donation
imaging every 6 months: (1) cirrhosis; (2) Asian descent plus

53
Disorders of the Liver

HCV results in chronic infection in 60% to 80%of patients, disease specialist because resistance-associated substitution-
with up to 30% progressing to cirrhosis over two to three dec- guided retreatment may be necessary. Patients with decom-
ades. Patients with cirrhosis have a 2% to 4% risk per year for pensated cirrhosis should see a hepatologist before treatment
developing hepatocellular carcinoma. and be considered for liver transplantation. Post- liver trans-
The first step in the diagnosis of chronic HCV infection is plantation recurrence of HCV infection is universal in treat-
HCV antibody testing, and if positive, a HCV RNA quantifica- ment-naive patients. Success rates ofHCV treatment after liver
tion. Patients with positive HCV antibody and RNA tests have transplantation are excellent.
active infection, and a genotype test should be performed. Cure is defined by the absence of HCV RNA in blood 12
Asymptomatic patients with a positive HCV antibody and a weeks after completion of treatment. HCV antibodies remain
negative HCV RNA test, and without recent exposure to HCV. positive indefinitely and should not be rechecked. Patients can
do not have active infection and generally do not require fur- become reinfected after new exposures, and HCV RNA testing
ther testing. is appropriate to identify new infection. Cure rates exceed 90%
All patients infected with HCV should be tested for HBV in the majority of patients. Virologic cure reduces the risk for
and HIV because of the potential shared routes of transmis- progression to cirrhosis, complications of cirrhosis, hepatocel-
sion. HBV testing should include HBsAg to assess for active lular carcinoma, and liver-related mortality. Patients with stage
infection (followed by HBV DNA if positive) and antibodies to F3 fibrosis or cirrhosis require ongoing surveillance for hepa-
hepatitis B antigens (anti-HBs and anti-H&) to assess for past tocellular carcinoma even after virologic cure. Patients with
infection. Susceptible patients should receive HBV vaccina- cryoglobulinemic vasculitis and non-Hodgkin lymphoma are
tion. HBV reactivation can be seen during treatment of HCV more likely to experience remission when HCV is eradicated.
infection with direct-acting antiviral therapy. Patients who test
positive for HBsAg with detectable HBV DNA and who do not Hepatitis D
meet standard HBV treatment criteria should undergo HBV Hepatitis D virus (HDV) is a defective RNA virus that requires
DNA monitoring approximately every 4 weeks until 12 weeks HBV for human infection. HDV is endemic in the
after completion of treatment for HCV infection, or they can Mediterranean basin and Pacific islands and uncommon in
be treated with oral HBV therapy prophylactical ly. Western countries. The diagnosis of HDV infection is made
Patients with chronic HCV infection require a fibrosis through detection of HDV lgG. The clinical course can range
assessment with a transient or MRI elastography or liver biopsy, from inactive disease to progressive liver disease (in the case
unless they have a documented short duration of disease, of simultaneous HBV-HDV coinfection) to fulminant hepatitis
decompensated cirrhosis, or a radiologic diagnosis of cirrhosis. in HDV superinfection. Patients infected with HDV with evi-
All patients infected with HCV should be considered for dence of progressive liver disease should receive treatment
treatment unless there are significant life-limiting comorbidi- with pegylated interferon for 12 months; cure rates are 25%
ties or major barriers to adherence to treatment. to 45%.
Treatment regimens include a combination of direct-
acting antivirals, using different mechanisms to prevent viral Hepatitis E
reproduction (Table 31). Regimens are chosen based on geno- Hepatitis E virus (HEV) is an RNA virus with worldwide dis-
type, previous treatment experience and response, and fibro- tribution. There are four different genotypes: genotypes 1 and
sis status. Patients whose infection does not respond to newer 2 are more common in developing countries and are transmit-
regimens are generally managed by a hepatologist or infectious ted by the fecal-oral route through contaminated water; geno-
types 3 and 4 are more common in developed countries where
transmission occurs through contaminated food, mostly pork
TABLE 31 . Treatment Regimens for Hepatitis C Virus
or deer meat. Jn developing countries, HEV infection generally
Infection
occurs in young adults and can occur in large epidemics. In
Drug Treatment Regimens
developed countries, HEV generally affects males older than
Grazoprevir• + elbasvirb age 40 years. The incubation period is 2 to 5 weeks.
Paritaprevir• + ombitasvirb + dasabuvir<' Approximately 50% of cases are asymptomatic. Symptoms of
HEV infection are jaundice, malaise, nausea, vomiting, ano-
Simeprevir' + sofosbuvird
rexia, and right-upper-quadrant pain. Aminotransferase lev-
Daclatasvirb + sofosbuvird
els are usually elevated to 1000 to 3000 U/L. Diagnosis relies
Led ipasvirb + sofosbuvird on detection of HEV IgM or RNA. Treatment is supportive, and
Velpatasvirb + sofosbuvird recovery is expected within 4 to 6 weeks. HEV infection
•Drug class: NS3/4A protease inhib itor (-p revlr)
should be considered in patients with an unknown cause of
"Drug class: NSSA inhibitor (-asvir)
acute hepatitis and in immunocompromised patients with
chronic hepatitis- Solid-organ transplant recipients with
'NSSB non·nucleoside polymerase inhib itor (-buvir)
chronic hepatitis E have response rates of 70% with ribavirin
•NSSB nucleotid e polymerase inhibitor (·b uvir)
treatment.

54
Disorders of the Liver

Other Viruses Autoimmune Hepatitis


Other viruses can causes hepatitis, including Epstein-Barr
Autoimmune hepatitis is a chronic inflammatory hepatitis
virus (EBY), cytomegalovirus (CMY), herpes simplex virus
that is four times more common in women than in men and
(HSV), varice!la zoster virus, and parvovirus. Approximately
can be associated with other autoinunune diseases (most
90% ofEBV-intected individuals have mild hepatitis, occasion-
commonly autoimmune thyroiditis, synovitis, or ulcerative
ally developing jaundice and elevated ALP levels. Diagnostic
colitis). It can affect individuals at any age but most commonly
testing includes heterophile antibody testing or checking EBV
occurs in middle-aged adults. The presentation of autoim-
serologies, which may demonstrate a positive viral-capsid
mune hepatitis can vary from asymptomatic elevation of
antigen IgM. Treatment is supportive.
transaminase levels to extrahepatic symptoms such as myalgia
CMV can cause a syndrome that mimics EBY-related
and malaise to acute liver failure. Diagnosis is made based on
mononucleosis. CMV Infection can cause mild aminotrans-
laboratory results (including positive antinuclear and smooth-
ferase elevations. Diagnosis is made with CMV serologies in
muscle antibodies and elevated IgG levels), exclusion of other
the immunocompetent host. Treatment is supportive, and
diagnoses (such as Wilson disease, viral hepatitis, and drug-
spontaneous recovery is the norm. CMV infection in solid-
induced liver injury), and histologic findings on liver biopsy.
organ transplant recipients can cause CMV syndrome, marked
Treatment includes prednisone and azathioprine for most
by fever and myelosuppression, or tissue-invasive CMY infec-
patients. For uncomplicated autoimmune hepatitis without
tion involving the gastrointestinal tract, liver, lungs, and retina.
cirrhosis, budesonide can be considered in place ofprednisone,
Treatment with intravenous ganciclovir is required in immu-
but its role is not established. Prednisone monotherapy is less
nocomprornised hosts. Without treatment, CMV infection car-
preferable due to adverse effects. Azathloprlne requires moni-
ries a high mortality rate in immunocompromised hosts.
toring for cytopenia. Typically, biochemical response occurs
HSY-caused hepatitis in women in the third trimester of
within 3 to 8 months for the 85% of patients whose disease
pregnancy manifests with fever, altered mental status, right-
responds to standard treatment. Histologic response can lag by
upper-quadrant pain, hepatomegaly, and a presentation similar
many months. Duration of treatment should be 2 to 3 years
to sepsis. Aminotransferase levels are commonly elevated to
before consideration of withdrawal. A liver biopsy is recom-
5000 U/L or higher with a disproportionately low bilirubin level,
mended to determine histologic response before consideration
accompanied by coagulopathy. Diagnosis can be confirmed with
of drug withdrawal. High rates of relapse after discontinuation
polymerase chain reaction tei.iing for HSV (when available) or a
of treatment underscore the need for serial monitoring of liver
liver biopsy showing intranuclear inclusions, multinucleated
tests. Patients with a severe acute form of autoimmw1e hepati-
giant cells, and coagulative necrosis with minimal inflamma-
tis presenting with jaundice should be managed by a hepatolo-
tion. Intravenous acyclovir is the treatment of choice. The case
gist, and patients with features of acute liver failure require
fatality rate is approximately 80% in untreated patients.
urgent transfer to a transplant center.
In adults, primary varicella zoster virus is a very rare
cause of acute hepatitis. Primary infection in organ transplant KEY POINTS
recipients can cause acute liver failure. Diagnosis in irnmuno- • Diagnosis of autoimmune hepatitis is made based on
compromised patients requires a biopsy of the skin or of the the presence of elevate d aminot ransferase levels, posi-
affected organ. Treatment is intravenous acyclovir. tive antinuclear antibody and smooth-muscle antibody.
Parvovirus B19 can result in transient elevation of ami- elevated levels oflgG, and compatible findings on liver
notransferase levels but has also been associated with fulmi- biopsy.
nant hepatic failure in very rare instances. • Wilson disease, viral hepatitis, and drug-induced liver
Other viruses associated with elevated liver chemistries injury need to be excluded before making the diagnosis
include human herpes virus 6, 7, and 8, as well as adenoviruses. of au toimmune hepatitis.
KEY POINTS
• Treatment of hepatitis A virus infection is supportive,
a nd 90% of patients or more recover fully within 3 to 6 Alcohol-Induced Liver Disease
months of infection. Alcohol-induced liver disease is the second most common
• Treatment of hepatitis B virus infection is advised for reason for liver transplantation in the United States. Alcohol
patients with acute liver failure, infection in the injury to the liver may take the form of steatosis, steatohepati-
immune-active phase or the reactivation phase, cirrho- tis, or severe steatohepatitis, also known as alcoholic hepatitis.
sis, and in immunosuppressed patients. Approximately 25% of heavy drinkers develop cirrhosis.
• Individuals born between 1945 and 1965 require one- History of alcohol use is the most important component in
time testing for hepatitis C virus. making the diagnosis, although not all patients are forthcom-
ing about alcohol use. See MKSAP 18 General Internal Medicine
• Up to 95% of patients with hepatitis C vims infection
for more information about screening for alcohol abuse. Most
can be cured with direct-acting antiviral agents.
patients with alcoholic liver disease have consumed more than

55
Disorders of the Liver

100 g of alcohol daily for 20 years. The Alcoholic Liver Disease/ hypertension and a history of consistent alcohol intake.
Nonalcoholic Fatty Liver Dlsease Index score (www.mayoclinic. Occasionally a llver biopsy is necessary in cases with diagnos-
org/medical-professlonals/model-end-stage-liver-disease/ tic ambiguity. Tn patients who drink alcohol, Inflammation of
alcoholic-liver-disease-nonalcoholic-fatty-liver-disease-index) the liver increases stiffness, making transient and MR elastog-
can be helpful In distinguishing alcoholic liver disease from raphy inaccurate. Nutritional assessments and alcohol treat-
nonalcoholic fatty liver disease. It includes the following vari- ment are required. Alcohol abstinence can result in significant
ables: AST, ALT, mean corpuscular volume, age, height, weight, stabilization of liver function and reversal of portal hyperten-
and sex. Physical examination may show evidence of hepato- sion. Liver transplantation is reserved for appropriate candi-
megaly in patients with steatosis or steatohepatitis. In patients dates who are abstinent from alcohol for approximately
with alcoholic hepatitis or cirrhosis, findings of advanced liver 6months.
disease may be present, including muscle wasting, scleral
KEY POINTS
icterus, jaundice, spider angiomata, gynecomastia, left hepatic
lobe hypertrophy, testicular atrophy, or palmar erythema. • The Alcoholic Liver Disease/ Nonalcoholic Fatty Liver
Laboratory evaluation of alcohol-related liver disease may Disease Index score can be helpful in distinguishing
show an elevated mean corpuscular volume, AST:ALT ratio alcoholic liver disease from nonalcoholic fatty liver
greater than 2, elevated y-glutamyl transferase level, and in disease.
advanced cases, elevated TNR and thrombocytopenia. • Manifestations of alcoholic hepatitis include fever,
Manifestations of alcoholic hepatitis include fever, jaun- jaundice, tender hepatomegaly, and leukocytosis.
Cl dice, tender hepatomegaly, and leukocytosis. Ultrasound or • Severity of alcoholic hepatitis is determined by the
cross-sectional Imaging may reveal evidence of steatosis, cir- Maddrey discriminant function score; patients with a
rhosis. and findings consistent with portal hypertension. The score of 32 or greater or with hepatic encephalopathy
diagnosis is most often made clinically, with liver biopsy may be considered for prednisone therapy.
reserved for cases with diagnostic uncertainty. Severity of alco-
holic hepatitis is determined by the Maddrey discriminant
function (MDF) score, which is calculated as follows:
Drug-Induced Liver Injury
MDF score : 4.6 (prothrombin time [s]
- control prothrombin time [s])
Drug-induced liver injury encompasses a spectrum of liver
injury and can be induced by many medications. Prescription.
CJ
+total bilirubin (mg/dL)
over-the-counter, and herbal medications have been impli-
In severe cases, as defined by an MDF score of 32 or greater cated. Acetaminophen is the most recognized medication to
or the presence of hepatic encephalopathy, treatment with pred- have intrinsic hepatotoxicity. and its pattern of liver injury is
nisolone is recommended by the American College of easily recognizable. People who chronically drink alcohol can
Gastroenterology (ACG) guideline. The STOPAH trial showed a develop acetaminophen hepatotoxicity even when taking
trend toward improvement in 28-day mortality with the use of therapeutic doses of acetaminophen. The early recognition of
prednlsolone, but results were not statistically significant. acetaminophen- induced liver injury is critical so that
However, a meta-analysis of randomized studies (including the N-acetylcysteine can be administered promptly to prevent
STOPAH study) showed that glucocorticoids were effective in liver failure.
reducing short-term mortality by 46%. PentoxifylJine can be Antibiotic (particularly amoxicillin-clavulanate) and
used in patients with contraindications to steroids, including antiepileptic medications (phenytoin and valproate) are
active infection, gastrointestinal bleeding, or kidney failure, but among the most common medications associated with drug-
data supporting its benefit are inconclusive. Pentoxifylline is not induced liver injury, representing 60% of cases. The most
effective in patients whose symptoms do not respond to pred- common drugs that cause acute liver failure in the United
nisone. The ACG guideline makes a conditional recommenda- States are antituberculosis drugs, sulfa-containing antimicro-
tion against the use of pentoxlfylline for patients with severe bial agents, and antlfungal agents. The pattern of liver test
alcoholic hepatitis based on a low level of evidence. Due to risk abnormalities and histological injury can be unpredictable.
for infection, prednisolone should be discontinued ilthe biliru- Some medications, such as amoxicillin-clavulanate, can cause
bin level does not decrease by day 7. If the bilirubin level a cholestatic hepatitis, whereas others, such as erythromycin,
decreases, treatment should be continued for 28 days. Nonsevere can cause differing patterns of liver pathology. Patients should
alcoholic hepatitis (MDF score <32) requires supportive meas- be asked about exposure within the past 6 months to medica-
ures and should not be treated with prednisolone or pentoxifyl- tions, both prescription and nonprescription as well as herbal
line. All patients require assessment for nutritional deficiencies, and dietary supplements. Withdrawal of the potentially
thiamine replacement, and alcohol treatment with a goal of offending drug with resolution of liver injury supports the
abstinence. Cl diagnosis. Re-exposure to the drug to demonstrate recurrence
Alcoholic cirrhosis can be diagnosed on clinical and radi- is not advisable, due to the risk of severe hepatitis upon
ologic grounds in patients with obvious evidence of portal rechallenge. CJ

56
Disorders of the Liver

Jaundice that occurs in the setting of cholestatic drug KEY POINTS


reactions can take months to resolve. An elevated bilirubin
• The most common cause of acute liver failure in the
level In the setting of increased hepatic transaminases greater
United States is acetaminophen overdose.
than three times the upper limit of normal is associated with
mortality rates as high as 14%. Until resolution occurs, there is • The diagnosis of acute liver failure requires immediate
potential for progression to liver failure and a need for liver referral to a liver transplant center.
transplantation.
KEY POINTS
• Acetaminophen, antibiotics (particularly amoxicillin- Metabolic Liver Diseases
davulanate). and antiepileptic agents (phenytoin and Nonalcoholic Fatty Liver Disease
valproate) are the most common causes of drug-induced Nonalcoholic fatty liver disease (NAFLD) is the most common
liver injury. cause of liver disease in the world. Some 30% of the U.S. popu-
• Liver injury typically resolves after discontinuation of lation may be affected by this condition. Most patients with
the offending drug. NAFLD do not have liver inflammation. In these patients, the
presence of fat in the liver without inflammation or fibrosis is
considered a benign condition. Patients who have fatty liver
CJ Acute Liver Failure with inflammation (nonalcoholic steatohepatitis [NASH]) have
Acute liver failure is defined as the onset of hepatic encepha- progressive disease that can lead to fibrosis and cirrhosis.
lopathy and a prolonged prothrombin time within 26 weeks of NAFLD usually develops due to the metabolic syndrome, lead-
occurrence of jaundice or other symptoms of liver inflamma- ing to triglyceride accumulation within the liver. The mecha-
tion in the absence of chronic liver disease. The most common nisms by which patients with NAFLD develop NASH are not
cause of acute liver failure in the United States is acetami- fully understood. Metabolic, genetic, and environmental fac-
nophen overdose. Other widely recognized causes are pre- tors likely play a role. Patients with NAFLD are often diagnosed
sented in Table 32. The diagnosis requires immediate referral by abdominal imaging. Ultrasound, CT, and MRI can detect the
to a liver transplant center. presence of hepatic fat. Symptoms of NAFLD are nonspecific
Patients with acute liver failure requjre careful monitor- and include fatigue and vague right-upper-quadrant abdomi-
ing of mental status because progressive hepatic encephalopa- nal pain. NASH is characterized by inflammation, risk for
thy can result in cerebral edema (see Hepatic Encephalopathy). progressive fibrosis, and development of cirrhosis. NASH may
Kidney injury is common, and continuous renal replacement affect 5% of the U.S. population. Mildly elevated hepatic
therapy is better tolerated than intermittent hemodialysis. transaminases are common in NASH. Unrecognized NASH is
Patients with acute liver failure are at risk for hypoglycemia likely a major cause of cryptogenic cirrhosis.
and infection. There are no tests that can diagnose NASH as a cause of
Specific treatment is based on the cause: N-acetylcysteine chronically elevated liver chemistries. Patients with elevated
is used for acetaminophen intoxication; antiviral medications liver chemistries, a negative serological evaluation for alterna-
for HBV infection and herpes simplex hepatitis; penicillin G tive causes, clinical features of the metabolic syndrome, and
for Amanita mushroom poisorung; and delivery of the fetus characteristic abdominal imaging are presumed to have NASH.
for acute fatty liver of pregnancy. Prompt treatment is crucial. Low titers of autoantibodies are observed in 20% of patients
For example, mortality rates for acetaminophen intoxication with NAFLD. The NAFLD fibrosis score (www.nafldscore.com)
are lowest when N-acetylcysteine is administered within 12 uses clinical data to identify patients at risk for severe disease.
hours of ingestion. CJ Transient elastography can be used to deterrrune whether
patients with NASH have developed significant hepatic fibro-
TABLE 32. Causes of Acute Liver Failure
sis. Liver biopsy is indicated when the diagnosis is in doubt, or
if the presence of hepatic fibrosis cannot otherwise be
Acetam inophen determined.
Hepatitis A virus The management of NAFLD is focused on weight loss
Herpes simplex virus through diet and lifestyle modification. No specific diet for
Autoimmune hepatitis NAFLD is recommended, although carbohydrate-restricted
diets may result in greater reduction in liver fat than other
Other medications (antituberculosis drugs, sulfa-containing
antimicrobial agents, antifungal agents, and herbal diets. Bariatric surgery and concomitant weight loss results in
supplements) improvement of inflammation and fibrosis associated with
Hepatitis 8 virus NAFLD. No drugs have been approved by the FDA for the treat-
Amanita pha//oides mushrooms ment of NAFLD. Cardiovascular disease is the leading cause of
death in patients with NASH, and therapy with statins should
Acute fatty liver of pregnancy
be considered.

57
Disorders of the Liver

a.1-Antitrypsin Deficiency of copper can result in sudden liver failure and the need for
a 1-Antitrypsin deficiency is an autosomal recessive genetic emergent liver transplantation, especially in younger patients.
disorder that results in accumulation of a variant protein in Unexplained liver disease or liver failure In any patient younger
the liver. Homozygosity for this condition may result in liver than age 40 years should prompt an investigation for Wilson
injury and eventual cinhosis. The hepatic accumulation of disease, although older patients with Wilson disease have also
a 1-antitrypsin results in decreased circulating a 1-antitrypsin, been described.
leading to lung disease. Supplemental a 1-antitrypsin prevents Decompensated Wilson disease presents with Coombs-
negative hemolytic anemia due to the sudden release of copper
CJ
progressive Jung injury but does not aftect the progression of
cirrhosis. Patients with heterozygosity for a 1-antitrypsin defi- from hepatocytes. Laboratory findings include high levels of
ciency are at increased risk for liver injury in the setting of urinary copper and low levels of ceruloplasmin and ALP.
other liver diseases, including viral hepatitis or fatty liver dis- Neurological changes can be seen in patients with Wilson dis-
ease. Liver transplantation is required to treat liver failure ease (tremor. early-onset Parkinson disease, dystonia). Kayser-
resulting from a 1-antitrypsin deficiency. Fleischer rings can be seen on slit-lamp examination of patients
with Wilson disease and neurologic findings. Histological
changes on liver biopsy can be nonspecific, although levels of
Hereditary Hemochromatosis
hepatic copper are typically high. Genetic testing for mutations
Hereditary hemochromatosis is a condition characterized by
in the ATP7B gene confirms Wilson disease. Cl
excessive accumulation of iron in the liver due to a mutation
Treatment is lifelong and involves administration of cop-
in the genes that control the synthesis of hepcidin. Several
per chelators. Trientlne is preferred over peniclllamine due to
genetic conditions can cause clinically significant iron over-
a lower rate of adverse effects. Zinc supplements can be
load; hereditary hemochromatosis typically results from
administered to decrease the intestinal absorption of copper.
homozygosity of the C282Y polymorphism of the HFE gene.
Cirrhosis can develop in untreated patients with hereditary KEY POINTS
hemochromatosis and is associated with an increased risk for • onalcoholic fatty liver disease is the commonest cause
hepatocellular carcinoma. of liver disease in the world.
Elevated transferrin saturation and elevated serum ferritin • Management of nonalcoholic liver disease is focused on
levels can suggest a diagnosis of hereditary hemochromatosis; weight loss through dietary and lifestyle modification.
transterrin saturation is recommended as the initial diagnostic
test. Elevations in either test can be seen in other liver diseases, • a 1-Antitrypsin deficiency is an autosomal recessive
so confirmation of the diagnosis requires genetic testing. genetic disorder that results in accumulation of a vari-
Removal of excessive iron, usually by phlebotomy, effec- ant protein in the liver; homozygosity can result in liver
tively prevents the development of cirrhosis. Cirrhosis may be injury and cirrhosis.
clinically apparent. If it is not apparent based on imaging or • Hereditary hemochromatosis is a condition character
physical examination findings, it should be suspected in all ized by excessive iron absorption that results in accu-
patients with hereditary hemochromatosis who have a serum mulation of iron in the liver and the development of
ferritin level greater than 1000 ng/mL (1000 µg/L). cirrhosis.
Confirmation of cirrhosis is important because surveillance • Wilson disease is a rare autosomal recessive disorder
for hepatocellular cancer is recommended In these patients. that causes accumulation of copper in the liver.
Patients with cirrhosis should undergo iron-lowering therapy
to stabili7e liver disease and to prevent other organ manifesta-
tions of iron overload. Liver failure resulting from hereditary
hemochromatosis is treated by liver transplantation. Cholestatic Liver Disease
First-degree relatives of patients with hereditary hemo- Primary Biliary Cholangitis
chromatosis should be screened with Iron studies as well as Primary blllary cholangitis (PBC), previously termed primary
testing for the HFE gene mutation. Children of affected biliary cirrhosis, is an autoimmune disease affecting the small
patients can be reassured that they do not have hemochroma- and medium bile ducts with a female to male predominance
tosis if the other parent is tested for the HFE gene mutation of 9 to 1. It is more common in individuals of European
and has a normal genotype, as this would mean that the chil- descent. PBC can present with fatigue and prurltus, but many
dren are obligate heterozygotes. patients are asymptomatic, with the diagnosis suggested by
Hemochromatosis is discussed further in MKSAP 18 elevated ALP levels. Diagnosis does not require liver biopsy
Hematology and Oncology. when the ALP is at least 1.5 times the upper limit of normal
and antimitochondrial antibody testing ls positive. In patients
Wilson Disease with negative antimitochondrial antibody test results and
Wilson disease is a rare autosomal recessive disorder that strong suspicion for PBC, a liver biopsy is necessary. Transient
causes accumulation of copper in the liver. The accumulation or MRI elastography can be used for fibrosis staging.

58
Disorders of the Liver

The initial treatment is ursodeoxycholic acid. Response to


treatment ls defined by improvement of ALP level to less than
1.67 times the upper limit of normal. Patients whose disease
does not respond to ursodeoxycholic acid should receive obet-
icholic acid. Dose reductions are required for obeticholic acid
use in patients with decompensated cirrhos.is to avoid worsen-
ing liver failure. Ursodeoxycholic acid treatment results in his-
tologic improvement, better survival rates, and diminished need
for liver transplantation. Patients who present with normal bili-
rubin and albumin levels and respond to treatment have a life
expectancy similar to that of Individuals without PBC.
PBC is associated with other autoimmune conditions,
particularly autoimmune thyroid disease. Therefore, in
patients with PBC, thyroid-stimulating hormone level should
be checked on a yearly basis. In patients with a PBC score of FIGURE 31 . Magnetic resonance cholangiopancreatography showing
4.1 or greater (www.mayoclinic.org/medical-professionals/ multifocal intra-and extrahepatic bile-duct stricturing with a dominant left-lobe
model-end- stage- IIver-disease/updated -natural- history - stricture {arrow) and upstream bile-duct dilation consistent with the diagnosis of
model-for-primary-biliary-cirrhosis), upper endoscopy is primary sclerosing cholangitis.
indicated to assess for esophageal varices. First-degree rela-
tives of patients with PBC, especially women, should be
Patients with PSC have a 15% lifetime risk for cholangio-
screened by checking their ALP level periodically. Patients
carcinoma. Yearly MRCP and measurement of the carbohy-
with advanced disease should be managed like other patients
drate 19-9 level is recommended for cholangiocarcinoma
with cirrhosis and portal hypertension (see Complications of
surveillance. The incidence of cholangiocarcinoma is highest
Advanced Liver Disease). Liver transplant outcomes for
in the first 2 years after diagnosis of PSC is made.
patients with PBC are excellent, with a 1-year survival rate
Liver transplantation should be considered for patients
greater than 90% and a recurrence rate of approximately 20'Yo
with decompensated cirrhosis, recurrent bacterial cholangitis,
at 5 years after liver transplantation.
and hilar cholangiocarcinoma. Transplant outcomes for
Prima ry Scle rosing Cholangitis patients with PSC are excellent, with 1-year survival rates of at
least 90% and recurrence rates of approximately 20% at 5 years
Primary sclerosing cholangitis (PSC) is an autoimmune fibro-
after liver transplantation.
intlammatory disease of the large bile ducts, but it can also
affect the small bile ducts (small-duct PSC). It is more com- KEY POINTS
mon in men than in women, which is unique among the • In patients with primary biliary cholangitis, ursodeoxy-
autoimmune liver diseases. PSC is associated with inflamma- cholic acid treatment results in histologic improvement.
tory bowel disease (IBD) in about 85% of cases; up to 7.5% of better survival rates, and diminished need to for liver
patients with ulcemtive colitis have PSC. All patients with PSC transplantation.
and without known IBD should have a colonoscopy at the time
• Primary sclerosing cholangitis is associated with inflam-
of PSC diagnosis. Patients with concomitant JBD may have a
matory bowel disease in about 85% of cases; these
unique PSC-IBD phenotype, characterized by rectal sparing,
patients have an increased risk for colorectal cancer and
mild pancolitis, and backwash ileitis; this carries a higher risk
require surveillance colonoscopy every l to 2 years.
for colon cancer, requiring colonoscopy with surveillance
biopsies every 1 to 2 years, as we 11 as a higher 1isk for pouchitis • Primary sclerosing cholangitis often requires liver
after total colectomy. The diagnosis of PSC can be made non- transplantation and has the highest case-based mortality
invasively through MR cholangiopancreatography (MRCP) rate among the autoimmune liver diseases.
(Figure 31). Endoscopic retrograde cholangiopancreatography
(ERCP) should be considered in patients with jaundice, wors-
ening pruritus, bacterial cholangitis, or a dominant stricture Determining Prognosis CJ
or bile duct mass seen on MRCP. Diagnosis of PSC does not The Child-Turcotte-Pugh (CTP) score (Table 33) and Model for
usually require liver biopsy; small-duct PSC, which cannot be End-Stage Liver Disease (MELD) score are prognostic in
diagnosed by MRCP, is an exception . patients with cirrhosis. The I-year survival rates for patients
PSC often requires liver transplantation and has the highest with CTP class A, B. and C cirrhosis are 100':1. . so·x.. and 45%,
case-based mortallty rate among the autoimmune liver diseases. respectively.
Median transplant-free survival for patients with PSC is 12 years. The MELD fo rmula includes bilirubin level, INR, and
There is no current medical therapy for PSC. ERCP to dilate creatinine level. and is accurate in predicting 3-month mor-
strictures and remove stones is used in symptomatic patients. tality. Another version of the MELD score, the MELD-Na

59
Disorders of the liver

TABLE 33. Child-Turcotte-Pugh Score•


1 Point 2 Points 3Polnts

Encephalopathy None Grade 1-11 Grade Ill-IV


Ascites None Mild/moderate Severe
Bilirubin <2 mg/dl (34.2 µmol/L) 2-3 mg/dl (34.2-51 .3 µmol/L) >3 mg/ dl (51 .3 µmol/L)
Albumin >3.5 g/dl (35 g/l) 2.8-3.5 g/dl(28-3 5 g/L) <2.8 g/d L (28 g/l )
Prothrombin time/INR <4s/<1 .7 4-6 s/1 .7-2.3 >6s/>2.3

' 5·6 points• Child -Turcotte-Pugh class A; 7.9 poi nts - Child -Turcotte-Pugh class B; 10-15 points • Child-Turcotte-Pugh class C.
- - - - - - - - - - - - - - - - - - - - --------

(https://optn. transplant.hrsa.gov/resources/allocation-calculators/ and increased intrahepatic vascular resistance. Prehepatlc


CJ meld-calculator/),
CONT.
adds sodium to the formula. The MELD score causes, including portal vein thrombosis, and posthepatic
has been the basis for liver transplant allocation since 2002. In causes, such as Budd-Chiari syndrome, can also result in portal
2016, the allocation system was changed to include the MELD-Na hypertension in the absence of cirrhosis. Hypersplenism, often
score, which was found to be a better predictive model for also associated with thrombocytopenia, further increases flow
3-month mortality. within the portal vein, exacerbating portal hypertension.
The MELD score can estimate postoperative mortality Complications of portal hypertension include gastroesophageal
in patients with cirrhosis (www.mayoclinic.org/medical- varices, ascites, and hepatic encephalopathy due to poor
professionals/model-end-stage-liver-disease/post-operative- hepatic clearance of absorbed nitrogenous compounds from
mortality-risk-patients-cirrhosis) . CJ the intestines. These complications of portal hypertension her-
ald a high rate of further complications and mortality, leading
KEY POINT
to consideration for liver transplantation.
• The Model for End-Stage Liver Disease-sodium formula
accurately predicts 3-month mortality and is used for
liver transplant allocation.
Esophageal Varices
Esophageal varices are enlarged vessels within the lumen of
CJ
the lower esophagus that provide extrahepatic pathways of
blood flow from the portal circulation to the systemic circula-
Complications of Advanced tion. Over time, esophageal varices enlarge and may spontane-
Liver Disease ously rupture, leading to bleeding and possible death. The
Patients with chronic liver disease from any cause are at risk for mortality rate for acute variceal hemorrhage has been signifi-
the development of cirrhosis. Uncomplicated cirrhosis is cantly reduced with treatment. Upper endoscopy should be
referred to as "compensated cirrhosis" and may be asympto- performed on all patients with cirrhosis to assess for the pres-
matic or associated with nonspecific symptoms. Decompensated ence of varices. Management of varices In patients with cir-
cirrhosis includes patients with compllcations such as ascites, rhosis depends on size of the varix and whether the cirrhosis
jaundice, hepatocellular carcinoma, hepatorenal syndrome, is compensated (Table 34) .
variceal hemorrhage, spontaneous bacterial peritonitis, and See Gastrointestinal Bleeding for discussion of manage-
hepatic encephalopathy. ment and secondary prophylaxis ofvariceal hemorrhage.

Portal Hypertension Gastric Varices and Portal


Portal venous hypertension develops in the setting of advanced Hypertensive Gastropathy
cirrhosis due to obstruction of blood flow caused by intrahe- Gastric varices are seen in as many as 20% of patients with
patic fibrosis, the development of regenerating liver nodules, cirrhosis and are responsible for 10% to 30% of variceal

Findings Management

No va rices Rep eat upper endoscopy in 3 years (unless d ecompensation occurs)


Small varices (<5 mm) Repeat upper endoscopy in 2 years (unless decompensation occurs)
Small varices with red wale marks (erythematous raised areas) Initiate nonselective ~- b l ocke r therapy (p ro pranolol, nadolol, or ca rved ilol)
Large varices (>5 mm) Initiate nonselective ~blocker therapy or endoscopic variceal ligation
Va rices in patients unable to tol erate ~-b l o c kers Endoscopic variceal ligation

60
Disorders of the Liver

hemorrhage. Varices that extend from the esophagus into the


Cl cardia of the stomach are treated with band ligation, like
absorption of nitrogenous substances. If a suboptimal
response is seen with lactulose therapy, rifaximin can be
CONT. esophageal varices. Varices of the gastric fundus, or isolated administered to alter bacterial flora in the intestinal lumen.
varices in other parts of the stomach, are not treated with band While rifaximin is better tolerated than lactulose, it is more
ligation. Bleeding varices in these areas should be treated with expensive. Initial treatment with lactulose is preferred.
hemodynamic resuscitation, antibiotic therapy, and octreo-
tide. Cyanoacrylate glue injection can be injected into bleeding As cites
gastric varices but is not FDA-approved for this indication. A Ascites occurs in 50% of patients within 10 years of diagnosis
transjugular intrahepatic portosystemic shunt (TIPS) should of cirrhosis and is the most common complication of cirrhosis.
be considered for bleeding from gastric varices. Splenic vein The finding of a fluid wave on physical examination can detect
thrombosis can cause isolated gastric varices and is treated ascites; other findings are less reliable. Ultrasound is useful for
with splenectomy. identifying the presence of ascites. When ascites is first
detected, diagnostic paracentesis should be performed. The
Hepatic Encephalopathy
serum-ascites albumin gradient is measured by subtracting
Hepatic encephalopathy is characterized by brain dysfunction
the level of albumin in the ascitic fluid from a concurrent
ranging from minimal abnormalities to frank coma. It occurs
serum albumin measurement:
due to insufficient hepatic function and porto-systemic shunt-
ing of blood caused by portal hypertension or vascular shunt- serum-ascites albumin gradient = serum albumin
ing. The development of hepatic encephalopathy can be - ascites albumin
spontaneous or precipitated by other conditions such as infec- Total protein levels in ascitic fluid also assist in determin-
tion, sedating medications, volume depletion, or gastrointesti- ing the cause of the ascites. The differential diagnosis of ascites
nal bleeding, and heralds worsening of liver function. is presented in Table 36.
Minimal manifestations of hepatic encephalopathy can be Cirrhosis is the cause in 85% of patients with ascites. The
detected on neuropsychiatric testing, although overt changes management of ascites involves a sodium-restricted diet
of hepatic encephalopathy; ranging from personality changes (<2 g/d) and diuretic therapy. typically with spironolactone and
to frank coma, are common in the setting of advanced cirrho- furosemide. Kidney function and electrolyte levels are typically
sis (Table 35). Asterixis, a flapping hand tremor. is typically checked 7 to 10 days after changes to diuretic regimens.
present in middle stages of hepatic encephalopathy. Asterixis Serial paracenteses can be performed if ascites does not
is typically absent in subclinical stages of hepatic encepha- resolve with sodium restriction and diuretic therapy. If more
lopathy and is lost as patients progress to later stages of hepatic than 5 L of ascitic fluid is removed at one time, supplemental
encephalopathy, including coma. 25% albumin, at a dose of 7 to 9 g/L of ascitic fluid removed,
Patients with hepatic encephalopathy should be moni- should be administered to prevent circulatory dysfunction
tored in an ICU. Diminishing consciousness should prompt after paracentesis.
consideration of intubation because airway-protective reflexes Patients with ascites should discontinue ACE inhibitors and
can be lost in the advanced stage of hepatic encephalopathy. NSAIDs. TIPS or intermittent paracentesis can be considered in
Measures to reduce intracranial hypertension include eleva- patients with refractory ascites and a low MELD score. Indwelling
tion of the head of the bed and increasing serum osmolality drains in ascites due to portal hypertension are associated with
with mannitol infusions. high rates of complications and are not recommended.
The treatment of hepatic encephalopathy is multifaceted.
Precipitating causes, such as infection or gastrointestinal Spontaneous Bacterial Peritonitis
bleeding, should be promptly treated. The nonabsorbed Spontaneous bacterial peritonitis (SBP) is an infection of
disaccharide lactulose is typically administered to decrease ascitic fluid in the setting of portal hypertension. SBP has a

TABLE 35. Hepatic Encephalopathy Classification


Severity of Description Findings
Encephalopathy

Minimal Detected only on neuropsychiatric testing Few or absent clinical manifestations


Grade 1 Mild lack of awareness, sleep disturbances Oriented to time and place, but with psychomotor slowing
Grade 2 Lethargic, disoriented, and with personality change Disoriented to time, with asterixis
Grade 3 Somnolence, confusion, and significant disorientation Disoriented to time, place, and situation, with asterixis
Grade4 Coma Lack of response even to painful stimuli

Adapted with permission from Vilstrup H, Amodio P, Bajaj J, Cordoba J, Ferenci P. Mullen KD, et al. Hepatic encephalopathy in chronic liver disease: 2014 practice guideline by
the American Association for the Study of Liver Diseases and the European Association for the Study of the Liver. Hepatology. 2014;60:715-35. IPMID: 25042402] doi:10.1002/
hep.27210.

61
Disorders of the Liver

TABLE 36. Characteristics of Ascites


SAAG Ascitic Fluid Total Other Characteristics
Protein Level

Cirrhosis High• Lowe Clear, straw-colored


Heart failure High• Highd
Chylous ascites Lowb or high• Highd High triglycerides
SAAG can be high if caused by portal hypertension
SAAG is low if caused by lymphatic disruption
Tuberculosis Lowb Peritoneal biopsy may be required to confirm diagnosis of
tuberculosis
Malignancy Lowb Highd Positive cytology
Nephrotic syndrome Lowb Low< Proteinuria and edema

SAAG = serum-ascites albumin gradient


·'2:1.1 g/dL ( 11 g/L)

''<1.1 g/dL(11 g/L)

'<2.5 g/dL(25 g/L)


0 2:2.5 g/dl (25 g/L)

high mortality rate, and patients who develop SBP should be creatinine of at least 0.3 mg/dL (26.5 µmol/L) and/or ~50%
CJ considered for liver transplantation. SBP can present with from baseline within 48 hours, bland urinalysis, normal kidney
CONT. tiever, abdomma . and kid ney ct·1sease, but its
. I pam, . presence ultrasound, and exclusion of other causes of acute kidney
should be considered in any patient with ascites who experi- injury. Often patients also have low fractional excretion of
ences a decline In condition. Patients with very low total sodium and oliguria. Type 2 HRS is characterized by a more
protein levels in their ascitic fluid (<LS g/dL [15 g/ L]) in gradual decline in kidney function associated with refractory
conjunction with hyponatremia. kidney dysfunction, or CTP ascites.
class B or C cirrhosis are at high risk for development of SBP Serum creatinine level is one of the most important
and should be given long- term primary prophylaxis with a predictors of death in patients with cirrhosis, and a rapid
fluoroquinolone antibiotic (for example, norfloxacin or rise should prompt evaluation for infections and causes of
ciprofloxacin). hypovolemia.
The diagnosis ofSBP is established by an ascitic-fluid neu- Treatment of HRS involves withdrawal of diuretics, vol-
trophil count of 250/µL or higher on diagnostic paracentesis. ume expansion. with intravenous albumin, and vasoconstric-
Bacterial culture of ascitic fluid should also be obtained. Prompt tors. Treatment may also include midodrine and octreotide
initiation of therapy with a third-generation cephalosporin (for with intravenous albumin to raise mean arterial pressure and
example, cefotaxime) is the initial treatment of SBP. If patients improve kidney perfusion. Hemodialysis while awaiting liver
have kidney dysfunction, or significant hepatic dysfunction as transplantation may be required for patients whose kidney
measured by a serum bilirubin level greater than 4 mg/dL function does not improve with therapy.
(68 µmol/L), adjunctive therapy with albumin (LS g/kg body
weight on day 1. as well as l g/kg on day 3) should be adminis- Hepatopulmonary Syndrome
tered; such treatment has a demonstrated survival benefit. Hepatopulmonary syndrome is characterized by dilation of
Follow-up paracentesis to demonstrate improvement in intrapulmonary vessels in patients with portal hypertension,
inflammation can be performed if clinical improvement is not resulting in right-to-left shunting of blood and hypoxemia.
obvious. Indefinite secondary prophylactic therapy should he Features of hepatopulmonary syndrome include orthodeoxia
offered to patients after resolution of SBP and typically con- (worsening oxygen saturation while upright) and platypnea
sists of once-daily fluoroquinolone therapy. (worsening sense of dyspnea when upright). The diagnosis
should be suspected in patients with portal hypertension who
Hepatorenal Syndrome have symptoms of dyspnea and evidence of hypoxia. Findings
Hepatorenal syndrome (HRS) occurs in the setting of portal of intrapulmonary shunting with agitated saline administra-
hypertension and results from a reduction in renal blood flow tion during echocardiography confirm the diagnosis.
during simultaneous dilatation of the splanchnic vasculature. Macroaggregated albumin perfusion studies can also aid in
Most patients who develop HRS have cirrhosis, although alco- diagnosis. Hepatopulmonary syndrome is initially treated
holic hepatitis and acute liver failure are also associated with with supplemental oxygen, but it is uniformly fatal without
this syndrome. Type l HRS is characterized by a rise in serum liver transplantation.

62
Disorders of the Liver

Cl Portopulmonary Hypertension
Patients with cirrhosis and portal hypertension who present
cirrhosis, and postvaccination immunity should be assessed.
Patients with chronic liver disease should receive annual influ-
CONT. wit• h dyspnea on exert10n. shou Id be suspected of having. enza vaccination and recommended pneumococcal vaccina-
portopulmonary hypertension. While less common than tions. Other vaccinations should be given based on· cunent
hepatopulmonary syndrome, portopulmonary hypertension recommendations (see MKSAP 18 General Internal Medicine).
is a complication of advanced cirrhosis with a high mortality Patients v.rith cirrhosis should be counseled to avoid the
rate. Echocardiography demonstrates right ventricular systolic use of alcohol. Sedating medications such as opiates and ben-
pressures greater than SO mm Hg, which should prompt right- zodiazepines should be avoided because they can precipitate
heart catheterization to confirm the diagnosis. Although por- symptoms of hepatic encephalopathy. Raw shellfish can trans-
topulmonary hypertension was formerly considered a mit Vibrio uulnificus, which can be fatal in patients with cir-
contraindication to liver transplantation, patients with pre- rhosis; therefore, patients with cirrhosis should be counseled
served right ventricular function who can attain a mean pul - to avoid the consumption of raw shellfish.
monary artery pressure of less than 35 mm Hg with the use of Patients with cirrhosis should be screened for osteoporo-
vasodilator therapies can benefit from liver transplantation. sis. Cholestasis results in decreased absorption of vitamin D,
Prostacyclin analogues. phosphodiesterase inhibitors, and and all causes of cirrhosis can be associated with osteoporosis.
endothelin receptor antagonists have been used to successfully The cause of osteoporosis in patients with cirrhosis is multi-
treat portopulmonary hypertension. Cl factorial and is associated with jaundice, hypogonadism, and
decreased levels of insulin-like growth factor that are common
KEY POINTS
in cirrhosis. Patients with osteoporosis should be treated with
• Upper endoscopy should be performed on all patients bisphosphonates. Patients with esophageal varices should
with cirrhosis to assess for the presence of varices ; pri- receive intravenous rather than oral bisphosphonates. Patients
mary prophylaxis to prevent bleeding includes nonse- with identified osteopenia should be treated with supplemen-
lective P-blocker therapy (propranolol, nadolol, or tal calcium plus vitamin D. Weight-bearing exercise should be
carvedilol) or endoscopic variceal ligation. encouraged in all patients with cirrhosis.
• Precipitating causes of hepatic encephalopathy may Protein-calorie malnutrition is common in patients with
include infection, sedating medications, volume deple- cirrhosis. Protein consumption is essential to prevent the cata-
tion, or gastrointestinal bleeding; treatment involves bolic effects of chronic liver disease. Daily protein intake of
addressing the precipitating cause and lactulose. 1.5 g/kg body weight should be administered to patients with
HVC • Ultrasound is the most effective means of ascertaining chronic liver disease.
the presence of ascites; the serum-ascites albumin gra- Medications that are metabolized by the liver may require
dient and total protein levels in ascitic fluid assist in dosage adjustments depending on the severity of the cirrhosis.
determining the cause of ascites. Patients with advanced cirrhosis (CTP class B or C) often
require dosage adjustments, whereas patients with well-
• The management ofascites involves a sodium-restricted
compensated cirrhosis (CTP class A) may not.
diet and diuretic therapy with spironolactone.
KEY POINTS
• An ascitic-fluid neutrophil count of 250/µ Lor greater
confirms the diagnosis of spontaneous bacterial perito- • Patients with cirrhosis should avoid alcohol. sedating
nitis; patients who develop this condition have a high drugs including opioids, and raw shellfish.
mortality rate and should be considered for liver • Patients with cirrhosis should be screened for osteopo-
transplantation. rosis; confirmed osteoporosis should be treated with
• Hepatopulmonary syndrome should be suspected in either oral or intervenous bisphosphonates.
patients with portal hypertension who have symptoms • Patients with chronic liver disease should be vaccinated
of dyspnea and evidence of hypoxia; the diagnosis is against hepatitis A and B vimses and should receive
confirmed by demonstrating intrapulmonary shunting annual influenza vaccination and recommended pneu-
with agitated saline administration during mococcal vaccinations.
echocardiography.

Hepatic Tumors, Cysts,


Health Care Maintenance in and Abscesses
Patients with Chronic Liver Disease Benign liver masses are typically discovered incidentally on
Patients with chronic liver disease are at risk for severe com- abdominal imaging. These findings are usually asymptomatic,
plications if they develop acute hepatitis and should be vacci- with the exception of liver abscesses or lesions greater than
nated against HAV and HBV if they are not immune. 5 cm in size. Most liver lesions can be characterized and
Nonresponse to vaccinations is more common in patients with diagnosed noninvasively with CT or MRI. Percutaneous

63
Disorders of the Liver

fine-needle aspiration of lesions is reserved for cases in which with hepatic adenomas are at increased risk for mallgnant
imaging is nondiagnostic and in which the Information transformation, and resection ls recommended.
obtained will prompt a change in management. Risks associ-
ated with liver biopsy, including a l'X, risk for serious compli- Focal Nodular Hyperplasia
cation, such as bleeding or perforation, and a 1-in-10,000 risk Focal nodular hyperplasia is the most common benign liver
for death, need to be considered. tumor and a frequent incidental finding. It is caused by a con-
genital arterial anomaly leading to a focal area of regeneration
Hepatic Cysts that can cause the formation of a stellate scar seen on CT or
Hepatic cysts are a common radiological finding on abdominal MRI. CT or MRI with and without contrast is recommended to
imaging. Simple cysts are smooth with thin walls and ane- confirm the diagnosis. Focal nodular hyperplasia does not
choic features on ultrasonography. Patients with polycystic have malignant potential or a risk for bleeding and does not
liver disease or autosomal dominant polycystic kidney disease require follow-up. In women with focal nodular hyperplasia
can present with numerous liver cysts. Asymptomatic cysts who continue to use oral contraceptives, there is limited evi-
are benign and require no follow-up. Large cysts, rarely pre- dence to reconunend liver ultrasonography every 2 to 3 years
senting with abdominal pain, can be treated with surgical to assess for growth. MRI with a hepatobiliary contrast agent
defenestration or, when surgery is contraindicated, with cyst can be used to distinguish between focal nodular hyperplasia
aspiration and sderotherapy. Rarely, polycystic liver disease and hepatic adenornas.
with extensive cystic liver involvement may lead to portal
hypertension and /or malnutrition. Obstruction of the portal Hepatic Hemangiomas
vein or inferior vena cava can occur due to mass effect. Options
Hepatic hemangiomas are common and more frequent in
for management include surgery, and if that is not technically
women. Up to 20'Yu of affected patients have multiple heman-
feasible, liver transplantation. Cystadenomas with thick,
giomas. In most cases, symptoms that might be ascribed to
irregular walls can be distinguished from simple cysts by
hemangioma are due to other causes. Kasabach-Merritt syn-
ultrasonography; they require surgical resection due to risk
drome is a rare syndrome that develops with particularly large
for malignancy.
hemangiomas that develop consumptive coagulopathy with
thrombocytopenia and sometimes disseminated intravascular
Hepatic Adenomas
coagulation. Hemangiomas do not have potential for malig-
Hepatic adenomas are rare liver neoplasms. Despite the
nancy, and spontaneous bleeding Is rare. The diagnosis should
benign nature of these lesions, adenomas that are greater
be confirmed with MRI or CT with contrast, which typically
than S cm in size have potential for risk for hemorrhage or
shows peripheral nodular enhancement and progressive cen-
malignant transformation. Hepatic adenomas are associated
tripetal fill-in. Biopsies should be avoided due to bleeding risk.
with oral contraception use and occur eight times more fre-
Hepatic hemangiomas are benign lesions that do not require
quently in women than in men. Other risk factors include
intervention or follow-up except in the rare instance when
androgen treatment, type 1 and 3 glycogen storage disease,
they cause symptoms.
and obesity. Increasingly, hepatic adenoma is an incidental
finding on abdominal ultrasound, CT, or MRI. Hepatic adeno-
mas can be differentiated from focal nodular hyperplasia Hepatic Abscesses [::J
using MRI with gadobenate dimeglurnine or gadoxetate diso- Pyogenic Liver Abscesses
dium contrast. Biopsy is indicated when there is diagnostic Pyogenic liver abscesses are complications of biliary tract
uncertainty but may increase the risk for bleeding; therefore, infections or portal venous spread of intra-abdominal infec-
it should only be performed if the diagnosis will result in a tions such as diverticulitis. Patients with pyogenic liver
meaningful change in management. Hepatic adenomas S cm abscesses present with fever, right-upper-quadrant abdominal
in size or smaller can be managed with serial imaging every 6 pain, and malaise. Liver abscesses are typically polymicrobial,
months for a 2-year period. For patients with hepatic adeno- and the diagnosis is confirmed by radiologically guided aspira-
mas larger than s cm in size, the risk for hemorrhage or tion. Small abscesses (<3 cm) can be successfully treated
malignant transformation is elevated and surgical resection by administration of broad-spectrum antibiotics. Larger
should be considered. abscesses are treated by aspiration or longer-term percutane-
Oral contraceptives should be discontinued in all patients ous tube drainage in addition to broad-spectrum antibiotics.
with hepatic adenomas with follow-up CT or MRI at 6- to The success of radiologically guided aspiration and tube drain-
12-month intervals to confirm stability or regression in the siz.e age has resulted in only rare need for surgical excision of
of the lesion. The duration of surveillance depends on subse- pyogenic hepatic abscesses.
quent Imaging findings. Biopsy to risk-stratify hepatic adeno-
mas can be considered on a case-by-case basis. Hepatic Amebic Liver Abscesses
adenomas with ~-catenin activation are at higher risk for Amebic abscesses of the liver are found in developing areas of
malignant transformation into hepatocellular carcinoma. Men the world or in migrants from countries in which amebiasis is

64
Disorders of the Liver

e_ndemic. Intestinal infection with amoebae can result in inva-


Cl
CONT.
s10n of the portal vein and migration to the liver. Typical symp-
t oms o f ameb"1c 1·1ver abscesses me
. Iude right-upper-quadrant
.
KEV POINTS
• Asymptomatic hepatic cysts are benign and require no HVC
follow-up.
abdominal pain and fever. Hepatic imaging and serological
testing make the diagnosis of amebic liver abscess. Treatment • Focal nodular hyperplasia, a common incidental find- HVC
of the hepatic infection with metronidazole or tinidazole ing, does not have malignant potential or a risk for
should be accompanied by eradication of the coexistent intes- bleeding, and does not require follow-up.
tinal infection with paromomycin. CJ • Hepatic adenomas s cm in size or smaller can be man-
aged with serial imaging; hepatic adenomas larger than
S cm in size should be considered for surgical resection.
Hepatocellular Carcinoma
• Patients with cirrhosis and high-risk patients with hep-
Hepatocellular carcinoma is the most common liver tumor atitis B virus infection require hepatocellular carcinoma
arising in patients with cirrhosis, the fifth most common surveillance using liver ultrasound with or without
cause of cancer worldwide, and the second most common a -fetoprotein measurement every 6 months.
cause of cancer-related deaths worldwide. Approximately 85%
of patients with hepatocellular carcinoma have cirrhosis.
Patients with cirrhosis, regardless of the cause, require
surveillance liver ultrasound with or without a -fetoprotein
Liver Transplantation CJ
measurement every 6 months, which leads to early diagnosis, Referral for liver transplantation is indicated in patients whose
increased rate of curative treatment, and improved 3-year MELD score is 15 or greater because liver transplantation pro-
survival rates. Patients with HBV infection can develop hepa- vides a survival advantage in these patients. Patients with
tocellular carcinoma even in the absence of cirrhosis; there- decompensated cirrhosis, including ascites, esophageal
fore, surveillance for hepatocellular carcinoma is variceal bleeding, hepatic encephalopathy, jaundice, or hepa-
recommended even in the absence of cirrhosis in individuals tocellular carcinoma, should also be considered for referral.
at high risk (individuals with HBV who are from sub-Saharan Patients should generally abstain from alcohol for at least 6
Africa and older than age 20 years, Asian men older than age months, although transplant centers may differ in their
40 years, and Asian women older than age so years). The diag- requirements for sobriety duration and chemical-dependency
nosis of hepatocellular carcinoma can usually be made with- treatment. Other factors important in candidate selection are
out biopsy using multiphase contrast-enhanced CT or MRI in adequate social support, adherence to treatment, and absence
patients with cirrhosis and a lesion at least 1 cm in size. Biopsy of significant cardiopulmonary, psychiatric, and active infec-
may be needed for lesions that are large, growing, or indeter- tious diseases.
minant on imaging. Appropriate candidates are placed on the national waiting
Many specialty societies have issued guidelines for the List, with the highest priority given to patients with acute liver
management of hepatocellular carcinoma. Patients with CTP failure and high MELD-Na scores. In some conditions, the most
class A cirrhosis, without significant portal hypertension or common being hepatocellular carcinoma, exception points are
jaundice, and a singular lesions cm in size or smaller should be added to the MELD score, resulting in increased scores every 3
considered for resection. Liver transplantation should be con- months. Other conditions eligible for MELD exception points
sidered in patients with hepatocellular carcinoma within the include portopulmonary hypertension, hepatopulmonary syn-
Milan criteria and with portal hypertension or jaundice. The drome, familial amyloidotic polyneuropathy, primary hyperox-
Milan criteria are as follows: up to three liver lesions 3 cm in aluria, cystic fibrosis, and cholangiocarcinoma.
size or smaller or one lesion 5 cm in size or smaller, without The average 1- and 5-year survival rates after liver trans-
macrovascular invasion, and without extrahepatic spread. plantation are 92% and 75 % to 85%, respectively. Recipients
Patients who are not surgical or liver transplantation candi- require lifelong immunosuppression, most commonly using
dates should be considered for locoregional therapy in the tacrolimus, and less often, cyclosporine. Both drugs are
absence of macrovascular invasion (tumor thrombus) or extra- metabolized by cytochrome P450 3A isozymes and pose a risk
hepatic spread. Locoregional therapies include radiofrequency for drug-drug interactions. Recipients are at increased risk for
ablation, microwave ablation, transarterial chemoemboliza- developing diabetes, hypertension, hyperlipidemia, chronic
tion, and transarterial radioembolization. Decisions about kidney disease, and malignancy secondary to immunosup-
locoregional therapy are increasingly made through multidis- pressive medications. CJ
ciplinary tumor boards. Sorafenib is used for advanced hepato- KEV POINT
cellular carcinoma, which includes cases with macrovascular
• Referral to a transplant center is indicated for patients
invasion or extrahepatic spread. In 2017, the FDA approved two
with acute liver failure or for patients with cirrhosis
drugs for second-line therapy: regorafenib, an oral multi-kinase
with a Model for End-Stage Liver Disease score oflS or
inhibitor, and nivolumab, an intravenous humanized monoclo-
greater or decompensated cirrhosis.
nal antibody against the programmed cell death receptor.

65
Disorders of the Liver

Pregnancy-Related Liver Diseases The most serious pregnancy-related liver diseases occur
in the third trimester of pregnancy and are associated with
c:J
During pregnancy, physiological changes occur that mimic
high rates of maternal and fetal mortality. In these condi -
chronic liver disease. Altered hormonal states and increasing
tions, the only definitive therapy is delivery of the fetus.
circulatory volume can lead to lower-extremity edema, palmar
HELLP (Hemolysis, Elevated Liver enzymes, and Low
erythema, and spider angiomata.
Platelets) syndrome is a severe complication ofpreeclampsia.
In pregnant women with preexisting chronic live1- dis-
HELLP typically presents with nonspecific symptoms such as
ease, HBV and HCV can he transmitted to the newborn. There
abdominal pain, nausea with vomiting, pruritus. and jaun-
are no specific steps that can be taken to eliminate the risk for
dice. Rates of maternal and fetal morbidity and mortality are
vertical transmission ofHCV; however, vertical transmission is
high. Patients with HELLP syndrome should, therefore. be
relatively rare, with less than 5% of women with HCV viremia
man~lged in high -risk obstetrical units. Blood pressure. fluid
transmitting the virus to their children.
and electrolytes. kidney function , and coagulopathy may
HBV carries a higher risk for vertical transmission.
require careful management in the perinatal state. Although
Among women with replicating HBV infection, the vertical
delivery is the definitive therapy for HELLP syndrome, the
transmission rate can be as high as 90%. Pregnant women
maternal condition may continue to worsen in the immedi -
with an HBV DNA level greater than 200,000 IU/mL between
ate postpartum period. Resolution is typically seen within
gestational weeks 24 and 28 should start oral antiviral treat-
days after delivery. Rarely, liver transplantation may be
ment to prevent vertical transmission. Oral antiviral agents
required if liver recovery is not seen. HELLP can reoccur in as
approved in pregnancy include lamivudine, telb!vudine, and
many as 25% of subsequent pregnancies.
tenofovir. Breastfeeding is not contraindicated during treat-
Acute fatty liver of pregnancy is a rare but serious condi -
ment. Passive immunization with HBV immune globulin and
tion that also occurs late in pregnancy. It presents with symp-
active HBV vaccination should be administered to newborns
toms similar to those of HELLP syndrome. Indicators of liver
within 12 hours of delivery. These measures can reduce verti-
failure, including hypoglycemia and coagulopathy, are often
cal transmission rates by 95%. Administration of antiviral
worse in acute fatty liver of pregnancy than in HELLP syn-
medications in the third trimester to pregnant women with
drome; both conditions require close monitoring to prevent
high HBV viremia can further reduce the risk for vertical
adverse maternal and fetal outcomes. Acute fatty liver of preg-
transmission.
nancy is a potential cause of acute liver failure. Affected
Women with autoimmune hepatitis who become preg-
patients may require transfer to a liver transplant center.
nant typically continue the use ofprednisone and/or azathio-
Prompt delivery of the fetus once the diagnosis is recognized
prine, which are generally felt to be safe during pregnancy.
typically results in improvement of the mother's medical con-
Several diseases of the liver are unique to pregnancy and
dition within 48 to 72 hours. Acute fatty liver of pregnancy
can affect the health of both the mother and fetus. During the
can reoccur in subsequent pregnancies. It is also associated
first trimester, hyperemesis gravidarum occurs when pro-
with long-chain 3-hydroxyacyl CoA dehydrogenase defi-
longed, intractable vomiting results In fluid and electrolyte
ciency. and affected women and their offspring should be
abnormalities. Elevated hepatic transaminase levels are seen
screened for this deficiency. CJ
in up to 50% of cases of hyperemesis gravidarum, but jaundice
is rare. Laboratory abnormalities typically resolve when vom- KEY POINTS
iting abates. Pyridoxine and antiemetic medications can • Measures to reduce vertical transmission of hepatitis B
resolve symptoms ofhyperemesis gravidarum. virus include administration of hepatitis B virus
Intrahepatic cholestasis of pregnancy is presumed to immune globulin to newborns and immediate vaccina-
result from cholestatic effects of increased levels ofpregnancy- tion of newborns.
related honnones. Increased risk for this condition can be • The most serious liver complications of pregnancy
seen in women of South American descent, twin pregnancies, occur in the third trimester and include HELLP
and in women with a history of liver disease. Intrahepatic (Hemolysis, Elevated Liver enzymes, and Low Platelets)
cholestasis of pregnancy typically presents in the second to syndrome and acute fatty liver of pregnancy; both are
third trimesters of pregnancy. Symptoms include pruritus in managed in high-risk obstetrical units and with early
most patients and jaundice in 10% to 25"/o of patients. Serum delivery.
bile acid levels are elevated and help to establish the diagnosis.
Fetal complications, including placental insufficiency. prema-
ture labor, and sudden fetal death, are more common in intra-
Vascular Diseases of the Liver
hepatic cholestasis of pregnancy. Ursodeoxycholic acid
produces relief of prurftus and Improves fetal outcomes. Portal Vein Thrombosis
Increased fetal mortality is noted to occur late in gestation. Portal vein thrombosis is common in patients with decompen-
Delivery ts typically induced at 36 to 38 weeks of gestation in sated cirrhosis and is a consequence of poor flow through
women with proven disease. the portal veins. The diagnosis Is typically established by

66
Disorders of the Gallbladder and Bile Ducts

abdominal Doppler ultrasonography or contrast-enhanced CT calcium bilirubinate, can form in patients with chronic hemo-
or MRI. Hypercoagulable states are typically not present in lytic disease states, ineffective erythropoiesis, ileal disease such
patients with cirrhosis who develop portal vein thrombosis. as Crohn disease, and cirrhosis. Brown pigment stones are
Chronic portal vein thrombosis is typically asymptomatic and composed of unconjugated bilirubin and varying amounts of
does not require anticoagulation therapy. Acute portal vein other substances, such as cholesterol, but also contain bacteria.
thrombosis can be symptomatic with the development of These are typically found in patients with biliary stasis and bac-
ascites or variceal hemorrhage. Anticoagulation can be consid- terial biliary infection, as can be seen in chronic biliary
ered if there is concern that portal vein thrombosis has extended obstruction.
into the supe1ior mesenteric vein risking intestinal ischemia. Gallstones are commonly discovered when patients
undergo abdominal imaging for unrelated reasons. Ultrasound
Budd-Chiari Syndrome and CT can identify gallstones, whereas plain radiography
Budd-Chiari syndrome describes any disease process that identifies gallstones uncommonly.
obstructs the normal outflow of blood from the liver, usually Gallstones that are found incidentally do not typically
as thrombosis of the hepatic veins. Causes vary, but it may cause any symptoms, and cholecystectomy is generally not
occasionally be associated with hypercoagulable states such as recommended because most stones remain asymptomatic.
in patients with a myeloproliferative neoplasm, pregnancy, Cholecystectomy is indicated in asymptomatic patients at high
oral contraceptive use, IBD, or inherited thrombophilias. risk for gallbladder cancer, including patients with gallstones
Underlying malignancy, especially hepatocellular carcinoma, larger than 3 cm in size, porcelain gallbladder (intramural
must be considered. Typical symptoms of Budd-Chiari syn- calcification of the gallbladder wall), gallbladder adenomas or
drome include hepatomegaly, ascites, and right-upper-quad- polyps larger than 1 cm in size, or anomaly of pancreatic ductal
rant abdominal pain. Budd-Chiari syndrome is typically diag- drainage.
nosed by ultrasound with Doppler evaluation in the
KEY POINT
appropriate clinical setting. The caudate lobe of the liver is
hypertrophied due to the presence of venous outflow channels • Gallstones that are found incidentally do not typically HVC
distinct from the hepatic veins. Long-term anticoagulation is cause any symptoms, and cholecystectomy is generally
required in patients with Budd-Clliari syndrome, although not recommended because most stones remain
bleeding risks are significant in patients with acute or chronic asymptomatic.
liver disease, portal hypertension, and esophageal varices.
Angioplasty of the hepatic veins and/or TIPS placement can be Biliary Colic
used to reestablish adequate hepatic venous drainage. If liver
failure develops, liver transplantation should be considered. Biliary colic pain results from stimulation of the gallbladder in
the presence of an obstructive cystic duct from gallstones or
KEY POINTS sludge. This pain is typically of relatively acute onset, is fairly
• Portal vein thrombosis is common in patients with severe and steady, and is located in the right upper quadrant or
decompensated cirrhosis and is a consequence of poor epigastrium. Pain may radiate to the right scapula and can be
flow through the portal veins. associated with nausea, vomiting, and diaphoresis, lasting 2 to
• The classic presentation of Budd -Chiari syndrome 6 hours. This symptom complex may be precipitated by eating
includes hepatomegaly, ascites. and right-upper-quad- a fatty meal, which causes gallbladder contraction. Patients
rant abdominal pain. with unrelenting right-upper-quadrant or epigastric pain gen-
erally do not have biliary colic.
Patients with typical biliary colic symptoms and gall-
Disorders of the stones on imaging should undergo cholecystectomy, as the risk
for complications from gallstones is approximately 2% to 3'Yo
Gallbladder and Bile Ducts per year. Complications include choledocholithiasis; cholangi-
tis, and pancreatitis. Patients with atypical symptoms should
Asymptomatic Gallstones be evaluated for other causes.
Gallstones can be characterized as either cholesterol stones or
KEY POINT
pigment stones, which are black or brown. Cholesterol stones
are the result of supersaturation of the bile with cholesterol; • Patients with typical biliary colic symptoms and gall-
they account for approximately 75% of cases in the United stones on imaging should undergo cholecystectomy.
States. Risk factors for cholesterol cholelithiasis include older
age, female sex (twice as likely as in men), American Indian
ethnicity, Western diet, pregnancy, rapid weight loss, obesity, Acute Cholecystitis CJ
total parenteral nutrttion, and drugs such as estrogen and soma- Acute cholecystitls develops in the setting of cystic-duct
tostatin analogues. Black pigment stones, usually composed of obstruction and gallbladder inflammation. In many patients,

67
Disorders of the Gallbladder and Bile Ducts

infection of the gallbladder ensues. Patients typically present or gallbladder-wall pneumatosls in the absence of calculi. In
Cl with severe right-upper-quadrant or epigastric pain lasting this setting, the gallbladder may not be visualized on a hepa-
CONT. longer than 6 hours. accompanied by fever and localized peri- tobillary iminodiacetic acid scan.
toneal signs in the right upper quadrant. A positive Murphy Treatment consists of empiric intravenous antibiotics to
sign (arrested inspiration upon contact of the gallbladder wall cover enteric bacteria and cholecystectomy. A cholecystostomy
with the examiner's fingers) may be seen on physical exami- tube may be needed if the patient is unstable or a poor candi-
nation. Laboratory studies may show leukocytosis; liver chem- date for surgery. The role of endoscopic gallbladder drainage is
istries are not usually elevated in uncomplicated acute evolving. The mortality rate for untreated acalculous cholecys-
cholecystitis. The diagnosis can be made by ultrasonography titis is as high as 75%. CJ
showing gallbladder wall thickening and/or edema and a
KEY POINTS
sonographic Murphy sign. A hepatobiliary iminodiacetic acid
scan can be used when the diagnosis .is unclear, such as when • Acalculous cholecystitis typically occurs in critically ill
symptoms suggest cholecystitis but the sonogram is normal. patients and is a result of gallbladder ischemia that can
Treatment includes pain control, intravenous antibiotics be complicated by enteric bacterial infection.
with gram-negative and anaerobic coverage (monotherapy • Left untreated, the mortality rate for acalculous chole-
with a ~ - lactam or ~- lactamase. or combination therapy cystitis is as high as 75%.
with a third-generation cephalosporin plus metronidazole),
and cholecystectomy. Timing of cholecystectomy depends on
the patient's surgical risk and response to antibiotics. An Functional Gallbladder Disorder
emergency operation is necessary in cases of suspected gall- The Rome 4 criteria for gallbladder disorders suggest that eval-
bladder perforation or emphysematous cholecystitis (infec- uation for functional gallbladder disorder is appropriate in
tion of the gallbladder wall with gas-forming organisms such patients with typical biliary pain in the absence of gallstones or
as Clostridium perfringens). For patients with low surgical structural pathology. Typical biliary pain Is located in the epi-
risk. cholecystectomy, preferably laparoscopic, should be gastrium and/or right upper quadrant; is intermittent; lasts
performed during the initial hospitalization. Patients who ;o:30 minutes; crescendos and then plateaus; is severe enough to
are deemed high-risk for surgery and who respond to antibi- interrupt normal activities or results in an emergency depart-
otics can be reassessed at a later time to determine if their ment visit; and Is not relieved by bowel movements, positional
surgical risk has decreased. In patients who are not good changes, or acid suppression. Supportive features include a
candidates for cholecystectomy and do not respond to anti- gallbladder ejection fraction of less than 40% measured by
biotics, percutaneous cholecystostomy tube or endoscopic cholecystokinin-stimulated cholescintigraphy and normal
drainage can be pursued. CJ liver chemistry tests and pancreatic enzymes. Other causes of
KEY POINTS upper abdominal discomfort, including functional dyspepsia,
HVC • The diagnosis of acute cholecystitis can be made by peptic ulcer disease, and angina, should be carefully consid-
ultrasonography showing gallbladder wall thickening ered. Gallstones should be excluded by transabdorninal ultra-
and/or edema and sonographic Murphy sign. sonography. Endoscopic ultrasonography may be helpful ifthe
transabdominal ultrasound is normal. If no other cause of pain
• Treatment of acute cholecystltis includes pain control, is found and the gallbladder ejection fraction is less than 40%,
intravenous antibiotics with gram- negative and anaero- cholecystectomy can be considered. The data supporting chol-
bic coverage, and cholecystectomy during the initial ecystokinin-stlmulated cholescintigraphy is poor. ln clinical
hospitalization in good candidates for surgery. practice, it is not uncommon to see patients who have met
these criteria and undergo cholecystectomy, but continue to
have the same upper-abdominal symptoms postoperatively.
CJ Acalculous Cholecystitis
Acalculous cholecystitis typically occurs in critically ill patients
and is a result of gallbladder ischemia that can be complicated
Common Bile Duct Stones
by enteric bacterial infection. Risk factors for this condition and Cholangitis
include cardiac and aortic surgery, sepsis, bums, and vasculi- Stones in the common bile duct are a leading cause of obstruc-
tis. The presentation depends on whether the patient is alert or tive jaundice. Complications in patients with common bile
sedated and mechanically ventilated. In alert patients, the duct stones are more common than in those with gallstones,
presentation is pain, as seen in cholecystitis related to gall- but 20% of patients spontaneously pass the stones, and less
stones. In sedated or mechanically ventilated patients, it may than 50% of patients develop symptoms. Symptomatic com-
present with leukocytosis, jaundice, and sepsis. The diagnosis mon bile duct stones present with jaundice, abdominal pain,
is made using ultrasonography, which may show gallbladder or pruritus due to obstruction of bile flow. Abdominal imag-
wall thickening, pericholecystic fluid, gallbladder distention, ing, including CT, ultrasonography, or magnetic resonance

68
Disorders of the Gallbladder and Bile Ducts

cholangiopancreatography (MRCP), can detect dilation of


extrahepatic and intrahepatic bile ducts, indicating the pres-
Gallbladder Cancer
ence of stones. Endoscopic retrograde cholangiopancreatog- Gallbladder cancer is the most common biliary cancer in the
raphy (ERCP) is the preferred therapeutic method for reliev- United States, but it is rare, with 3700 new cases per year. Risk
ing obstruction of the common bile duct by stones. factors include female sex, ethnicity or race (American Indian,
The presence of cholangitis is heralded by the onset of the Alaskan native, black), cholelithiasis, gallbladder polyps, por-
CJ Charcot triad (fever, jaundice, and right-upper-quadrant
abdominal pain) or Reynold pentad (Charcot triad plus hypo-
celain gallbladder, anomalous pancreaticobiliary junction,
and obesity. The gallbladder serves as a reservoir for Salmonella
typhi in chronically infected patients, and patients with this
tension and altered mental status). Cholangitis is potentially
life-threatening, and antibiotic therapy targeting gram-nega- organism are at higher risk for gallbladder cancer.
tive Enterobacteriaceae should be administered. Identified Presenting symptoms may include right-upper-quadrant
common bile duct stones should be removed urgently with pain, nausea, vomiting, weight loss, or jaundice in advanced
ERCP in patients with cholangitis, after which elective chole- cancers, and biliary colic in early cancers. Gallbladder cancer
cystectomy should be performed within 6 weeks to reduce the should be suspected if an enhancing gallbladder mass is seen
risk for complications. In patients who are not surgical candi- on CT or MRI.
dates, endoscopic sphincterotomy can be performed to facili- Early gallbladder cancer is most commonly diagnosed
tate the passage of additional common bile duct stones. CJ incidentally at the time of cholecystectomy performed for bil-
iary colic. Incidental tumors found to invade the lamina pro-
KEY POINTS pria (stage Tla) do not require further treatment, whereas
• Endoscopic retrograde cholangiopancreatography is the more advanced lesions require extended cholecystectomy.
preferred therapeutic method for relieving obstruction The treatment of choice for gallbladder cancer is surgery.
due to a common bile duct stone. Treatment for unresectable disease can include chemotherapy
• Cholangitis (heralded by the onset of fever, jaundice, with or without radiation, or palliative care.
and right-upper-quadrant abdominal pain) is poten- Prophylactic cholecystectomy is recommended for
tially life-threatening; antibiotic therapy targeting patients with an anomalous pancreaticobiliary junction, gall-
gram-negative Enterobacteriaceae should be adminis- bladder polyps 1 cm in size or larger, or gallbladder polyp(s) in
tered, and identified common bile duct stones should the presence of biliary colic or gallstones; it is also recom-
be removed urgently. mended for patients with primruy sclerosing cholangitis and a
gallbladder polyp larger than 8 mm in size. Prophylactic chol-
ecystectomy can be considered for porcelain gallbladder or for
Gallbladder Polyps gallstones larger than 3 cm in size.
Gallbladder polyps, usually incidental findings, can be seen on Cholangiocarcinoma
1% to 5% of gallbladder ultrasounds. They can be neoplastic, Cholangiocarcinoma, although rare, is an increasingly recog-
such as an adenoma, or nonneoplastic, such as cholesterol nized malignancy. It is classified as (1) intrahepatic- seen in the
polyps or adenomyomas. The best predictor of a malignant or smaller bile ducts entirely within the liver parenchyma; (2)
premalignant lesion is size of the gallbladder polyp, with pol- hilar-most commonly arising from confluence of right and left
yps greater than I cm in size being more likely to be neoplastic. hepatic ducts (Klatskin tumor); or (3) distal-arising distal to
Gallbladder polyps associated with gallbladder stones or pri- the cystic-duct entrance. Risk factors include primary scleros-
mary sclerosing cholangitis are also more likely to be neoplas- ing cholangitis, choledochal cysts, liver flukes (Opisthorchis),
tic. Management of gallbladder polyps is outlined in Figure 32. exposure to thorium dioxide (contrast medium), and hepato-
KEY POINT lithiasis. Symptoms vary with tumor location and may include
• Gallbladder polyps greater than I cm in size and those jaundice, pain in the right upper quadrant, and constitutional
symptoms.
associated with gallbladder stones or primary sclerosing
cholangitis are more likely to he neoplastic. Intrahepatic, hilar, and distal cholangiocarcinomas can
occur in patients without liver disease. Intrahepatic

Gallbladder polyp sS mm Gallbladder polyp 6-9 mm Gallbladder polyp 2'1 cm Gallbladder polyp (any size)+
gallstones, biliary colic
or primary sclerosing cholangitis
+ gallbladder polyp >8 mm
Repeat ultrasound in 12 months Repeat ultrasound in 6 months Cholecystectomy
Cholecystectomy if t size then yearly
Cholecystectomy if t size
Cholecystectomy

FIGURE 3 2. Management of gallbladder polyps.

69
Gastrointestinal Bleeding

cholangiocarcinoma can complicate cirrhosis, and hilar chol- bleeding. is bleeding that does not appear to originate from the
angiocarcinoma can complicate primary sclerosing cholangitls. upper or lower gastrointestinal tract; it is relatively uncommon.
Diagnosis of intrahepatic cholanglocarcinoma requires
KEY POINT
imaging with CT or MRI, and usually a biopsy. An elevated CA
19-9 level is supportive but insufficient for diagnosis. First-line • Upper gastrointestinal bleeding is more common, more
therapy for intrahepatic cholangiocarcinoma is resection. severe, and has a higher mortality rate than lower gas-
Locoregional and/or systemic chemotherapy are appropriate trointestinal bleeding.
for patients who are not candidates for resection.
Diagnosis ofhilar cholangiocarcinoma can be challeng-
ing and is made by a combination of MRCP and ERCP. Upper Gastrointestinal Bleeding CJ
During ERCP. bile-duct brushings should be obtained for UGIB can present in various ways, including hematemesis,
cytologic examination and fluorescence in situ hybridization "coffee-ground" emesis, melena, or hematochezia. Hematemesis
testing. The latter test uses DNA probes to evaluate for gain is vomiting of bright red blood or clots. Coffee-ground emesis is
or loss of chromosomes or loci, which are often present in the vomiting of dark, digested blood. Melena is black, tarry stool
biliary cancer. An elevated CA 19- 9 level is supportive, but with a distinctive odor. Hematochezia is the passage of fresh
repeat ERCP is often required every 2 to 3 months to make blood or clots from the rectum.
the diagnosis. First-line therapy for hilar cholangiocarci-
noma is resection. Patients with obstructive jaundice may Causes
require ERCP with stent placement to allow for biliary drain- Common causes ofUGIB include peptic ulcer disease, gastroe-
age. Patients with unresectable hilar cholanglocarcinoma sophageal varices. and Mallory-Weiss tear. Peptic ulcer disease
smaller than 3 cm in size and without extrahepatic spread is the most common cause (50%), with most gastroduodenal
can be evaluated for liver transplantation at select centers ulcers caused by Hellcobacter pylori or NSAID use. Erosive
with neoadjuvant chemoradiation protocols. However, per- esophagitis is a common endoscopic finding but only rarely
cutaneous or transluminal biopsy of hilar cholangiocarcl- causes clinically important UGIB. Therefore, in a patient with
noma excludes a patient for liver transplantation due to the UGIB and erosive esophag!tis, alternative causes for the bleed-
risk for tumor seeding. ing should be excluded.
The preferred treatment for distal cholangiocarcinoma is Bleeding gastroesophageal varices typically occur In the
a Whipple resection. distal esophagus or proximal stomach in individuals with
Metastatic cholangiocarcinoma of any variety should be advanced liver disease. Bleeding risk of varices is proportional
treated with gemcitabine-cisplatin. to varix size.
The 5-year survival rate for patients with cholangiocarci- A Mallory-Weiss tear consists of a mucosal disruption at
noma (excluding liver transplant recipients), including those the gastroesophageal junction and typically forms after
who undergo resection, is 20% to 30"h.. repeated episodes of severe vomiting or retching.
KEY POINTS Less common causes of UGIB include Cameron erosions.
• Resection is first-line treatment for cholangiocarcinoma. Dieulafoy lesion, gastric antral vascular ectasia, aortoenteric
fistula, hemosuccus pancreaticus, hemobilia, and upper gas-
• Selected patients with unresectable hilar cholangio- trointestinal tumors (Table 37).
carcinoma smaller than 3 cm in size and without
extrahepatic spread may be candidates for liver
transplantation.
Evaluation
The initial step in the approach to UGlB is a risk assessment to
determine the severity of UGIB. Th Is risk assessment includes
the measurement of vital signs and reviewing patient factors.
Gastrointestinal Bleeding Tachycardia (pulse rate >100/min) , hypotension (systolic blood
pressure <100 mm Hg), age older than 60 years. and major
Overview comorbid medical conditions are all associated with increased
In the United States, gastrointestinal bleeding is a common gas- risk for rebleeding and death.
trointestinal cause of hospitalization. Upper gastrointestinal Findings of stigmata of chronic liver disease suggest a
bleeding (UGIB) is defined as bleeding from the esophagus, possible variceal source of bleeding.
stomach, or duodenum. The mortality rate for patients with
UGIB varies from 2% to 10% but is usually due to other factors Management
related to comorbid diseases. Lower gastrointestinal bleeding Patients with altered mental status, massive hematemesis, or
(LGIB) occurs from the colon or anorectum. It is less common, an increased risk for aspiration should undergo endotracheal
typically less severe, and has a lower mortality rate than UGlB. intubation. Hemoglobin levels should be measured. A restric-
Small-bowel bleeding, formerly called obscure gastrointestinal tive transfusion strategy is recommended and initiated when

70
Gastrointestinal Bleeding

TABLE 37. Less Common Causes of Upper Gastrointestinal Bleeding


Lesion Pathogenesis Presentation Treatment

Cameron erosion Mechanical trauma to mucosal Typically chronic GI bleeding Includes medical therapy with PPI
folds of hiatal hernia presenting as iron deficiency and iron, and surgical repair of
anemia hiatal hernia
Dieulafoy lesion Dilated, aberrant submucosal Included in differential diagnosis of Endoscopic
vessel recurrent, often massive b leeding
without clear source
Gastric antral Most idiopathic; associated with Acute bleeding or iron deficiency Endoscopic
vascular ectasia cirrhosis and systemic sclerosis anemia
Aortoenteric fist ula Direct communication between "Herald" bleed followed by Surgical
aorta and GI tract massive exsanguination
Hemosuccus Erosion of pancreatic pseudocyst Upper GI bleeding in setting of Mesenteric angiography with coil
pancreaticus or tumor into a vessel with pancreatic d isease embolization
bleeding into pancreatic duct
Hemobilia Bleeding from the hepatobiliary Triad of jaundice, biliary colic, and Angiography or surgical
tract often caused by arteriobiliary GI bleeding
fistula from trauma or liver biopsy
Upper GI tumors Benign or malignant neoplasms Slow or massive hemorrhage Palliative radiographic and
endoscopy for malignant tumors,
surgical resection for benign
tumors
Gl • gastrointestinal; PPI • proton pump inhibitor.

the hemoglobin level is below 7 g/dL (70 g/L) in hemodynami- gauge) are required with initiation of crystalloid fluids, either
Cl cally stable patients without preexisting cardiovascular dis- normal saline or lactated Ringer solution, to maintain ade-
CONT. ease. Patients with hypotension due to severe, ongoing UGIB quate blood pressure. See Disorders of the Liver for discussion
and those with concomitant cardiovascular disease should be ofvariceal bleeding. including prophylaxis.
transfused before the hemoglobin level decreases below 7 g/dL
(70 g/L) to prevent the decreases below 7 g/dL (70 g/L) that Pre-endoscopic Care
may occur with fluid resuscitation alone. Intravenous proton pump Inhibitor (PPI) therapy initiated
For variceal bleeding, resuscitative measures need to be before endoscopy decreases high-risk endoscopic stigmata
initiated with the goal of hemodynamic stabilization. Two seen (Figure 33 and Figure 34) but does not influence outcome.
large-bore peripheral intravenous catheters (minimum 18 Octreotide and antibiotics should be initiated if variceal

Fl GU RE 3 3. Left: Duodenal ulcer with non bleeding visible vessel (arrow) that is at high risk for re bleeding and must be treated endoscopically. Right: Active arterial
spurting (dotted arrow) from a duodenal ulcer (solid arrow). This lesion is at the highest risk for rebleeding and must be treated endoscopically.
Courtesy of Louis M.Wong Kee Song, MD, M•yo Cllni~

71
Gastrointestinal Bleeding

Glasgow-Blatchford score may be considered for early dis-


charge or outpatient treatment.

Endoscopic Evaluation and Treatment


Upper endoscopy is the primary diagnostic modality for evalu-
ating UGIB. For patients hospitalized with UGIB. endoscopy
should be performed within 24 hours of resuscitation; in those
with rapid bleeding or suspected variceal hemorrhage, It
should be done more emergently. The possibility of aortoen-
teric fistula should always be considered in patients who have
had previous aortic graft surgery and who present with gastro-
intestinal bleeding because aortoenteric fistula is life- threaten-
ing, with a mortality rate of50%even with surgical intervention.
When there is a high degree of suspicion for an aortoenteric
fistula, CT with intravenous contrast should be performed
before endoscopy or other types of gastrointestinal evaluation.
FIGURE 3 4. Duodenal ulcer with adherent clot (arrow) that is at risk for Endoscopy can determine the cause of bleeding and helps
rebleeding. This can be treated medically or by clot removal and endoscopic to risk-stratify the patient. Lesions at high risk for recurrent
therapy in addition to standard medical therapy. bleeding that require endoscopic treatment include: actively
Courtesy of l.ouis M.Wong K~ Song, MD. Mayo Clinic bleeding peptic ulcers, ulcers wlth nonbleeding visible vessels
(see Figure 33), and ulcers with adherent clots (see Figure 34).
An adherent clot should be irrigated with the intention of
. . , hemorrhage is suspected. Intravenous erythromycin given removing the clot; if clot removal is successful, the ulcer is
LA.I before endoscopy improves gastric visualization and decreases then considered low-risk for rebleeding. Lesions at low risk for
CONT. the need for repeat endoscopy, but it should be administered rebleeding (clean-based ulcers, ulcers with pigmented spots,
only when requested by the endoscopist, not routinely. and Mallory-Weiss tears) do not require endoscopic treatment
Nasogastric tube lavage is not required, as it has shown no (Figure 35). Most Mallory-Weiss tears (Figure 36) stop bleeding
evidence of clinical benefit. spontaneously. Endoscopic techniques such as injection ther-
Vitamin K and 4-factor prothrombin complex concen- apy. thermal devices, and endoclips can be used for actively
trate should be administered to patients on anticoagulation bleeding tears.
with a supratherapeutic INR. The risk for continued bleeding The initial therapy for acute esophageal variceal hemor-
on anticoagulation therapy should be weighed against the risk rhage (Figure 37) is resuscitation in an ICU with the goal of
associated with stopping therapy. Furthermore, endoscopy maintaining hemodynamic stability and a hemoglobin of
should not be delayed for anticoagulation reversal unless the 7 g/dL (70 g/L) . Overtransfusion can precipitate variceal
INR Is greater than 3.0. rebleeding due to increased portal pressure.
Decisions regarding discontinuing antiplatelet therapy The most effective approach for control of acute variceal
are based on whether the therapy is for primary or secondary hemorrhage is combined therapy with octreotide (somatosta-
prophylaxis. If aspirin is being taken for primary prophylaxis, tin analog) and endoscopic therapy. Octreotide decreases
then it should be discontinued because the risk for recurrent splanchnic blood flow and lowers portal pressure; it should be
bleeding outweighs the benefit. Aspirin for secondary prophy- initiated before endoscopic evaluation and continued for 3 to 5
laxis can be discontinued for 3 days but needs to be promptly days after variceal hemorrhage. Endoscopic variceal ligation
resumed when hemostasis is secure. Decisions regarding dis- within 12 hours of presentation is the endoscopic treatment of
continuing clopidogrel and other antiplatelet agents should be choice for hemostasls of active variceal hemorrhage, with a
made in conjunction with a cardiologist. success rate of 90%. Subsequent endoscopy with further band
Patients with hemodynamic instability or active bleed- ligation as needed to obliterate varices should be performed
ing (hematemesis or recurrent large-volume hematochezla) every 2 to 4 weeks.
should be admitted to an ICU for resuscitation. Other Patients who develop variceal hemorrhage are at high risk
patients can be admitted to a regular hospital ward. Several for infection, such as pneumonia and urinary tract infection, and
decision rules and predictive models have been developed to nearly 50% of patients with cirrhosis who are hospitaliz.ed with
identify patients who are at low risk for recurrent or life- UGIB have a bacterial infection. Rates of rebleeding and death are
threatening UGIB. The modified Glasgow-Blatchford bleed- reduced with prophylactic antibiotics (such as ceftnaxone or
ing score is calculated using the blood urea nitrogen level, quinolone). Initiation of antibiotics at the time of hospitalization
hemoglobin level, systolic blood pressure. and pulse rate. It is recommended in all patients with cirrhosis and gastrointesti-
predicts the need for clinical intervention, rebleeding, and nal bleeding, and antibiotic therapy should continue for 7 days
mortality. Patients at low risk according to the modified after varlceal hemorrhage, even in the absence of ascites.

72
Gastrointestinal Bleeding

FIGURE 3 5. Ulcers at low risk for rebleeding, for which endoscopic therapy is not ind icated. Left: Clean-based gastric ulcer with no blood vessels, pigmented spots/
protuberances, or clots noted in the base. Right: Non protuberant pigmented spot (arrow) in a duodenal ulcer bed.
Courtesyol Louis M. Wong Kee Song, MO, Mayo Oinic

FIGURE 3 7. Acute esophageal variceal hemorrhage.Avarix in the distal


esophagus is seen spurting bright red blood.

FIGURE 36. Mallory-Weiss tear.A superficial linear mucosa! tear(arrow) seen


on endoscopic retroflexion in the proximal stomach. Postendoscopic Care
Courtesy ol louis M. Wong Kee Song, MD,Mayo Clinic. Patients with low-risk stigmata (see Figure 35) can be fed
within 24 hours of endoscopy, receive once-daily oral PPI
therapy, and be discharged from the hospital. Patients with
Nonselective P-blocker therapy (propranolol, nadolol, or high-risk lesions and those with adherent clots requiring
CJ carvedilol) should be initiated in addition to endoscopic band endoscopic treatment should receive intravenous PPI therapy
CONT. ligation for secondary prophylaxis ofvariceal hemorrhage. The for 72 hours to decrease risk for rebleeding and remain in the
dosage of P-blockers should be increased as tolerated to obtain hospital for this interval.
a resting pulse rate of SS to 60/min. When hemostasis is secure, antithrombotic agents can be
If bleeding recurs, endoscopic therapy should be repeated, restarted while continuing high-dose oral PP! therapy twice
but routine second-look endoscopy is not recommended. daily. Patients with idiopathic peptic ulcer disease, unrelated
Interventional radiology or surgery is reserved for cases of to NSAID use or Helicobacter pylori infection, should con-
rebleeding despite endoscopic treatment. For variceal bleed- tinue once-daily oral PPI therapy indefinitely because of the
ing, placement of a transjugular intrahepatic portocaval shunt high risk for recurrent bleeding.
is reserved for bleeding that is not controlled by drug and For discussion of management of peptic ulcer disease, see
endoscopic therapy. Disorders of the Stomach and Duodenum. CJ

73
Gastrointestinal Bleed ing

KEY POINTS of patients with a presumed lower gastrointestinal source of


bleeding are found to have an upper gastrointestinal source.
• The most common causes of upper gastrointestinal
Dlverticular bleeding is arterial, usually painless, occurs
bleeding include peptic ulcer disease, gastroesophageal
varices, and Mallory-Weiss tear. in the neck or dome of a diverticulum, and stops spontane-
ously in 75% of cases. In patients with dlverticulosis, the risk
• Tachycardia (pulse rate >100/min), hypotension (sys- for bleeding is estimated at 0.5 per 1000 person-years. For
tolic blood pressure <100 mm Hg), age older than 60 further discussion of diverticular disease, see Disorders of the
years, and major comorbid medical conditions are all Small and Large Bowel.
associated with increased risk for rebleeding and death Angiodysplasia, also known as angiectasia or arterio-
in patients with upper gastrointestinal bleeding. venous malformation, can occur throughout the colon but is
• Upper endoscopy is the primary diagnostic modality for most common in the right colon. Elderly patients and patients
evaluating upper gastrointestinal bleeding. on anticoagulation therapy are at highest risk.
• Management of an ulcer depends on the endoscopic Postpolypectomy bleeding can occur immediately after
appearance and risk for rebleeding. polyp removal or days or weeks later. Risk is increased in
patients with polyps larger than 2 cm in size, with polyps
located in the right colon, and with resumption of antithrom-
CJ Lower Gastrointestinal Bleeding botic therapy.
Patients with ischemic colitis usually present with
Twenty percent of all cases of gastrointestinal bleeding originate
severe abdominal pain, often out of proportion to physical
in the colon or rectum. Most cases of LGIB stop spontaneously
findings. Diarrhea. abdominal pain, and hematochezia can
and have good outcomes; however, higher rates of morbidity and
occur with inflammatory bowel disease and infectious coli-
mortality are seen in older patients and in those with comorbid
tis. LGIB from a colon malignancy may be painless or associ-
conditions. Patients with LGIB usually present with sudden
ated with obstructive symptoms. Patients with cardiac dis-
onset of hematochezia (maroon or red blood per rectum) .
ease, such as valve dysfunction or dilated cardiomyopathy,
Occasionally, bleeding from the cecum or right colon may appear
are at risk for acquired von Willebnmd disease and gastroin-
black and tarry, like melena. LGrB may present with additional
testinal bleeding.
symptoms of pain, diarrhea, or change in bowel movements.

Causes Evaluation
The most common cause of minor LGIB is hemorrhoidal An initial patient assessment and hemodynamic resuscitation
bleeding. Hemorrhoidal bleeding is usually characterized by a should be performed simultaneously. The timing and quality
small volume of bright red blood and does not cause hemody- of any previous colonoscppy should be assessed, as should
namic instability or significant volume loss (see Disorders of whether or not polypectomy or biopsies were performed. The
the Small and Large Bowel for discussion of hemorrhoids) . patient's medication history, especially use and dosing of
Causes of severe LGIB that may lead to clinical instability ant!thrombotic agents, should be assessed, as well as personal
include divertlcular bleeding, colonic angiodysplasia, postpo- history, risk factors for liver disease, other comorbidities, and
lypectomy bleeding, Dieulafoy lesion, solitary rectal ulcer syn- recent illness.
drome, rectal varices, or malignancy (Table 38). Fifteen percent
Management
TABLE 38 . Causes of Severe Lower Gastrointestinal
Bleeding Resuscitation goals for patients with LGIB should be normali-
zation of blood pressure and heart rate, as well as the transfu-
Diverticulosis
sion of packed red blood cells if needed to maintain the
Aortoenteric fistu la hemoglobin level above 7 g/dL (70 g/L), with a threshold of
Colonic or rectal va ri ces 9 g/dL (90 g/L) in patients with massive bleeding, or when
Dieulafoy lesions treatment may be delayed. Platelet transfusion to maintain
Neoplasm
counts above 50.000 cells/µL (SO x 109 /L) is recommended in
patients with active bleeding. The decision to discontinue or
Colitis
reverse anticoagulant agents should balance the risk of ongo-
lschemic
ing bleeding with the risk of thromboembolic events and often
Inflammatory bowel disease requires a multidisciplinary approach.
Infectious Colonoscopy is the initial diagnostic test in the majority
lntussusception of patients with LGIB and should be performed within
Meckel diverticulum
24 hours of presentation after adequate colon preparation in
patients with significant bleeding. Colonoscopy identifies a
Ang iodysp las1a
source of LGIB in two thirds of patients. Hematochezia with

74
Gastrointestinal Bleeding

~ hemodynamic instability may indicate a rapid UGIB source. Causes


.... and upper endoscopy may be indicated. The likely underlying cause of small-bowel bleeding varies
CONT. Radiographic Interventions should be considered in with patient age (Table 39). Patients younger than age 40 years
patients with ongoing bleeding who do not respond to resus- are likely to have bleeding due to inflammatory bowel disease.
citation, patients who cannot tolerate colonoscopy or colon Dieulafoy lesions, neoplasia (leiomyoma, carcinoid, lym-
preparation, or patients in whom a source of bleeding is not phoma, or adenocarcinoma). Meckel diverticulum, or a poly-
identified endoscopically. Techniques include CT angiogra- posis syndrome. Patients older than age 40 years are likely to
phy. angiography, and, less frequently, tagged red blood cell have bleeding due to angiodysplasia, Dieulafoy lesion. neopla-
scintigraphy. sia, or NSAID-related ulcers. Angiodysplasia (Figure 38) is the
Angiography with embolization is frequently used to stop most common cause of small-bowel bleeding. It is found in
persistent or recurrent diverticular hleeding hecause endo- 40% of cases and is often seen in elderly patients. Rare causes
scopic approaches are limited due to the typical location ofthe of bleeding include Henoch -SchOnlein purpura, small-bowel
vessel inside a thin-walled diverticulum. Surgical consultation varices or portal hypertensive enteropathy, amyloidosis, blue
is usually reserved for patients who do not respond to endo- rubber bleb nevus syndrome, hematobilia, aortoenteric fistula,
scopic or radiographic measures. and hemosuccus entericus.
The risk for rebleeding is highest in patients with diverticu-
lar bleeding (9% to 47%) and angiodysplasia bleeding (37% to Evaluation
64%). For prevention of recurrent LGIB, nonaspirin NSAIDs A detailed medical history and physical examination are
should be avoided, particularly after diverticular or anglodys- needed to narrow the differential diagnosis to a small-bowel
plasia bleeding. The continued use of antiplatelet or anticoagu- source. Patients should be asked about NSAID use to evaluate
lants needs to be weighed against the risk for rebleeding. Aspirin for NSAID-induced small-bowel ulcers. aortic aneurysm
for secondary prevention in patients with high-risk cardiovas-
cular disease should not be discontinued. Decisions about dis-
continuation of dual antiplatelet therapy in patients with an TABLE 39. Causes of Small-Bowel Gastrointestinal
Bleeding
acute coronary syndrome or coronary stent placement should
be made in conjunction with a cardiologist. Anticoagulation use Differential Patient Clinical Clues
for other medical indications should be resumed as soon as pos- Diagnosis Age(Years)
sible, within at least 7 days for most patients. CJ Angiodysplasia >60 Intermittent, usually occult
bleeding; may also occur in
KEY POINTS the colon
• Patients with lower gastrointestinal bleeding usually Peutz-Jeghers <20 Perioral pigmentation,
present with sudden onset of hematochezia (maroon or syndrome obstructive symptoms
red blood per rectum). Meckel 20-60 Possible abdominal pain
diverticulum
• Most cases oflower gastrointestinal bleeding stop spon-
taneously and have good outcomes; however, higher Hemangioma <20 Possible cutaneous
hemangiomas
rates of morbidity and mortality are seen in older
patients and in those with comorbid conditions. Malignancy >50 Weight loss, abdominal pain
Hereditary >50 Mucocutaneous
• Colonoscopy identifies a source oflower gastrointestinal hemorrhagic telangiectasias
bleeding in two thirds of patients. telangiectasia ________ J

CJ Small-Bowel Bleeding
The term small- bowel bleeding is preferred to obsct1re gastro-
intestinal bleeding because in many cHnical situations the
cause of the bleeding can now be identified. Patients with
small-bowel bleeding often have nonnal results on upper
endoscopy and colonoscopy. Small-bowel bleeding can be
characterized as overt or occult. In patients with visible bleed-
ing (either melena or hematochezia), it is overt. In patients
who present with anemia but no gross signs of bleeding, the
bleeding is considered occult. It is estimated that 5% to 10°!.. of
gastrointestinal bleeding occurs between the ligament ofTreitz FIGURE 3 8. Capsule endoscopy image of angiodysplasia. The lesion (arrow)
and the ileocecal valve; this is also known as rnidgastrointesti- has a fern like pattern and is red in color.Angiodysplasia can have no bleeding or
nal bleeding. active bleeding.

75
Gastrointestinal Bleeding

repair (which raises concern for an aortoenteric fistula), Follow-up studies after a positive scan can include repeat
Cl necrotizing pancreatitis (which causes hemosuccus pancreati- endoscopy or angiography, both of which can offer more accu-
CONT. cus), or liver damage (such as trauma. tumor, or recent biopsy rate localization and therapy.
causing hemobilia). The presence of skin lesions may help
determine an underlying diagnosis, including mucocutaneous Wireless Capsule Endoscopy
telangiectasia (hereditary hemorrhagic telangiectasia, also Capsule endoscopy employs a wireless capsule camera
known as Osler-Weber-Rendu syndrome [Figure 39]) or der- (Figure 40) that is swallowed by the patient to take images of
matitis herpetiformis (celiac disease). the small bowel. The images are transmitted to a radiofre-
If the bleeding source is not identified, but clinical suspi- quency receiver worn by the patient. Capsule endoscopy is the
cion suggests that the cause of the bleeding is discoverable by preferred test for evaluating stable patients for causes of small-
a conventional endoscopic examination, a second-look endos- bowel bleeding after normal results on upper endoscopy and
copy or colonoscopy should be done. The diagnostic yield is up colonoscopy. Capsule endoscopy is able to visualize the entire
to 25% with this approach. small bowel in up to 90% of cases, with a diagnostic yield as
high as 83%. Limitations of capsule endoscopy include the
Angiography inability for therapeutic intervention and difficulty with local-
Conventional angiography is a diagnostic and therapeutic test. ization of the lesion. The primary complication is possibility of
However, it is limited to detecting bleeding at rates greater capsule retention due to obstruction or strictures. The capsule
than 0.5 ml/min. Clinical predictors of successful angiogra- can be retrieved by deep enteroscopy or surgery.
phy include hemodynamic instability and the need for trans- If there is continued concern for bleeding from the small
fusion of more than 5 units of blood. Potential complications bowel, specialized types of enteroscopy may be considered,
of angiography include acute kidney injury, systemic embo- including push, spiral, and balloon enteroscopy. These tech-
lism, hematoma, and vascular dissection or aneurysm. niques allow visualization beyond the ligament of Treitz for
CT angiography uses multiple phases of contrast enhance- diagnosis and the opportunity for therapeutic intervention. In
ment, including arterial enhancement. It can identify bleeding at general, the rates of complications are low, but complications
rates as low as 0.3 ml/min, but its usefulness is limited because can include perforation and in the case of balloon enteroscopy,
the patient must have active bleeding to identify the location. ileus and pancreatitis.

Technetium-Labeled Nuclear Scan Small-Bowel Imaging


Technetium 99m- labeled red blood cell or sulfur colloid Endoscopy has replaced imaging for the initial evaluation of
nuclear scans are able to detect bleeding rates between 0.1 to suspected bleeding from the small bowel.
0.4 ml/min. Their accuracy in identifying a source of bleeding Barium-based examinations are no longer recommended
varies, ranging from 24% to 91%, and they do not provide for for the evaluation of small-bowel bleeding because of low
therapeutic intervention. A nuclear scan is often done before diagnostic yields. CT enterography is beneficial in diagnosing
angiography to confirm the presence of active bleeding. small-bowel masses and has shown a diagnostic yield of 40%
for bleeding. Due to insufficient data, MR enterography is not
recommended for the evaluation of small-bowel bleeding.
However. it can be considered in patients younger than age 40
years, and it offers lower exposure to radiation than CT.

FIGURE 3 9. Heredital)' hemorrhagic telangiectasia {Osler-Weber-Rendu


0 1 cm 2 3 4
syndrome) is a disorder of development of the vasculature characterized by
telangiectases and arteriovenous malformations in specific locations. It is one of
the most common monogenic disorders, but affected individuals are frequently
I
not diagnosed. The most common features of the disorder-nosebleeds and f I GU RE 4 0. Endoscopy capsule.
telangiectases on the lips, hands, and oral mucosa- are often quite subtle. Courtesy of Elizabeth Rai-n, MD, Mayo Oink.

76
Bibliography

lntraoperative Endoscopy Am J Gastmenterol. 2016;lll:30 -50; quiz 51. (PMID: 26526079] dol:l0.1038/
CJ
CONT.
Intraoperative endoscopy occurring during laparotomy is
ajg.2015.322
Vae-.ll MF, Pandolfino JE. Vela ME ACG clinical guideline: diagnosis and man ·
often a last resort because it is the most invasive modality agement of achalasla. Am J Gastroenterol. 2013;108:1238-49; quiz 1250.
available. The diagnostic yield for small-bowel bleeding has [PMID: 23877351) dol:l0.1038/ajg.2013.196

been reported in the range of 58% to 88%; however, its use Zerbib F, Roman S. Current therapeutic options for esophageal motor disorders
as defined by the Chicago ClassJtlcation. J Clin Gastroentero]. 2015;49:451 -
should be reserved for patients in whom all other diagnostic 60. (PMIO: 25844840] dol:l0.1097/MCG.0000000000000317
modalities have failed. Disorders of the Stomach and Duodenum
American Cancer Society. Cancer facts & figures 2016. Atlanta: American
Management Cam.-er Society; 2016.
Berg P. McCallum R. Dumping Syndrome: A Review of the Current Concepts
After achieving hemodynamic stabilization, therapy is guided of Pathophysiology, Diagnosis, and Treatment. Dig Dis Sci. 2016;61:1!-8.
by the underlying source of bleeding. Vascular lesions (angio- [PMID: 26396002] dol:10.1007/s10620-015-3839-x
dysplasia) should be treated with electrocautery, argon plasma Camfllerl M, Parkman HP, Shafi MA, Abell TL. Gerson L; American College of
Gastroenterology. Clinical guideline: management of gastroparesis. Am J
coagulation, injection therapy, mechanical hemostasis (herno- Gastrnenterol. 2013;108:18-37; quiz 38. [PMID: 23147521] doi:l0.1038/
clips or banding), or a combination of these techniques. ajg.2012.373
Medical therapy for vascular lesions may require a sornatosta- Chey WD, 1.eontladls GI, Howden CW, Moss SF. ACG clinical guideline: treat-
ment of Helicobacter pylori infection. Am J Gastroenterol. 2017;112:212-
tin analog, such as octreotide. Hormonal thernpy no longer has 239. (PMID: 28071659] dol:l0.1038/ajg.2016.563
a role in the medical management of small-bowel bleeding. Evans JA. Chandrasekhara V. Chathadl KV, Decker GA, Early OS, Ff sher DA. et
al; ASGE Standards of Practice Committee. The role of endoscopy In the
Tumors or masses require surgical intervention, and if management of premalignant and mallgnant conditions of the stomach.
massive bleeding is present, embolization of the bleeding vessel Gastrolntest Endosc. 2015;82:1-8 . [PMID : 25935705] dol:l0 .1016/j.
gie.2015 .03.1967
may be needed with the assistance ofinterventionaJ radiology.
Ingle SB, Hinge Ingle CR. Eosf11ophfl!c gastroenterltls: an unusual type of
Anemia should be treated with blood transfusion acutely gastroenterttls. World J Gastroenterol. 2013:19:5061-6. (PMtD: 23964139)
if needed and iron supplementation. If a causative agent is dot:l0.3748/wjg.v19.131.5061
identified, such as an NSAID, the agent should be stopped. Laine L. Clinical Practice. Upper gastrointestinal bleeding due to a peptic ulcer.
N Engl J Med. 2016;374:2367-76 . [PMID: 27305194) doi:l0.1056/
Patients with angiodysplasia in the setting of aortic stenosis NEJMcp1514257
(known as Heyde syndrome) benefit from valve replacement Lanza FL, Chan FK, Quigley EM; Practice Parameters Committee of the
surgery. While not FDA approved, thalidomide. which inhibits American College ofGastroenterology. GuldeUnes for prevention of NSAID-
related ulcer complications. Am J Gastroenterol. 2009;104:728-38. [PMID:
vascular endothelial growth factor, has shown some benefit in 19240698] dol:I0.1038/ajg.2009.115
decreasing bleeding in patients with vascular malformations Moayyedl PM. Lacy BE, Andrews CN. Enns RA, Howden CW, Vakil N. ACG and
CAG Clinical guideline: management of dyspepsia. Am J Gastroenterol.
of the gut. CJ 2017;112:988-1013. [PM!D: 28631728] dof:l0.1038/ajg.2017.154
KEV POINTS Park N , l.am-Hlmlln D, Vemulapalli R. Review of autoimmune metaplastic
atrophic gastritis. Gastrointest Endosc. 2013;n:284- 92. (PMID: 23199649)
• Capsule endoscopy is the preferred test for evaluating doi:l0 .1016/j.gle.2012.09.033
stable patients for small-bowel bleeding after normal Stanghelllni V. Chan FK, Hasler WL, Malagelada JR. Sul.Oki H. Tack J, et al.
Gastroduodenal disorders. Gastroenterology. 2016;150:1380- 92. [PMID:
results on endoscopy and colonoscopy. 27147122] doi :l0.10S3/j.gastro.2016.02.0ll
• After achieving hemodynamic stabilization, therapy for Syngal S, Brand RE, Church JM, Glardlello FM, Hampel ML, Burt RW; Amerlcnn
College of Gastroenterology. ACG clinical guideline: genetic testing and
small-bowel bleeding is guided by the underlying management of hereditary gastrolntestfnal cancer syndromes. Am J
source of bleeding. Gastroenterol. 2015;110:223- 62; quiz 263. (PMID: 25645574] dot:l0.1038/
ajg.2014.435
Talley NJ, Ford AC. Functional dyspepsia. N Engl J Med. 2015;373:1853-63.
Bibliography (PM!D: 26535514] doi:l0.1056/NEJMral501505
Zong L, Chen P. Billroth I vs. Billroth II vs. Roux-en-Y following distal gastrec-
Disorders of the Esophagus tomy : a meta- analysis based on 15 studies. Hepatogastroenterology.
Bowers SP. Esophageal motil!ty disorders. Surg Clln North Am. 2015;95:467- 2011;58:1413-24. (PMID: 21937419) doi:I0.5754/hgel0567
82. [PMID: 25965124] doi:l0.1016/J.suc.2015.02.003
Disorders of the Pancreas
Ganz RA. A review of new surgical and endoscopic therapies for gastroe-
sophageal reflux disease. Gastroenterol Hepatol (N Y). 2016;12:424-31. Banks PA, Bollen TL, Dervenls C, Gooszen HG, Johnson CD, Sarr MG, et al;
[PMIO: 27489524] Acute Pancreatltls Classification Working Group. Classification of acute
pancreatftfs- 2012: revision of the Atlantn classification and definitions by
Gunaslng:im N, Perczuk A, Talbot M, Ka ffes A, Saxena P. Update on therapeutic International consensus. Gut. 2013;62:102-11. [PM!D: 23100216) doi:l0.1136/
interventions for the management of achalasla. J Gastroenterol Hepatol. gutjnl-2012-302779
2016:31:1422-8. (PM!D: 27060999] doi:l0.1111/jgh.13408
Canto Ml, Hruban RH, Fishman EK. Kamel IR, Schullck R, Zhang Z, et al;
Katz PO, Gerson LB, Vela MF. Guidelines for the diagnosis and management of American Cancer of the Pancreas Screening (CAPS) Consortium. Frequent
gastroesophageal reflux disease. Am J Gastroenterol. 2013;108:308-28; quiz detection of pancreatic lesions In asymptomatic high-risk Individuals.
329. [PM!D: 23419381] dol:l0.1038/ajg.2012.444 Gastroenterology. 2012:142:796-804; quiz e14-5. [PMID : 22245846]
Kavltt RT. Hirano J, Vaezl MF. Diagnosis and treatment of eoslnophllic doi:l0.1053/j.gastro.2012.01.005
esophagftis In adults. Am J Med. 2016;129:924-34 . (PMID: 27155108] Di MY, Liu 1-1, Yang 'l'l, Bonis PA, TangJL, Lau J. Prediction models of mortality
doi:l0.1016/).amjmed.2016.04.024 In acute pancreatltls In adults: a systematic review. Ann Intern Med.
Pamlolfino JE, Kahrllas PJ; American Gastroenterological Association. AGA 2016:165:482-490. [PM!D: 2745~310]
technical review on the clinical use of esophageal manometry. Elta GH, Enestvedt BK, Sauer BG, Marie Lennon A. ACG Clinical Guldellne:
Gastroenterology. 200.5;128:209-24. (PMID: 15633138} Diagnosis and management of pancreatic cysts. Am J Gastroenterol.
Shaheen NJ, Falk GW, Iyer PG, Gerson LB: American College of Gastroenterology. 2018 Feb 27. dol:l0.1038/ajg.2018.l~. [Epub ahead of print] (PM!D:
ACG cllnlcal guideline: diagnosis and management of Barrett's esophagus. 29485131]

77
Bibliography

Khonma AA, Mangu PB, Berlin J, Engebretson A. Hong TS, Maitra A, el al. Schiller LR. Pardi DS, Spiller R. et al. Gastro 2013 APDW/ WCOG Shanghai
Potentially curable pancreatic cancer: American Society of Clinical working party report: chronic diarrhea: detlnltlon, classification, diagno-
Oncology cllnlcnl prnctlce guideline. J Clln Oncol. 2016;34:2541-56. fPMID: sis. J Gastroenterol Hepatol. 2014 Jan;29(1):6- 25. [PMIO: 24117999]
27247221] doi:l0.1200/JC0.2016.67.5553 Shepherd SJ, Lomer MC. Gibson PR. Short-chain carbohydrates and functional
Kunz PL, Reidy-1..agunes D, Anthony LB, Bertino EM, Brendtro K, Chan JA. gastrointestinal disorders. Am J Gastroenterol. 2013 May;l08(5):707-17.
el al; North American Neuroendocrlne Tumor Society. Consensus guidelines (PMIO: 23588241]
for the management and treatment o!' neuroendocrine tumors. Pancreas. Tursi A, Papa A, Danese S. Review article: the pathophys!ology and medical
2013;42:557-77. (PMID: 23591432) dol:l0.1097/MPA.Ob013e31828e34a4 management of diverticulosis and divenlcular dis(.-ase of the colon. Ailment
Majumder S, Chari ST. Chronic pancreatltls. Lancet. 2016:387:1957-66. [PMID: Pharmacol Ther. 2015 Sep;~2(6) :664-84. [PMID: 26202723)
26948434) dol:10.1016/SOJ40-6736(16).00097-0 Wald A, Bharucha AE, Cosman BC, Whitehead WE. ACG cilnic:il guideline:
Okano K, Oshima M, Yachida S. Kushida Y. Kato K, Kamada H, et al. Factors management of benign anorectal disorders. Am J Gastroenterol. 2014
predicting surviv:il and pathological subtype in patients with ampullary Aug;l09(8) :ll41-57; (Quiz) 1058. [PM ID: 25022811)
adenocarcinoma. J Surg Oncol. 2014;110:156-62. [PMID: 24619853] doi:
10.1002/jso.23600 Colorcctal Neoplasla
Okazaki K. Uchida K. Autoimmune Pancreatltis: the past, present, and American cancer Society. Colorectul cancer Facts and Figures 2014-2016.
Future. Pancreas. 2015;44:1006-16. [PMID: 26355544) doi:l0.1097/MPA. Atlanta: American Cancer Society, 2014.
0000000000000382
American Gastroenterology Association. AGA institute guidelines for colonos-
Tenner S, Baillie J, DeWitt J, Vege SS; American College of Gastmenterology. copy surveillance after cancer resection: clinical decision tool.
American College of Gastroenterology guideline: management of acute Gastroenterology. 2014 May;146(S):l413-4 . [PMID: 24742563]
pancreatitis. Am J Gastroenterol. 2013;1 08:1400-15: 1416. (PMID: 23896955)
Bihbins-Oomlngo K; U.S. Preventive Services Task Force. Aspirin use for the
doi:T0.1038/ajg.2013.218
primary prevention of cardiovascular disease and colorectal cancer: U.S.
Vaughn VM. Shuster D, Rogers MAM. Mann J. Conte ML, Saint S. et al. Early Preventive Services Task l'orce recommendation statement. Ann Intern
versus delayed feeding In patients with acute pancreatitis: a sy:.tematlc Med. 2016 Jun 21;164(12):836-45. [PMID: 27064677)
review. Ann Intern Med. 2017;166:883-892. [PMIO: 28505667]
Bouvan:l V. Loomis D, Guyton KZ, et al.; International Agency for Research on
Disorders of the Small ond l.Jlrge Bowel cancer Monograph Working Group. carcinogenicity of mnsumptlon of red and
processed meat. Lancet Oncol. 2015 Dec:16(16):1599-600. [PM1D: 26514947]
Brandt U, Feuerstadt P, Longstreth CF, lloley SJ; American College of
Ga~tmenterology. ACG dinkai guideline: epidemiology, risk factors, pat- Glardlello FM, Allen JI, Axilbund JE, et al. Guidelines on genetic evaluation
terns of presentation, diagnosis, and management of colon ischemla (Cl). and management of Lynch syndrome: a consensus statement by the US
Am J Gastroenterol. 2015 Jan:ll0(1):18- 44; quiz 45. dol: 10.1038/ Multi-Society Task Force on Colorectal Cancer. Dis Colon Rectum. 2014
ajg.2014.395. [PMID: 25559486] Aug;57(8):1025-48. [PMID: 25003300J
Brenner DM, Shah M. Chronic constipation. Gastroenterol Clin North Am. Lieberman DA, Rex DK. Winawer SJ. et al.; United States Multi-Society Task
2016Jun:4S(2):205-16. doi: 10.1016/J.gtc.2016.02.013. [PMID: 27261894) Force on Colorec:tal Cancer. Guidelines for coionoscopy surveillance after
screening and polypectomy: a consensus update by the US Multl-Soclely
Clair DG, Beach JM. Mesenteric ischemia. N Engl J Med. 2016 Mar 10: Task Force on Colorectal Cancer. Gastroenterology. 2012 Sep;143(3):844-57.
374(10):959- 68. [PMlD: 26962730) fPMID: 22763141]
Downey L, Houten R. Murch S, et al.: Guldellne Development Group. Provenzale 0, Gupta S, Ahnen DJ. Bray T, Cannon JA, Cooper G, el al. Genetic/
Recognition, assessment, and management of coeiiac: disease: summary of Familial High-Risk Assessment : Colorectal Version 1.2016, NCCN Cllnlcal
updated NICE guidance. BMJ. 2015 Sep 2;35l :h4513. doi: 10.1136/bmj. Practice Guidelines in Oncology. J Natl ComprCanc Netw. 2016;14:1010 -30.
h4513. [PMID: 26333593] [PMID: 27~96117]
Ford AC, Moayyedi P, lacy llf', et al.; Task Force on the Management of Rubenstein JH, Enns R, Heidelbaugh J, Barkun A; Clinical Guidelines
Functional Bowel Disorders. American College of Gastroenteroiogy mono- Committee. American Gastroenterologicai Association institute guideline
graph on the management of lrritahle bowel syndrome and chronic idio- on the diagnosis and management oflJnch Syndrome. Gastroenterology.
pathic constipation. Am J GastroenteroJ. 2014 Aug:l09 Suppl l:S2-26; qui7. 2015 Sep:149(3):777-82: qui7. e16-7. [PMm: 262265n)
S27. l.PMIO: 25091148]
Syngal S. Brand RJ::, Church IM, et al.; American College ofGastroenterology.
Freeman A. Menees S. Fec:il incontinence and pelvic floor dysfunction In ACG clinical guideline: genetic testing and management of hereditary gns-
women: a review. Gastroenterol Clin North Am. 2016 Jun:45(2):217-37. trointestinal cancer syndromes. /\m J Gastroenterol. 2015 Feb;U0(2):223-
(PMID: 27261895] 62: quiz 263. [PMID: 25645574]
Keefer L. Drossman DA, Guthrie E, et al. Centrally mediated disorders of gas-
trointestinal pain. Gastroenterology. 2016Feb19. pii: S0016-5085(16)0022S- Disorders of the Liver
O. [PMID: 27144 628J AASLD-IDSA. Recommendations for testing, managing. and treating hepatitis
Kornbluth A, Sachar OB; Pr.lclice Parameters Committee of the American C. https://www.hcvguidelines.org/. Accessed February 14, 2018.
College ofGastruenterology. Ulcerative colitis practice guidelines in adults: C.'halasanl NP, Hayashi PH, Bonkovsky HL. Navarro VJ, Lee WM, Fontana RJ;
American College ofGastroenterology, Practice Parameters Committee. Am Practice Parameters Committee of the American College ofGastroenterology.
J Gastrocnterol. 2010 Mar;IOS(3):S01 - 23; quiz 52~ . Erratum In: Am J ACG dinlcai guideline: the diagnosis and management of !dlosyncr.itic
Ga~troenterol. 2010 Mar;l05(3):500. [PMID: 20068560)
drug- Induced liver injury. Am J Gastroenlerol. 2014 ;109:950-66; qui7. 967.
Leeds IL, Fang SH. Anal t'llncer and lntraepllhellal neoplasia screening: a [PMID: 24935270] dol:l0.1038/ajg.2014.131
review. World J Gastrolncest Surg. 2016 Jan 27;8(1):41 -51. fPMID: 26843912] Chalasani N, Younossi Z. Lavine JE, Charlton M. Cusi K, Rinella M. et al. The
l.lchtensleln GR, Hanauer SB, Sandborn WJ; Practice Parameters CommJttee diagnosis and management of nonalcoholic Fatty liver disease: practke
ofAmerican College of Gastroenterology. Management ofCrohn's disease In guidance From the American Association for the Study of Liver Diseases.
adults. Am J Gastroenterol. 2009 Feb;J04(2):465-83; quiz 464, 484. [PMID: Hcpatology. 2017. [PMID: 28714183] dol:I0.1002/hep.29367
19174807) European Association tor the Study of the !.Iver. EASL clinical practice guide-
Mearin F, Lacy BE, Chang L, et al. Bowel disorders. Gastroenterology. 2016 Feb lines: autoimmune hepatitis. J Hepa to!. 2015;63:971 -1004. [PMID: 26341719]
18. pii: S0016-5085(16)00222-S. [PMID: 27144627) doi:I0.1016/j.jhep.2015.06.030
Nlkaki K, Gupte GL Assessment of intestinal malabsorption. Best Prac:t Res Heimbach J, Kulik LM, f'lnn R. Sirlin CB. Abe(.'3Ssis M, Roberts LR, et al. AASL.D
Clin Gastroenterol. 2016 Apr;30(2):225- 3S. [PMID: 27086887] guidelines for the treatment of hepatocellular carcinoma. Hepatology. 2017.
(PMm: 28130846] dol:10.10021hep.29086
Pardi DS, Tremaine WJ. Carrasco-Labrn A. American GastroenterologlC'JI
Association Institute technical review on the medlC'JI management or micro- llyasJA, Kanwal F. Primary prophylaxisofvariceal bleed Ing. Gastmenterol Clin
sc.upic colitis. Gastroenterology. 2016Jan;150(1):247-274.ell. [PMID: 26584602) North Am . 2014;43:783-94. [PMID: 25440925] doi:I0.1016/j.gtc.2014.08.008
Riddle MS. DuPont HI. Connor BA. ACG cllnlcal guideline: diagnosis, treat· Kamar N, llendall R. Legrand-Abravanel F. Xia NS, ljaz S, lzopet J, et al.
ment, and prevenllon of acute diarrheal lntectlons In adults. Am J 1lepatltis E. Lancet. 2012;379:2477-88. [PMID: 22549046J dol:l0.1016/
Gastroenteml. 2016 May;111 (5):602-22. [PMID: 27068718) SOI40- 6736(11)61849-7
Rubio-Tapia A, Hill ID, Kelly CP, et al: American College ofGastroentcrology. Undor KD. Gershwln ME, Poupon R. Kaplan M, Bergasa NV. Heathcote El;
ACG clinical guldel!nes: diagnosis and management of ceilac disease. Am J American Association for Study of Llver Diseases. Primary biliary cirrhosis.
Gastrocnteml. 2013 May;I08(5):656-76; quiz 6n. [PMID: 23609613) Hepalology. 2009;50:291- 308. [PMID: 19554543] dol:I0.1002/hep.22906

78
Bibliography

Lindnr KD, !<owdley KV, l-l~rrison ME; American College of Gastroenlerology. Gastrointestinal Bleeding
ACG dlnlcal guideline: primary sclerosing cholangitis. Am J Gaslrnenterol.
Fisher I., Lee Krinsky M, Anderson MA. Appalaneni V. Banerjee S. l:len-
2015;110:646-59; quiz 660. [PMID: 25869391) doi:l0.1038/ajg.2015.112
Menachem T. et al: ASGE Standards of Pmctlce Committee. The role of
Radeke S. OlllonJ~~ Murray AL. A systematic review of the prevalence of mildly endoscopy In the management ofobscure GI bleeding. Gaslrolntest Endosc.
abnormal liver function tests and associated health outmmes. Eur J 2010;72:471 -9. [PMID: 20801285) doi:l0.1016/j.gle.2010.04.032
Gastroenterol 1-lepalol. 2015;27:1 ·7. [PMID: 25380394] dof:l0.1097/ Gerson LB. Fidler JL, (',ave DR. Leighton JA. ACG clinical guideline: diagnosis
MEG.0000000000000233 and management of small bowel bleeding. Am J Gastroenterol.
Singal AK, Bataller R. Ahn J, Karnath PS, Shah VH. ACG Clinical Guideline: 2015:110:1265-87; quiz 1288. [PMID: 26303132] dol:l0.1038/ajg.2015.246
Alcoholic Liver Disease. Am J Gastroenterol. 2018. [PMJI): 2933643~J Laine L. Clinical practice. Upper gastrointestinal bleeding due to a peptic ulcer.
dol:I0.1038/ajg.2017.469 N Engl J Med. 2016;374:2367-76. [PMrD: 27305194] dol:I0.1056/
Terrault NA. l\zowej NH. Chang KM, I lwang JP. Jonas MM. Mumd MH: NEJMcp\514257
American Association for Lhe Study of Liver Diseases. AASLD guidelines lbr Laine L. Jensen DM. Management of patients wilh ulcer bleeding. Am J
treatment of chronic hepatitis l\. Hepatology. 2016;63:261-83. [PM!f): Gaslroenterol. 2012;107:345-60; quiz 361. [PMJD: 22310222J doi:I0.1038/
26566064) dol:l0.1002/hep.28156 ajg.2011.480
Thursz MR, Richardson P, Allison M, Austin A. Bowers M. Day CP, et al; Leighton JA. The role of endoscopic imaging of the small bowel in clinical
STOPAH Trial. Prednlsolone or pentoxlt'ylline for alcoholic hepatitis. N Engl prnctlce. Am J Gastroenterol. 201];]06:27-36; quiz 37. [PMlrl: 20978483)
J Med. 2015;372:1619-28. [PMID: 25901427] doi:I0.1056/NEJMoa1412278 dol:l0.l038/ajg.20I0.410
Westbrook RH, Dushelko G. Williamson C. Pregnancy and liver disease. J Liu K, Kafles AJ. Review article: the diagnosis and investigatlo11 of obscure
Hepatol. 2016;64:933-45. [PMID: 26658682) doi:l0.1016/j.jhep.2015.11.030 gastrointestinal bleeding. Aliment Pharmaco!Ther. 2011 ;34:~16-23. [PMID:
21692820] doi:l0.1111/j.1365-2036.201l.04744.x
Disorders of the Gallbladder and Bile Ducts StTate LL Gralnek IM. ACG clinical guideline: management of patients with
Cotton PS, Elta GH. Carter CR, Pasrlcha PJ, Cordzziari ES. Rome IV. Gallbladder acute lower gastrointestinal bleeding. Am J Gastroenterol. 2016;111:459 -74.
and sphincter of Oddi disorders. Gastroenterology. 2016. (PMID: 27144629] [PMID: 26925883] doi:l0.1038/ajg.2016.41
dol:l0.1053/j.gastro.2016.02.033 Teshima CW. Small bowel endoscopy tor obscure GI bleeding. !lest Pract Res
European Association for the Study of the Liver (El\Sl.). Clinical Practice Clln Gastroenterol. 2012:26:247- 61. [PMID: 227045681 dol:l0.1016/j.
Guldelfnes on the prevenllon, diagnosis and treatment of gallstones. J bpg.2012 .01.020
Hepatol. 2016;65:146-181. [PMID: 27085810] doi:I0.1016/j.jhep.2016.03.00S VIiianueva C. Colomo A, Bosch A, Con~-epcl6n M, Hernandez-Gen V, Amell C.
Razumllava N, Gores GJ. Cholanglocarclnoma. Lancet. 2014:383:2168-79. et al. Transfusion strategies for acute upper ga~-troinLeslinal bleeding. N
[PMID: 2458l682J doi:l0.J016/S0140-6736(13)61903- 0 Eng!J Med. 2013;368:11 - 21. [PMID: 23281973] doi:IO.I056/NEJMoal211801

79
Gastroenterology and Hepatology
Self-Assessment Test
This self-assessment test contains one-best-answer multiple-choice questions. Please read these directions carefully
before answering the questions. Answers, critiques, and bibliographies immediately follow these multiple-choice
questions. The American College of Physicians (ACP) is accredited by the Accreditation Council for Continuing Med-
ical Education (ACCME) to provide continuing medical education for physicians.

The American College of Physicians designates MKSAP 18 Gastroenterology and Hepatology for a maximum of
22 AMA PRA Category 1 CreditsrM. Physicians should claim only the credit commensurate with the extent of their
participation in the activity.

Successful completion of the CME activity, which includes participation in the evaluation component, enables the
participant to earn up to 22 medical knowledge MOC points in the American Board of Internal Medicine's Mainte-
nance of Certification (MOC) program. It is the CME activity provider's responsibility to submit participant comple-
tion information to ACCME for the purpose of granting MOC credit.

Earn Instantaneous CME Credits or MOC Points Online


Print subscribers can enter their answers online to earn instantaneous CME credits or MOC points. You can submit
your answers using on line answer sheets that are provided at mksap.acponline.org, where a record of your MKSAP
18 credits will be available. To earn CME credits or to apply for MOC points, you need to answer all of the questions in
a test and earn a score of at least 50°/c, correct (number of correct answers divided by the total number of questions).
Please note that ifyou are applying for MOC points, you must also enter your birth date andABIM candidate number.

Take either of the following approaches:


• Use the printed answer sheet at the back of this book to record your answers. Go to mksap.acponline.org,
access the appropriate online answer sheet, transcribe your answers, and submit your test for instantaneous
CME credits or MOC points. There is no additional fee for this service.
• Go to mksap.acponline.org, access the appropriate online answer sheet, directly enter your answers, and
submit your test for instantaneous CME credits or MOC points. There is no additional fee for this service.

Earn CME Credits or MOC Points by Mail or Fax


Pay a $20 processing fee per answer sheet and submit the printed answer sheet at the back of this book by mail or
fax, as instructed on the answer sheet. Make sure you calculate your score and enter your birth date and ABIM can-
didate number, and fax the answer sheet to 215-351-2799 or mail the answer sheet to Member and Customer Service,
American College of Physicians, 190 N. Independence Mall West, Philadelphia, PA 19106-1572, using the courtesy
envelope provided in your MKSAP 18 slipcase. You will need your IO-digit order number and 8-digit ACP ID number,
which are printed on your packing slip. Please allow 4 to 6 weeks for your score report to be emailed back to you. Be
sure to include your email address for a response.

lfyou do not have a 10-digit order number and 8-digit ACP ID munber, or if you need help creating a usemarne and
password to access the MKSAP 18 online answer sheets, go to mksap.acponline.org or email custserv@acponline.org.

CME credits and MOC points are available from the publication date of July 31, 2018, until July 31, 2021. You may
submit your answer sheet or enter your answers online at any time during this period.

81
Directions
Each of the numbered items is followed by lettered answers. Select the ONE lettered answer that is BEST in each case.

Item 1 Which of the following is the most appropriate manage-


A 60-year-old woman is evaluated for persistent consti- ment of his aspirin therapy?
pation symptoms of 2 years' duration. She has reflex sym- (A) Continue aspirin use until the day of the polypectomy;
pathetic dystrophy syndrome involving the right arm and resume in 48 hours
neck that began 3 years earlier and requires chronic opi- (B) Discontinue aspirin use 7 days before the polypectomy;
oid analgesic therapy. She reports passing two hard bowel resume immediately after
movements per week. Trials of several fiber supplements
caused severe bloating and abdominal distention without (C) Discontinue aspirin use 7 days before the polypectomy;
relieving symptoms. A trial of polyethylene glycol (PEG) has resume in 48 hours
not been effective; the patient reports that her stool is soft, (D) Do not discontinue aspirin
but she still has no more than two bowel movements per
week. Adding bisacodyl caused severe abdominal cramping,
prompting its discontinuation. Colonoscopy at the onset of ltem4
constipation was unremarkable. In addition to PEG, her A 48-year-old woman is evaluated at a follow- up appoint-
medications are gabapentin, hydrocodone, fentanyl patch, ment for elevated liver chemistry tests over the preceding
and a calcium supplement with vitamin D. 6 months. She reports no symptoms. She has type 2 diabetes
On physical examination, vital signs and other findings mellitus and hypertension. Her family history is unremark-
are normal. able. She drinks two glasses of wine one or two times per
month. Her medications are metformin and lisinopril.
Which of the following is the most appropriate treatment On physical examination, vital signs are normal; BMI is
for her constipation? 31. No spider angiomata, palmar erythema, splenomegaly,
(A) Add docusate sodium abdominal distention, or lower-extremity edema is noted.
(B) Add lactulose Laboratory studies:
(C) Increase PEG dosage Alkaline phosphatase 96 U/L
(D) Switch to naloxegol Alanine aminotransferase 85U/L
Aspartate aminotransferase 74 U/L
Anti- smooth muscle antibody Positive (1:20 titer)
Item 2 Results of other studies, including hepatitis C antibody,
A 29-year-old woman is evaluated during a new-patient hepatitis B surface antigen, iron saturation, al-antitryp-
appointment. She was diagnosed with hepatitis B virus sin phenotype, tissue transglutaminase IgA antibody, anti-
(HBV) infection 10 years ago; her mother had HBV infection, mitochond1ial antibody, and lgG, are within normal limits.
and it was presumed that the patient acquired the infection On ultrasonography, the liver is hyperechoic and
at birth. She reports feeling well. Her medical history is enlarged.
otherwise unremarkable and she takes no medication.
On physical examination, vital signs are normal, as is Which of the following is the most likely diagnosis?
the remainder of the examination. (A) Antirnitochondrial antibody- negative primary biliary
Laboratory studies are positive for hepatitis B surface cholangitis
antigen and positive for hepatitis B e antigen. The serum HBV
(B) Autoimmune hepatitis
DNA level is 20,000,000 IU/mL. Alanine aminotransferase and
aspartate aminotransferase levels are within normal limits. (C) Nonalcoholic fatty liver disease
(D) Primary sclerosing cholangitis
Which of the following is the most appropriate next step in
management?
(A) Entecavir
Item 5
(B) Hepatic ultrasonography A 75-year-old man is evaluated for progressive dysphagia of
8 months' duration for both solid food and water, and the
(C) Pegylated interferon necessity to induce vomiting several times each month to
(D) Repeat liver chemistry tests in 6 months relieve his symptoms. He also has experienced chest pain
(E) Tenofovir and heartburn symptoms. He has lost approximately 6 kg
(13 lb) of weight over the precedlng 3 months and a total
of 9 kg (20 lb) since his symptoms began. He has a long
Item 3 hi.story of cigarette and alcohol use. His medical history and
A 58-year-old man is counseled before undergoing colonos- review of systems is otherwise negative. He has no travel
copy and polypectomy In 10 days' time. A routine screening history outside the northeastern United States. He takes no
CT colonography showed two polyps in the descending medication.
colon, 10 mm and 8 mm in size. Two years earlier, he had On physical examination, vital signs are normal; BMI is
an inferior wall myocardial infarction. His medications are 23. He appears thin and tired. The remainder of the physical
low-dose aspirin, atorvastatin, metoprolol, and enalapril. examination is unremarkable.

83
Self-Assessment Test

Upper endoscopic findings reveal retained saliva, liquid, supine, oxygen saturation is 98%breathing ambient air, but
and food in the esophagus without mechanical obstruction. oxygen saturation drops to 92% with standing.
Manometry demonstrates incomplete lower esophageal A radiograph of the chest is normal.
relaxation and aperistalsis.
Which of the following is the most appropriate diagnostic
Which of the following is the most likely diagnosis? test?
(A) Achalasia (A) Bronchoscopy
(B) Chagas disease (B) CT angiography
(C) Eosinophilic esophagitis (C) Echocardiography with agitated saline
(D) Pseudoachalasia (D) Pulmonary function testing

Item 6
A 35-year-old man is evaluated for a 1-year history of near-
Item 9 c::J
A 72-year-old woman is evaluated in the hospital for new-
daily postprandial diarrhea, episodic abdominal cramping onset abdominal pain in the right upper quadrant and fever
relieved with a bowel movement, and abdominal bloating. He that developed abruptly on hospital day 5. She was hospital-
is otherwise healthy, and his only medication is loperamide. ized 5 days earlier for altered mental status. Jn the emergency
This treatment has not been consistently effective in reducing department, she was found to be confused, hypotensive, and
diarrhea symptoms and has had no effect on the cramping tachycardic. She was transferred to the ICU, where she was
and bloating, despite increased frequency of dosing. diagnosed and treated for urosepsis. Within 24 hours, she was
Vital signs are normal. Diffuse tenderness to abdominal hemodynamically stable and the sepsis syndrome resolved.
palpation is noted. Other physical examination findings are She also has type 2 diabetes mellitus, hypertension, and
normal. hyperlipidemia. Her medications are insulin glargine, insulin
Stool testing for infection and celiac antibody testing aspart, lisinopril, atorvastatin, and piperacillin-tazobactam.
are negative. On physical examination, the patient is alert. Tempera-
Colonoscopy findings are unremarkable. ture is 38.2 °C (100.8 °F), blood pressure is 90/62 mm Hg,
Which of the following is the most appropriate treatment? pulse rate is 110/ min, and respiration rate is 20/min. Abdom-
inal examination is notable for tenderness to palpation of the
(A) Alosetron right upper quadrant. A soft palpable mass is felt in this area.
(B) Linaclotide Laboratory studies show a leukocyte count of 20,000/
(C) Low-FODMAP (Fermentable Oligosaccharides, µL (20 x 109 / L) and a serum total bilirubin level of 5 g/dL
Disaccharides, Monosaccharides, And Polyols) diet (85.5 µmol!L).
Ultrasonography shows a distended gallbladder with
(D) Lubiprostone
wall thickening and pericholecystic fluid. No gallstones are
seen and bile ducts are normal.
Item 7 Which of the following is the most appropriate next step in
A 52-year-old man is evaluated for dysphagia of 3 months' management?
duration. He reports regurgitating undigested food soon after
(A) Cholecystostomy tube placement
eating solid food, occasional coughing and choking after swal-
lowing, and chronic halitosis. He reports no weight loss or (B) Endoscopic retrograde cholangiopancreatography
chest pain. He drinks two beers weekly and does not smoke. (C) Hepatoblliary iminodiacetic acid scan
On physical examination, vital signs are normal; BMI is (D) MR cholangiopancreatography
25. The remainder of the examination, including abdominal
examination, is unremarkable.
Which of the following is the most appropriate diagnostic Item 10
test to perform next? A 45-year-old man is evaluated for severe lower back and hip
pain related to degenerative joint disease. His pain responds
(A) Barium esophagography to naproxen, but he was hospitalized 6 months earlier for
(B) Esophageal manometry a bleeding gastric ulcer attributed to daily naproxen use.
(C) 24-Hour esophageal pH monitoring Naproxen was stopped, and twice-daily omeprazole was
(D) Upper endoscopy initiated at that time. Three months later, the ulcer had
healed completely and omeprazole was discontinued. The
patient's pain did not respond to trials of acetaminophen.
Item 8 He did not tolerate tramadol. The patient has no other med-
A 48-year-old woman is evaluated for shortness of breath of ical problems and currently takes only acetaminophen.
6 weeks' duration. She has cirrhosis due to primary biliary Which of the following is the most appropriate treatment
cholangitis. regimen?
On physical examination, vital signs are normal. Spider
nevi are present on the skin. The cardiopulmonary exam- (A) Celecoxib
ination is normal. There is no edema. When the patient is (B) Celecoxib and omeprazole

84
Self-Assessment Test

(C) Ibuprofen and misoprostol Upper endoscopy shows an area of high-grade stenosis
(D) Naproxen and ranitidine in the distal esophagus.
Which of the following is the most appropriate treatment?
Item 11 (A) Increase fluticasone
A 55-year-old woman is evaluated after screening colonos- (B) Endoscopy with dilation
copy showed three polyps at the rectosigmoid junction. The (C) Omeprazole
three polyps were 3 mm, s mm, and 6 mm in size. All three (D) Oral prednisone
polyps were completely excised and pathology showed
them to be hyperplastic. Colonoscopy preparation was
excellent and the procedure was complete to the cecum.
Family history is signlflcant for colon cancer diagnosed in
Item 14 c::J
her paternal grandfather at age 80 years. A 24-year-old man is evaluated in the emergency depart-
All physical examination findings, including vital signs, ment for a 2-week history of worsening bloody diarrhea
are normal. with up to 10 bloody bowel movements per day. He also
reports increasing lower abdominal pain and distension
When should this patient next undergo colonoscopy? and that stool frequency has decreased over the past day.
He has extensive ulcerative colitis of 5 years' duration. His
(A) 1 year
medications are infliximab and azathioprine.
(B) 3 years On physical examination, the patient appears ill. Tem-
(C) 5 years perature is 38.3 °C (101 °F), blood pressure is 90/60 mm Hg,
(D) 10 years and pulse rate is 110/min; other vital signs are normal. The
abdomen is distended with guarding. Bowel sounds are
hypoactive.
CJ Item 12 Laboratory studies show a hemoglobin level of 10 g/dL
(100 g/ L), leukocyte count of 16,000/µL (16 x 109 / L), and
A 62-year-old woman is evaluated in the emergency depart-
ment for a 12-hour history of fever, dark urine, and abdomi- blood urea nitrogen level of26 mg/dL (9.3 mmol/L).
nal pain in the right upper quadrant. An abdominal radiograph is shown.
On physical examination, temperature is 39.7 °C (103.S °F);
other vital signs are normal. Her mental status is normal.
Sciera! icterus is noted. The abdomen is tender to palpation
in the right upper quadrant.
Laboratory studies:
Hematocrit 36%
Leukocyte count 14,600/µL (14.6 x 109 / L)
Platelet count 350,000/µL (350 x 109 / L)
Alanine aminotransferase 453 U/L
Total bilirubin 3.5 mg/dL (59.9 µmol/L)
An abdominal ultrasound shows a stone in the com-
mon bile duct and intra hepatic biliary dilation. Gallbladder
stones are also noted.
Antibiotic therapy is initiated.
Which of the following is the most appropriate next step in
management?
(A) Cholecystectomy
(B) Endoscopic retrograde cholangiopancreatography
(C) MR cholangiopancreatography Which of the following is the most appropriate next step in
(D) Percutaneous cholecystostomy management?
(A) Colectomy
Item 13 (B) Colonoscopy
A 30-year-old man is evaluated for ongoing symptoms of (C) CT of the abdomen
dysphagia. He was previously diagnosed with eosinophilic (D) Stool culture
esophagitis on upper endoscopy and has completed ans-week
course of swallowed aerosolized flutlcasone, which did not
alleviate his symptoms. He takes no other medications. Item 15
On physical examination, vital signs are normal; BMI is A 65-year-old woman is evaluated for watery, nonbloody
25. Other findings, including those of an abdominal exam- diarrhea of 8 months' duration. She reports up to six bowel
ination, are unremarkable. movements per day with nocturnal awakenings, but no

85
Self-Assessment Test

abdominal pain, bloating, flatulence, or weight loss. Her rate is 108/min, and respiration rate is 22/min. Jaundice is
last screening colonoscopy at age 60 years was normal. She apparent. Abdominal examination shows tenderness to pal-
has no personal or family history of colon cancer or polyps. pation in the right upper quadrant. The uterus is of appro-
She also has osteoarthritis, and her only medication is ibu- priate size for gestation.
profen approximately three times weekly. The patient is
Laboratory studies:
not exposed to children and has no recent travel history or 34%
Hematocrit
engagement in outdoor recreational activities. 6000/µL (6x109/ L)
Leukocyte count
On physical examination, vital signs are nonnal. The 155,000/µL (155x10 9 /L)
Platelet count
remainder of the physical examination, including abdomi- 2.2
INR
nal examination, is normal. Alanine aminotransferase 115 U/ L
Laboratory studies show hemoglobin, serum albumin, Aspartate aminotransferase 130 U/L
total lgA, and tissue transglutaminase IgA levels to be within 6.2 mg/dL (106.0 µmoi / L)
Total bilirubin
normal limits. Glucose 55 mg/dL (3.1 µmol / L)
Which of the following is the most likely diagnosis?
On abdominal ultrasonography, the liver is hypere-
(A) Giardia lamblia infection choic. Hepatic vasculature is patent, and there is no bile-
(B) Microscopic colitis duct dilation.
She is transferred to an ICU setting, and intravenous
(C) Small intestinal bacterial overgrowth
fluids with glucose are administered.
(D) Ulcerative colitis
Which of the following is the most appropriate next step in
management?
c::J Item 16 (A) Endoscopic retrograde cholangiopancreatography
A 78-year-old man is evaluated in the emergency depart- (B) Immediate delivery of the fetus
ment for worsening jaundice of 2 weeks' duration, dry (C) Lactulose
mouth, dark urine, and light stools. He has also noticed
(D) Ursodeoxycholic acid
swelling under his jaw bilaterally. He reports no abdominal
pain or weight loss.
On physical examination, vital signs are normal. Swol-
len submandibular glands bilaterally and jaundice are Item 1 8
noted. A 65-year-old man is evaluated after a screening ultrasound
for abdominal aortic aneurysm showed incidental gallblad-
Laboratory studies: der findings. He reports no symptoms. He continues to
Alkaline phosphatase 180 U/L smoke cigarettes, 1 pack per day. He has no other medical
Alanine aminotransferase 66 U/L problems and takes no medications.
Aspartate aminotransferase 55 U/L On physical examination, vital signs are nonnal, as is
Total bilirubin 6.2 mgldL (106.0 µmol/L) the remainder of the examination.
Direct bilirubin 4.8 mg/dL (82.1 µmol/L) The results of all laboratory studies, including a com-
A CT scan of the abdomen shows a narrowed pancre- plete blood count and alkaline phosphatase, alanine amino-
atic duct and enlargement of the pancreas parenchyma, transferase, aspartate aminotransferase, and total billrubin
levels, are within normal limits.
described as a ~sausage - shaped" pancreas, as well as
The abdominal ultrasound shows numerous layering
findings consistent with retroperitoneal fibrosis. Endo-
gallstones and an immobile 8-mm gallbladder polyp.
scopic ultrasound- guided biopsy of the pancreas shows
more than 10 IgG4-positive cells/hpf and no evidence of Which of the following is the most appropriate next step in
malignancy. management?
Which of the following is the most appropriate treatment? (A) Cholecystectomy
(A) Azathioprine (B) MR cholangiopancreatography
(B) Endoscopic retrograde cholangiopancreatography (C) Repeat ultrasonography in 6 months
with bile-duct stenting (D) Ursodeoxycholic acid
(C) Pancreaticoduodenectomy
(D) Prednisone
Item 19
A 60-year-old woman is evaluated 1 month after com-
c::J Item 17 pleting a 14-day course of Heltcobacter pylori eradication
A 25-year-old woman is evaluated in the hospital for therapy consisting of amoxicillin, clarithromycin, and ome-
right-upper-quadrant abdominal pain, jaundice, and nau- prazole. Initial upper endoscopy before treatment showed
sea of 10 days' duration. She is in her 35th week of preg- patchy gastric erythema with no ulcers or erosions, and
nancy. Her only medication is a prenatal vitamin. biopsies revealed H. pylori gastritis. Currently, she reports
On physical examination, the patient is drowsy. Tem- alleviated symptoms. She is otherwise healthy and takes no
perature is normal. Blood pressure is 95/60 mm Hg, pulse medication.

86
Self-Assessment Test

Which of the following is the most appropriate test to 2.3-kg (5.1-lb) weight loss since her symptoms started. She
perform next? has no personal or family history of colon cancer or other
cancers. She takes no medication.
(A) Serologic antibody testing for H. pylori
On physical examination, her pulse rate is 102/min;
(B) Upper endoscopy with gastric biopsy other vital signs are normal. BM! is 22. Abdominal exam-
(C) Urea breath test ination reveals mild tenderness in the left lower quadrant
(D) No further testing with no rebound or guarding. Rectal examination shows no
masses. Large, friable hemorrhoids are present.
Laboratory studies show a hemoglobin level ofl0.5 g/dL
CJ Item 20 (105 g/L) and a serum ferritin level ors ng/mL (5 µg/L).
A 36-year-old woman is evaluated 3 days after being hos- Which of the following is the most appropriate next step in
pitalized for gallstone pancreatitis. An abdominal ultra- management?
sound showed multiple gallstones in the gallbladder and
a normal-diameter common bile duct. She was treated (A) Anoscopy
with intravenous hydration and pain medication. Over the (B) Colonoscopy
course of 3 days, she tolerated eating and her pain subsided. (C) Flexible sigmoidoscopy
On physical examination, vital signs are normal; BM! is (D) Topical hemorrhoid treatment
32. Minimal tenderness to palpation is noted on abdominal
examination. All other findings are unremarkable.
All laboratory studies have returned to baseline normal
values. Item 23
A 52-year-old woman is evaluated for midabdominal pain
Which ofthe following is the most appropriate treatment? and nausea. These symptoms worsen after eating large
(A) Endoscopic retrograde cholangiopancreatography meals. The pain does not alleviate after bowel movements,
(B) Laparoscopic cholecystectomy before discharge from and she reports frequent constipation. She has gained 2.3 kg
the hospital (5.1 lb) over the past year. The pain began 1 year earlier, after
she was hospitalized with an episode of gallstone pancreatl-
(C) Laparoscopic cholecystectomy 4 weeks after dis- tis for which she underwent laparoscopic cholecystectomy.
charge from the hospital After cholecystectomy, she had persistent abdominal pain.
(D) MR cholangiopancreatography Over the next several weeks she had upper and lower endos-
(E) Ursodeoxycholic acid copies, a CT scan of the abdomen and pelvis, and multiple
laboratory tests; all were normal. She states that the only
thing that helps the pain is hydrocodone, but as the dose of
CJ Item 21 hydrocodone has increased, so has her pain intensity.
On physical examination, vital signs are normal; BMI
A 48-year-old man ls hospitalized with a fever and rigors.
He has primary sclerosing cholangitis and ulcerative colitis. is 26. Tenderness to palpation in all quadrants is noted on
His only medication is mesalamine. abdominal examination. Other findings are unremarkable.
On physical examination, temperature is 38.6 °C Which of the following is the most likely diagnosis?
(101.5 °F), blood pressure is 118/75 mm Hg, pulse rate is
95/ min, and respiration rate is 18/ min. Abdominal exam- (A) Chronic pancreatitis
ination is notable for right-upper-quadrant tenderness to (B) Distal intestinal obstruction syndrome
palpation. The remainder of the examination is normal. (C) Irritable bowel syndrome
Laboratory studies show a leukocyte count of14,000/µL
(D) Narcotic bowel syndrome
(14x10 9 / L) . Blood cultures are pending.
A 1-L bolus of intravenous normal saline is adminis-
tered, and intravenous piperacillin- tazobactam is given.
Item 24
Which of the following is the most appropriate test to
A 60-year-old man is evaluated for constant low-grade
perform next?
eplgastric pain radiating to his back that worsens after he
(A) CA 19-9 measurement eats fatty foods . He has a 2-year history of chronic pan-
(B) Endoscopic retrograde cholangiopancreatography creatitis. The pain has progressively worsened over the
preceding 6 months. His weight is stable. He has a normal
(C) IgG4 measurement
bowel movement once every other day. He does not drink
(D) Percutaneous transhepatic biliary tube placement alcohol. He continues to smoke 2 packs of cigarettes per
(E) PETscan day as he has done for 30 years. The patient has a remote
history of opioid abuse.
On physical examination, vital signs are normal.
Item 22 Abdominal examination shows tenderness to palpation in
A 39-year-old woman ls evaluated for fatigue, intermittent the epigastrium.
rectal bleeding, and abdominal pain over the past 2 months. A contrast-enhanced CT scan of the abdomen shows
She reports that the bleeding is not accompanied by anal calcifications throughout the pancreas with no pancreatic
pain or itching. She has experienced an unintentional mass, no dilated pancreatic duct, and no cystic lesions.

87
Self-Assessment Test

Which of the following is the most appropriate initial


treatment?
Item 26 CJ
A 34-year-old woman is evaluated 4 days after being hospi-
(A) Celiac plexus block talized for severe epigastric abdominal pain associated with
(B) NSAIDs, low-fat diet, and smoking cessation nausea and vomiting. At the time of hospitalization, her
serum lipase level was 4650 U/L and an abdominal ultra-
(C) Oxycodone
sound examination showed cholelithiasis. She has taken
(D) Pancreatic enzyme replacement therapy nothing by mouth (NPO) since hospitalization and reports
having no appetite. She is being treated with intravenous
fluids and opioid pain medication. A CT scan of the abdo-
Item 25 men and pelvis with contrast showed changes consistent
A 43-year-old man is evaluated for 4 days of fever and with pancreatic necrosis. Her medical history is unremark-
arthralgla, as well as a raised purple rash over his lower able and she took no medication before hospitalization.
extremities ofl week's duration. He is a current intravenous On physical examination, temperature is 38 °C (100.4 °F),
drug user. He has no other medical problems and takes no blood pressure is 130/70 mm Hg, pulse rate is 90/min,
medications. and respiration rate is 15/min. Abdominal examination
On physical examination, temperature is 37.8 °C (100 °F) shows tenderness in the epigastrium with no abdominal
and blood pressure is 132/85 mm Hg; other vital signs are distention.
normal. The large and small joints are tender to palpation
without evidence of synovitis. The abdomen is not tender to Which of the following is the most appropriate next step in
palpation. The liver edge is palpable below the right costal management of her nutrition?
margin. The skin findings are shown. (A) Initiate enteral feeding
(B) Initiate total parenteral nutrition
(C) Maintain NPO status until cholecystectomy is
performed
(D) Maintain NPO status until the lipase level normalizes

Item 27 CJ
A SS-year-old man is evaluated in the hospital for new-onset
ascites. He has a history of cirrhosis due to hepatitis C viral
infection. His only medication is propranolol.
On physical examination, pulse rate is 58/min; other
vital signs are normal. The abdomen is soft and distended
consistent with ascites.
Laboratory studies show a serum albumin level of2.5 g/ dL
(25 g/L), serum total bilirubin level of3.6 mg/dL (61.6 µmol/L),
and serum creatinine level ofl.4 mg/dL (123.8 µmol/L).
Laboratory studies: Paracentesis with analysis of ascitic fluid shows a leu-
Alanine aminotransferase 143 U/L kocyte count of200/µL with 30% neutrophils, albumin level
Aspartate aminotransferase 130 U/L of0.4 g/ dL (4 g/ L), and total protein level of0.9 g/dL (9 g/L).
Total bilirubin 2.2 mg/dL (37.6 µmol/L)
Creatinine 1.0 mg/dL (88.4 µmol/L) Which of the following is the most appropriate next step in
Hepatitis B surface antigen management?
Positive
IgM antibody to hepatitis B core Positive (A} Increase propranolol therapy
antigen (B) Initiate albumin infusion
Hepatitis B e antigen Positive
(C) Initiate ciprofloxacin therapy
HBVDNA 10,000,000 IU/mL
Hepatitis C antibody Negative (D) Initiate lisinopril therapy
HIV -1/HIV-2 antigen and Negative
antibody combination
immunoassay Item 28
Cryoglobulin Normal A 53-year-old woman is evaluated for an 8-month history
Urinalysis Normal of palpitations, tachycardia, lightheadedness, and sweating
with explosive diarrhea and nausea within 15 to 30 min-
Which of the following is the most appropriate treatment? utes of eating. She has lost 6.8 kg (15 lb) during this time
(A) Entecavir period and describes feeling better if she eats smaller por-
(B) Plasma exchange tions. She reports no vomiting. Upper endoscopy performed
1 year earlier for refractory gastroesophageal reflux disease
(C) Prednisone and cyclophosphamide incidentally found a small gastrointestinal stromal tumor in
(D) Sofosbuvir and ledipasvir her stomach, and she underwent a partial gastrectomy with
(E) Tenofovir, emtricitabine, and raltegravir gastrojejunostomy. Her only medication is omeprazole.

88
Self-Assessment Test

On physical examination, vital signs are normal; BM1 is preceding 6 months. He does not smoke or drink alcohol.
28. Abdominal examination shows a well-healed scar with He takes no medication.
no abdominal pain on palpation and no organomegaly. On physical examination, vital signs are normal; BMI
is 20. Abdominal examination shows a thin abdomen with
Which of the following is the most likely diagnosis?
tenderness to deep palpation in the epigastrium. No jaun-
(A) Dumping syndrome dice is noted.
(B) Gastroparesis Which of the following is the most appropriate next step ln
(C) Irritable bowel syndrome management?
(D) Small intestinal bacterial overgrowth
(A) er of the abdomen wit h contrast
(B) Endoscopic retrograde cholangiopancreatography
Item 29 (C) Endoscopic ultrasound
A 35-year-old woman is evaluated for constipation. She (D) Clinical observation
reports passage of hard stool every 3 to 4 days and associ-
ated bloating. Her symptoms have been present for more
than 10 years and have progressed gradually. Trials of over-
the-counter fiber supplementation and polyethylene glycol
Item 32 CJ
A 75-year-old man is evaluated after hospitalization for an
worsened the bloating, prompting discontinuation. Senna episode of hematemesis. Upon admission, his iNR was 2.5.
tea was ineffective. Bisacodyl caused abdominal cramping. Intravenous proton pump inhibitor therapy was initiated, and
She is otherwise healthy and currently takes no medication. his warfarin was held. Endoscopy showed a large duodenal
On physical examination, vital signs and other findings bulb ulcer with an actively bleeding visible vessel; endoscopic
are normal. hemostasis was achieved. Vital signs have been normal for
Which of the following is the most appropriate treatment? 24 hours after endoscopy. The patient also has hypertension
and atrial fibrillation and a history of transient ischemic attack.
(A) Lactulose His daily medications are warfarin, losartan, and metoprolol.
(B) Llnaclotide On physical examination, vital signs are normal; BM!
(C) Methylnaltrexone is 32. Other than an irregularly irregular heart rhythm, the
physical examination is normal.
(D) Rifaximin
Which of the following is the most appropriate manage-
ment of the patient's warfarin therapy?
Item 30
(A) Restart warfarin therapy now
A 28-year-old woman is evaluated at 28 weeks' gestation.
This is her first pregnancy. She has chronic hepatitis B virus (B) Restart warfarin therapy in 10 days
(HBV) infection acquired through vertical transmission. The (C) Restart warfarin therapy in 30 days
patient reports feeling well. Her only medication is a prena- (D) Hold warfarin therapy indefinitely
tal vitamin.
On physical examination, vital signs are normal. The
uterus is enlarged, consistent with 28-week intrauterine Item 33
gestation. No stigmata of chronic liver disease are noted.
Laboratory studies are positive for hepatitis B surface A 32-year-old woman is evaluated for a 6-month history of
antigen and hepatitis B e antigen. The HBV DNA level is loose stools, bloating, and a 3.2-kg (7-lb) weight loss. Her
300,000 IU/mL. The results of other studies, including ala- medical history is otherwise unremarkable. Her brother has
nine aminotransferase, aspartate aminotransferase, and type 1 diabetes mellitus, and her mother has autoimmune
total bilirubin levels, are within normal limits. thyroid disease. She reports no other symptoms and takes
no medication.
Which of the following is the most appropriate next step On physical examination, vital signs and other findings
in management? are normal; BMI Is 19.
(A) Cesarean delivery at term I..aboratocy studies:
Hemoglobin 11.0 g/dL (110 g/L)
(B) HBV DNA measurement in 3 months
Alanine aminotransferase 60 U/L
(C) Pegylated interferon Aspartate aminotransferase 42U/L
(D) Tenofovir Total bilirubin 0.9 mg/dL (15.4 µmol /L)
Ferri tin 6 ng/mL (6 µg/L)
Item 31 Which of the following is the most appropriate test to
A 60-year-old man is evaluated following the diagnosis of perform next?
acute pancreatltis. Abdominal ultrasonography showed a (A) Anti-gliadln JgA antibody
normal gallbladder, biliary system, and liver, but the pan-
creas was not well visualized due to overlying loops of (B) Anti- Saccharomyces cereuisiae lgA antibody
bowel. He reports decreased appetite, vague new back pain, (C) Anti- smooth muscle antibody
and an unintentional 6.9-kg (15.2-lb) weight loss over the (D) Anti- tissue transglutaminase IgA antibody

89
Self-Assessment Test

Item 34 Which of the following is the most appropriate test to


A 63-year-old woman is evaluated at a follow-up appoint- perfonn next?
ment after undergoing right hemicolectomy for colon can- (A) Angiography
cer 1 year earlier. Her paternal grandfather had colon cancer
(B) Capsule endoscopy
diagnosed at age 7S years. She reports that she has been
feeling well and takes no medication_ (C) Colonoscopy
All physical examination findings, including vital signs, (D) Upper endoscopy
are normal.
When should this patient's next surveillance colonoscopy Item 37
take place? A 72-year--0ld woman is evaluated for a 3-month history
(A) Now of large-volume, watery, nonbloody diarrhea with mucus,
which occurs even when she is fasting. She reports no abdom-
(B) In lyear
inal pain or flushing but has experienced an unintentional
(C) In 3 years 3-kg (6.6-lb) weight loss. She has never had a colonoscopy.
(D) In S years On physical examination, blood pressure is 100/SO mm
Hg and pulse rate is 95/min; other vital signs and examina-
tion findings are normal.
Item 35 Laboratory studies show a serum sodium level of
A 60-year-old man is evaluated for epigastric pain that 130 mEq/L (130 mmol/L) and a serum potassium level of
occurs intermittently after eating. He was hospitalized twice 3.3 mEq/L (3.3 mmol/L). Hemoglobin and serum creatinine
in the preceding year for idiopathic recurrent acute pancre- levels are within normal limits.
atitis_ He is a former smoker and reports no alcohol use. Which of the following types of diarrhea is most compatible
On physical examination, vital signs are normal, as is with the patient's findings?
the remainder of the examination_
Contrast-enhanced CT of the abdomen shows a dif- (A) Infectious
fusely dilated main pancreatic duct with normal intrahe- (B) Inflammatory
patic and extrahepatic bile ducts and a normal gallbladder. (C) Osmotic
No tumor is seen in the pancreas or liver, and there is no
(D) Secretory
peripancreatic inflammation or necrosis. Endoscopic ultra-
sonography confirms no evidence of a tumor or gallstone,
but the main pancreatic duct appears dilated throughout Item 38
the pancreas. Mucin is seen exuding from the ampulla of
A 2S-year-old woman is evaluated for a I-month history
Yater during the endoscopy.
of intermittent, loose, bloody bowel movements associated
Which of the following is the most appropriate next step in with pain in the left lower quadrant before defecation. The
management? bleeding occurs with and without bowel movements. She
reports no fever, chills, night sweats, arthralgia, eye pain, or
(A) Endoscopic ultrasonography in 1 year rash. The patient works in a day care center and has not trav-
(B) MRI of the abdomen in 1 year eled or used antibiotics recently. She takes no medication.
(C) Oral prednisone On physical examination, vital signs are normal. The
(D) Pancreatic resection abdomen is scaphoid and soft, with suprapubic tenderness.
The remainder of the physical examination is normal.
Stool testing for bacterial enteropathogens, including
CJ Ite m 36 Clostridium difficile, is negative.
Colonoscopy results show patchy erythema and ulcer-
A 36-year-old man is evaluated in the emergency depart- ation in the cecum, ascending colon, descending colon, and
ment after passing three bowel movements of red to sigmoid colon with no involvement of the transverse colon
maroon- colored blood. He recently injured his knee playing or rectum. Mucosal biopsies of the involved mucosa reveal
soccer and has been taking ibuprofen three times a day for distorted and branching colonic crypts with lymphocytlc
a week. He has no other relevant medical history and does and neutrophilic infiltration. Heal examination is normal.
not smoke or drink alcohol.
On physical examination, he is lightheaded but alert. Which of the following is the most likely diagnosis?
His blood pressure is 80/60 mm Hg, pulse rate is 126/min,
(A) Crohn disease
respiratory rate is 12/min, and oxygen saturation is 98%
breathing ambient air. Cardiac examination shows tachy- (B) Giardiasis
cardia. Rectal examination shows maroon-colored stool. All (C) Microscopic colitis
other findings are unremarkable. (D) Ulcerative colitis
The patient's vital signs improve after he is given intra-
venous hydration.
Laboratory studies show a hemoglobin level of 9 g/dL Item 39
(90 g/ L) upon presentation. Six hours later, the hemoglobin A SS-year-old man is evaluated at a follow-up appointment
level is 7.4 g/dL (74 g/ L). 4 days after being diagnosed with acute sigmoid diverticulitis.

90
Self-Assessment Test

He is on day 4 of oral antibiotics and reports that his pain is


almost gone. He has no other symptoms and has never had
a colonoscopy. He takes no other medication.
His vital signs are normal. Abdominal examination is
positive for minimal tenderness on deep palpation in the left
lower quadrant without rebound or guarding.
The CT scan of the abdomen from the emergency
department shows mild sigmoid diverticulitis with no
abscess.
When should this patient undergo colonoscopy?
(A)Now
(B)In 1 to 2 months
(C) In 6 months
(D) In 12 months

Item 40
A 65-year-old woman is reevaluated following an initial
evaluation for anemia. Other than a gradually increasing (C) Prednisone suppository
sense of fatigue, she has no symptoms. Her only other med- (D) Sitz baths and psyllium
ical problem is autoimmune thyroid disease, which is being (E) Topical hydrocortisone and lubiprostone
treated with levothyroxine. Her last screening colonoscopy,
done 4 years earlier, was normal.
Physical examination reveals normal vital signs. There Item 42
is evidence of conjunctiva] rim pallor. The remainder of the A 25-year-old man is evaluated for worsening heartburn
examination, including thyroid and neurologic examina- of 4 months' duration, despite treatment with twice-daily
tions, is normal. omeprazole. He has no pertinent personal or family medical
At the time of her initial evaluation, laboratory stud- history and takes no other medication.
ies showed a hemoglobin level of 10 g/dL (100 g/L) and On physical examination, vital signs and other findings
mean corpuscular volume of104 fL. Serum cobalamin and are normal.
ferrltin levels were low. An antiparietal cell antibody test Upper endoscopy shows a normal esophagus with
was positive. Stool testing revealed no evidence of blood. a normal gastroesophageal junction. The stomach has
Which of the following is the most appropriate test to 30 small (<10 nun) sessile polyps seen in the fundus. The
perform next? duodenum is normal. Pathology of a polyp shows it to be a
fundic gland polyp.
(A) Capsule endoscopy
Which of the following is the most appropriate next step in
(B) Glucose hydrogen breath test
evaluation of this patient?
(C) Serum gastrin measurement
(D) Upper endoscopy (A) Colonoscopy
(B) Gastrectomy
(C) Repeat upper endoscopy in 3 months
Item 41 (D) No further evaluation
A 28-year-old woman is evaluated for 2 weeks of intense
rectal pain that occurs with most bowel movements. She
also reports periodic rectal bleeding and occasional con- Item 43
stipation symptoms that have worsened since a recent A 44-year-old man is evaluated after his routine yearly
shoulder surgery. Over-the-counter hemorrhoid cream follow-up for ulcerative colitis showed elevated liver chem-
has not alleviated her symptoms. Family history includes istry test results. He was diagnosed with ulcerative colitis
colon cancer diagnosed in her grandmother at age 5 years ago. He reports that he passes solid, formed stool
74 years. The patient's medications are acetaminophen every 1 to 2 days without abdominal pain. His only medica-
with codeine and an over-the-counter stool softener as tion Is mesalamine.
needed. On physical examination, vital signs are normal; BMI is
On physical examination, vital signs are normal. Rectal 26. Hepatomegaly and palmar erythema are noted.
examination findings are shown (see top of next column). Laboratory studies:
Which of the foJlowlng Is the most appropriate next step in Alkaline phosphatase 412 U/L
management? Alanine aminotransferase 78 U/L
Aspartate aminotransferase 62 U/L
(A) Anal botulinum toxin injection Total bilirubin 2 mg/dL (34.2 µmol/L)
(B) Flexible sigmoidoscopy Direct bilirubin 1.5 mg/dL (25.7 µmol/L)

91
Self-Assessment Test

A complete blood count and serum creatinine level


are normal, and an antimitochondrial antibody titer is
Item 4 6
A 30-year-old man is evaluated after being hospitalized
CJ
negative.
for an acute flare of extensive ulcerative colitis. He reports
MR cholangiopancreatography shows hepatomegaly six to eight bloody bowel movements daily with promi-
but is otherwise normal. nent urgency and lower abdominal cramping for the past
Liver biopsy is pending. 2 weeks. He has been taking prednisone daily for 1 week. His
Which of the following is the most likely diagnosis? only other medication is mesalamine.
On physical examination, vital signs are normal.
(A) Antimitochondrial antibody- negative primary biliary Abdominal examination reveals lower abdominal tender-
cholangitis ness. The abdomen is not distended and bowel sounds are
(B) Drug-induced liver injury normal. Blood is seen on digital rectal examination.
(C) Nonalcoholic fatty liver disease Hemoglobin level is 10 g/dL (100 g/L). He remains
(D) Primary sclerosing cholangitis hemodynamically stable.
Which of the following is the most appropriate venous
thromboembolism prophylaxis for this patient?
Item 44 (A) Aspirin
A 45-year-old man is evaluated for a 2-month history of (B) Graduated compression stockings
a burning sensation starting in his stomach and radiating
(C) Heparin
into his chest, usually occurring 4 to 5 times weekly. He says
that he usually eats dinner late and then goes to sleep. He (D) Intermittent pneumatic compression
often wakes up with a sour taste in his mouth. He reports (E) Low-dose warfarin
no dysphagia or unintentional weight loss. He takes no
medication.
On physical examination, vital signs are normal; BMI is Item 47
34. The remainder of the examination, including abdominal A 37-year-old man is evaluated during a follow-up appoint-
examination, is unremarkable. ment after resection of a well-differentiated adenocarcinoma
Which of the following is the most appropriate next step in of the ascending colon, which was diagnosed 1 month earlier.
management? His family history is significant for endometrial cancer diag-
nosed in his mother at age 50 years and colon cancer diag-
(A) Ambulatory pH testing nosed in his maternal grandfather at age 49 years. The patient
(B) Barium esophagography reports that he has been feeling well and takes no medication.
(C) Empiric trial of a proton pump inhibitor All physical examination findings, including vital signs,
(D) Upper endoscopy
are normal.
Which of the following is the most appropriate next step in
management? ·
Item 45 (A) Colonoscopy in 3 years
A 45-year-old woman is evaluated during a routine (B) Fecal immunochemical testing in 1 year
follow-up appointment for cirrhosis due to primary bili- (C) Genetic counseling
ary cholangitis. She has chronic symptoms of fatigue that
are unchanged since her previous visits. She has no other (D) Upper endoscopy and capsule endoscopy
new symptoms of advancing liver dysfunction, including
no ascites, jaundice, or hepatic encephalopathy. A recent
upper endoscopy showed small esophageal varices with- Item 48
out red wale marks. Her only medication is ursodeoxy- A 35-year-old man is evaluated during a follow-up appoint-
cholic acid. She is up to date for her influenza, 23-valent ment for persistent heartburn with chronic cough. He has a
polysaccharide pneumococcal, and hepatitis A and B 1-year history of gastroesophageal reflux disease and takes
immunizations. pantoprazole twice daily. He reports no nausea, vomiting,
On physical examination, vital signs are normal. Scat- or dysphagia. Upper endoscopy performed 1 year earlier
tered spider angiomas are seen on her chest and upper back. showed no abnormal findings.
The remainder of the examination is normal. His vital signs and physical examination are normal.
An abdominal ultrasound shows changes in the liver Results of an ear, nose, and throat evaluation are noncon-
consistent with cirrhosis. No liver masses are seen. tributory.
Which of the following is the most appropriate intervention Which of the following is the most appropriate next diag-
to perform next? nostic test?
(A) Bone densitometry (A) Ambulatory pH testing
(B) Colonoscopy (B) Barium esophagography
(C) Contrast-enhanced abdominal CT (C) Esophageal manometry
(D) Herpes zoster immunization (D) Upper endoscopy

92
Self-Assessment Test

Item 49 Item 51
A SS-year-old man is evaluated for ascites. He recently went A 21-year-old woman is evaluated for a 6-week history of
to the emergency department, where paracentesis was per- frequent bowel movements (three times daily) with inter-
formed. He was then discharged for outpatient follow-up. mittent passage of small amounts of blood and mucus. She
He has a history of cirrhosis due to nonalcoholic steatohepa- also reports suprapubic cramping pain that is relieved with
tit!s and also has hypertension. Endoscopy 3 months earlier passage of stool. She reports no fever, chills, nausea, vom-
showed small varices without stigmata, making prophy- iting, or weight loss. She is otherwise healthy and tal<es no
laxis for esophageal variceal bleeding unnecessary. His only medication.
medication is lisinopril. On physical examination, vital signs are normal. The
On physical examination, vital signs are normal; BMI abdomen is scaphoid, with tenderness to palpation in the
is 28. Abdominal examination shows abdominal distention suprapubic area. Rectal examination is notable for a small
without tenderness. amount of blood on the examining finger.
Laboratory studies of the ascitic fluid show a leuko- Laboratory studies show a normal complete blood
cyte count of 80/µL with 20% neutrophils and protein level count and C-reactive protein level. Stool testing for entero-
of 1.6 g/dL (16 g/L). Serum studies show a creatinine level pathogens, including Clostridium difficile, is negative.
of 1.3 mg/dL (114. 9 µmol/L) and sodium level of134 mEq/L Results of colonoscopy show continuous, symmetric
(134 mmol/L). rectal and sigmoid inflammation characterized by ery-
An abdominal ultrasound from the emergency depart- thema, edema, and friable mucosa. The remainder of the
ment shows changes consistent with cirrhosis. The portal colonic mucosa and distal ileum is normal. Biopsy speci-
vein and hepatic veins are patent with normal flow direc- mens from the rectum and sigmoid colon show evidence of
tion. A moderate amount of free-flowing ascites ls seen. mildly active chronic colitis.
In addition to initiating a sodium-restricted diet, which of Which of the following is the most appropriate
the following is the most appropriate next step in manage- treatment?
ment?
(A) Mesalarnine enema
(A) Discontinue lisinopril (B) Mesalamine suppository
(B) Initiate free-water restriction (C) Oral mesalamine
(C) Initiate propranolol (D) Oral mesalarnine and mesalamine enema
(D) Insert an indwelling drain into the peritoneal cavity

Ite m 52
CJ Item 50 A 20-year-old man is evaluated for epigastric pain that has
gradually increased in severity over 8 months. The pain
A 30-year-old man is evaluated in the emergency depart-
ment for the recent onset of confusion and jaundice of worsens with eating and is not relieved by antacids. The
6 weeks' duration. His medical history is unremarkable. patient reports no melena, diarrhea, or constipation. The
On physical examination, the patient is confused. His patient's personal medical history is unremarkable and
temperature is 36.9 °C (98.4 °F), blood pressure is 102/ he takes no medication.
60 mm Hg, pulse rate is US/min, and respiration rate is 22/min. On physical examination, vital signs are normal. Epi-
Jaundice and asterixis are noted. No organomegaly is noted. gastric tenderness to palpation is noted. Other findings are
normal.
Laboratory studies: A complete blood count is normal.
Hematocrit 38%
Leukocyte count 11,000/µL (11x10 9 /L) Which of the following is the most appropriate next step in
Platelet count 3SO,OOO/µL (350x10 9 / L) management?
INR 2.6 (A) Initiation of omeprazole
Alanine aminotransferase 200 U/ L
lSO U/L (B) Stool antigen testing for Helicobacter pylori
Aspartate aminotransferase
Total bilirubin 18.6 mg/dL (318.1 µmol/L) (C) Ultrasonography of the right upper quadrant
Creatinine 3.0 mg/dL (26S.2 µmol /L) (D) Upper endoscopy

On abdominal ultrasound, the liver appears normal


with intact vasculature. No splenomegaly or dilation of bile Item 53
ducts is noted.
A 46-year-old man is evaluated for abdominal pain in the
Which of the following is the most appropriate next step in right upper quadrant and fever of 1 month's duration. He
management? recently emigrated from Mexico. His medical history is
unremarkable, and he takes no medication.
(A) Administer fresh frozen plasma On physical examination, temperature is 37. 7 °C (99. 9 °F);
(B) Initiate hemodialysis other vital signs are normal. Abdominal examination shows
(C) Perform endoscopic retrograde cholangiopancreatog- tenderness to palpation of the right upper quadrant. No
raphy scleral icterus is noted. The remainder of the examination
(D) Refer for liver transplantation is normal.

93
Self-Assessment Test

An ultrasound of the liver shows a fluid-containing Which of the following is the most appropriate test to
structure and complex wall consistent with hepatic abscess. perform next?
Laboratory studies show a leukocyte count of' 10,600/µL
(10.6 x 109 /L) . Testing for Entomoeba histolytica JgG is (A) AntimitochondriaJ antibody test
positive. (B) Hepatitis B virus DNA measurement
(C) Serum ceruloplasmin measurement
Which of the following is the most appropriate treatment?
(D) Transferrin saturation measurement
(A) Meropenem
(B) Metronidazole and paromomycin
(C) Percutaneous drainage of the abscess Item 56
(D) Surgical resection A SS-year-old man is evaluated for progressive dysphagia
of 2 years' duration. He reports dysphagia to both solid
food and liquids. He has cardiomyopathy, with an ejection
Item 54 fraction of 15%. His medications are pantoprazole, furose-
A 26-year-old woman is evaluated for a 2-month history of mide, valsartan, digoxin, metoprolol, low-dose aspirin, and
diarrhea characterized by two to three semibloody stools amiodarone. He is unable to walk up two flights of stairs
per day associated with cramping lower abdominal pain. without stopping. He does not drink or smoke.
She reports no fever, chills, nausea, vomiting, or weight On physical examination, his blood pressure is 100/65
loss. She has not traveled internationally. She takes no mm Hg, pulse rate is 90/min, and respiratory rate is 22/min;
medication. BMI is 32. His examination is remarkable for fine crackles
On physical examination, vital signs are normal. at the lung bases posteriorly, a third heart sound, and 2+
Abdominal examination shows left and right lower abdom- pitting edema to the knees.
inal tenderness to palpation. Rectal examination is remark- Upper endoscopy shows no masses. A barium esopha-
able for bright red blood. gram is shown.
Stool testing for enteric pathogens is negative.
Results of colonoscopy show inflamed mucosa char-
acterized by granularity, erythema, friability, and loss
of vascular pattern that starts at the anorectal verge and
extends proximally in a continuous and symmetric fash-
ion to the splenic flexure where there is an abrupt transi-
tion from affected to normal mucosa. The terminal ileum
is nom1al. Biopsy results for the inflamed mucosa reveal
crypt abscesses along with distorted and branching colonic
crypts.
Which of the following is the most likely diagnosis?
(A) Chronic Entomoeba histolytica infection
(B)Crohn colitis
(C) Cytomegalovtrus infection
(D) Ulcerative colitis

Item 55
A 26-year-old woman is evaluated for the new onset of
jaundice. She reports no fever or abdominal pain. Her med-
ical history is unremarkable and she takes no medication.
On physical examination, vital signs are normal. Sciera!
lcterus and psychomotor slowing are noted.
Laboratory studies:
Hemoglobin 10.2 g/dL (102 g/L)
Reticulocyte count 8% of total erythrocyte count
Alkaline phosphatase 26 U/L
Alanine aminotransferase 78U/L
Aspartate aminotransferase 156 U/L
Total b!lirubin 6.4 mg/dL (109.4 µmol/L)
Conjugated bilirubin 2.6 mg/dL (44.S µmol/L)
Testing for hepatitis A, B, and C viral infections is neg-
ative for acute infection.
On abdominal ultrasonography, the liver is small and
nodular. Splenomegaly is noted.

94
Self-Assessment Test

Which of the following is the most appropriate treatment? Item 59


(A) Botulinum toxin injection A 75-year-old woman with longstanding gastroesophageal
(B) Calcium channel blockers reflux disease (GERD) comes to the office with concerns
related to her new diagnosis of osteopenia. Her GERO is well
(C) Endoscopic pneumatic dilation
controlled with once-daily pantoprazole, which she has
(D) Laparoscopic surgical myotomy taken without side effects for 1 year. She also takes calcium
and vitamin D supplements. Other than her age, she has no
additional risk factors for osteoporosis.
Item 57
Her vital signs and the remainder of the physical exam-
A 56-year-old woman is evaluated for chest discomfort after ination are normal.
meals occurring intermittently over the preceding month.
She describes a sensation of heaviness on her chest, and says Which of the following ls the most appropriate next step in
that she also notices this pain sometimes while walking up management?
stairs. She reports no nausea, dysphagia, or reflux. She has (A) Attempt to discontinue or reduce pantoprazole
been taking ranitidine with minimal relief of symptoms.
She also takes atorvastatin for hyperlipidemia. She smokes (8) Continue pantoprazole at current dose
half a pack of cigarettes daily. (C) Obtain an upper endoscopy
On physical examination, her blood pressure is 140/ (D) Switch to ranitidine and metoclopramide
90 mm Hg and other vital signs are normal; BMI is 34. The (E) Switch to sucralfate
remainder of the examination, including abdominal exam-
ination, is unremarkable.
Item 60
Which of the following is the most appropriate next step in
management? A 22-year-old woman is evaluated for intractable pruritus that
keeps her awake at night. She is in her 25th week of pregnancy.
(A) Barium esophagography Her medications are a prenatal vitamin and a folate supplement.
(B) Electrocardiography On physical examination, vital signs are normal. Sciera!
(C) Empiric trial ofa proton pump inhibitor icterus is noted. Excoriations are seen on the arms, chest,
(D) Upper endoscopy abdomen, and legs.
Laboratory studies:
Hematocrit 35%
Item 58 Platelet count 370,000/µL (370 x 109 / L)
A 32-year-old woman is evaluated for arthralgia and jaun- Alanine aminotransferase SSU/L
dice. Three months earlier, she traveled to Mexico for a Aspartate aminotransferase 30U/L
2-week vacation and developed nausea, vomiting, abdom- Bile acids Elevated
inal pain, fever, and jaundice approximately 3 weeks after Total bilirubin 2.5 mg/dL (42.8 µmol!L)
returning to the United States. Testing for hepatitis A virus
IgM was positive, and she was treated with supportive mea- Testing for hepatitis C and hepatis B viral infections is
sures. Her symptoms and jaundice resolved within 3 weeks. negative.
A few weeks later, jaundice reappeared along with arthral- An ultrasound shows a normal liver with no dilated
gia. The patient's medical history is otherwise unremark- bile ducts.
able, and she takes no medication. Which of the following is the most appropriate next step in
On physical examination, vital signs are normal; BMI management?
is 27. Jaundice is noted. The remainder of the examination
is unremarkable. (A) Llver biopsy
Laboratory studies: (B) Peripheral blood smear
Alkaline phosphatase 272 U/L (C) Topical glucocorticoids
Alanine aminotransferase 775 U/L (D) Ursodeoxycholic acid
Aspartate aminotransferase 672 U/L
Total bilirubin 5.8 mg/dL (99.2 µmol/L)

The results of other studies, including a complete blood


Item 61 CJ
count, INR, serum creatinine level, antinuclear antibody, A 60-year-old woman is admitted to the hospital with
anti- smooth muscle antibody, and total IgG level, are within sudden-onset, cramping abdominal pain of moderate sever-
normal limits. ity in the right lower quadrant, followed several hours later
Ultrasonography of the liver is normal. by a bloody bowel movement. She has coronary artery dis-
ease; medications are atorvastatin, metoprolol. sublingual
Which of the following is the most likely diagnosis? nitroglycerin, and low-dose aspirin.
On physical examination, the patient appears comfort-
(A) Autoimmune hepatitis able. Pulse rate is 110/min; BM! is 35. Other vital signs are
(B) Leptospirosis normal. The abdomen is nondistended with normal bowel
(C) Malaria sounds. Deep palpation elicits tenderness in the right lower
(D) Relapsing, remitting hepatitis A viral infection quadrant with no rebound or guarding.

95
Self-Assessment Test

A CT scan without contrast shows thickening of the (C) Fundoplication


Cl ascending colon. Colonoscopy results show a segment of
subepithelial hemorrhage, edema, and erythema from the
CONT.
(D) Repeat endoscopy in 6 months

cecum to the hepatic flexure.


Which of the following is the most appropriate test to Item 64
perform next? A 45-year-old man is evaluated for watery diarrhea accom-
panied by nausea and bloating. Symptoms began 4 weeks
(A) CT angiography earlier with abdominal cramping and explosive watery
(B) Doppler ultrasonography of mesenteric vessels stools. Now, he reports up to five loose bowel movements
(C) MR angiography per day, with no blood and no nocturnal symptoms. He
(D) Selective catheter angiography also describes generalized abdominal discomfort that is not
relieved after a bowel movement. He works at a child care
center. He is otherwise healthy and takes no medication.
c:J Ite m 62 On physical examination, vital signs are normal.
Abdominal examination shows periumbilical tenderness
A 73-year-old man is evaluated in the hospital for Jight- with no rebound or guarding. The remainder of the exam-
headedness. He also reports nonbloody, watery diarrhea ination is unremarkable.
of 4 months' duration and an unintentional 4.5- kg (9.9-lb) Results of laboratory studies, including hemoglobin
weight loss over the same time period. He has hypertension level and a comprehensive metabolic panel, are within nor-
and hyperlipidemia. His medications are olmesartan and mal limits.
atorvastatin.
On physical examination, blood pressure is 100/50 mm Which of the following is the most appropriate next step in
Hg and pulse rate is 108/min; other vital signs are normal. management?
Physical examination findings are unremarkable.
(A) Colonoscopy
Results of laboratory studies, including serum creati-
nine, total lgA, and tissue transglutaminase lgA, are within (B) CT scan of the abdomen and pelvis
normal limits. (C) 24-Hour urine 5-hydroxyindoleacetic acid measurement
The patient responds to fluid resuscitation with nor- (D) Stool testing for Giardia lamblia
malization of his pulse and blood pressure. Colonoscopy is
grossly normal, and biopsy samples show no evidence of
microscopic colitis. Upper endoscopy with duodenal biop- Item 65
sies shows villous flattening and increased intraepithelial A 45-year-old woman is evaluated for episodic nausea,
lymphocytes. bloating, and epigastric pain of 5 years' duration. In the past
Which of the following is the most appropriate next step in 3 months, the nausea has been accompanied by occasional
management? vomiting. She also reports near-daily heartbw-n symptoms
that have not responded to daily omeprazole. She has a
(A) Discontinue atorvastatin 10-year history of type 2 diabetes mellitus that is treated
(B) Discontinue olmesartan with metforrnin and glyburide.
(C) Start a gluten-free diet On physical examination, vital signs are normal; BM!
(D) Start prednisone is 29. Abdominal examination shows diffuse tenderness to
deep palpation with no guarding. Other findings are normal.
Laboratory studies show a blood hemoglobin A1c level
Item 63 of 7.5%. The basic metabolic panel ls normal. A complete
blood count and liver chemistry tests are normal.
A 65-year-old man is evaluated with upper endoscopy in Upper endoscopy shows a moderate amount of retained
follow-up for Barrett esophagus. He has had heartburn for food in the stomach and patchy erythema of the gastric
more than 15 years, but his symptoms have been well controlled mucosa. Biopsies of the stomach are normal.
with daily omeprazole. He reports no weight loss or pain with
swallowing and has no history of anemia. He stopped smoking Which of the following is the most appropriate next step in
5 years earlier, but has a 40-pack-year history. management?
Vital signs and the remainder of the physical examina-
(A) Gastric emptying scintlgraphy
tion are normal.
On upper endoscopy, an area of salmon-colored (B) 24-Hour pH probe
mucosa is seen in the esophagus. Biopsies confirm evidence (C) Initiation of domperidone
of Barrett esophagus with low-grade dysplasia. The pathol- (D) Initiation ofmetoclopramide
ogy slides were reviewed by a second pathologist, confirm-
ing the presence oflow-grade dysplasia.
Which of the following is the most appropriate next step in Item 66 CJ
management? A SS-year-old man is evaluated after emergent treatment
for an episode of hematemesis. Emergency endoscopy was
(A) Endoscopic ablation performed in the emergency department, and the bleeding
(B) Esophagectomy was successfully treated with band ligation. The endoscopy

96
Self-Assessment Test

rrl revealed esophageal varices, one of which had stigmata of Item 69


IAI recent hemorrhage. Treatment with octreotide and a proton A 64-year-old man is evaluated for hepatitis Cvirus-related
CONT. pump inhibitor was initiated. The patient has a history of cirrhosis with decompensation, including previous variceal
cirrhosis due to hepatitis C viral infection. He has no other hemorrhage, and ascites. His medications are furosemide,
medical problems and takes no medication. spironolactone, and nadolol.
On physical examination, vital signs and other findings On physical examination, vital signs are normal; BMI
are normal. is 25. Spider angiomata are seen over the chest, and palmar
Laboratory studies show a hemoglobin level of 8.9 g/dL erythema is noted. The abdomen is distended with flank
(89 g/ L), leukocyte countof3600/µL (3 .6 x 109 / L) , and plate- dullness to percussion. The left liver lobe is palpable 5 cm
let count of80.000/µL (80x109 / L). below the xiphoid process. The spleen is palpable. Bilat-
Which of the following is the most appropriate next eral lower-extremity edema is noted. The remainder of the
treatment? examination is normal.
A screening ultrasound examination shows a 3-cm
(A) Blood transfusion mass with poorly defined margins and coarse, irregular
(B) Ciprofloxacin internal echoes in the right hepatic lobe. A CT scan of the
(C) Platelet transfusion abdomen with contrast shows a 3-cm arterial enhancing
(D) Transjugular intrahepatic portosystemic shunt placement lesion with portal venous phase washout in the periphery
of the right lobe. The chest is normal on CT.

CJ Item 67 Which of the following is the most appropriate next step in


management?
A 56-year-old man is evaluated in the emergency depart-
ment for altered mental status of18 hours' duration. He has (A) Biopsy of the lesion
a history of cirrhosis due to hepatitis C viral infection and (B) Liver transplantation
also has anxiety. He has not changed his diet recently, and (C) Sorafenib
he has no symptoms suggestive of gastrointestinal bleeding. (D) Surgical resection
His bowel movements have been regular and unchanged.
His only medication is alprazolam started 2 weeks earlier,
after a visit to an urgent care center. Item 70
On physical examination, vital signs are normal. Oxygen A 25-year-old man is evaluated for a sensation of solid food
saturation is 96'Yu breathing ambient air. Abdominal exam- "sticking" several times per week. He reports that he some-
ination is unremarkable; there is no evidence of ascites. times forces himself to vomit when he feels food "stuck"
Psychomotor slowing and asterixis are noted. There are no in the esophagus, but he has never gone to the emergency
focal neurologic findings. The remainder of the examination department. He talces a multivitamin and is generally healthy.
is unremarkable. On physical examination, vital signs and other find-
Complete blood count, serum electrolytes and creati- ings, including those of an abdominal examination, are
nine. and blood glucose are normal. unremarkable.
In addition to starting lactulose, which of the following is Upper endoscopy findings are shown.
the most appropriate next step in management?
(A) CT of the head
(B) Initiate a protein -restricted diet
(C) Initiate rifaximin
(D) Withdraw alprazolam

Item 68
A 38-year-old man is evaluated during a new-patient
appointment. The patient reports no rectal bleeding or other
gastrointestinal symptoms. His family history incl_udes colon
cancer diagnosed in his father at age 52 years. Hts pers~nal
medical history is unremarkable and he takes no medication.
All physical examination findings, including vital signs,
are normal.
When should this patient undergo his first screening
colonoscopy?
(A) Now
(B) Age 40 years
(C) Age 42 years Biopsies of the esophagus show more than 18 eosino-
(D) Age 50 years phils/hpf.

97
Self-Assessment Test

Which of the following is the most likely diagnosis? second bloody bowel movement in the emergency depart-
ment. She is otherwise healthy and takes no medication.
(A) Achalasia
Findings on physical examination, including vital
(B) Candida esophagitis signs, are nom1al.
(C) Eosinophilic esophagitis Laboratory studies show a hemoglobin level of 9 g/dL
(D) Pill-induced esophagitis (90 g/ L).
Which of the following is the most appropriate next step In
management?
Item 71
A 65-year-old man undergoes screening with ultrasonogra- (A) Angiography and arterial embolization
phy for abdominal aortic aneurysm. He has no symptoms, his (B) Colonoscopy within 8 hours with cleansing enemas
medical history is unremarkable, and he takes no medication. (C) Colonoscopy within 24 hours with oral bowel prepa-
Vital signs and other findings of the physical examina- ration
tion are nomlal. (D) Tagged red blood cell scintigraphy
On the abdominal ultrasound, the abdominal aorta (E) Transfusion of packed red blood cells
appears normal. Multiple gallstones smaller than 1 cm in
size are seen in the gallbladder. The liver has normal echoge-
nicity, and there are no dilated bile ducts. No abnormalities
are seen in the gallbladder wall.
Item 74 CJ
A 58-year-old woman is evaluated in the emergency depart-
Which of the following is the most appropriate manage- ment for sharp. nonradiating pain in the left lower quadrant
ment ofthls patient's gallstone disease? that has persisted for 2 days. She reports no fever. diarrhea,
or blood in stools, and has not traveled recently. Her last colo-
(A) Cholecystectomy noscopy, performed I year earlier, showed pandiverticulosis.
(B) Endoscopic retrograde cholangiopancreatography On physical examination, her temperature is 37.2 °C
(C) Repeat ultrasonography in 6 months (99.0 °F). blood pressure is 130/85 mm Hg, pulse rate is
(D) Clinical observation 88/ min, and respiratory rate is 18/min; BM! is 26. Abdomi-
nal examination is positive for left-lower-quadrant tender-
ness on palpation with no rebound or guarding.
CJ Item 72 Results of laboratory studies show an elevated leuko-
cyte count; other findings are nonnal.
An80-year-old man is evaluated in the emergency department
Abdominal CT shows focal diverticulitis in the sigmoid
after a bowel movement with initial passage of brown soft stool
colon, without abscess.
followed by a large volume of red blood. He reports intermit-
tent chills and fevers over the past week. He also has peripheral Which of the following is the most appropriate next step in
vascular disease, hypertension, and hypercholesterolemia. His management?
history includes an aortoiliac aneurysm treated with an aortic
bifurcation graft 3 years earlier. His medications are atorvasta- (A) Colonoscopy
tin, hydrochlorothiazide, losartan, and low-dose aspirin. (B) Intravenous antibiotics
On physical examination, the patient is comfortable. (C) Oral antibiotics
His temperature is 38 °C (100.4 °F), blood pressure is 108/ (D) Surgery
60 mm Hg, pulse rate is 112/min, and respiration rate is 18/
min. Cardiopulmonary examination is unremarkable. There
is midabdominal tenderness to palpation. Rectal examina- Item 75
tion reveals bright red blood mixed with brown stool. A 66-year-old man is evaluated for breakthrough symp-
Laboratory studies show a hemoglobin level of 9 g/dL toms of gastroesophageal reflux disease (GERD) despite
(90 g/L) . Leukocyte count is 14,000/µL (14 x 109 /L) with once-daily pantoprazole. He has a 5-year history of GERO.
neutrophilia. On physical examination, his blood pressure is 118/
Which of the following is the most appropriate test to per- 70 mm Hg, pulse rate is 76/min, and respiratory rate is
form next? 18/min; BMI is 30. The abdomen Is soft, nontender, and non-
distended. Bowel sounds are hyperactive. The remainder of
(A) CT scan with contrast the examination is unremarkable.
(B) Mesenteric angiogram Upper endoscopy shows Barrett esophagus. Biopsy
(C) Tagged red blood cell scintigraphy results are indefinite for dysplasia.
(D) Upper endoscopy Which of the following Is the most appropriate next step in
management?

CJ Item 73 (A) Endoscopic ablation therapy


(B) Esophagectomy
A 72-year-old woman is evaluated after being hospitalized
for gastrointestinal bleeding. She went to the emergency (C) Optimization of medical therapy followed by repeat
department after passing a large amount of bright red blood upper endoscopy
per rectum at home and became lightheaded. She had a (D) Repeat upper endoscopy in 1 year

98
Self-Assessment Test

Item 76 Which of the following Is the most likely diagnosis?


A78-year-old woman is evaluated for frequent rectal urgency (A) Acute mesenteric ischemia
with the passage of explosive, loose to watery stool. She says
(B) Diverticular bleeding
she is often unable to get to the bathroom in time. Episodes
have occurred after meals and in the early morning over the (C) Enterohemorrhagic Escherichia coli
past 3 months. She has Alzheimer dementia and lives with her (D) lschemic colitis
daughter. She also has hypertension and chronic back pain.
Her history includes occasional constipation that resolves
when treated with docusate. Her last screening colonoscopy Item 78
at age 70 years showed diffuse diverticulosis. Her medica- A 26-year-old woman with Crohn disease is evaluated for
tions are lisinopril, acetaminophen, memantine, docusate as a 2-week history of worsening abdominal pain in the right
needed, and a calcium supplement with vitamin D. lower quadrant. She reports passage of one to two formed
On physical examination, vital signs are normal. Fecal and nonbloody stools per day with no changes in bowel
soiling is noted. Rectal examination shows normal anal tone habits. The patient has required three courses of prednisone
and brown stool in the rectal vault. for disease flares over the past year. Her only medication is
A complete blood count and thyroid-stimulating hor- azathioprine.
mone level are normal. On physical examination, temperature is 37.7 °C
Which of the following is the most appropriate next step in (99.9 °F) and pulse rate is llS/min; other vital signs are nor-
management? mal. Abdominal examination shows fullness and tender-
ness in the right lower quadrant with no distinct mass. The
(A) Abdominal radiograph remainder of the examination Is unremarkable.
(B) Anorectal manometry Laboratory studies show a hemoglobin level oflO.S g/dL
(C) Loperamide (lOS g/L) and a C-reactive protein level of 32 mg/dL
(320 mg/L). Leukocyte count and liver chemistry tests are
(D) Psyllium
normal.
CT enterography shows asymmetric mural thickening
CJ Item 77 and mucosa! inflammation of a long segment of distal ileum
without luminal narrowing.
A 68-year-old man is evaluated in the ICU for cramping
abdominal pain in the left lower quadrant, which began Which of the following is the most appropriate treatment?
approximately 3 hours before the evaluation. Twenty min- (A) Budesonide
utes before the evaluation, he had a loose stool with hema-
(B) Infliximab
tochezia. The patient has been in the ICU for 24 hours, after
being admitted with fever and hypotension secondary to (C) Mesalamine
urosepsis. He was treated with intravenous fluids, vaso- (D) Prednisone
pressors, and piperacillin-tazobactam. His medical history
includes hypertension, benign prostatic hypertrophy, and
hyperlipidemia. His daily medications are chlorthalidone, Item 79
atorvastatin, and low-dose aspirin. A 30-year-old woman is evaluated during a new-patient
On physical examination, the patient is alert. Tempera- appointment. Her personal medical history is unremarkable
ture is 38.7 °C (101.7 °F), blood pressure is 106/60 mm Hg, and she takes no medication. Her family history includes
pulse rate is 90/min. respiration rate is 18/min, and oxygen colon cancer diagnosed in her mother at age 48 years, endo-
saturation is 96'Y., breathing ambient air. There is tenderness metrial cancer diagnosed in a maternal aunt at age Sl years,
to palpation over the left side of the abdomen without guard- and colon cancer diagnosed in her maternal grandfather at
ing. The remainder of the examination is unremarkable. age SS years.
Colonoscopy findings are shown. All physical examination findings, including vital signs,
are normal.
Genetic testing for MSH2 mutation is positive, consis-
tent with Lynch syndrome.
Which of the following is the most appropriate next step?
(A) Colectomy
(B) Colonoscopy at age 38
(C) Colonoscopy at age 40
(D) Colonoscopy now

Item 80
A 38-year-old man is evaluated for epigastric discomfort
and early satiety associated with an unintentional 4.5-kg
(9.9- lb) weight loss over the preceding 5 months. His

99
Self-Assessment Test

family history includes lobular breast cancer diagnosed in without guarding. The remainder of the examination is
his mother at age 45 years, stomach cancer diagnosed in normal.
his maternal grandfather at age 48 years, and stomach can- Results of laboratory studies, including a complete
cer diagnosed in his maternal uncle at age 52 years. The blood count and liver chemistry tests, are normal.
patient's medical history is unremarkable, and he takes no
medication. Which of the following is the most appropriate treatment?
On physical examination, vital signs are normal. (A) Alosetron
Abdominal examination shows epigastric tenderness to (B) Budesonide
palpation and normal bowel sounds.
(C) Cognitive-behavioral therapy
Which of the following is the most appropriate diagnostic (D) Linaclotide
test to perform next?
(A)
(B)
Colonoscopy
Gastric emptying study Item 83 CJ
(C) Helicobacter pylori serology
A 55-year-old man is evaluated after being hospitalized for
epigastric pain of 1 month's duration and melenic stools
(D) Upper endoscopy over the past 3 days associated with fatigue. He reports
(E) Upper gastrointestinal radiograph series no hematochezia. hematemesis, chest pain, or shortness
of breath. He has osteoarthritis treated with ibuprofen.
Item 81 He received an intravenous fluid bolus in the emergency
department.
A 32-year-old man is evaluated for a 1-week history of On physical examination, blood pressure is 135/75 mm Hg
jaundice and pruritus. One month earlier, he completed and other vital signs are normal, with no orthostatic changes.
treatment for sinusitis with amoxicillin-clavulanate. His Abdominal examination reveals epigastric tenderness but is
sinusitis symptoms resolved with therapy. otherwise unremarkable. No stigmata of chronic liver dis-
On physical examination, vital signs are normal. BMT is ease are seen.
26. Jaundice is noted. Excoriations are seen on the extremi- Laboratory studies show a hemoglobin level of7.3 g/dL
ties. The abdominal examination is normal. (73 g/L).
Laboratory studies: Upper endoscopy shows a 1.5-cm duodenal bulb ulcer
INR . 1.0 with a clean base.
Albumin 3.8 g/dL (38 g/L)
Which of the following is the most appropriate resuscita-
Alkaline phosphatase 580U/L
Alanine aminotransferase 42U/L tion measure?
Aspartate aminotransferase 38U/L (A) Transfuse red blood cells to a goal hemoglobin level
Total bilirubin 8.6 mg/dL (147.l µmol/L) of 8 g/dL (80 g/ L)
(B) Transfuse red blood cells to a goal hemoglobin level
Hepatitis A, B, and C virus and Epstein-Barr virus stud-
ies show no evidence of current or previous infection. Test- of9 g/dL (90 g/ L)
ing for antinuclear antibody and anti-smooth muscle anti- (C) Transfuse red blood cells to a goal hemoglobin level
body is negative. The ceruloplasrnin level is normal. oflO g/dL (100 g/ L)
Results of abdominal ultrasonography are normal, (D) No transfusion
with no evidence of biliary dilation. gallstones, or hepatic
parenchymal abnormalities.
Item 84
Which of the following is the most appropriate next step in
management? A 28-year-old man is evaluated after hospitalization for
lower gastrointestinal bleeding 6 weeks earlier. At that time,
(A) Endoscopic retrograde cholangiopancreatography he was found to have innumerable adenomatous colon
(B) Liver biopsy polyps during colonoscopy. He underwent total colectomy,
(C) Predntsone administration and pathology showed a 2-cm adenocarcinoma in the trans-
(D) Continued observation verse colon. There was no evidence of metastatic disease.
Genetic testing confirmed a germline mutation in the ade-
nomatous polyposis coli (APO gene consistent with familial
Item 82 adenomatous polyposis syndrome. Family medical history
A 54-year-old woman is evaluated for a 1-year history of is unremarkable.
generalized abdominal pain that is constant throughout the On physical examination, vital signs are normal.
day, every day. The pain is not triggered by eating and is not Abdominal examination shows a well-healed scar and no
relieved by bowel movements. She reports no weight loss tenderness to palpation.
and no change in bowel habits. She also has depression. A Which of the following ts the most appropriate test to per-
screening colonoscopy done 8 months earlier was normal. fonn next?
Her only medication is escitalopram.
On physical examination, vital signs are normal; BMI (A) Barium upper gastrointestinal series
is 28. The abdomen is tender to palpation in all quadrants (B) Double-balloon enteroscopy

100
Self-Assessment Test

(C) Upper endoscopy


(D) No further testing

Item 85
A 52-year-old man is evaluated after screening colonoscopy
revealed three polyps in the descending colon, measuring
s mm, 8 mm, and 3 mm in size. Colonoscopy prepara-
tion was excellent, and the procedure was complete to the
cecum. The polyps were completely excised, and pathology
showed all three to be tubular adenomas. The patient has no
personal or family history of colorectal cancer. He reports
no symptoms and takes no medication.
All physical examination findings, including vital signs,
are normal.
Based on his colonoscopy findings, when should this patient noted on musculoskeletal exam. The liver edge is palpable
next undergo surveillance colonoscopy? at the costal margin. The remainder of the physical exam-
(A) !year ination is unremarkable.
(B) 3 years Laboratory studies:
Complete blood count Normal
(C) syears
Alanine aminotransferase 112U/L
(D) 10 years Aspartate aminotransferase 84U/L
Total bilirubin I mg/dL (17.1 µrnol/L)
Creatinine Normal
Item 86 Urinalysis Normal
A SS-year-old woman is evaluated for a 6-month history
of throbbing and sometimes burning epigastric pain. The Which of the following is the most appropriate treatment?
pain occurs 2 to 3 times per week and often subsides with (A) Cyclophosphamide
eating. She reports no weight loss, nausea, or vomiting, and
no bowel symptoms. She tested negative for Helicobacter (B) Glucocorticoids
pylori infection. Her pain did not respond to a 4-week trial (C) Ledipasvir and sofosbuvir
of omeprazole. The patient also has anxiety, hypothyroid- (D) Pegylated interferon and ribavirin
ism, and type 2 diabetes mellitus controlled by diet. Her (E) Rituximab
family medical history is unremarkable. Her medications
are lorazepam and levothyroxine.
On physical examination, vital signs are normal. Epi- Item 88
gastric tenderness to palpation is noted. Other findings are A 63-year-old woman is evaluated for diarrhea charac-
normal. terized by three to four large-volume, watery stools per
A complete blood count, liver chemistry tests, and thy- day over a period of 14 months with gradually increasing
roid-stimulating hom10ne level are normal. Hemoglobin severity and frequency. She now reports occasional urge
~c level is 6.7%. fecal incontinence and nocturnal diarrhea but no abdomi-
Upper endoscopy findings are normal. Gastric and nal pain, bloody stools, or weight loss. She has been taking
small-bowel biopsies are normal. loperamide up to five times daily, but symptoms have
Which of the following is the most appropriate next step in persisted.
management? On physical examination, vital signs are normal; BMI is
26. Abdominal examination is normal with no tenderness or
(A) CT scan of the abdomen distention. Rectal examination reveals no blood or masses.
(B) Gastric emptying test Results of routine laboratory studies are normal. Poly-
(C) Initiation of a tricyclic antidepressant merase chain reaction testing of the stool for Clostridium
(D) Twice-daily proton pump inhibitor therapy difficile is negative.
COlonoscopy results are normal. Random colon biopsy
(E) Ultrasonography of the right upper quadrant specimens show lymphocytic infiltration of the mucosa
with a subepithelial collagen band.
Item 87 Which of the following is the most appropriate treatment?
A 61-year-old woman is evaluated for joint pain, a rash on (A) Bismuth subsalicylate
her legs, and weakness. She has chronic genotype 1 hepatitis (B) Budesonide
C viral infection. She takes no medication.
On physical examination, vital signs are normal. The (C) Mesalamine
lower extremities are shown (see top of next column). No (D) Prednisone
evidence of muscle weakness, joint swelling or warmth is (E) Probiotics

101
Self-Assessment Test

Item 89 (D) Tenofovir


A 33-year-old woman is evaluated following an inciden- (E) No treatment
tally discovered hepatic adenoma. One week earlier, she
was evaluated for nephrolithiasis with CT urogram. A 4-cm
right-lobe liver lesion was also seen. Follow-up MRI iden- Item 92
tified findings consistent with hepatic adenoma. She is A 40-year-old woman is evaluated for intermittent lower
experiencing no symptoms now but has noted a sense of abdominal pain, daily bloating, and constipation. Her
right-upper-quadrant abdominal discomfort in the past. lower abdominal pain initially worsens then subsides after
Her only medication is an oral contraceptive agent. a bowel movement; it occurs nearly daily. She has bowel
All physical examination findings are normal. movements every 2 to 3 days and reports that most bowel
Which of the following Js the most appropriate manage- movements are hard. The patient reports that adherence
ment? to a gluten-free diet has helped the bloating, but other
symptoms have not responded. A twice-daily soluble
(A) Discontinue the oral contraceptive agent fiber supplement has not affected symptoms. Her last
(B) Radiofrequency ablation thyroid-stimulating hormone measurement and complete
(C) Surgical resection blood count 6 months earlier were normal. Family history
includes colon cancer diagnosed in her grandfather at age
(D) Observation
60 years and a first cousin with celiac disease.
On physical examination, BMI is 29; all other findings
are unremarkable.
Item 90
A 75-year-old man is evaluated during a follow-up appoint- Which of the following is the most appropriate next step in
ment for newly diagnosed iron deficiency anemia and stool management?
testing positive for occult blood. He has undergone upper (A) Colonoscopy
endoscopy and colonoscopy twice with no cause found for (B) Glucose breath test
his gastrointestinal bleeding.
On physical examination, vital signs and other findings (C) Polyethylene glycol 3350
are unremarkable. (D) Serum tissue transglutaminase antibody
Laboratory studies show a serum ferritin level of 6 ng/mL measurement
(6 µg/L), a hemoglobin levelof9.9g/dL (99 g/L), anda mean
corpuscular volume of 78 fL.
Item 93
Which of the following is the most appropriate next step in
A 65-year-old man is evaluated after a positive stool antigen
management?
test for Helicobacter pylori infection obtained to confirm
(A) Capsule endoscopy eradication after therapy. H. pylori gastritis was diagnosed
(B) CT enterography in the setting of a duodenal ulcer. Four weeks ago, he com-
(C) Push enteroscopy pleted a 10-day course of eradication therapy consisting of
amoxicillin, clarithromycin, and omeprazole. He reports
(D) Small-bowel follow-through radiography
taking all medications as prescribed during treatment and
reports no upper gastrointestinal symptoms or melena. The
patient does not smoke cigarettes or drink alcohol. He has
Item 91 no known drug allergies.
A 40-year-old woman is evaluated at a follow-up appoint-
ment for hepatitis B virus (HBV) infection, which was diag- Which of the following is the most appropriate 14-day
nosed at age 23 years. The patient also has psoriasis. She treatment regimen?
is married and is sexually active. Medications are topical (A) Amoxicillin, clarithromycin, and omeprazole
clobetasol and an oral contraceptive agent. (B) Amoxicillin, metronidazole, and omeprazole
On physical examination, vital signs are normal. Psori-
(C) Bismuth, metronidazole, omeprazole, and tetracy-
atic lesions are present on the elbows and knees. No hepa-
cline
tosplenomegaly is noted.
(D) Clarithromycin, metronidazole, and omeprazole
Laboratory studies:
Alanine aminotransferase 125 U/L
Aspartate aminotransferase
Hepatitis B surface antigen
112 U/L
Positive
Item 94 [:J
Hepatitis B e antigen Positive A 61-year-old woman is evaluated in the hospital for abdom-
HBVDNA 41,326 IU/mL inal discomfort and worsening ascites. She reports a signifi-
cant decrease in urine output over the preceding 3 days. She
Which of the following is the most appropriate treatment? has a history of cirrhosis due to primary biliary cholangitis.
Her medications are furosemide and spironolactone.
(A) Adefovir On physical examination, blood pressure is 90/58 mm
(B) Lamivudine Hg; other vital signs are normal. The abdomen is distended
(C) Pegylated interferon with a positive fluid wave.

102
Self-Assessment Test

Laboratory studies show a serum creatinine level


Cl of[106.l
CONT.
2.8 mg / dL (247.5 µmol/L) (3 weeks ago, 1. 2 mg /dL
µmol/L)) and a serum sodium level of 133 mEq / L
Urinalysis shows hematuria.
A CT scan identifies nephrolithiasis and a small stone
In the left ureter. The CT scan also shows a 6-cm fluid col-
(133 mmol / L). lection with solid debris in the body of the pancreas with a
Urine studies show a sodium level of less than 10 mEq/L well-defined wall.
(10 mmol/L) and no protein, leukocytes, erythrocytes, or
casts. Which of the following is the most appropriate manage-
Analysis of ascitic fluid shows a leukocyte count of ment of the fluid collection?
180/µL with 30% neutrophils. Cultures of ascitic fluid are (A) Antibiotic therapy
negative. (B) CT-guided fine-needle aspiration
A kidney ultrasound shows no evidence of obstruction
(C) Drainage procedure
or kidney parenchymal disease.
(D) Observation
Which of the following is the most likely diagnosis?
(A) Acute interstitial nephritis
Item 96
(B) Acute tubular necrosis
A 62-year-old woman is evaluated during a follow-up
(C) Hepatorenal syndrome appointment 12 weeks after she completed treatment for
(D) Membranous glomerulonephritis genotype la hepatitis C virus (HCV) infection. She has
Child-Turcotte-Pugh Class A cirrhosis (well-compensated
[::J Item 95 cirrhosis). Small esophageal varices were noted on upper
endoscopy I year earlier.
A 48-year-old man is evaluated in the emergency depart- On physical examination, vital signs are normal; BMI
ment for left flank pain and dysuria. Six months earlier. the is 26. Palmar erythema, spider angiornata over the chest, a
patient was hospitalized for severe acute gallstone pancre- firm liver edge 3 cm below the costal margin, and a palpable
atitis. Contrast-enhanced CT of the pancreas showed lack spleen tip are noted. The examination is otherwise normal.
of perfusion in the body of the pancreas. He recovered with Her HCV RNA is undetectable and her calculated Model
supportive care and was discharged 2 weeks later. He had an for End-Stage Liver Disease score is 8.
uncomplicated laparoscopic cholecystectomy 4 weeks after
discharge. He reports that he has felt well until the sudden Which of the following is the most appropriate manage-
onset ofleft flank pain today. ment for this patient?
On physical examination, blood pressure is 130/80 mm Hg (A) Liver transplantation evaluation
and pulse rate is 90/min; other vital signs are normal.
Abdominal examination is notable for pain in the left lower (B) Measurement of HCV RNA in 12 weeks
quadrant on palpation. The remainder of the examination (C) Ultrasonography of the liver every 6 months
is normal. (D) Upper endoscopy

103
Answers and Critiques

Item 1 Answer: D ltem2 Answer: D


Educational Objective: Treat opioid-induced Educational Objective: Manage the immune-tolerant
constipation. phase of hepatitis B viral infection.
Switching to naloxegol is the most appropriate treatment Repeat liver chemistry testing in 6 months is the most
for this patient. The clinical definition of constipation (as appropriate next step in the management of this patient.
defined by the Rome 4 international working group) is a The patient has hepatitis B virus (HBV) infection in the
symptom complex that includes at least two of the fol - immune-tolerant phase, which can be determined by the
lowing: straining during defecation, passage of lumpy or likely vertical transmission and the patient's young age,
hard stool, sensation of incomplete defecation, use of man- positive hepatitis 8 e antigen (HBeAg), high viral load, and
ual maneuvers to facilitate a bowel movement, and/or fre- normal aminotransferase levels. Therefore, the patient only
quency of fewer than three bowel movements per week. requires serial monitoring of aminotransferase levels. There
Oral naloxegol is a peripherally acting µ-opioid receptor are four typical phases of HBV infection: (1) immune toler-
antagonist that is FDA-approved for the treatment of opioid- ant; (2) immune active, HBeAg positive; (3) immune con-
induced constipation in adults with chronic noncancer trol (inactive); and (4) reactivation, HBeAg negative. Not all
pain. This patient's constipation can be classified as opi- patients go through each phase. Patients with infection in
oid-induced because her constipation symptoms developed the immune-tolerant phase do not have significant hepatic
after the initiation of chronic opioid analgesic therapy for inflammation and have no fibrosis, and, therefore, do not
reflex sympathetic dystrophy syndrome. First-line laxative require treatment. However, infection can progress to the
therapies, including over-the-counter stool softeners, bulk immune-active, HBeAg-positive phase, in which hepatic
laxatives (fiber supplements), a stimulant laxative (bisaco- inflammation, elevated aminotransferase levels, and fibrosis
dyl), and an osmotic laxative (polyethylene glycol [PEGJ), develop, underscoring the need for surveillance of amino-
have all been ineffective. Current maintenance laxative transferase levels.
therapy should be stopped before the initiation of naloxegol Patients with HBV infection in the immune-active,
and can be added to the naloxegol after 3 days of monother- HBeAg-positive and reactivation, HBeAg-negative phases
apy as symptoms dictate. require treatment if the alanine aminotransferase level is
SUrfactants such as docusate sodium or docusate cal- elevated. Antiviral therapy is also required for patients who
cium are weak laxatives with an excellent safety profile. As present with acute liver failure, all patients with cirrhosis, and
such, they are most appropriate for very mild, intermittent patients undergoing treatment with certain immunosuppres-
constipation and will not be effective in this patient sive or chemotherapy regimens. None of these scenarios apply
Adding a second osmotic agent such as lactulose is to this patient, so she does not require antiviral treatment
unlikely to improve the patient's reduced stool frequency, such as tenofovir, entecavir, or pegylated interferon.
which is the result of slowed colonic motility caused by her Patients with HBV infection are at increased risk for
chronic opioid analgesic use. Also, lactulose use is likely to hepatocellular carcinoma, even in the absence of cirrhosis.
lead to bloating. Patients from Southeast Asia should undergo hepatocellular
Increasing the dose of PEG will not provide any addi- carcinoma surveillance with ultrasonography starting at age
tional benefit because the patient reports soft stool with the 40 years for men and at age SO years for women, and patients
current dose. The patient's altered colonic motility will not from sub-Saharan Africa should begin at age 20 years. Other
improve with additional PEG. Furthermore, an increased indications include persistent inflammatory activity (defined
dose of PEG is likely to cause bloating. as an elevated alanine aminotransferase level and HBV DNA
levels greater than 10,000 JU/ml for at least a few years) and
KEY POINT
a family history of hepatocellular carcinoma. The preferred
• Oral naloxegol is a peripherally acting µ-opioid recep- surveillance strategy is liver ultrasonography with or with-
tor antagonist that is FDA-approved for the treatment out a-fetoprotein measurement. This patient is not yet old
of opioid-induced constipation in adults with chronic enough to warrant hepatocellular carcinoma surveillance,
noncancer pain. so hepatic ultrasonography is not indicated.
KEY POINT
Bibliography
Chey WD. Webster L. Sostek M, Lappalalnen J, Barker PN. TackJ. Naloxegol • Patients with hepatitis B infection in the immune-
for opioid-induced constipation in patlents with noncancer pain. N tolerant phase require serial monitoring of
Engl J Med. 2014;370:2387- 96. [PMlD: 24896818] doi:l0.1056/
NEJMoal310246
aminotransferase levels.

105
Answers and Critiques

Bibliography diseases may also result in hepatic steatosis, patients with


Temmlt NA, Bzowej NH, Chang KM. Hwang JP. Jonas MM, Murad MH; elevated liver chemistries and suspected nonalcoholic ste-
American Association for the Study of Liver Diseases. AASLD guidelines
for treatment of chronic hepatitis ll l·lepatology. 2016;63:261-83. [PMID:
atohepatitis should be evaluated to exclude other causes of
26566064] do!:l0.1002/hep.28156 chronic liver disease.
The diagnosis of primary biliary cholangitis (PBC) is
generally made on the basis of a cholestatic liver enzyme
Item 3 Answer: D profile in the setting of a positive antimitochondrial anti-
Educational Objective: Manage aspirin use before and body test. The predominant liver enzyme abnormality is
after polypectomy. an increase in serum ALP levels, but a mild to moderate
increase in aminotransferase levels is also seen. Antimito-
This patient should not discontinue aspirin use. Aspirin
chondrlal antibody-negative PBC accounts for about 10% of
does not need to be discontinued before colonoscopy in
cases of PBC. This patient's liver chemistry profile is hepato-
any scenario, and data from studies of patients who have
cellular (elevations primarily of aminotransarninase levels),
undergone polypectomy show no difference in the risk for
not cholestatic, making PBC unlikely.
postprocedure bleeding with discontinuation or continu-
Although this patient has a positive anti-smooth mus-
ation of aspirin use. The American College of Gastroenter-
cle antibody test, the low titer alone is not diagnostic of
ology's 2016 guidelines for management of lower gastro-
autoimmune hepatitis. Autoimmune hepatitis is typically
intestinal bleeding (LGIB) recommend the continuation
accompanied by higher autoantibody titers and elevated
of aspirin for secondary cardiovascular prophylaxis after
y-globulin levels. It requires a liver biopsy to establish the
polypectomy. Discontinuing aspirin is recommended
diagnosis. Between 20'Yo and 30% of patients with NAFLD
after polypectomy in patients without established car-
exhibit low-titer autoantibodies.
diovascular disease who are using aspirin as primary
This patient has a minimally elevated ALP level, which
prophylaxis. In patients with establlshed cardiovascular
helps to exclude primary sclerosing cholangitis (PSC) or
disease, such as this patient, the risks of a potential car-
PBC. In addition, the patient's risk factors and ultrasound
diovascular event outweigh those of potential gastroin-
findings make these diagnoses unlikely. MR cholangiopan-
testinal bleeding.
creatography is required to diagnose PSC.
Holding aspirin for a period of time after a polypectomy,
such as 48 hours, has not been shown to reduce postproce- KEY POINT
dure LGIB and may increase risk for a thromboembolic event • Nonalcoholic fatty liver disease is the most common
in a patient with established cardiovascular disease. cause of abnormal liver test results in the United
KEY POINT States.
• Aspirin for secondary prophylaxis in patients with
established cardiovascular disease should be contin- Bibliography
Chalasani N, Younossi Z, Lavine JE, Diehl AM, Brunt EM, Cusi K, et al. The
ued after colonoscopy with polypectomy. diagnosis and management of non-alcoholic fatty liver disease: prac-
tice guideline by the American Association for the Study of Liver
Diseases, American College of Castroenterology, and the American
Bibliography Castroenterologlcal Association. Hepatology. 2012;S5:2005-23. [PMID:
Strate LL, Gralnek IM. ACG clinical guideline: management of patients with 22188764] doi:l0.1002/hep.25762
acute lower gastrointestinal bleeding. Am J Gastrocnterol. 2016;111:459-
74. [PMID: 26925883] doi:l0.!038/ajg.2016.41

Items Answer: D
ltem4 Answer: C Educational Objective: Diagnose pseudoachaJasia.
Educational Objective: Diagnose nonalcoholic fatty
This patient likely has pseudoachalasia, and endoscopic
liver disease.
ultrasound should be used to diagnose a possible tumor
The most likely diagnosis Is nonalcoholic fatty liver disease of the distal esophagus or gastric cardia. The other causes
(NAFLD). Up to 108 million Americans have NAFLD, which of pseudoachalasia are benign disease, such as amyloido-
far exceeds the prevalence of any other form of chronic sis and sarcoidosis, and postsurgical status (for example,
liver disease. Risk factors include obesity, diabetes melli- after Nissen fundoplication or bariatric surgery). The clin-
tus, Insulin resistance, hypertension, and hyperlipidemia. ical presentation of achalasia consists of dysphagia to both
This patient has three risk factors for NAFLD, as well as solids and liquids along with regurgitation of undigested
an enlarged liver on physical examination. Her elevated food and saliva. Other conditions can mimic achalasia and
alanine aminotransferase and aspartate aminotransferase can have identical clinical, barium-imaging, and manomet-
levels are within the typical range for patients with NAFLD. ric findings, as well as identical endoscopic appearance.
Alkaline phosphatase (ALP) levels may be slightly elevated These include pseudoachalasia, secondary achalasia, and
as well, typically less than 2 to 2.5 times the upper limit of Chagas disease. Pseudoachalasia is caused by a tumor at the
normal. The finding of a hyperechoic liver on ultrasonog- gastroesophageal junction infiltrating the myenteric plexus
raphy is also consistent with NAFW. Because other liver causing esophageal motor abnormalities. Tumors capable

106
Answers and Critiques

of infiltrating the myenteric plexus include those of the or change in bowel consistency. IBS can then be further sub-
distal esophageus, gastric cardia, pancreatic, breast, lung, typed into IBS with predominant constipation (IBS-C), pre-
and hepatocellular. Patients with pseudoachalasia are often dominant diarrhea, mixed bowel habits, or unclassified.
in their sixth decade of life or older, have a short duration 1l1e effects of dietary FODMAPs may contribute to the
of symptoms, and experience sudden and profound weight symptoms of IBS-D. A randomized controlled trial involving
loss. Endoscopic ultrasonography can exclude an infiltrating 84 adults with IBS-D compared a low-FODMAP diet to a diet
tumor, and guidelines recommend its use in patients with a based on modified National Institute for Health and Care
strong suspicion for malignancy. Excellence (mNICE) guidelines. In this study, more patients
Achalasia affects men and women equally, with an on the low-FODMAP diet reported adequate relief of their
annual incidence of 1 in 100,000 individuals. It tends to IBS-D symptoms (52% versus 41 %) and response in abdom-
occur between the ages of 30 and 60 years. Typical achalasia inal pain (51% versus 23%) than those on the mNICE diet.
has an insidious onset and long duration of symptoms, often Alosetron is a peripherally acting serotonin type 3-
measured in years, before patients seek medical attention. receptor antagonist approved by the FDA for the treatment
This patient's age, short duration of symptoms, and rapid of IBS-D in women aged 18 years or older after failure of
weight loss argue against the diagnosis of achalasia. conventional therapy: Due to the risk for adverse events
Chagas disease is caused by infection with the vector- with the use of alosetron, including serious complications
borne parasite Trypanosoma cruzi in rural areas of Latin of constipation and ischemic colitis, a prescriber must first
America. The major manifestations of Chagas disease include complete an FDA-mandated Risk Evaluation and Mitigation
enteric myenteric destruction, resulting in achalasia, rnega- Strategy training program that ls available online (www.
colon, heart disease, and other neurologic disorders. The alosetronrems.com). Alosetron ls not FDA-approved for the
lack of travel and absence of other manifestations of Chagas treatment of men with IBS-D due to the small number of
disease make this diagnosis unlikely: men Involved in the pivotal clinical trials, and it would not
Eosinophilic esophagitis is inconsistent with this be appropriate as an initial treatment for IBS Jn this male
patient's symptoms and endoscopic examination. Its typical patient.
presentation is in younger patients with food bolus obstruc- Linadotide is a peripherally acting guanylate
tion. Endoscopy findings in eosinophilic esophagitis include cyclase-C activator approved by the FDA for the treatment
rings and furrows. of IBS-C. Lubiprostone is a peripherally acting chloride
channel activator that is approved by the FDA for the
KEY POINT
treatment ofIBS-C in women aged 18 years or older. In this
• Pseudoachalasia is caused by a tumor at the gastroe- patient whose predominant bowel symptom is diarrhea,
sophageal junction infiltrating the myenteric plexus use of either of these agents will worsen his diarrhea and
causing esophageal motor abnormalities; symptoms, is therefore not indicated.
barium-imaging and manometric findings, and endo-
KEY POINT
scopic appearance are similar to achalasia.
• A low-FODMAP (Fermentable Oligosaccharide ,
Bibliography Disaccharides, Monosaccharides, And Polyols) diet
Vaezl MF, Pandoltlno JE. Vela MF. II.CG clinical guideline: diagnosis and can reduce abdominal pain and bloating and improve
management of achalasla. Am J Gastroenterol. 2013;108:1238-49; quiz stool consistency. frequency, and urgency in patients
1250. [PMIO: 23Bn3Sl] doi:l0.1038/ajg.2013.196
with diarrhea-predominant irritable bowel syndrome.

ltem6 Answer: C Bibliography


Eswaran SL, Chey WD. Han -Markey T, Ball s. Jackson K. /\. randomized
Educational Objective: Treat diarrhea-predominant controlled trial comparing the low FODMAP diet vs. modified NICE
irritable bowel syndrome. guidelines in US adults with lBS-0. Am J Gastroenterol. 2016;111:1824·
1832. [PMlt:>: 2n2s6s2J dol:l0.1038/ajg.2016.434
A low-FODMAP (Fermentable Oligosaccharfdes, Dlsaccha-
rides, Monosaccharldes, And Polyols) diet is the most appro-
priate treatment for this patient. FODMAPs consist of short- ltem7 Answer: A
chain carbohydrates that are poorly absorbed and rapidly
Educational Objective: Diagnose a Zenker diverticulum.
fermented by gut bacteria, resulting in the production of gas
and an increased osmotic fluid load within the gut lumen. Barium esophagram ls the most appropriate next diagnostic
The patient has diarrhea-predominant irritable bowel syn- test for this patient. The patient's primary symptom of dys-
drome (IBS-D). IBS is a heterogeneous symptom complex phagia associated with regurgitation of undigested food Is
characterized by abdominal pain and altered bowel habits. the classic presentation of a Zenker diverticulum. Other com-
The diagnosis of IBS requires symptoms of recurrent abdom- monly reported symptoms include halitosis, aspiration, and
inal pain or discomfort at least 1 day a week for a period of gurgling in the chest, but esophageal dysphagia is the most
3 months, along with two of the following three additional common symptom, reported by the majority of patients with
criteria: pain relieved by defecation, change in stool frequency, a Zenker diverticulum. This type of diverticulum is located

107
Answers and Critiques

in the cervical esophagus and may lead to complications of breath. Dilation of pulmonary vasculature occurs at
such as aspiration and pneumonia. The best initial test is a the base of the lungs, so hypoxemia is most noted when
barium esophagram, which will Identify the diverticulum. patients are upright or sitting, when shunting is maximal.
Treatment is reserved for symptomatic patients and endo- Classic features are platypnea (worsening shortness of
scopic diverticulectomy is favored where surgical expertise is breath in the upright position) and orthodeoxia (wors-
available. In medical centers without such expertise, surgery ening arterial oxygen sattuation irl the upright position).
by external neck incision is used. Pulse oximetry is often used to screen for changes in the
Esophageal manometry is used when there is concern arterial oxygen saturation level w ith changes of position.
for a motility disorder, such as achalasia. Patients with The diagnosis is made by demonstrating an arterial oxygen
motility disorders commonly report dysphagia to liquids or tension less than 80 mm Hg (10.7 kPa) breathing ambient
both solids and liquids; this patient's dysphagia to solid food air, or an alveolar-arterial gradient oflS mm Hg (2 kPa) or
does not suggest a motility disorder. greater, along with evidence of intrapulmonary shunting
Ambulatory pH testing and impedance-pH testing can on echocardiography with agitated saline or macroag-
be valuable tools in identifying acid exposure within the gregated albumin study. The detection of intrapulmonary
esophagus. Impedance-pH testing can identify both acid shunting of blood is best confirmed by echocardiography
and nonacid reflux. Testing can be done with a 48- hour with agitated saline (also known as a bubble study) , during
wireless capsule or 24-hour transnasal catheter to detect which bubbles are identified in the left side of the heart
active acid reflux. The wireless capsule has been shown after 5 beats, demonstrating that the shunting of blood
to have better patient tolerability. A 24-hour pH manom- is not intracardiac. Clinically significant hepatopulmo-
etry test is often used to further evaluate a patient with nary syndrome is treated with supplemental oxygen and
symptoms of gastroesophageal reflux disease that have not liver transplantation. Hepatopulmonary syndrome is a
responded to medical therapy (usually a trial of an acid-re- progressive condition that is ultimately fatal without liver
ducing agent such as a proton pump inhibitor). A pep- transplantation.
tic stricture causing solid-food dysphagia may result from Bronchoscopy is of no value in the diagnosis of platy-
untreated reflux, but this patient reports dysphagia and pnea or shunting disorders. It is potentially useful in the
regurgitation of undigested food, symptoms strongly sug- diagnosis of a pulmonary infiltrate or relief of an airway
gestive of Zenker d!verticulum. obstruction.
Endoscopy is used to Inspect the mucosa! surface of the CT angiography can demonstrate the presence of large
esophagus, stomach, and duodenum to identify conditions vascular shunts in the lungs but ls rarely required to estab-
within the upper gastrointestinal tract Because this patient's lish the diagnosis of hepatopulmonary syndrome. An addi-
symptoms are consistent with the presentation of a Zenker tional benefit of CT angiography is its ability to show pul-
diverticulum, a barium esophagram is a more appropriate monary emboli. In this patient, the presence of orthopnea
choice than upper endoscopy. Although endoscopy may is not consistent with pulmonary embolism. Transthoracic
identify the diverticulum, there is risk for perforation if the echocardiography with agitated saline is the gold standard
endoscope enters the diverticulum. for detecting pulmonary vascular dilatation and diagnosing
hepatopulmonary syndrome.
KEY POINT
Pulmonary function testing ls useful for evaluating the
• Patients with dysphagia associated with regurgitation presence of obstructive lung disease as well as restrictive
ofundigested food should be evaluated with a barium lung disease. The normal pulmonary examination and nor-
esophagram for the presence of a Zenker diverticulum. mal chest radiography suggest that neither restrictive nor
obstructive lung disease is contributing to this patient's
Bibliography presentation.
Smith CD. Esophageal strictures and dlverttcula. Surg Clln North Am.
2015;95:669-81. [PMID: 25965138] doi:l0.1016/j.suc.2015.02.017 KEY POINT
• The diagnosis of hepatopulmonary syndrome is made
by demonstrating an arterial oxygen tension less than
Item 8 Answer: C
80 mm Hg (10.7 kPa) breathing ambient air, or an
Educational Objective: Diagnose hepatopulmonary alveolar-arterial gradient oflS mm Hg (2 kPa) or
syndrome. greater, along with evidence of intrapulmonary
Echocardiography with agitated saline is the most appro- shunting on echocardiography with agitated saline or
priate next test for this patient. Hepatopulmonary syn- macroaggregated albumin study.
drome is a complication of cirrhosis caused by dilation of
the pulmonary vasculature in the setting of advanced liver Bibliography
disease and portal hypertension. A high alveolar-arterial Krowka MJ. FJllon MB, Kawut SM, Fuhrmann V. Heimbach JK, Ramsay MA,
oxygen gradient results from functional shunting. Patients et al International Liver Transplant Society practk'e guidelines: diagnosis
and management of hepatopulmonary syndrome and portopulmonary
with hepatopulmonary syndrome usually have a preexist- hypertension. Transplantation. 2016;100:1440-52. IPMlD: 27326810]
ing diagnosis of liver disease and present with shortness do!:l0.1097/TP.0000000000001229

108
Answers and Critiques

CJ Item 9 Answer: A Item 10 Answer: B


Educational Objective: Treat acalculous cholecystitis. Educational Objective: Prevent recurrent NSAID-
related peptic ulcer disease.
Cholecystostomy tube placement is the most appropriate
next step in management for this patient. She has risk factors Celecoxib plus omeprawle is the most appropriate treat-
as well as clinical and radiologic findings that are consis- ment regimen for this patient. Patients who have bleeding
tent with acalculous cholecystitis. Cholecystectomy is the ulcers while taking NSAIDs should discontinue NSAIDs
definitive treatment for acalculous cholecystitis in stable permanently if possible. In cases where this is not pos-
patients. However, this patient is now hemodynamically sible, strategies to reduce the risk for recurrent bleeding
unstable and, therefore, requires a temporizing cholecys- should be instituted. Selective cyclooxygenase-2 (COX-2)
tostomy tube to allow time for her to stabilize and for gall- inhibitors, such as celecoxib, preferentially inhibit the
bladder inflammation to improve before cholecystectomy. COX-2 isoenzyme, which prlmarlly modulates pain and
Risk factors for acalculous cholecystitis include diabetes inflammation, and minimally inhibit the COX-1 isoen-
mellitus, sepsis, trauma. burns, vasculitis, cardiovascular zyme, which promotes generation of the gastric mucosal
disease, mechanical ventilation, and total parenteral nutri- protective barrier, decreases gastric acid secretion, and
tion. Acalculous cholecystitis can present with biliary colic helps to maintain good mucosal blood flow. The risk for
symptoms in the alert patient or with unexplained leuko- gastroduodenal ulcers and ulcer complications is signifi-
cytosis, sepsis, and jaundice in the critically ill patient. On cantly lower in patients taking COX-2 inhibitors compared
physical examination, a mass may be palpated in the right wlth nonselectlve NSAIDs such as naproxen; however, Jn
upper quadrant. Ultrasound may show some of the features high-risk patients, such as those with previous peptic ulcer
seen in this patient (distended gallbladder with wall thick- disease, a COX-2 inhibitor alone is no better than a nonse-
ening and pericholecystic fluid), gas bubbles in the fundus lective NSAID coadministered with a proton pump inhibi-
("champagne sign"), or nonvisualization of the gallbladder tor in preventing ulcer complications, with rebleeding rates
altogether. No stones or sludge are present in the gallblad- of approximately 4 % to 6% within 6 months.
der. Management includes bacterial cultures, intravenous A single 12-month randomized study showed an ulcer
antibiotics to cover gram-negative organisms, and cholecys- rebleeding rate of 0% in patients treated with celecoxib
tostomy tube placement in the unstable patient or cholecys - plus omeprazole compared with 9% in patients treated
tectomy in the stable patient. with a celecoxib alone. A direct comparison of naproxen
The role of endoscopic retrograde cholangiopancrea - plus omeprazole to celecox:ib plus omeprazole in the pre-
tography with stenting is evolving. It has been used in case vention of NSAID bleeding in high-risk patients has not
series to decompress the gallbladder in patients with acal- been performed. There is also evidence that COX- 2 inhibi-
culous cholecystitis, but it is not yet considered a first- or tors and nonselective NSAIDs, with the possible exception
second-line treatment option due to less robust evidence to of naproxen, increase the risk for cardiovascular compli-
support this practice and the need for experienced endosco- cations; therefore, the decision to use a COX-2 inhibitor
pists to perform the procedure. requires a harm-benefit analysis that weighs the gastroin-
A hepatobiliary iminodiacetic acid scan may be used testinal risks of an NSAID with the potential cardiovascular
when ultrasonography is equivocal, and it would show non- risks of a COX-2 Inhibitor.
opacification of the gallbladder in cases of cholecystitis. It is The use of ibuprofen, a nonselective NSAID, is no safer
unnecessary in this patient who has clinical and radiologic than the use of naproxen for lessening the likelihood of
features consistent with acalculous cholecystitis. development of peptic ulcers.
This patient's ultrasonographic findings of a distended Other gastroprotective agents, such as H2 blockers
and thick-walled gallbladder with associated perichole- (including ranitidine) and misoprostol, have been ineffec-
cystic fluid and a lack of gallstones, in addition to her risk tive in preventing NSAID-related peptic ulcers in patients
factors for acalculous cholecystitis. make further imaging, at low or moderate risk; however, their efficacy in high-
such as MR cholangiopancreatography, unnecessary and risk patients (such as this patient wlth a previous NSAID-
possibly dangerous due to the need for urgent gallbladder induced gastric ulcer) has not been demonstrated. Misopros-
decompression. tol is associated wlth adverse effects, including diarrhea and
abdominal pain. Also, the required twice-daily dosing for
KEY POINT
ranitidine and four-times-daily dosing for misoprostol may
• Acalculous cholecystitis can present with biliary colic lead to patient nonadherence, which further reduces the
symptoms in the alert patient or with unexplained efficacy of such therapy.
Jeukocytosis, sepsis, and jaundice in the critically ill
KEY POINT
patient.
• In patients requiring NSAIDs, an evidence-based
Bibliography treatment strategy to prevent recurrent NSAID-
Huffman JL, Schenker S. Acute acalculous cholecystltis: a review. CUn induced peptic ulcers is the use of a cyclooxygenase-2
Gastroenterol Hepatol. 2010;8:1S· 22. [PMID: 19747982) doi:l0.1016/j.
cgh.2009.08.034
selective NSAID plus a proton pump inhibitor.

109
Answers and Critiques

Bibliography KEY POINT


Laine L. CLINICAL PRACTICE. Upper gastrolntestlnal bleeding due to a
peplk ulcer. N Engl J Med. 2016;374:2367-76. [PMID: 27305194] • Patients with small (<10 mm) hyperplastic polyps on
doi:10.lOS6/NJ:JMcp1514257 baseline colonoscopic examination should undergo
surveillance colonoscopy in 10 years.

Item 11 Answer: D Bibliography


Educational Objective: Manage follow-up colonoscopy Lieberman DA. Rex DK, Winawer SJ. Giardiello FM, Johnson DA. Levin TR;
United States Multi-Society Task Force on Colorectal Cancer. Guidelines
for hyperplastic polyps. for colonoscopy survei Ila nee after screening and polypectomy: a consen-
sus update by the US Multi-Society Task Force on Colorectal Cancer.
This patient should undergo colonoscopy in 10 years. Gastroenterology. 2012;143:8'1'1-57. [PMID: 22763141] dol:l0.1053/
Serrated polyps are classified Into three histologic types: j.gastro.2012.06.001
hyperplastic polyps, sessile serrated polyps, and tra-
ditional serrated adenomas. Hyperplastic polyps are
the most common type of serrated polyp. They are
non-neoplastic and are composed of normal mucosal
Item 1 2 Answer: B
Educatio nal Objective: Treat acute cholangitis with
Cl
elements; small hyperplastic polyps, often found in the
biliary obstruction.
rectosigmoid colon, are believed to have no clinical sig-
nificance. As a result, the interval until the next screen- Endoscopic retrograde cholangiopancreatography (ERCP)
ing examination is 10 years, the same as for patients is the most appropriate next step in the management of
who do not have polyps found on baseline examination. this patient. The patient presents with the Charcot triad of
Sessile serrated polyps (also known as sessile serrated symptoms-fever, abdominal pain in the right upper quad-
adenomas) and traditional serrated adenomas are both rant, and jaundice- which is consistent with a diagnosis of
neoplastic and are precursors to colorectal cancer; they cholangitis. In patients with evidence of biliary obstruction
should be completely excised. Substantial variability (as seen in this patient's findings on ultrasonography) and
has been demonstrated in the ability of a pathologist to more than mild disease, biliary decompression with ERCP is
differentiate a hyperplastic polyp from a sessile serrated an essential component of therapy. Obstruction is typically
polyp; therefore, guidelines recommend managing large indicated by a dilated bile duct and persistently elevated liver
(>10 mm) hyperplastic polyps as If they are sessile ser- enzyme levels. Indications for urgent ERCP include ongoing
rated polyps. septic physiology (persistently elevated leukocyte count and
The patient's family history of a second-degree rela- temperature greater than 38.9 °C [102 °F]) despite resuscita-
tive with colon cancer diagnosed at age 80 years does not tive measures and antibiotics, hyperbilirubinemia (>S mg/
increase her risk for colon cancer or indicate the need for dL [85.5 µmol /L]), and altered mental status, which may also
more frequent or early colonoscopy. herald a worsened prognosis.
A 1-year surveillance interval is not appropriate for When duct decompression with ERCP is not possible,
this patient. It is indicated in patients with more than percutaneous cholangiography with biJiary tube placement
10 adenomas found on colonoscopy, those with a diag- can be performed. Most patients with cholangitis also have
nosed polyposls syndrome, or those with Lynch syndrome. cholelithiasis, elevated aminotransferase levels, and hyper-
Lynch syndrome is the term used to describe patients who bilirubinemia. Acute cholangitis is usually caused by Esch-
meet the Amsterdam II criteria for hereditary nonpol- erichia coli, Klebsiella species, Pseudomonas species, and
yposis colorectal cancer and have an identified germline enterococci and can progress to septic shock with or without
mutation in one of the four mismatch repair genes (MLHl , liver abscess formation. The mainstay of therapy for cholan-
MSH2, MSH6, PMS2) or the epithelial cell adhesion mole- gitis is the initiation of antibiotic therapy targeting enteric
cule gene (EPCAM) . organisms; for mild to moderate community-acquired dis-
A 3-year surveillance interval is recommended for ease, options include cefawlin, cefuroxime, or ceftriaxone.
patients who have three or more adenomas (or sessile ser- Patients who have complicated gallstone disease with
rated polyps) found on baseline colonoscopy, one adenoma evidence of choledocholithiasis should be considered for
larger than 10 mm in size, or an adenoma with any degree of cholecystectomy; however, in the acute setting, this inter-
villous or high-grade dysp!asia. vention should be deferred until after antibiotic therapy
A surveillance interval of S years is recommended for and biliary decompression with ERCP. Cholecystectomy will
patients with two or fewer adenomas (or sessile serrated remove the source of future gallstones, but it will not decom -
polyps) found on baseline colonoscopy and for patients with press this patient's obstructed common bile duct or reduce
a first-degree relative with colon cancer diagnosed at an the risk for sepsis.
age younger than 60 years. Sessile serrated polyps are more MR cholangiopancreatography Is a diagnostic test that is
frequently found in the proximal colon and may be difficult unlikely to yield additional information other than what this
to detect on colonoscopy due to their flat appearance. Like patient's ultrasound has already shown. In this patient with
tubular adenomas, surveillance colonoscopy is based on size a very high likelihood of finding a common bile duct stone,
and presence of dysplasia. a therapeutic procedure such as ERCP is more appropriate.

110
Answers and Critiques

CJ
CONT.
A percutaneous cholecystostomy can be used to treat
cholecystitis, but in the setting of cholangitls, as in this patient,
than inflammatory, so oral preclnisone would not be effective
for his dysphagia symptoms.
percutaneous cholecystostomy would not necessarily provide
KEY POINT
decompression of the biliary obstruction if the bile duct is
obstructed above the insertion point of the cystic duct. Due • Esophageal stricture in patients with eosinophilic
to their higher rates of morbidity, percutaneous approaches to esophagitis requires treatment with endoscopic dilation
decompression of the biliary system should only be considered when symptoms do not respond to medical therapy.
if endoscopic approaches are unavailable or contraindicated.
Bibliography
KEY POINT
Dellon f'.S·
Gonsalves N, Hirano T, Furuta GT, Liacouras CA, Katzka DA;
• Patients who have cholangitis with evidence of biliary American College ofGastroenterology. ACG clinical guideline: evidenced
base_d approach to the ?!agnos!s and management of esophageal eosino-
obstruction should be treated with antibiotic therapy phllta and eoslnophlhc esophagitls (EoE). Am J Gastroenterol. 2013:
and biliary decompression with endoscopic retro- 108:679-92; quiz 603 fPMID: 23567357] doi:l0.1038/ajg.2013.71
grade cholangiopancreatography.
Item 14 Answer: A
Bibliography Educational Objective: Treat toxic megacolon.
Cl
Demehrl FR, Alam 1-JB. Evidence-based management of common gallstone-
related emergencies. J Intensive Care Med. 2016;31:3-13. [PMID: This patient requires colectomy. He has a history of ulcer-
25320159] dol:lo.nnto885066614554192
ative colitis and presents with fever, tachycardia, hypoten-
sion, and a dilated colon on abdominal radiography; the
diagnosis is toxic megacolon. a life-threatening condition
Item 13 Answer: B that complicates approximately 5% of acute, severe cases of
Educational Objective: Treat an esophageal stricture. ulcerative colitis. Toxic megacolon is defined by the pres-
ence of toxicity (fever, tachycardia, hypotension, and leu-
Endoscopy with dilation is the most appropriate treatment
kocytosis) along with evidence of colonic dilation. Patients
for this patient, who has eosinophilic esophagitis, refractory
with this condition have an increased risk for complications
symptoms of dysphagia despite fluticasone therapy, and the
such as colonic perforation. Intravenous fluid resuscitation
intravenous high-dose corticosteroids, and broad-spect~
finding of an esophageal stricture on endoscopy. Eosinophilic
esophagitis can cause patients to develop a ftbrostenotic
antibiotics (for example, a third-generation cephalosporin
esophageal stricture, which can be treated using endoscopy
with metronidazole) should be initiated in patients with
with dilation. Endoscopic dilation relieves the dysphagia but
toxic megacolon. Management requires close collaboration
has no effect on underlying inflammation; therefore, medical
with a surgeon; therefore, emergent surgical consultation
therapy must be maintained. For unclear reasons, patients
for consideration of subtotal colectomy is required because
with eosinophilic esophagitis are more prone to mucosa!
of the impending risk for perforation and peritonitis in
tears with dilation than are patients with other stricturing
patients with toxic megacolon. Some patients may respond
diseases. It is imperative that the extent of dilation be limited
to medical therapy with high-dose glucocorticoids (in addi-
in amount to avoid these complications; multiple dilations
tion to intravenous fluids and broad-spectrum antibiotics).
may be required to adequately treat the dysphagia.
but there should be a low threshold for surgical intervention
Most patients respond quickly after initiation of the flu-
due to the potential harms associated with toxic megacolon.
ticasone; therefore, continued or increased fluticasone alone
Colonoscopy is contraindicated because it would
will not alleviate the patient's dysphagia symptoms. Contin-
increase the risk for perforation and complications related
ued fluticasone may be necessary as maintenance therapy
to toxic megacolon.
for this patient. Eosinophilic esophagitis is a chronic disease
CT of the abdomen and pelvis is the optimal imaging
that often recurs after treatment is stopped; therefore, repeat
modality to evaluate suspected toxic megacolon and may
or maintenance therapy may be needed.
better assess for the presence of colonic necrosis; however,
Omeprazole and other proton pump inhibitors (PP!s)
in this case, the diagnosis of toxic megacolon can be confi-
are not effective in relieving dysphagia due to stricture. PP!s
dently made based on the patient's presentation and abdom-
can reduce inflammation and eosinophil count and are often
inal radiography, and urgent surgical evaluation is necessary.
used before initiating fluticasone therapy to detennine if the
Stool studies for enteric pathogens may identify a
patient has PPI-responsive eosinophilic esophagitis.
precipitant of toxic megacolon but require a minimum of
Limited data suggest that prednisone may be useful in
24 hours before test results are received. Surgical consulta-
patients with eosinophilic esophagitis who do not experi-
tion should not be delayed to wait for results of stool testing.
ence relief of symptoms with fluticasone therapy. However,
like the swallowed aerosolized glucocorticoids, relapse is KEY POINT
common when the medication is stopped, and relatively • Toxic megacolon is defined by the presence of toxicity
high doses are typically required, which carry associated and evidence of colonic dilation; it requires prompt
risks of immunosuppression and other side effects. Addi- surgical treatment.
tionally, this patient's esophageal stricture is fibrotic rather

111
Answers and Critiques

Bibliography KEY POINT


Seah D, De Cruz P. Review article: the practical management of acute severe
ulcerative colltls. Aliment Pharmacol Ther. 2016;43:482-513. [PMID: • Microscopic colitis is a cause of nonbloody, watery
26725569] doi:lO.llll/apl.13491 diarrhea in older adults and is diagnosed by colonos-
copy with random biopsies from multiple colonic
segments.
Item 15 Answer: B
Educational Objective: Diagnose microscopic colitis. Bibliography
Pardi DS. Diagnosis and management of microscoplc colitis. Am J
Based on the patient's age, sex, and clinical presenta- Gastroenterol. 2017;112:78-85. [PMID: 27897155) dol:l0.1038/ajg.2016.477
tion, microscopic colitis is the most likely diagnosis.
Microscopic colitis is the underlying cause in 10% to
15% of patients with chronic, watery diarrhea. In con- Item 1 6 Answer: D
trast to inflammatory bowel disease, microscopic colitis
Educational Objective: Treat autoimmune pancreatitls.
Cl
is more common in older persons and does not cause
endoscopically visible inflammation. The symptoms of Prednisone is the most appropriate treatment for this
microscopic colitis are similar to other chronic causes of patient. Based on his typical symptom of painless jaun-
nonbloody diarrhea, such as celiac disease and irritable dice and the characteristic "sausage-shaped" pancreas on
bowel syndrome; therefore, colonic mucosa! biopsies imaging, the patient has type 1 autoin1mune pancreatitis,
are required for diagnosis. Lymphocytic and collagenous a frequent manifestation of lgG4 disease. He also has asso-
colitis are the two subtypes of microscopic colitis, and ciated IgG4 -related conditions, sialadenitis, and probable
they are distinguishable only by histology. Random biop- retroperitoneal fibrosis. Autoimmune pancreatitis is rare
sies from multiple colonic segments are recommended to and has an unclear pathogenesis. Type 1 autoimmune pan-
establish the diagnosis because the disease can be patchy. creatitis is a systemic fibroinflammatory disease, defined
In some patients, certain medications (such as NSAIDs as an inflammatory condition causing tissue damage and
and proton pump inhibitors) have been implicated as scarring. Pancreatic involvement is only one manifesta-
causative agents. Microscopic colitis is associated with tion of lgG4 disease, which is characterized by abundant
other autoimmune diseases such as diabetes mellitus lgG4-producing plasma cells seen on tissue biopsy. Most
and psoriasis. The association with celiac disease is of lgG4-related conditions are characterized by plasma-cell
particular clinical importance because the symptoms of infiltration of the affected tissue and the clinical conse-
these conditions are similar. Therefore, in patients with quences that infiltration entails. Almost any organ can be
celiac disease or microscopic colitis whose symptoms do involved; lymph nodes are frequently affected. The ini-
not respond to appropriate therapy, the other condition tial treatment is oral prednisone with a taper over 2 to
must be ruled out. 3 months, and symptoms usually resolve within 2 to 4
Giardia lamblia is a common infectious cause of per- weeks with treatment. Almost all patients (>90%) enter
sistent diarrhea in immunocompetent patients in developed clinical remission in response to glucocorticoids. Treat-
countries and should be considered in patients with expo- ment response may be limited by the amount of fibrosis
sure to young children or potentially contaminated water, present before initiation of therapy. The relapse rate is
such as lakes and streams. This patient has no risk factors for approximately 30% in type 1 autoimmune pancreatitis.
Giardia infection. Azathioprine is an immunosuppressive drug that has
Small intestinal bacterial overgrowth (SIBO) causes been used to treat relapsing lgG4-related disease and can
diarrhea, often with bloating, flatulence, and weight loss. be used as a glucocorticoid-sparing agent. It may take 6 to
Several conditions can predispose patients to SIBO due to 8 weeks to reach a therapeutic drug level and provoke a clin-
effects on stomach acid, intestinal transit, or disruption of ical response. Therefore, it is not considered initial first-line
normal antibacterial defense mechanisms. Gastric bypass therapy for symptomatic IgG4-related disease.
surgery is an increasingly common cause of SIBO. The Endoscopic retrograde cholangiopancreatography with
absence of malabsorption symptoms and weight loss make bile-duct stenting is usually not required in patients with
this diagnosis unlikely. autoimmune pancreatitis because most patients' symptoms
Ulcerative colitis typically presents with bloody diar- respond quickly to oral prednisone. Glucocorticoid therapy
rhea and abdominal discomfort, the severity of which is also treats the underlying immunologic disorder, making It
related to the extent and severity of inflammation. Because the preferred first-line treatment.
ulcerative colitis typically involves the rectum, patients with Pancreaticoduodenectomy (Whipple surgery) is not
this condition commonly present with tenesmus, urgency; indicated in patients with autoimmune pancreatitis. It may
rectal pain, and fecal incontinence. Some patients with distal be considered to treat pancreatic adenocarcinoma occur-
inflammation can present with constipation owing to rec- ring in the head of the pancreas, which can be mistaken for
tal spasm and stasis of stool. Fever and weight loss suggest autoimmune pancreatitis. Because this patient's histology
severe disease. The patient's symptoms are not typical for findings confirm lgG4-related disease, a trial of immuno-
ulcerative colitis. suppressive therapy is the most appropriate treatment.

112
Answers and Critiques

KEY POINT most common laboratory findings are elevated bilirubin


and alkaline phosphatase levels. The condition is believed
• Almost all patients (>90%) with autoimmune pancre-
to result from sex hormone- induced inhibition of bile salt
atitis enter clinical remission in response to glucocor-
export from hepatocytes. It is treated with ursodeoxycholic
ticoids.
acid, which can result in alleviation of symptoms. Although
the maternal effects ofintrahepatic cholestasis of pregnancy
Bibliography are mild, it can cause fetal distress and premature labor.
Okazaki K. Uchida K. J\utolmmune pancreatltis: the past. present, and
future. Pancreas. 2015;4~ : 1006 - 16. [PM!D: 26355544) doi :l0.1097/ Because this patient's clinical profile is not compatible with
MPA.0000000000000382 intrahepatic cholestasis of pregnancy, ursodeoxycholic acid
therapy is not indicated.
KEY POINT
:J [:J Item 17 Answer: B • The fetus should be delivered immediately upon rec-
Ed ucat io nal Objective: Treat acute fatty liver of ognition of acute fatty liver of pregnancy.
pregnancy with immediate delivery of the fetus.
Immediate delivery of the fetus is the most appropriate next Bibliography
step in management. This patient has findings of acute fatty Bacak SJ, Thornburg LL Liver failure In pregnancy. Crit Care Clin.
2016;32:61 -72. [PMID: 26600444) dol:l0.1016/j.ccc.2015.08.005
liver of pregnancy, which is a rare but serious condition
occurring most commonly in the third trimester. Women
with this condition typically present with a 1- to 2-week Item 18 Answer: A
history of nausea and vomiting, right-upper-quadrant or
Educational Objective: Treat a gallbladder polyp.
epigastric pain, headache, jaundice, anorexia, and /or poly-
uria and polydipsia (due to associated transient diabetes Cholecystectomy is indicated for this patient with a gall-
insipidus). Maternal and neonatal mortality rates are high bladder polyp and gallstones because of the increased risk
in this setting. This patient's presentation with coagulopa- for gallbladder cancer when the two conditions coexist.
thy, hypoglycemia, and somnolence consistent with hepatic The finding of a gallbladder polyp larger than 1 cm in size
encephalopathy are indications of acute liver failure, which is an indication for cholecystectomy, even if the patient
can result from acute fatty liver of pregnancy. Immediate is asymptomatic. An additional indication for prophy-
delivery of the fetus is indicated to prevent fetal mortality lactic cholecystectomy is the presence of a gallbladder
and to reverse the mother's liver failure and improve her polyp larger than 8 mm in size in the setting of primary
condition. Prompt delivery typically results in improvement sclerosing cholangitis. Gallbladder polyps are found on
of the mother's medical condition within 48 to 72 hours. approximately 5% of ultrasounds. Although only a small
Acute fatty liver of pregnancy can reoccur in subsequent preg- percentage of gallbladder polyps are neoplastic (adenoma
nancies. It is also associated with long-chain 3-hydroxyacyl or adenocarcinoma), the risk for neoplasia increases as
CoA dehydrogenase deficiency, and affected women and polyp size increases.
their children should be screened for this deficiency. HELLP Further evaluation of the polyp with abdominal CT,
(Hemolysis, Elevated Liver enzymes, and Low Platelets) endoscopic retrograde cholangiopancreatography, or MR
syndrome can also occur in the third trimester of preg- cholangiopancreatography is not indicated because gallblad-
nancy. It has similarly life-threatening consequences and der ultrasonography is adequately sensitive for the detection
also requires emergent delivery of the fetus to resolve the of gallbladder lesions. These tests could be considered if this
condition. HELLP syndrome can present similarly to acute patient had symptoms or elevated liver chemistry tests sug-
fatty liver of pregnancy in that manifestations of liver failure gesting bile-duct obstruction or malignancy.
are present. In a patient with an 8-mm gallbladder polyp in the
This patient's ultrasound showed no dilation of bile absence of gallstones or primary scleroslng cholangitls,
ducts, and no stones were seen in the common bile duct; repeat ultrasonography in 6 months would be indicated.
therefore, endoscopic retrograde cholangiopancreatography However, follow-up ultrasonography ls not appropriate for
is not indicated. this patient with a gallbladder polyp and gallstones, which
This patient's drowsiness is due to impending liver increase the risk for gallbladder cancer.
failure. Lactulose, which is used for treatment of hepatic Bile acids, such as ursodeoxycholic acid, work by
encephalopathy in patients with chronic liver disease, is reducing biliary cholesterol secretion, thereby increasing
not indicated for patients with acute liver failure because it biliary bile-acid concentrations and, as a result, reducing
may exacerbate symptoms of ileus, and there is no evidence the cholesterol saturation index and gallstone size. Bile-
of benefit in the acute setting. Any symptoms referable to acid therapy works best for small, primarily cholesterol
hepatic encephalopathy will resolve with delivery of the gallstones and is indicated in patients who cannot or will
fetus, which resolves acute fatty liver of pregnancy. not undergo laparoscopic cholecystectomy. More impor-
Intrahepatic cholestasis of pregnancy occurs during the tantly, ursodeoxycholic acid therapy would not address this
second or third trimester and resolves after delivery. The patient's gallbladder polyp.

113
Answers and Critiques

KEY POINT Bibliography


Lopes J\I. Vale FF. Oleastro M. Heltcobocter pylori lntectlon-recent develop-
• The finding of a gallbladder polyp larger than 1 cm in ments In diagnosis. World J Gastroenterol. 2014:20:9299- 313. [PMID:
size, or a polyp of any size associated with gallstones, 2507132~ I doi:I0.3748/wjg.v20.128.9299

is an indication for cholecystectomy even if the


patient is asymptomatic.

Bibliography
Item 20 Answer: B CJ
Educational Objective: Treat gallstone pancreatitis with
Gallahan WC, Conway JD. Diagnosis and management of gallbladder polyps.
Gastroenterol Clin North Am. 2010;39:359-67, x. (PMJD: 20478491] prompt cholecystectomy.
dol:l0.1016/j.gtc.2010.02.001
Laparoscopic cholecystectomy before discharge from
the hospital is the most appropriate treatment. Gallstone
Item 19 Answer: C acute pancreatitis can be diagnosed based on elevated
liver transaminases on presentation, a lipase level elevated
Educational Objective: Evaluate for Heltcobacter pylori
to more than three times the upper limit of normal, char-
eradication following treatment.
acteristic severe abdominal pain, and ultrasonographic
A urea breath test is the most appropriate next t~t for this evidence of cholelithiasis. This patient showed clinical
patient. Testing to confirm eradication should be pursued in improvement within 3 days of hospitalization. Her lab-
all cases of' identified and treated Helicobacter pylori infection oratory values have normalized, suggesting spontaneous
because of the established risks for peptic ulcer disease and passage of a gallstone through the common bile duct.
gastric malignancy ln patients with chronic H. pylori infection. which occurs in most patients with gallstone pancreatitis.
To maximize the accuracy of testing to confirm eradication, In a multicenter randomized controlled trial. same-admis-
testing should be performed a minimum of 4 weeks after sion cholecystectomy reduced rates of gallstone-related
completion of H. pylori eradication therapy and after proton complications compared with interval cholecystectomy 25
pump inhibitor therapy has been discontinued for 1 to 2 weeks to 30 days after hospital discharge for patients with mild
and H2-blockers for 1 to 2 days. The test chosen should be gallstone pancreatitis.
highly accurate in identifying active infection; appropriate tests Endoscopic retrograde cholangiopancreatography
include the urea breath test, fecal antigen test, or biopsy-based (ERCP) is indicated within the first 24 hours for patients
testing. The urea breath test ls limited by need for specialized with acute pancreatilis and ascending cholangitls (fever,
equipment and personnel and by its cost. The fecal antigen test abdominal pain, and jaundice) due to choledocholith!-
is limited by the collection ofstool but is less expensive than the asis. If there is evidence of ongoing biliary obstruction
urea breath test. Biopsy-based testing is expensive and invasive. in patients hospitalized with acute pancreatitis, ERCP
Unless upper endoscopy is indicated for other reasons, nonin- may be indicated to remove a retained gallstone from the
vasive testing modalities (the urea breath test or the fecal anti- common bile duct. This patient's symptoms and labora-
gen test) are more appropriate for confirmation of eradication tory abnormalities resolved quickly, which supports the
or assessment for reinfection. Both testing modalities are equiv- spontaneous passage of a gallstone without evidence of
alent with regard to accuracy; therefore, the specific test chosen ongoing biliary obstruction.
should be based on patient preference and/or test availability. MR cholangiopancreatography (MRCP) can be used
Serologic antibody testing is an inaccurate means to identify causes of biliary obstruction. MRCP is not
of testing to confirm eradication because antibodies can needed in this patient because she has normal -caliber
remain present despite successful eradication of active infec- bile ducts on abdominal ultrasonography and normal
tion; therefore, serologic testing cannot distinguish between liver chemistry test results, Indicating that a biliary
past and current H. pylori infection. obstruction is unlikely.
Invasive (endoscopic) tests for H. pylori include the Ursodeoxycholic acid has been used to medically dis-
rapid urease test, histology, and culture; all invasive testing solve small cholesterol gallstones in patients who are not
modaltties identify active infection. Due to its expense and candidates for surgery. The medication works slowly and
invasive nature, biopsy-based testing should be reserved for may take longer than 1 year to dissolve small stones, leaving
patients requiring a repeat upper endoscopy for other rea- patients at risk for recurrent attacks of gallstone pancreatl-
sons (for example, follow-up endoscopy for high-risk gastric tis or other gallstone-related complications. This patient is
ulcer). In this patient with no indications for repeat upper young and without comorbidities, making surgery a more
endoscopy, one of the two noninvasive tests, either the urea appropriate treatment.
breath test or fecal antigen test, is preferable.
KEY POINT
KEY POINT • Same-admission cholecystectomy reduces rates of
• After eradication therapy for Helicobacter pylori gallstone-related complications compared with chole-
infection, eradication should be confirmed using the cystectomy after hospital discharge for patients with
urea breath test or feca l antigen test. mild gallstone pancreatitis.

114
Answers and Critiques

Bibliography Bibliography
da Costa DW, Bouwense SA, Schepers NJ, Besse link MG. van Santvoort HC, Lindor KD, Kowdley KV, Harrison ME; American CollegeofGastroenterology.
van Brunschot S, et al; Dutch Pancreatills Study Group. Sarne-admission ACG clinical guideline: prlrnary sclerosfng cholangftls. Am J
versus Interval cholecystectomy for mild gallstone pancreatitfs Gastroenterol. 2015;110:646-59; quiz660. [PMID: 25869391] doi:I0.1038/
(PONCHO): a multicentre randomised controlled trial. Lancet. ajg.2015.112
20l5;3B6:1261-8. [PMJO: 26460661] doi:l0.10J6/SOJ40·-6736(15)00274-3

Item 22 Answer: B
CJ Item 21 Answer: B Educational Objective: Evaluate rectal bleeding in a
Educational Objective: Treat cholangitis using yowig patient.
endoscopic retrograde cholangiopancreatography.
Colonoscopy is the most appropriate next step in manage-
Endoscopic retrograde cholangiopancreatography (ERCP) is ment for this patient. Her symptoms of weight loss, abdom-
the most appropriate next test for this patient. This patient inal pain, and rectal bleeding with iron deficiency anemia
with primary sclerosing cholangitis (PSC) presents with fever, warrant further evaluation with colonoscopy. Inflammatory
rigors, right-upper-quadrant pain, and leukocytosis, all of bowel disease is a possibility in this patient; however, colon
which are consistent with bacterial cholangitis. Indications cancer also needs to be excluded. In the United States, the
for ERCP in patients with PSC are bacterial cholangitis (as overall incidence of colon cancer has decreased by 3% to 4%
in this patient), increasing jaundice, increasing pruritus, or since the early 2000s; however, in adults younger than age
a dominant stricture seen on imaging. Symptoms of bacte- SO years, the incidence of colorectal cancer is increasing at a
rial cholangitis, increasing jaundice, and pruritus can signify rate of 2.1 % per year.
strictures that may improve with dilation or stentlng, or, Anoscopy enables a limited examination and would not
alternatively, removing sludge or stone debris in the bile ducts assist in evaluating the patient's iron deficiency anemia and
via ERCP. A dominant stricture in a patient with PSC must be weight Joss.
evaluated for cholangiocarcinoma by obtaining biliary brush- Flexible sigmoidoscopy allows examination of only the
ings for cytologic examination, and, if available, fluorescent lower third of the colon, and if negative, a full colonoscopy
in situ hybridization to evaluate chromosomal abnormalities. would still be required. There has been a gradual shift from
The CA 19-9 level will be elevated in the setting of bac- left-sided to right-sided colon cancers. A number of factors
terial cholangitis. The risk for false -positive results makes have been suggested to explain this, including inadequate
CA 19-9 measurement inappropriate in this context. CA colon preparation, incomplete colonoscopy, and difficulty
19-9 levels can be used as an adjunctive tool in the diagnosis recognizing serrated polyps that are typically flat and occur
of cholangiocarcinoma, but the diagnosis cannot be made more often on the right side. However, there also appears to
based only on this marker. be a true increase in tumors of the proximal and right colon
lgG4 levels should be checked in patients with a new that may be missed if only a sigmoidoscopy is performed.
diagnosis of presumed PSC because IgG4 cholangitis Is a Hemorrhoids are arteriovenous communications cov-
steroid-responsive condition, whereas PSC Is not. Testing ered by cushions of connective tissue in the anal canal.
for lgG4 does not assist in the management of this patient's Internal hemorrhoids cause most hemorrhoidaJ symptoms
cholangitis. (bright red blood dripping in the toilet bowl or seen on toilet
A percutaneous transhepatic biliary tube can be paper, with no accompanying pain; a protrusion of tissue;
employed when ERCP is unsuccessful at traversing a biliary itching; and pain). Patients with alarm features (such as
stricture. but because of its invasiveness and inconvenience, unexplained weight loss, change in bowel movements, iron
it would not be a first-line tool for assessing and treating a deficiency anemia, age older than SO years, or personal or
patient with bacterial cholangitis. family history of colorectal cancer or inflammatory bowel
The role of PET in the evaluation and management of disease) warrant colonoscopy. Initial treatment of internal
cholangiocarcinoma is evolving. However, this is not the and external hemorrhoids consists of dietary and lifestyle
test of choice in a patient with bacterial cholangitis because modifications to soften bowel movements and avoid consti-
it does not allow for biliary intervention. Furthermore. PET pation, straining, and prolonged time on the toilet. Increased
scans can be associated with false-positive results for malig- fiber intake has been shown to reduce symptomatic prolapse
nancy in the setting of bacterial cholangitis, and also with and bleeding. Local therapy such as topical anesthetics and
false-negative results due to the desmoplastic reaction of glucocorticoids may relieve pain and itching, but data to
cholangiocarcinoma tumors. support their use are scant.
K EY POINT KEY POINT
• Indications for endoscopic retrograde cholangiopan- • Red-flag symptoms such as rectal bleeding with iron
creatography in patients with primary sclerosing deficiency anemia, abdominal pain, and weight loss
cholangitis are bacterial cholangitis, increasing jaun- should prompt evaluation by colonoscopy for colorec-
dice. increasing pruritus, or a dominant stricture on tal cancer regardless of the patient's age or the pres-
imaging. ence of bleeding hemorrhoids.

115
Answers and Critiques

Bibliography Item 24 Answer: B


lnra JA. Syngal S. Colorectal cancer in young adults. Dig Dis Sci. 2015 ·60·722-
33. [PMID: 25~80~03] dol:I0.1007/sl0620- 014-3464-0 ' . Educational Objective: Treat pain in chronic pancreatitis.
The most appropriate initial treatment is the use of NSAIDs,
a low-fat diet, and smoking cessation. The hallmark symp-
Item 23 Answer: D
tom of chronic pancreatitis is abdominal pain that often
Educational Objective: Diagnose narcotic bowel radiates to the back; however, pain can be absent. Pain is
syndrome. typically intermittent, with attacks interrupted by varying
Narcotic bowel syndrome is the most likely diagnosis in this pain-free intervals. Constant pain may occur from local ana-
patient Narcotic bowel syndrome, also known as opiate-induced tomic causes (compressing pseudocyst, biliary or pancre-
gastrointestinal hyperalgesia, is a centrally mediated disorder atic duct stricture) or from visceral hyperalgesia (increased
of gastrointestinal pain characterized by a pamdoxical increase sensation in response to stimuli) from chronic narcotic use
in abdominal pain with increasing doses of narcotics. Many and centralization of pain. Management focuses on reducing
patients are prescribed opioid pain medication for an unrelated pain and detecting and treating complications. Unfortu-
medical condition but have increasing pain over time despite nately, the treatment of persistent pain is difficult, and the
clinical evidence of improvement or resolution of the initial evidence supporting most treatment modalities is of low
condition. Patients often fear tapering off opioids and believe the quality and often contradictory. Most autho1ities recom-
opioids are the only treatment that alleviates their pain. mend that persistent pain be treated in a stepwise approach
Chronic pancreatitis may result from a severe episode of beginning with lifestyle modification (discontinue alcohol
acute pancreatitis or recurrent episodes of acute pancreati- and cigarettes), use of simple analgesics, adding low-dose
tis. lt may require opioid pain medication for flares of pain or tricyclic antidepressants, and gabapentinoids (gabapentin
for constant, daily pain, but the pain is usually relieved with and pregabalin). Smoking has been identified as an import-
opioids. This patient only had one episode of uncomplicated ant and independent risk factor for chronic pancreatitis, and
pancreatitis and has no other risk factors for chronic pan- cessation of smoking and alcohol use is recommended to
creatitis and no evidence of chronic pancreatitis (pancreatic prevent recurrent attacks of pancreatitis. An important goal
calcifications) on abdominal imaging. is to control pain with opioid-sparing adjunctive agents to
Distal intestinal obstruction syndrome is caused by minimire chronic opioid use, owing to concerns for opioid
thickened intestinal contents that completely or partially dependence and gastrointestinal side effects. This would
block the small intestinal lumen in patients with cystic especially be true in a patient with a history of opioid or
fibrosis. It is characterired by progressive, crampy abdomi- other substance abuse, such as this patient.
nal pain in the right lower quadrant, sometimes progressing Nerve blocks, such as celiac plexus blocks, and neuroly-
to vomiting. This patient has no history of cystic fibrosis and sis procedures are not recommended for the management of
no symptoms of a small-bowel obstruction making this an pain related to chronic pancreatitis because the response rate
unlikely diagnosis. is low (15%), and pain relief, if achieved at all, is short-lived.
The diagnosis of irritable bowel syndrome is typically Opioid pain medications, such as oxycodone, are used
made in the presence of recurrent abdominal pain or dis- in acute pancreatitis and during acute flares of chronic pan-
comfort at least 3 days per month in the last 3 months that creatitis, but they should be avoided in the long-term man-
is associated with two or more of the following: relief with agement of ongoing pain due to the iisk for hyperesthesia
defecation, onset associated with a change in frequency of and the development of tolerance and/or addiction.
stool, or onset associated with a change in form (appearance) Pancreatic enzyme replacement therapy is often rec-
of stool. While this patient has abdominal pain and constipa- ommended for the treatment of persistent pain associated
tion, her symptoms are worsened with eating, not relieved with chronic pancreatitis. A large systematic review found
with defecation, and the onset of pain Is clearly tied to an conflicting evidence for the efficacy of pancreatic enzymes
acute abdominal diagnosis and treatment with an opioid, in relieving pain in patients with chronic pancreatitis but
making JBS an unlikely diagnosis. found that they may improve fat absorption. If used, acid
suppression therapy with a proton pump inhibitor should
KEY POINT be given as an adjunct to uncoated pancreatic enzymes in
• Narcotic bowel syndrome, also known as opiate- order to reduce the inactivation of enzymes by gastric acid.
induced gastrointestinal hyperalgesia, is a centrally Simple analgesics and lifestyle modifications can be effective
mediated disorder of gastrointestinal pain character- for some patients and should be initiated first.
ized by a paradoxical increase in abdominal pain with KEY POINT
increasing doses of opioids.
• Treatment of chronic pancreatitis-related persistent
pain should proceed in a stepwise approach begin-
Bibliography
ning with lifestyle modifications (discontinue alcohol
Keerer '~ Drossman DA, Guthrie E, Simren M, Tllllsch K, Olden K, et al.
Centrally mediated disorders or gastrointestinal pain. Gastroenterology. and cigarettes) and the use of simple analgesics
2016. [PMID: 271446281 dol:I0.1053/j.gastro.2016.02.034 (acetaminophen, NSAIDs).

116
Answers and Critiques

Bibliography
Majumder S, Chari ST. Chronic pancreatitis. Lancet. 2016;387:1957-66.
[PMID: 26948434] doi:l0.1016/SOM0-6736(16)00097-0
It em 2 6 A nswer : A
Educational Objective: Treat acute necrotic pancreatitis
Cl
with enteral feeding.
Placement of a nasoenteric tube for enteral feeding is the
Item 25 Answer: A most appropriate next step in this patient's management.
Educational Objective: Treat hepatitis B virus-related The patient has taken nothing by mouth (NPO) since hos-
polyarteritis nodosa pitalization 4 days earlier and requires nutritional support.
She continues to have abdominal pain requiring intravenous
The most appropriate treatment is entecavir. Hepatitis opioid pain medication, as well as nausea and no appetite.
B virus (HBV) - related polyarteritis nodosa (PAN) is the Enteral nutrition is preferred in patients with acute pancre-
most likely diagnosis in this patient. The patient has fever, atitis because of the benefit of maintaining a healthy muco-
arthralgia, and evidence of cutaneous vasculitis (rash). sa! barrier in the gut to prevent translocation of bacteria.
There is also evidence of hepatitis (elevated bilirubin and Both nasogastric and nasojejunal enteral feeding are safe
aminotransferase levels) likely related to his use of intrave- and have comparable effectiveness. Studies show nasogastric
nous drugs. Testing for hepatitis B surface antigen and hep- tube feeding is well tolerated in these patients. and place-
atitis B e antigen is positive, as is IgM antibody to hepatitis ment ofa nasogastric tube may be easier, more cost effective,
B core antigen, with a significant HBV DNA viral load; all and faster than placement of a nasojejunal tube.
of these results are consistent with a recent HBV infection. Total parenteral nutrition (TPN) is discouraged in
PAN and mixed cryoglobulinemia are rare HBV-associated patients with acute pancreatitis because the mucosa! barrier
vasculitides. This patient has undetectable serum cryo- is not maintained when patients are NPO for prolonged peri-
globulins, excluding the diagnosis of mixed cryoglobulin- ods, which may lead to higher rates of bacterial translocation
emia. There are no data from randomized trials to guide into necrotic pancreatic tissue. TPN also increases the risk
therapy in patients with HBV-related PAN. Patients with for bacterial and fungal bloodstream infections, as well as
HBV infection and mild manifestations of PAN are usually venous thrombosis associated with more proximal indwell-
treated with antiviral agents such as entecavir. Patients ing intravenous catheters required for TPN administration.
(such as this one) with mild PAN have constitutional symp- Cholecystectomy will not be an option for this patient
toms, arthritis or arthralgia, anemia, and skin lesions. More in the near future (likely not during this hospitalization)
severe disease is characterized by organ dysfunction (for because she has severe necrotic acute pancreatitis. Main-
example, myocarditis, kidney insufficiency and hyperten- taining NPO status for a prolonged period of time before
sion, mononeuritis multiplex) or life-threatening systemic surgery is not appropriate clinical management of her
manifestations. Patients with severe manifestations of PAN nutritional needs.
are often treated with an antiviral agent and a short course There are no data to support withholding feeding in
of prednisone and plasma exchange. patients with acute pancreatitis based on enzyme levels.
Prednisone and cyclophosphamide are agents typi- Lipase levels three to five times the upper level of normal are
cally used to treat mild and severe manifestations of idio- used as diagnostic criteria for acute pancreatitls but have no
pathic PAN, the most commonly recognized form of PAN. impact on clinical management decisions.
Because this patient has HBV-related PAN, treating the
causative viral infection with an antiviral agent is the pre- KEY POINT
ferred management. • Enteral nutrition is preferred in patients with acute
Sofosbuvir and ledipasvir are direct-acting antiviral pancreatitis because of the benefit of maintaining a
agents used to treat hepatitis C virus (HCV) infection and healthy gut mucosa! barrier to prevent translocation
would be an appropriate choice for mild HCV-related PAN. of bacteria.
However, this patient's hepatitis C antibody is negative and
these agents are not indicated. Bibliography
Tenofovir, emtricitabine, and raltegravir are used to Forsmark CE, Vege SS, Wilcox CM. Acute pnncreatilis. N Engl J Med.
treat HIV infection. This drug combination would be an 2016;375:1972-1981. [PMID: 27959604]
appropriate choice for HIV-related PAN, but the patient's
HIV test is negative and these agents are not indicated. Teno-
It em 2 7 Answer: C
fovir alone would be a reasonable treatment for this patient.
KEY POINT
Educationa l O bjective: Prevent spontaneous bacterial
Cl
peritonitis.
• Mild hepatitis B virus-related polyarteritis nodosa is
treated with antiviral agents. [nitiation of indefinite primary prophylaxis with ciprofloxa-
cin is the most appropriate next step in the management of
this patient with ascites. Patients with ascites are at risk for
Bibliography
Forbess L, Bannykh S. Polyarteritis nodosa. Rheum Dis Clin North Am.
developing spontaneous bacterial peritonitis (SBP), a com-
2015;41:33- 46, vii. [PMID: 25399938] doi:l0.1016/j.rdc.2014.09.005 mon infection in patients with cirrhosis. SBP has a mortality

117
Answers and Critiques
- -

rate of 20%. Long-term primary antibiotic prophylaxis may


Cl reduce mortality in patients at high risk for SBP. Criteria for
into the small intestine after partial gastric resection
can lead to postprandial vasomotor symptoms known as
CONT. patients at high risk include an ascitic-fluid total protein dumping syndrome. This patient had a gastrojejunostomy
level less than 1.5 g/dL (15 g/L) in conjunction with any 1 year earlier, and her symptoms started after the surgery.
of the following: serum sodium level less than or equal to Common early symptoms of dumping symptoms are palpi-
130 mEq/L (130 mmol/L), serum creatinine level greater tations, tachycardia, diaphoresis, and lightheadedness with
than or equal to 1.2 mg/dL (106.1 µmol/L), blood urea nitro- abdominal pain and diarrl1ea presenting within 30 min-
gen level greater than or equal to 25 mg/dL (8. 9 mmol/L), utes of eating. Late symptoms can occur 1 to 3 hours after
serum bilirubin level greater than or equal to 3 mg/dL eating in 25% of patients and include sweating, tremor,
(51.3 µmol /L). or Child-Turcotte-Pugh class B or C cirrhosis. hunger, and difficulty with concentration and cognition
Patients who have had a bout of SBP should also receive (hypoglycemia). A minority of patients have both early
lifelong antibiotic prophylaxis to reduce the risk for recur- and late symptoms. The diagnosis is usually made based on
rence. In the setting ofvariceal hemorrhage, a limited 7-day clinical findings. Initial treatment for dumping syndrome
course of antibiotics initiated at the time of bleeding is indi- is dietary: eating small frequent meals and saving liquids
cated to prevent infectious complications from intestinal until after a meal, which this patient has already found to
bacterial translocation. help her symptoms.
Increasing this patient's propranolol is not indicated Dumping syndrome is misdiagnosed as gastroparesis in
because, although P-blocker therapy can reduce the risk up to 37% of patients. Gastroparesis involves delayed gastric
for variceal bleeding, the patient already has a pulse rate of emptying and is typically associated with nausea and vom-
less than 60/min. Nonselective P-blockers such as propran- iting after eating. The prominent vasomotor symptoms this
olol may be associated with higher transplant-free survival patient describes are uncommon in gastroparesis.
in patients with cirrhosis overall but may decrease trans- Irritable bowel syndrome is more common in younger
plant-free survival in the first 6 months after SBP or in patients, and rarely causes weight loss. Symptoms that occur
patients with refractory ascites, and discontinuation should after a gastrointestinal surgery should suggest an alternative
be considered at that time. diagnosis.
Albumin infusion may decrease the frequency ofhepa- Small intestinal bacterial overgrowth (SIBO) is an excess
torenal syndrome and improves survival in patients with number and alteration in type of bacteria cultured from the
SBP. but its role in primary prevention of SBP is undefined small intestine. Digestive enzymes and intestinal motility
and its use for primary prevention not recommended. normally limit the growth of excessive bacteria, but SIBO
Systemic blood pressure decreases in patients with can occur in conditions in which these functions are dis-
decompensated cirrhosis resulting in reductions in renal rupted, including gastrojejunostomy, which creates a blind
perfusion and glomerular filtration rate. This leads to ele- loop of small intestine. Clinical features ofSIBO are diarrhea,
vated levels of vasopressin, angiotensin, and aldosterone. bloating, and weight loss. Patients may have a combination
ACE inhibitors, such as lisinopril, and angiotensin recep- of fat, protein, or carbohydrate malabsorption. Unlike in
tor blockers impair these compensatory efforts to maintain dumping syndrome, symptoms are not immediately related
blood pressure and can worsen kidney perfusion in the to eating and are not associated with prominent vasomotor
setting of ascites due to portal hypertension; therefore. initi- symptoms sucl1 as palpitations, tachycardia, diaphoresis,
ating lisinopril is inappropriate in this patient. and lightheadedness.
KEY POINT KEY POINT
• Primary prophylactic antibiotic therapy is indicated • Rapid gastric emptying of hyperosmolar chyme into
for patients at high risk for the development of spon- the small intestine after partial gastric resection can
taneous bacterial peritonitis, including patients with lead to postprandial vasomotor symptoms, abdominal
very low ascitic-fluid protein levels and those with pain, and diarrhea. collectively known as dumping
advanced liver failure. syndrome.

Bibliography Bibliography
Sofa E, Sole C, Gines P. Management of uninfected and Infected ascites in Berg P. McCallum R. Dumping syndrome: a review of the current concepts
cirrhosis. Liver Int. 2016;36 Suppl 1:109-15. [PMID: 26725907) dol:l0.1111/ of pathophyslology, diagnosis, and treatment. Dig Dis ScL 2016;61:11-8.
liv.13015 [PMID: 26396002) doi:l0.1007/sl0620-015-3839-x

Item 28 Answer: A Item 29 Answer: B


Educational Objective: Diagnose dumping syndrome Educational Objective: Treat chronic idiopathic
after gastrojejunostomy. constipation unresponsive to first-line treatment.
Dumping syndrome is the most likely diagnosis in this Linaclotide is the most appropriate treatment for this
patient. Rapid gastric emptying of hyperosmolar chyme patient. Linaclotide is a peripherally acting guanylate

118
Answers and Critiques

cyclase-C receptor agonist that is FDA approved for the Bibliography


treatment of chronic idiopathic constipation in adults. Lembo AJ, Schneier HA, ShiffSJ, Kurtz CB, MacDougall JE. Jla XD, et al. Two
Linaclotide increases intracellular and extracellular cyclic randomized trials of linaclotide for chronic constipation. N Engl J Med.
2011;365:527-36. [PMID: 21830967] doi:l0.1056 /NEJMoa10!0863
guanosine monophosphate, which results in chloride and
bicarbonate secretion into intestinal lumen, increasing
intestinal fluid content and accelerated transit time. Its Item 30 Answer: D
superiority to placebo in the treatment of constipation
Educational Objective: Prevent vertical transmission of
was demonstrated in two 12-week, high-quality random-
hepatitis B viral infection.
ized controlled trials. Diarrhea occurred in 16% of patients
receiving linaclotide (compared to 4.7% in those receiving Tenofovir is the most appropriate next step in management
placebo) in the 12-week clinical trials. The potential for of this patient, with the goal of preventing vertical trans-
diarrhea can be minimized by tal<ing linaclotide on an mission of hepatitis B virus (HBV) infection from mother to
empty stomach, ideally 30 minutes before the first meal child during the course of delivery. Guidelines recommend
of the day. Plecanatide is the second guanylate cyclase-C treatment with lamivudine, telbivudine, or tenofovir for the
receptor agonist to receive FDA approval for the treatment prevention of vertical transmission in pregnant women who
of chronic idiopathic constipation. Diarrhea was also the have HBV DNA levels greater than 200,000 IU/mL at 24 to
most commonly repo1ied side effect of plecanatide ther- 28 weeks' gestation. There are no head-to-head data compar-
apy In the 12-week clinical trials (S'Yo versus l'X. in the ing these regimens, but tenofovir is preferred over telbivudine
placebo group). Lubiprostone, a chloride channel agonist, and lamivudine due to lower rates ofresistance. Tenofovir and
is a third agent with FDA approval for the treatment of telbivudine are the only FDA pregnancy category B agents.
chronic idiopathic constipation. The most common side Lamivudine and other oral drugs used to treat HBV are cate-
effects reported in clinical trials were nausea, reported by gory C agents, though there are reasonable data on the safety
29% of patients taking lubiprostone, and diarrhea, reported of larnivudine use in pregnancy in the HIV population. Only a
by 12%. The efficacy and safety of linaclotide, plecanatide, few patients will become hepatitis B surface antigen- negative
and lubiprostone have not been established in patients aged with treatment; therefore, cure ofHBV infection is an unreal-
younger than 18 years nor in pregnant patients. Because of istic goal for most chronically infected patients.
cost effectiveness, over-the-counter laxatives, such as fiber There are no data suggesting that cesarean delivery
supplements, polyethylene glycol, and bisacodyl, should be is effective at preventing vertical transmission of HBV. All
pursued first in patients with constipation, but they were babies born to mothers with chronic HBV infection should
ineffective in this patient. receive active HBV vaccination and passive immunization
Osmotic laxatives include magnesium hydroxide, (HBV immune globulin). The risk for developing chronic
lactulose, sorbitol, and polyethylene glycol (PEG); clin- HBV infection is high (90%) in newborns who acquire HBV.
ical trials have demonstrated the superiority and safety Because the patient's HBV DNA level is high enough
of PEG. The patient tried over-the-counter PEG, and (>200,000 IU/mL) to warrant treatment to prevent vertical
it was poorly tolerated due to bloating. Substituting transmission of HBV. measuring her HBV DNA level again in
another, less effective, osmotic agent (for example, lact- 3 months would not be appropriate without first instituting
ulose) will lead to similar adverse effects without reliev- treatment.
ing constipation. Pegylated interferon is not considered safe in pregnancy
Methylnaltrexone, an injectable peripheral opioid and, therefore, would be an inappropriate choice for this
antagonist that does not cross the blood-brain barrier, is patient. Pegylated interferon can be used to treat HBV infec-
very effective in treating opioid-induced constipation with- tion in patients with high alanine aminotransferase levels,
out adversely affecting analgesia. Bowel obstruction is an low HBV DNA levels, and without cirrhosis. Candidates for
absolute contraindication to methylnaltrexone. Because this interferon have a desire for finite therapy, do not have cirrho-
patient does not have opioid-induced constipation, methyln- sis, are not pregnant, and do not have significant psychiatric
altrexone is an inappropriate choice. disease, cardiac disease, seizure disorder, cytopenia, or auto-
Rifaximin is a nonabsorbable antibiotic used in the immune disease.
treatment of irritable bowel syndrome predominated by
KEY POINT
diarrhea, hepatic encephalopathy, and travelers' diarrhea.
It has no proven efficacy in chronic idiopathic constipation • Pregnant women who have hepatitis B virus DNA
and is therefore not indicated in this patient. levels greater than 200,000 IU/mL at 24 to 28 weeks'
gestation should be treated with tenofovir to prevent
KEY POINT
vertical transmission during delivery.
• Linaclotide is a peripherally acting guanylate cyclase-C
receptor agonist that is FDA approved for the treat- Bibliography
ment of chronic idiopathic constipation in adults with Terrault NA, Bwwej NH, Chang KM, Hwang JP. Jonas MM, Murad MH;
symptoms refractory to first-line therapies. American Association for the Study of Liver Diseases. AASLD guidelines
for treatment of chronic hepatitis B. Hepatology. 2016;63:261-83. [PMID:
26566064] doi:l0.1002/hep.28156

119
Answers and Critiques

Item 31 Answer: A INR, patient comorbid conditions, and duration of therapy.


Educational Objective: Diagnose pancreatic cancer. For patients with an INR in or slightly above the therapeutic
range (up to 2.7), normalization of the INR does not reduce
Contrast-enhanced CT of the abdomen is the most appro- rebleeding risk but delays endoscopy and decreases the
priate next step in this patient's management. Pancreatic sensitivity of endoscopy to identify important prognostic
neoplasm must be considered as a cause of acute pancreatltis indicators related to risk for rebleeding. There are few data to
in patients older than age 40 years when no other cause has guide the clinician's decision of when to resume antithrom-
been identified and/or when worrisome features such as botic therapy after gastrointestinal bleeding. In general, once
weight loss or new onset of diabetes mellitus are present. endoscopic hemostasis has been achieved, anticoagulation
This patient had no identifiable cause and has worrisome should be reinitiated, and in most cases, this can be done
features of decreased appetite, back pain, and weight loss. on the same day as the procedure. After temporary discon-
Contrast-enhanced multidetector CT has a 90% sensitivity tinuation of warfarin, antithrombotic therapy should be
for detecting malignancy in patients in whom pancreatic reinitiated within 7 days of initial drug discontinuation to
cancer is suspected. CT also provides staging information. avoid an increased risk for a thromboembolic event. Hemo-
Endoscopic retrograde cholangiopancreatography (ERCP) stasis has been achieved in this patient, with no evidence of
is not appropriate for this patient because laboratory data, ongoing gastrointestinal bleeding seen for 24 hours after the
abdominal ultrasound, and physical examination show no evi- procedure; therefore, it is appropriate to reinitiate warfarin
dence ofbiliary obstruction. ERCP may be indicated in patients therapy now.
with biliary obstruction for stenting of the bile duct. This may Retrospective studies have shown that resumption of
occur in the setting of a neoplasm in the head of the pancreas. warfarin therapy is associated with a decreased risk for
Endoscopic ultrasound (EUS) examination resembles mortality and thrombosis without a significantly increased
standard endoscopy, but the endoscope has an ultrasound risk for recurrent hemorrhage in patients with gastroin-
transducer at the tip that emits acoustic waves to the sur- testinal bleeding. Waiting 10 or 30 days to resume warfarin
rounding tissues, allowing for the acquisition of real-time therapy increases the risk for a thromboembolic event with
views of the pancreas, bile ducts, and blood vessels. In the no reduction in risk for recurrent gastrointestinal bleeding.
diagnosis of pancreatic lesions, EUS is useful for the detec- This patient's CHA2DS 2 -VASc (Congestive heart failure,
tion of small tumors that may be below the resolution of the Hypertension, Age <!:.75 years, Diabetes mellitus, Stroke or
CT scan and allows tissue diagnosis by fine-needle aspiration transient ischemic attack or thromboembolism, Vascular
when required. In patients in whom pancreatic cancer is disease, Age 65-74 years, Sex category) score is 5, predicting
suspected, contrast-enhanced multidetector CT is the pre- a nearly 7% annual risk for stroke and thromboembolism
ferred initial diagnostic study. event at 1-year follow-up evaluation. This is a moderately
No further testing is indicated for patients younger than high annual risk for thromboembolism; therefore, holding
age 40 years who are asymptomatic after treatment for a first warfarin therapy indefinitely is inappropriate.
case of mild acute pancreatitis in the absence of worrisome
KEY POINT
features. This patient's age, lack of an identifiable cause,
and worrisome clinical features warrant further evaluation • Once endoscopic hemostasis has been achieved in a
rather than continuing current management. patient with gastrointestinal bleeding, anticoagulation
should be reinitiated, and in most cases, this can be
KEY POINT
done on the same day as the procedure.
• Pancreatic neoplasm must be considered as a cause of
acute pancreatitis in patients older than age 40 years Bibliography
when no other cause has been identified and/or when Abmham NS. Management of ant!platelet agents and anticoagulants in
worrisome features, such as weight loss or new onset patients with gastrointestinal bleeding. Gastrolntest Endosc Clin N Am
2015;25:449-62. [PMID: 26142031] dol:l0.1016/j.giec.2015.02.002
of diabetes mellitus, are present.

Bibliography
Item 33 Answer: D
Tenner S, Baillie J, DeWitt J, Vege SS; American College ofGastroenterology.
American College of Gastroenterology guideline: management of acute Educational Objective: Diagnose celiac disease.
pancreat!tis. Am J Gastroenterol. 2013;108:1400-15: 1416. [PMID:
23896955] doi:J0.1038/ajg.2013.218 Testing for anti-tissue transglutaminase (tTG) IgA antibody
is the most appropriate next test for this patient, who has
symptoms of celiac disease, including diarrhea, bloating,
CJ Item 32 Answer: A and weight loss. She is underweight due to malabsorption
related to immune destruction of small intestinal villi, and
Educational Objective: Manage antithrombotic therapy
she has laboratory abnormalities associated with cellac dis-
after gastrointestinal bleeding.
ease, including iron deficiency anemia and elevated liver
This patient's warfarin therapy should be restarted now. The transaminases. Her family history of autoimmune diseases
risk for warfarin-associated bleeding is determined by the is also consistent with celiac disease due to common genetic

120
Answers and Critiques

predisposition (HLA-DQ2 and DQB). The best initial screen- colon cancer should have a subsequent surveillance colo-
ing test for celiac disease is anti- tTg lgA antibody testing. noscopy within 1 year. If the colonoscopy is normal, the
Given increased prevalence of lgA deficiency in celiac dis- AGA recommends repeat examination in 3 years; ASCO
ease, total IgA levels should be measured in patients with recommends repeat examination in 5 years. If normal, colo-
high suspicion for celiac disease. If patients are lgA deficient, noscopy should be repeated every 5 years thereafter until
IgG-based antibody testing (such as deamidated gliadin the benefit of continued surveillance is outweighed by risks
peptide [DGP] IgG) should be performed. Positive antibody and diminished life expectancy. If neoplasms are detected
screening for celiac disease should prompt upper endoscopy during any follow-up examination, then the surveillance
and biopsies of the duodenum for definitive diagnosis. Treat- interval should be adjusted based on polyp size, number,
ment is a lifelong, strict gluten-free diet. and Wstology.
Gliadin antibodies are neither sensitive nor specific for Colonoscopy is the preferred modality for surveillance
the diagnosis of celiac disease and are no longer recommended in patients with a personal history of colon cancer. There
for the evaluation of celiac disease. Newer gliadin-based serol- is insufficient evidence to support CT colonography, fecal
ogy tests, anti- DGP IgA and anti- DGP lgG, have greater immunochernical testing, or fecal DNA testing for surveil-
sensitivity and specificity than older antigliadin antibodies. lance after colon cancer.
Anti- DGP IgG is particularly useful in cases of total lgA defi- Surveillance colonoscopy every 1 to 2 years is recom-
ciency for diagnosing celiac disease. mended in individuals with Lynch syndrome, regardless
Anti- Saccharomyces cerevisiae antibodies have been of whether the patient has a personal history of colon
proposed as a serologic method for differentiating Crohn cancer. Lynch syndrome is unlikely because the patient's
disease from ulcerative colitis, but they are neither ade- family history does not meet the Amsterdam II criteria,
quately sensitive nor specific and can lead to false-positive which require that three family members are affected by
results if used as a screening test for gastrointestinal symp- a Lynch syndrome- associated cancer, at least two succes-
toms. In addition, the absence of fever, bloody diarrhea, or sive generations are affected, one of the affected family
abdominal pain makes the diagnosis of inflammatory bowel members is a first-degree relative of the other two affected
disease unlikely in this patient. family members, and at least one cancer was diagnosed in
Anti-smooth muscle antibody testing can be helpful in a family member younger than age 50 years, with famil-
the diagnosis of autoimmune hepatitis. Autoimmune hepati- ial adenomatous polyposis excluded and tumors verified
tis is a chronic inflammatory liver disease that is usually seen histologically.
in women and can be associated with other autoimmune A 5-year or 10-year interval is not appropriate for this
diseases (most commonly autoimmune thyroiditis, syno- patient because the risk for metachronous cancer (multiple
vitis, or ulcerative colitis). The disease presentation ranges primary tumors developing at different time intervals) is
from asymptomatic to acute liver failure. Serum amino- increased for about 2 to 3 years after colon cancer resection.
transferase levels are elevated and range from mild eleva- According the American Cancer Society's Colorectal
tions to greater than 1000 U/L Serum lgG levels are also Cancer Survivorship Care Guidelines published in 2015,
elevated. Patients with autoimmune hepatitis do not present a history and physical examination should be performed
with loose stools, bloating, and weight loss. every 3 to 6 months for the first 2 years, then every 6 months
for 5 years. After treatment for stages l , 2, or 3 colorectal
KEY PO I NT
cancer, patients at high risk for recurrence (for example,
• Anti- tissue transglutaminase IgA antibody testing is with poorly differentiated histology, lymphatic or vascular
the best screening test for celiac disease. invasions, or positive resection margins) should receive
annual abdominal-pelvic and chest CT scans for 5 years
Bibliography after resection. A history and physical examination should
Rubio-Tapia A, Hill ID, Kelly CP, Calderwood AH, Murray JA; American also be performed every 3 to 6 months for 5 years. Carci-
College ofGastroenterology. ACG clinical guidelines: diagnosis and man-
agement of cellac disease. Am J Gastroenterol. 2013;108:656-76; quiz 677. noembryonic antigen measurement is recommended every
[PMrD: 23609613] dol:l0.1038/ajg.2013.79 3 to 6 months for the first 2 years, then every 6 months to
5 years if the patient is a potential candidate for further
intervention. Follow-up evaluation recommendations from
other expert organizations vary.
Item 34 Answer: A
Educational Objective: Monitor for colorectal neoplasia KEY POINT
after colon cancer resection. • After treatment of colon cancer, patients should
This patient's next surveillance colonoscopy should take undergo surveillance colonoscopy 1 year after diagnosis.
place now (1 year after diagnosis) according to the American
Gastroenterological Association (AGA) and American Soci- Bibliography
ety of Clinical Oncology (ASCO) guidelines. Patients who American Gaslmenterology Association. AGA institute guidelines for colo-
noscopy surveillance after cancer resection: clinical decision tool.
undergo a complete perioperative colonoscopy with clearing Gastroenterology. 2014;146:1413-4. [PMID: 24742563] doi:l0.1053/
of synchronous neoplasia and curative surgical resection for J.gastro.2014.03.029

121
Answers and Critiques

Item 35 Answer: D upper gastrointestinal bleeding (UGIB) , likely from an


Educational Objective: Treat main-duct intraductal NSAJD-induced peptic ulcer. Fifteen percent of patients with
papillary mucinous neoplasm of the pancreas. presumed lower gastrointestinal bleeding are found to have
an upper gastrointestinal source. Factors in this patient that
Pancreatic resection is the most appropriate next step in favor a diagnosis of UGIB include young age, hemodynamic
management. Pancreatic cysts are classified as pancreatic instability on presentation, and NSAID use. Approximately
cystic neoplasms (the most common). nonneoplastic pan- 80% of UGIB is due to four causes: peptic ulcer disease,
creatic cysts, and pseudocysts. The two most common pan- esophagogastric varices, esophagitis, and Mallory-Weiss
creatic cystic neoplasms are mucinous cystic neoplasms tear. Bleeding typically stops spontaneously; however, 20%
(MCNs) and intraductal papillary mucinous neoplasms of patients have persistent or recurrent bleeding, which
(IPMNs), which involve the main duct, branch ducts, or increases mortality. Guidelines recommend upper endos-
both. Most pancreatic cysts are branch-duct IPMNs. Main- copy within 24 hours of presentation in patients w ith fea-
duct IPMNs can be diagnosed based on diffuse dilation of the tures ofUGIB.
main pancreatic duct and the characteristic feature of rnucin Angiography is used to diagnose the cause of obscure
exuding from the arnpulla during endoscopic visualization. gastrointestinal bleeding when more common sources are
Main-duct IPMNs carry a greater than 65% risk for malig- not found on routine upper and lower endoscopy. It is also
nant transformation. Surgical resection is the only option for used for treatment, such as embolization, when a bleeding
treatment of these high-risk cystic lesions of the pancreas in source has been identified.
patients who are appropriate surgical candidates. Capsule endoscopy employs a wireless capsule camera
Diagnosis of solitary cysts Is challenging. It may require that is swallowed by the patient to take images of the small
endoscopic ultrasonography and fine-needle aspiration to bowel. These images are transmitted to a radiofrequency
distinguish between cystic neoplasms (branch-duct JPMNs, receiver worn by the patient. Capsule endoscopy allows
MCNs, and serous tumors) and pseudocysts based on cytol- visualization of the entire small bowel. This patient's bleed-
ogy. This patient has endoscopic ultrasonographic findings ing is very likely to be accessible by upper endoscopy, the
showing characteristic features of main-duct IPMN. The best preferred diagnostic test.
course of action is pancreatic resection and not endoscopic Colonoscopy requires bowel preparation for adequate
ultrasound surveillance in 1 year. visualization of bleeding sources, and in the setting of acute
MRI of the abdomen In I year ls the standard surveil- bleeding with hemodynamlc instability, a more urgent diag-
lance recommendation for asymptomatic, low-risk cystic nostic and therapeutic test is indicated. Colonoscopy would
lesions of the pancreas. Main-duct IPMN is considered a be indicated if the source of bleeding were not identified on
high-risk cystic lesion and, therefore, MRI surveillance is upper endoscopy.
not appropriate.
KEY POINT
Oral prednisone is used to treat autoimmune pancre-
atitis, a frequent manifestation of IgG4-related disease. The • Hematochezia associated with hemodynamic instabil-
characteristic finding of autoimmune pancreatitis on imag- ity in a young patient is likely due to an upper gastro-
ing is a narrowed main pancreatic duct with parenchymal intestinal source.
swelling, known as the "sausage-shaped" pancreas, which Is
not consistent with this patient's findings. Bibliography
Str:ice LL, Gralnek JM. ACG clinical guideline: management of patients with
KEY POINT acute lower gastrointestinal bleeding. Am J Gastroenterol. 2016;lll:459-
74. [PMID: 26925883] dol:10.1038/ajg.2016Al
• Surgical resection is the best management option for
high-risk cystic lesions of the pancreas, such as intra-
ductal papillary mucinous neoplasms that involve the
main duct.
Item 37 Answer: D
Educational Objective: Diagnose secretory diarrhea.
Bibliography Secretory diarrhea is the diagnosis most compatible with the
Vege SS, Ziring B, Jain R, Moayyedl P; Clinical Guidelines Committee.
American Gastroenterological Association Institute guideline on the
patient's presentation. In the evaluation of chronic diarrhea,
diagnosis and management of asymptomatic neoplastic pancreatic cysts. it is often useful to categorize the condition as secretory or
Gastroenterology. 2015;148:819-22; quize12-3. [PMID: 25805375] osmotic, as well as to determine whether the cause is due
doi:l0.1053/j.gnstro.2015.0l.015
to chronic infection or an inflammatory condition, which
will guide further evaluation and treatment. Secretory and
c::J Item 36 Answer: D osmotic diarrhea can often be distinguished by clinical his-
tory. Patients with secretory diarrhea may pass liters of
Educational Objective: Diagnose the cause
stool daily, causing severe dehydration and electrolyte dis-
of hematochezia.
turbances, with persistent stooling despite fasting. Patients
Upper endoscopy is the most appropriate test to perform with osmotic diarrhea often have stool volumes of less than
next. The patient's presentation is consistent with rapid 1 Lid and have cessation of stooling when they are fasting. This

122
Answers and Critiques

patient's large-volume, watery diarrhea that persists despite Item 38 Answer: A


fasting is compatible With secretory diarrhea. Another clue Educational Objective: Diagnose Crohn disease.
to this patient's diagnosis is hypokalemia. Hypokalemia
from lower-gastrointestinal losses is most common when The patient has Crohn colitis presenting as a chronic
the losses occur over a prolonged period. Fecal electrolytes inflammatory diarrhea. Common symptoms are abdomi-
can be used to calculate the fecal osmotic gap: nal pain, diarrhea, and weight loss; overt gastrointestinal
bleeding is a less common manifestation. Crohn disease
290 - (2 x [stool sodium+ stool potassium]) has varied presentations that can make diagnosis diffi-
An osmotic gap of less than 50 mOsm/kg suggests cult. The pattern of inflammation and histologic features
secretory diarrhea, and a gap greater than 75 mOsm/kg sug- in this case allow distinction of Crohn disease from other
gests osmotic diarrhea. inflammatory conditions. Colonoscopy findings are con-
Examples of conditions that can cause a chronic secre- sistent with Crohn colitis based on patchy distribution
tory diarrhea include medications (colchicine, NSA!Ds), of mucosa! inflammatory changes with normal inter-
hormone-producing tumors (carcinoid, gastrinoma, vening mucosa, called "skip areas." The biopsy results
VIPoma), small intestinal bacterial overgrowth, bile acid show distorted and branching colonic crypts, which
malabsorption (short-bowel syndrome), and villous ade- are indicative of chronic colitis. The skip areas of nor-
noma. Additional testing may be warranted in patients with mal intervening mucosa and biopsy results for involved
secretory diarrhea. Patients found to have a small-bowel mucosa showing features of chronicity (distorted and
tumor in the setting of diarrhea as well as symptoms such branching colonic crypts) make Crohn disease the most
as flushing should be considered for additional testing, likely diagnosis.
including radioimmunoassays for peptides and/or 24-hour The patient is at risk for Giardia lamblia infection
urine 5-hydroxyindoleacetic acid measurement for carci- because she works in a child care center. However, Glardia
noid tumors. Colonoscopy can be used to evaluate for villous is a small-bowel protozoa! pathogen and does not cause
adenomas and is also indicated if the patient is not up-to- the symptoms of colonic inflammation with bloody diar-
date for colon cancer screening. rhea. Although most patients with giardiasis are asymp-
Infections leading to chronic diarrhea can be caused by tomatic, those with symptoms note abdominal pain and
parasites such as Giardia. Giardia leads to a malabsorptive cramping with diarrhea, often consisting of large-volume,
condition; it would not present with large-volume diarrhea watery, and foul-smelling stools. Symptoms may be acute
that persists despite fasting. or chronic.
Inflammatory causes of chronic diarrhea include Microscopic colitis does not cause bloody diarrhea
inflammatory bowel diseases (ulcerative colitis and Crohn but rather is a cause of chronic watery diarrhea pre-
disease) and microscopic colitis. Patients with inflammatory dominantly in women, with a peak onset between ages
causes of chronic diarrhea present with bloody stools, noc- 60 and 70 years. Also, colonoscopy results are normal in
turnal symptoms, and sometimes anemia. This patient is less patients with this condition, with inflammation only seen
likely to have an inflammatory cause for her chronic diar- on biopsy.
rhea given the absence of these signs and symptoms. Blood Ulcerative colitis typically presents with bloody diar-
or leukocytes in the stool suggest an inflammatory cause. rhea and abdominal discomfort, the severity of which is
Causes of chronic osmotic diarrhea include med- related to the extent and severity of inflammation. Because
ications (laxatives), undigested sugars (lactose), celiac ulcerative colitis typically involves the rectum, tenesmus,
disease, and fat maldigestion or absorption leading to urgency, rectal pain, and fecal incontinence are com-
steatorrhea (chronic pancreatitis). When laxative abuse is mon. Patients with ulcerative colitis have distorted and
suspected, a stool or urine laxative screen can aid in the branching colonic crypts on biopsy, but the distribution
diagnosis. Celiac disease can be evaluated with serology of inflammation begins in the rectum and progresses up
testing and small intestinal biopsy. A positive 72-hour the colon in a continuous and symmetric pattern, without
stool collection for fecal fat confirms steatorrhea; a ran- skip areas.
dom fecal fat assessment may be helpful if a timed collec- KEY POINT
tion is not possible.
• Colonoscopy results in Crohn disease show patchy
KEY POINT distribution of mucosa! inflammatory changes
• Patients with secretory diarrhea may pass liters of with "skip areas" of normal intervening mucosa,
stool daily, causing severe dehydration and electrolyte and biopsy results for involved mucosa show
disturbances, with persistent stooling despite fasting. features of chronicity (distor ted and branching
colonic crypts) .
Bibliography
Schiller LR, ~ardi DS, Spiller R, Semrad CE, Surawicz CM. Giannella RA,
Bibliography
et al. Gastro 2013 APDW/WCOG Shanghai working party report: chronic Lee JM, Lee KM. Endoscopic diagnosis and differentiation of inflammatory
diarrhea: definition, classUicatlon, diagnosis. J Gastroenterol Hepatol. bowel disease. Clln Endosc. 2016;49:370-S. fPMID: 27484813]
2014;29:6-25. [PM!D: 24117999] dol:IO.llll/jgh.12392 doi:l0.5946/ce.2016.090

123
Answers and Critiques

Item 39 Answer: B autoimmune thyroid disease. Autoimmune gastritis is asso-


Educational Objective: Manage follow-up colonoscopy ciated with parietal cell loss, reduced gastric acid produc-
after uncomplicated diverticulitis. tion, and secondary hypergastrinemia. Hypergastrinemia
is associated with an increased risk for the development
This patient should undergo colonoscopy in 1 to 2 months of gastric carcinoid and adenocarcinoma. Jt is prudent to
following treatment of uncomplicated diverticulitis. perform upper endoscopy and gastric biopsy at the time of
According to the American Gastroenterological Association pernicious anemia diagnosis to evaluate for these cancers;
Institute Technical Review on the Management of Acute however, the benefit on ongoing surveillance endoscopy is
Diverticulitis, patients with a recent episode of acute diver- unclear. Autoimmune-related pernicious anemia and asso-
ticulitis should undergo colonoscopy 4 to 8 weeks after an ciated iron deficiency anemia are likely to require lifelong
episode of acute diverticulitis. Follow-up colonoscopy may vitamin Bn and iron replacement, respectively.
identify a few cases of colorectal carcinoma. The risk for per- Capsule endoscopy is typically used to evaluate obscure
foration after a case of diverticulitis is uncertain. The tech- causes of gastrointestinal bleeding. This patient had a normal
nical review estimated that 1 in 67 patients with confirmed colonoscopy 4 years earlier, and stool evaluation is negative
acute diverticulitis would have a misdiagnosed colorectal for blood. Finally, this patient's iron deficiency anemia is
cancer found on follow-up colonoscopy. Almost all of the most likely due to atrophic gastritis- related achlorhydria,
misdiagnosed colorectal cancers included in the technical which decreases iron absorption by impairing the con-
review were located in the area of diverticulitis. Assessing version of ferric iron to absorbable ferrous iron; in many
the patient for underlying inflammatory bowel disease is patients, iron deficiency precedes vitamin B12 deficiency.
another reason for performing follow-up colonoscopy. Additional testing for iron deficiency beyond upper endos-
Colonoscopy during the acute phase of illness is contra- copy is likely unnecessary.
indicated because acute diverticulitis causes acute colonic Although small intestinal bacterial overgrowth (SIBO)
inflammation, which may increase the risk for perforation. can be associated with anemia, the diagnosis is unlikely
The risk for perforation has been reported to be as high as in this patient with no risk factors or typical symptoms of
0 .3'Yo, but it is generally thought to be less than 0.1%, based SIBO; therefore, a glucose breath test to assess for SIBO is
on data from large populations undergoing screening colo- likely to be oflow yield.
noscopy. If the results of colonoscopy performed within the The serum gastrin level is elevated in patients with any
past 12 to 24 months are negative, then repeat colonoscopy form of atrophic gastritis and has no diagnostic or prognostic
is not needed. value in this setting.
Delaying colonoscopy beyond 1 to 2 months places the
KEY POINT
patient at risk for a missed diagnosis that could be important
and time sensitive, such as a diagnosis of colon cancer. • Patients w ith newly diagnosed pernicious anemia
should be evaluated for gastric adenocarcinoma and
KEY POINT
gastric carcinoid with upper endoscopy and gastric
• Patients with uncomplicated diverticulitis should undergo biopsy.
colonoscopy 1 to 2 months after the episode of acute
diverticulitis, when colonic inflammation has resolved. Bibliography
Minalyan A, Benhammou JN, Artashe~y.rn A, Lewls MS, Pisegna JR.
Bibliography Autoimmune atrophic gastritis: current perspectives. Clin Exp
Gastroenterol. 2017;10:19 -27. [PMID: 28223833] doi:l0.2M7/CEG.S109123
Strate LL, Peery AF, Neumann I. American Gastroenterolog!cal Association
Institute technical review on the management of acute di~rticulitis.
Gastroenterology. 201S;l~9:1950-1976.e12. [PMID: 26453776] dol:
10.1053/j.gastro.2015.IO.OOJ
Item 41 Answer: D
Educational Objective: Treat acute anal fissure.

ltem40 Answer: D Sitz baths and psylllum are the most appropriate next step
in the management of this patient, who has an acute anal
Educational Objective: Diagnose atrophic gastritis.
fissure that is most likely the result of worsening constipa-
Upper endoscopy with gastric biopsy is the most appropri- tion induced by the recent use of opioid analgesic therapy.
ate next diagnostic study. The two forms of atrophic gas- Anal fissures are tears in the anoderm below the dentate
tritis are Helicobacter pylori- associated and autoimmune. line that can be seen on inspection of the perianal area, often
H. pylori- associated atrophic gastritis typically resolves with unaided by the use of an anoscope. They are usually in the
H. pylori eradication, whereas autoimmune atrophic gas- posterior position and are less often in the anterior midline.
tritis has no cure. This patient's hematologic findings and The classic symptom is pain with and after defecation, which
positive testing for serum antiparietal antibodies confirm may be associated with bright red blood on the toilet tissue.
the diagnosis of pernicious anemia and suggest the presence Most acute fissures heal spontaneously. The most effective
of autoimmune atrophic gastritis. Iron deficiency anemia treatment approach is daily warm-water sitz baths and the
is a common comorbldity of autoimmune gastritis, as is use of the bulk laxative psyllium. This combination therapy

124
Answers and Critiques

was superior to topical lidocaine or hydrocortisone in a ran- pattern with multiple generations affected, but de novo
domized clinical trial. mutations account for up to 25% of cases. JfFAP is confirmed
Anal botullnum toxin injection is reserved for chronic in this patient, it Is likely due to a de novo mutation because
anal fissures that do not respond to more conservative treat- of his unremarkable family history.
ment measures, including sitz baths, fiber supplementa- There is no role for surgical management of fundic
tion, and topical therapy (nitroglycerin or calcium channel gland polyps because they rarely progress to gastric cancer,
blocker) applied to the anal canal. This initial acute presen- so gastrectomy is not indicated for this patient.
tation of anal fissure is best managed by more conservative, An upper endoscopy in 3 months is not indicated for
less invasive treatment measures first. diagnosis ofFAP. JfFAP is confirmed, the patient should have
A flexible sigmoidoscopy is not indicated because the regular surveillance of the upper gastrointestinal tract with
diagnosis of anal fissure can be accurately made by history a forward and side-viewing scope to assess for duodenal
and examination. Furthermore, the patient's family history polyps. Duodenal polyps occur commonly in FAP, and risk
is not significant enough to warrant endoscopic evaluation. for duodenal cancer is elevated. The interval for surveillance
Although topical anti-inflammatory therapy can allevi- upper endoscopy in FAP depends on the number, size, and
ate the paJn associated with anal fissure, the placement of a pathology of the duodenal polyps. This upper endoscopy also
suppository may be uncomfortable. In addition, the patient includes surveillance for gastric adenomas and cancer.
has already tried over-the-counter topical hemorrhoid ther- For sporadic fundic gland polyps, no further evaluation
apy with no relief of symptoms, so a glucocorticoid suppos- would be needed. Sporadic fundic gland polyps can arise in
itory is unlikely to help. the setting of proton pump inhibitor (PPI) therapy; however,
Topical glucocorticoid therapy is less effective in the this usually occurs with prolonged therapy. Because this
treatment of anal fissure than sitz baths and fiber. Lubipros- patient has only been taking a PPI for 4 months, his fundic
tone is a peripherally acting chloride channel activator with gland polyposis is unlikely to be caused by PP! therapy. The
FDA approval for the treatment of opioid-induced constipa- patient's young age and the number of gastric polyps should
tion. This patient's opioid analgesic use is temporary after prompt evaluation by colonoscopy.
shoulder surgery, and lubiprostone use is more appropriate
KEY POINT
in the setting of more chronic opioid use.
• Patients with multiple fundic gland polyps found at a
KEY POINT young age should be evaluated for familial adenoma-
• The most effective treatment approach for anal fissure tous polyposis.
is daily warm-water sitz baths and the use of the bulk
laxative psyllium. Bibliography
Syngal S, Drand RE, Church JM, Giardiello FM, Hampel HI, Burt RW:
American College of Gastroenterology. ACG clinical guideltne: genettc
Bibliography testing and management of hereditary g;istrolntestlnal cancer syn-
Beaty JS, Shashidharan M. Amil fissure. Clin Colon Rel1al Surg. 2016;29:30-7. dromes. AmJ Gastroenterol. 2015;110:223-62; quiz 263. [PMID: 256455741
[PMID: 26929749] doi:10.IOSS/s- 003S- l570390 doi:l0.1038/ajg.2014.435

ltem42 Answer: A Item 43 Answer: D


Educational Objective: Diagnose familial adenomatous Educational Objective: Diagnose primary sclerosing
polyposis. cholangitis.
Colonoscopy is the best next step in the evaluation of this Primary sclerosing cholangitis (PSC) is the most likely diag-
patient. The patient has multiple fundic gland polyps, nosis in this patient. The patient has elevated liver chemis-
whlch can be associated with familial adenomatous polyp- try test results in a predominantly cholestatic pattern and
osis (FAP) . Fundic gland polyps related to FAP are usually concomitant inflammatory bowel disease (JBD) . Disorders
numerous (>30 polyps) and frequently harbor low-grade are considered cholestatic when the most prominent abnor-
dysplasia, but they rarely progress to gastric cancer. Colo- mality of liver chemistry testing is an elevation of the serum
noscopy to rule out FAP is recommended in patients younger alkaline phosphatase level. Approximately 5% of patients
than age 40 years with dysplastic or nwnerous fundic gland with IBD will develop PSC during the course of their disease.
polyps. Classic FAP results in the development of hundreds Jn most patients, PSC presents as a stricturing process in the
to thousands of colorectal adenomas that often manifest medium to large bile ducts, readily identifiable by MR chol-
by the second decade of life. Without treatment, colorectal angiopancreatography (alternating strictures and dilations
cancer typically develops in all patients by age 40 years. resulting in a M string of beads" pattern). However, a minority
Therefore, a colonoscopy is indicated in this patient to eval- of patients may present with involvement of only the small
uate for colonic polyposis. If colonic polyposis identified, the bile ducts, ca.lied small-duct PSC. The cholangiogram is nor-
patient should undergo genetic testing to make the definitive mal in these patients; the diagnosis of small-duct PSC can
diagnosis of FAP. FAP is caused by mutations in the APC only be made by liver biopsy. Small-duct PSC is associated
gene. Mutations can be inherited in an autosomal dominant with a better prognosis than typical PSC, although 10% to

125
Answers and Critiques

20% of patients with small-duct PSC eventually develop frequent symptoms could include famotidine or other H2
changes in the medium and large ducts. blockers. Symptom relief with the use of a PPI confirms the
The diagnosis of primary biliary cholangitis (PBC) is diagnosis of GERD.
generally made on the basis of a cholestatic liver enzyme Ambulatory pH testing is used to determine acid expo-
profile in the setting of a positive antimitochondrial antibody sure in the esophagus in patients considering antireflux
test. Case reports have described PBC associated with IBD, surgery. Indications for surgery include patient preference
but the association between the two conditions is not well to stop taking medication, medication side effects, and
accepted, making the diagnosis of PBC unlikely. Although refractory symptoms despite optimized medical therapy.
10% of patients with PBC may test negative for antimi- Laparoscopic fundoplication and bariatric surgery (in obese
tochondrial antibody; the absence of antibody makes this patients) are surgical methods used to treat GERD. This
diagnosis less likely. A liver biopsy Is required to establish patient has no surgical indications.
the diagnosis of antimitochondrial antibody-negative PBC. A barium esophagram is the initial test for evaluation
Drug-induced liver injury can occur in patients tak- of achalasia. The main symptom of achalasia is dyspha-
ing medications used to treat IBD, such as azathioprine gia to both solids and liquids along with regurgitation of
and tumor necrosis factor-a inhibitors. Drug-induced liver undigested food and saliva. Patients may also report unin-
injury is not a common adverse effect ofmesalamine; there- tentional weight loss, chest pain, and heartburn. A barium
fore, this diagnosis is unlikely in this patient. esophagram is not indicated because this patient's symp-
Nonalcoholic fatty liver disease is the most common toms are more consistent with GERD than with achalasia.
liver disease in the United States. It is unlikely in this patient Upper endoscopy is indicated in patients with alarm
because it usually does not present with alkaline phos- symptoms, such as dysphagia or weight loss, and in patients
phatase levels greater than 2 to 2.5 times the upper limit whose symptoms do not respond to a PPL Upper endoscopy
of normal. Furthermore, MRI has a reasonable sensitivity is not indicated in this patient with typical GERO symptoms
(>80'Yo) for the detection of fatty liver, which was not seen because most patients with typical symptoms have normal
in this patient. upper-endoscopy findings. Upper endoscopy is useful for
the evaluation of the esophagus to identify damage in the
KEY POINT
form of erosive esophagitis, stricture, or Barrett esophagus.
• Approximately 5% of patients with inflammatory
KEY POINT
bowel disease will develop primary sclerosing cholan-
gitis during the course of their disease, typically pre- • Patients with a clinical diagnosis of gastroesophageal
senting as cholestatic liver injury with a characteristic reflux disease should start an empiric trial of a proton
imaging study showing bile duct strictures and dila- pump inhibitor in conjunction with lifestyle and die-
tions ("string of beads"). tary changes, \Ni th no further testing.

Bibliography Bibliography
Lindor KD. Kowdley KV, Harrison ME; American CollegeofGastroenterology. Katz PO, Gerson LB, Vela MF. Guidelines for the diagnosis and management
ACG clinical guideline : primary scleroslng cholangitis. Am J of gastroesophageal reflux disease. Am J Gastroenterol. 2013
Gastroenterol. 2015;110:646-59; quiz 660. [PMID: 25869391] dol:J0.1038/ Mar:IOB(3):308-28; quiz 329. Erratum In: i\m J Gastroenterol. 2013
ujg.2015.112 Oct;l08(10):1672. [PMID: 23419381]

ltem44 Answer: C ltem45 Answer: A


Educational Objective: Treat gastroesopbageal reflux Educational Objective: Screen for osteoporosis in a
disease. patient with cirrhosis.
The most appropriate next step in the management of this Bone densitometry to screen for osteoporosis is the most
patient is the Initiation of a proton pump Inhibitor (PPI) In appropriate next intervention for this patient with cirrhosis.
conjunction with lifestyle and dietary changes. The diagnosis Patients with cirrhosis are at twice the risk for the develop-
of gastroesophageal retlux disease (GERD) is made clinically. ment of osteoporosis as demographically similar patients
This patient's symptoms of burning pain in his abdomen without cirrhosis, and women (especially those who are
and chest and a sour taste in his mouth occurring more than postmenopausal) are at higher risk than men. Initial studies
a few times a week are consistent with GERD. Due to the of metabolic bone disease in patients with cirrhosis focused
frequency of his symptoms, the best initial treatment is a on those with primary biliary cholangitis (PBC) or alcoholic
once-daily PPI for a pe1iod of up to 8 weeks. Weight reduc- liver disease. Subsequently, it has been recognized that all
tion is suggested for patients with recent weight gain or who patients with cirrhosis are at increased risk for bone frac-
are overweight. Interventions such as raising the head of tures due to osteoporosis. Standard evaluation should include
the bed and eliminating meals within 2 to 3 hours of bed- measurement of serum calcium, phosphate, and vitamin
time are helpful for nocturnal GERD. Cessation of alcohol D levels. Up to two thirds of patients with cirrhosis are
and tobacco use is universally supported. Treatment for less deficient in vitamin D. Dual-energy x-ray absorptiometry

126
Answers and Critiques

is recommended for patients with cirrhosis or PBC (even if old. Patients with IBD who develop VTE have an increased
noncirrhotic). mortality rate, and the most important risk factor for devel-
Patients with primary sclerosing cholangitis (PSC) have opment of VTE is active IBD. For these reasons. prevention
a high likelihood of ulcerative colitis with associated risk of VTE in patients with IBD is essential. All hospitalized
for colonic dysplasia and colon cancer. Eighty percent of patients with IBD should be given pharmacologic VTE pro-
patients with PSC have inflammatory bowel disease, usually phylaxis with subcutaneous heparin. Systematic reviews of
ulcerative colitis. This association with ulcerative colitis is trials comparing prophylactic low-molecular-weight hepa-
not seen in patients with PBC. This patient is at average risk rin with unfractionated heparin have not shown a statisti-
for colorectal cancer, and colonoscopy is not indicated until cally significant difference for mortality or major bleeding
age 50 years. events, although there was a nonsignificant trend favoring
Contrast-enhanced abdominal CT with arterial and low-molecular-weight heparin in the prevention of PE.
portal venous phase imaging can be helpful in evaluating for Mechanical VTE prophylaxis with graduated com-
hepatocellular cancer; however, in this patient whose ultra- pression stockings or intermittent pneumatic compression
sound showed no hepatic mass, a CT scan is not necessary. devices is not recommended either with or in place ofphar-
Patients with cirrhosis who develop an infection have a macologic prophylaxis, although intermittent pneumatic
30% mortality rate at 1 month, and another 30% die within compression has shown some efficacy in surgical patients
12 months; this underscores the importance of preventing and may be an option for nonsurgical patients with a contra-
infection. However, the herpes zoster vaccine is not Indi- indication to pharmacologic therapy. such as severe gastro-
cated in patients younger than age 60 years. It is recom- intestinal bleeding. Compression stocking are no more effec-
mended for individuals aged 60 years and older, regardless tive than placebo in preventing VTE and are associated with
of whether they have had a previous zoster episode, but harm in the form of increased incidence of skin breakdown.
vaccination should be avoided in immunocomprornised Aspirin and low-dose warfarin have no role in VTE
patients. Patients with decompensated cirrhosis are con- prophylaxis for hospitalized medical patients or patients
sidered immunocompromised due to impaired reticuloen- with IBD.
dothelial cell function, impaired neutrophil function, and
KEY POINT
possible concomitant malnutrition.
• All hospitalized patients with inflammatory bowel
KEY POINT
disease should be given pharmacologic venous throm-
• Patients with cirrhosis are at increased risk for the boembolism prophylaxis with subcutaneous heparin.
development of osteoporosis and should be screened
using bone densitometry. Bibliography
Seah 0, De Cruz P. Review article: the practical management of acute severe
ulcerative colitis. Aliment Pharmacol Ther. 2016;43:482-513. [PMID:
Bibliography 26725569] dol:IO.lllllapt.13491
Santos LA, Romeiro FC. Diagnosis and management of cirrhosis-related
osteoporosis. Biomed Res lnt. 2016;2016:1423462. [PMID: 27840821]

ltem47 Answer: C
CJ Item 46 Answer: C Educational Objedive: Diagnose Lynch syndrome.
Educational Objedive: Prevent venous
This patient should be referred to a genetic counselor for
thrornboembolism in a hospitalized patient with
genetic testing. The patient could have I,ynch syndrome based
inflammatory bowel disease.
on his personal history of colon cancer diagnosed before age
Subcutaneous heparin is the most appropriate venous 50 years and his family history of colon cancers. His family
thromboembolism (VTE) prophylaxis for this patient. VTE is history meets the Amsterdam II criteria for Lynch syndrome
a common clinical problem and is associated with substan- because three Individuals in the family are affected with a
tial morbidity and mortality among hospitalized patients. Lynch syndrome-associated cancer, at least two successive
Most medical patients have one or more risk factors for generations are affected, one of the affected family members
VTE. More than a quarter of patients with undiagnosed and is a first-degree relative of the other two affected family
untreated pulmonary embolism (PE) will have a subsequent members, and at least one cancer was diagnosed in a family
fatal PE. and between 5% and 10"/o of all in-hospital deaths are member younger than age SO years (the "3-2-1-1" pattern).
a direct result of PE. VTE is one of the most common extrain- Additional criteria are that familial adenomatous polyposis
testinal manifestations of inflammatory bowel disease (IBO); has been excluded and tumors have been verified histolog-
patients with IBO have a three-fold risk for VTE compared ically. Guidelines recommend tumor testing (microsatellite
to patients without IBD. VTE represents a significant cause instability testing or immunohistochernlstry) to screen for
of morbidity and mortality in patients with IBD, and risk Lynch syndrome in all colon cancers. The results of tumor
for VTE is highest at the time of disease flare. Although the testing can help detennine if Lynch syndrome is likely and
incidence ofVTE increases with age, the highest relative risk inform genetic testing. Lynch syndrome is only diagnosed in
for VTE in IBD is observed in patients younger than 40 years individuals who meet the Amsterdam criteria and have an

127
Answers and Critiques

identified constitutional mutation in a mismatch repair gene amounts of reflux may produce symptoms. The selection of
(MLHl. MSH2, MSH6, or PMS2) or the epithelial cell adhe- a diagnostic test to confirm or exclude laryngopharyngeal
sion molecuJe (EPCAM) gene. Olagnosis of Lynch syndrome reflux is controversial. Ambulatory pH testing, if' positive,
will inform colonoscopy surveillance recommendations for can help to confirm the diagnosis of GERO and supports the
this patient. Identification of a Lynch syndrome mutation diagnosis of laryngopharyngeal reflux. Negative ambulatory
will also affect the recommendation for screening of other pH testing suggests that the patient does not have GERO and
cancers (gastric, skin, urinary trac t, and, in women, ovarian that proton pump inhibitor therapy should be discontin-
and endometrial cancers). In addition, if he has an identi- ued and another cause of the persistent hoarseness sought.
fiable mutation associated with Lynch syndrome, genetic Other experts propose laryngoscopy as the gold standard
testing should be offered to his first-degree relatives. to diagnose laryngopharyngeal reflux. Common findings
A 3-year interval for colonoscopy cannot be recom- during laryngoscopy include edema and erythema, but
mended until the results of the genetic evaluation are avail- these findings are also seen in 80% of healthy controls. While
able. If Lynch syndrome is confirmed, the screening interval laryngoscopy should not be used as the sole test to diagnose
is every 1 to 2 years, not every 3 years. extraesophageal GERO, abnormal findings in patients with
Colonoscopy is preferred for screening high-risk an appropriate clinical history suggest that cough is related
patients such as those with a previous colorectal cancer; to GERO. Other causes of cough may include allergy, smok-
therefore, screening using fecal immunochemical test- ing, and voice abuse, and these should be ruled out with an
ing, other fecal-based testing, or colonic imaging is not ear, nose. and throat evaluation.
appropriate. Barium esophagography should not be used as the ini-
rn patients with Lynch syndrome, upper endoscopy is tial diagnostic test for GERD and ts not useful for evaluating
recommended to screen for upper gastrointestinal cancers this patient's symptoms, which primarily involve the laryn-
and to sample for Helicobacter pylori. However, this can- gopharynx.
not be recommended until results from genetic testing are Esophageal manometry is used for patients suspected
available. Patients with Lynch syndrome have an increased of having an underlying motility disorder involving peristal-
risk for small-intestinal cancer. However, routine cancer sis or lower esophageal sphincter dysfunction. This patient
screening with capsule endoscopy is controversial and not reports no dysphagia, a common presenting symptom of
routinely recommended even In patients with confirmed motility disorders, and his heartburn symptoms are con-
Lynch syndrome. trolled, so esophageal manometry is not indicated.
Upper endoscopy is the primary tool used to evaluate
KEY POINT
patients with GERD for complications such as erosive esoph-
• Patients with a family history suggesting Lynch syn - agitis, stricture, Barrett esophagus, and esophageal cancer.
drome (three family members are affected with a Because this patient does not have esophageal symptoms
Lynch syndrome- associated cancer, at least two suc- and had a normal endoscopy 1 year ago, upper endoscopy is
cessive generations are affected, one of the affected not indicated.
family members is a first-degree relative of the other
KEY POINT
two affected family members, and at least one cancer
was diagnosed in a family member younger than age • Ambulatory pH testing can be a helpful diagnostic
SO years) should be referred for genetic counseling. test in patients with suspected extraesophageal mani-
festations of gastroesophageal reflux disease.
Bibliography
Syngal S, Brand RE, Church JM, Glardlello FM, Hampel HL, Burt RW; Bibliography
American College of Gastroenterology. ACG clinical guideline: genetic Katz PO, Gerson LB, Vela MF. Guidelines for the diagnosis and management
testing and management of hereditary gastrointestinal cancer syn- of gastroesophageal reflux disease. Am J Ga.~troenterol. 2013:108:308-28;
dromes. Am J Gastroenterol. 2015;ll0:223-62;quiz 263. [PMID: 25645574] quii. 329. fPMID: 23419381] doi:l0.1038/ajg.2012.444
doi:l0.1038/ajg.2014.435

Item 48 Answer: A Item 49 Answer: A


Educatio na l Objedive: Evaluate persistent Educatio nal Objed lve: Treat ascites caused by portal
gastroesophageal reflux disease. hypertension.
Ambulatory pH testing is the most appropriate next diag- Discontinuing lisinopril is the most appropriate next step
nostic test for this patient. The patient has persistent cough, in the management of this patient with ascites. Blood pres-
which may be an extraesophageal symptom of gastroesoph- sure falls with worsening cirrhosis, resulting in reduced
ageal reflux disease (GERD) resulting from laryngopharyn- renal blood flow and glomerular fl.ltration. A compensatory
geal reflux. Additional extraesophageal symptoms of GERD upregulatlon of the renln-angiotensln system results in
include asthma, globus sensation, hoarseness, throat clear- increased levels of vasoconstrictors, including vasopres-
ing, and chronic laryngitis. It appears that the laryngophar- sin, angiotensin, and aldosterone, which support systemic
ynx is more sensitive to the erosive effects of acid, and small blood pressure and kidney function. ACE inhibitors and

128
Answers and Critiques

angiotensin receptor blockers impair the compensatory of acute liver failure are medications, especially acetamino-
response to cirrhosis-related hypotension and thereby phen, and viral infections. although many cases are due to
impair the ability to excrete excess sodium and water and indeterminate causes. The prompt recognition of acute liver
may also affect survival. Medications that decrease kid- failure is essential due to high rates of mortality. Patients
ney perfusion, including NSA!Ds, ACE inhibitors such as require frequent monitoring for hypoglycemia, hypophos-
lisinopril, and angiotensin receptor blockers, should be phatemia, acute kidney injury, infections, and progressive
discontinued because their use often worsens ascites due hepatic encephalopathy, which can be accompanied by cere-
to portal hypertension. The mainstay of therapy of ascites bral edema and intracranial hypertension. Liver transplanta-
is to initiate dietary changes, restricting sodium intake to tion improves the survival rate; therefore, referral to a liver
less than 2000 mg (87 mEq) daily. If sodium restriction transplantation center is essential upon recognition of acute
does not result in significant improvement of ascites, the liver failure.
initiation of diuretic therapy with spironolactone with or The presence of an elevated INR reflects decreased syn-
without furosemide can be effective in increasing urinary thesis of liver-derived clotting factors, and the administra-
sodium excretion. tion of fresh frozen plasma is not indicated in the absence of
Free-water restriction can be useful for the manage- demonstrable bleeding or the need for invasive procedures.
ment of dilutional hyponatremia that is sometimes seen in Kidney failure is common in the setting of acute liver
patients with advanced liver dysfunction. This patient has a failure, and many patients require renal replacement ther-
normal serum sodium concentration, so free-water restric- apy, which should be performed in a liver transplant center.
tion is not indicated. Most patients requiring hemodialysis in this setting tolerate
Propranolol and other nonselective (}-blockers are often continuous renal replacement therapy better than intermit-
used prophylactically for the prevention of variceal hem- tent hemodialysis.
orrhage, but they do not have a role in the management of There are several appropriate indications for endoscopic
ascites. Furthermore, in some patients with ascites that is retrograde cholangiopancreatography (ERCP) , and one of
refractory to medical management, 13-blockers may worsen the most common is the evaluation of biliary obstruction.
clinical outcomes, including survival. In these cases, ERCP can both be diagnostic and therapeu-
Indwelling drains for ascites have been used for patients tic if the obstruction can be removed or bypassed. In this
with malignant ascites, but in the setting of portal hyperten- patient, ERCP is not indicated because the ultrasound shows
sive ascites, such as seen in this patient, indwelling drains no signs of biliary obstruction.
are associated with a high risk for infection and their use is
KEY POINT
contraindicated.
• Acute liver failure is an indication for immediate
KEY POINT
referral to a liver transplantation center.
• Medications that decrease kidney perfusion, includ-
ing NSA!Ds, ACE inhibitors, and angiotensin receptor Bibliography
blockers, should be discontinued in patients with Lee WM. Stravitz RT, Larson AM. Introduction to the revised American
Association for the Study of Liver Diseases position paper on acute liver
ascites. failure 2011. Hepatology. 2012;55:965-7. [PMID: 22213561] doi:l0.1002/
hep.25551
Bibliography
Runyon BA; AASLD. rntroduction to the revised American Association for
the Study of Liver Diseases Practice Guideline management of adult
patients with ascltes due to cirrhosis 2012. Hepatology. 2013;57:1651-3. Item 51 Answer: D
[PMID: 23463403] dol:l0.1002/hep.26359
Educational Objective: Treat left-sided ulcerative
colitis.

c::J Item 50 Answer: D Oral mesalamine with mesalamine enema is the most appro-
priate treatment. Ulcerative colitis typically presents with
Educational Objective: Treat acute liver failure.
bloody diarrhea and abdominal discomfort, the severity of
Referral for liver transplantation is the most appropriate whlch is related to the extent and severity of inflammation.
choice for this patient with acute liver failure. Acute liver The distribution of ulcerative colitis is generally divided
failure is defined by the manifestation of hepatic enceph- into proctitis (involving the rectum only), left-sided colitis
alopathy within 26 weeks of developing symptoms of liver (inflammation does not extend beyond the splenic flexure),
disease. The development of jaundice was this patient's first and pancolitis (inflammation extends above the splenic flex-
symptom of liver disease. Within 6 weeks. he developed ure). The choice of treatment depends on the extent and
coagulopathy, with an INR of 2.6, as well as symptoms severity of inflammation. This patient's stool passage fre-
of hepatic encephalopathy (confusion and asterixis). Liver quency (three per day), intermittent hematochezla, normal
injury is distinct from acute liver failure and presents with blood count and C-reactive protein level, and normal vital
elevated liver test results and/or jaundice in the absence of signs categorize her disease as mild. Results of colonoscopy
evidence of liver function failure. The most common causes with biopsy show moderate inflammation of the rectum and

129
Answers and Critiques

a sigmoid colon consistent with chronic idiopathic left-sided Upper endoscopy should be performed in patients older
ulcerative colitis. than 60 years with persistent dyspeptic symptoms despite
Mesalamine preparations are the mainstay of treat- eradication of H. pylori and/or a trial of' PPI therapy. Upper
ment in patients with mild to moderate disease. Mesalamine endoscopy could also be considered, regardless of age, in
given orally or topically is effective in inducing and main- patients with a combination of alarm features, including a
taining remission in ulcerative colitis. Evidence suggests family history of gastric cancer, immigration from a region
that combined mesalamine therapy (oral and topical) is with increased risk for gastric cancer (Asia, Russia, and
superior for induction of remission in mild to moderately South America), and severe symptoms. Upper endoscopy
active disease compared with oral or topical therapies alone; is considered the gold standard for the exclusion of upper
however, patient adherence to this dual treatment regimen gastrointestinal structural causes of dyspepsia.
can be difficult. No other biochemical, structural (including right upper
A mesalamine enema has topical effects up to the quadrant ultrasonography), or physiologic studies should be
splenic flexure of the colon and would be a more effective performed routinely for the evaluation of dyspepsia.
topical agent when combined with oral mesalamine in
KEY POINT
this patient.
A mesalamine suppository is appropriate for treatment • Patients younger than age 60 years presenting with
of ulcerative proctitis but is not effective for topical treat- dyspepsia should first undergo a noninvasive test for
ment of inflammation above the rectum. Helicobacter pylori followed by eradication therapy if
testing is positive.
KEY POINT
• Combined mesalamine therapy (oral and topical) is Bibliography
superior for induction of remission in mild to moder- Moayyedi PM, L.a~y BE, Andrews CN, Enns RA, Howden ON, Vakil N. ACG and
ately active ulcerative colitis compared with oral or CAG clinical guideline: management of dyspepsia. Am J Gastroenterol.
2017;112:988-1013. [PMID: 28631728] dol:l0.1038/ajg.2017.154
topical therapies alone.

Bibliography
Item 53 Answer: B
Bressler B. Marshall JI<, Bernstein CN, Bitton A. Jones J, Leont!adls GI, et al;
Toronto Ulcerative Colitis Consensus Group. Clinical pr.ictice guidelines Educational Objective: Treat amebic liver abscess.
for the medical management of nonhospltallzed ulcerative colitis: the
Toronto mnsensus. Gastroenterotogy. 201S;l48:1035-10S8.e3. [PMID: Metronidazole plus paromomycin is the most appropriate
25747596] dol:J0.1053/j.gnstro.2015.03.001
treatment for this patient with an amebic liver abscess. The
two most common types of hepatic abscess are pyogenic
and arnebic. Pyogenic liver abscesses are the most com-
Item 52 Answer: B mon hepatic abscesses in the United States, whereas amebic
abscesses are seen more frequently in the developing world.
Educational Objective: Evaluate dyspepsia.
Pyogenic abscesses are typically polyrnicrobial, originating
Stool antigen testing for Hellcobacter pylori is the most from gastrointestinal flora in the setting of intra-abdominal
appropriate next step In the management of this patient The infections, malignancies, or procedures. Amebic abscesses
American College of Gastroenterology and Canadian Asso- are usually caused by the organism Entamoeba histolytica,
ciation of Gastroenterology have issued a joint recommen- and the mechanism of formation involves enteric infection
dation that patients younger than age 60 years presenting with the invasion of amoeba through the intestinal mucosa
with dyspepsia should first undergo a noninvasive test for and via the portal vein. With amebic abscesses, infection
H. pylori followed by eradication therapy if testing is posi- of the liver is asymptomatic until hepatic necrosis results
tive. The committee gave a strong recommendation based on in abscess development, at which time abdominal pain,
high-quality evidence for the "test and treat" strategy. fever, and leukocytosis develop. Amebic liver abscesses are
The pooled data from four trials including a total of 1608 seen in patients from endemic areas, such as India, Africa,
patients with dyspepsia found no difference in the preva- or Central or South America, or in patients who have trav-
lence of dyspepsia at the end of 1 year of follow-up between eled to endemic areas. The typical presentation in travelers
patients undergoing the "test and treat" strategy versus those occurs 3 to S months after infection. Nearly all patients
receiving a course of empirical proton pump inhibitor (PPI) with an amebic abscess have positive serological studies for
therapy. Furthermore, there was a trend toward a reduction E. histolytica. The diagnosis Is usually established with com-
in cost for the "test and treat" strategy compared to empir- patible imaging and serologic testing. Ultrasonography is less
ical PPI therapy. For these reasons, testing for H. pylori is sensitive for the detection of pyogenic liver abscesses (CT is
a more appropriate initial strategy for this patient than a preferred) but may be equally sensitive for the detection of
course of omeprazole. ln patients whose testing is negative amebtc abscesses. The mainstay of therapy for an amebic
for H. pylori, an empiric trial of PPI therapy (for example, liver abscess is antibiotic therapy; such as metronidazole,
omeprazole) can be considered before proceeding to any plus a luminal agent, such as paromomycin, to eradicate the
other testing. coexisting intestinal infection.

130
Answers and Critiques

Meropenem is an effective broad-spectrum antibiotic symptoms are diarrhea, fever, and abdominal pain; half
that is often used to treat pyogenic liver abscesses. It has no of patients have grossly bloody stools. The most common
role in the treatment of an amebic abscess. finding on endoscopy is well-demarcated ulcerations.
Percutaneous drainage and surgical resection of an On biopsy, histological findings include multinucleated
abscess are treatment options indicated for pyogenic giants cells with eosinophilic inclusions. 'Ihe rarity of
liver abscesses. Surgical intervention is usually necessary this condition coupled with the patient's colonoscopy
for abscesses 5 cm or larger in size, complex abscesses, and biopsy findings make cytomegalovirus infection an
presence of gas-forming organisms, hemodynamic insta- unlikely diagnosis.
bility, biliary fistulization, or presence of a foreign body.
KEY POINT
These procedures are typically not needed in the treatment
of amebic liver abscesses, which usually resolve with anti- • Chronic bloody diarrhea and abdominal discomfort
microbial therapy. arc typical presenting symptoms of inflammatory
bowel disease; endoscopic findings help distinguish
KEY POINT
ulcerative colitis from Crohn disease.
• The mainstay of therapy for amebic liver abscesses is
antibiotic therapy, such as metronidazole, plus a Bibliography
luminal agent, such as paromomycin, to eradicate the Danese S, Fiocchi C. Ulcerative colitis. N Engl J Med. 2011;365:1713-25.
coexisting intestinal infection. [PMID: 22047562] doi:l0.1056/NEJMral102942

Bibliography Item 55 Answer: C


Bonder A. Afdhal N. Evaluation ofliver leslons. Clin Liver Dis. 2012;16:271-
83. [PMIO: 22541698] doi:l0.1016/j.cld.2012.03.001 Educational Objective: Test for Wilson disease.
Serum ceruloplasmin measurement is the most appro-
Item 54 Answer: D priate test for this patient. Wilson disease is an autosomal
recessive disorder of copper excretion with subsequent
Educational Objective: Diagnose ulcerative colitis.
accumulation of copper in the liver and central nervous
Ulcerative colitis is the most likely diagnosis in this patient. system, cornea, kidney, joints, and cardiac muscle resulting
The differential diagnosis of chronic bloody diarrhea includes in organ dysfunction. Children aged 10 years or younger
inflammatory bowel disease (ulcerative colitis or Crohn coli- with Wilson disease tend to present with acute liver fail-
tis) and chronic ulcerating infections such as cytomegalo- ure; older patients (aged 30 years or younger) present
virus or Entamoeba histolytica. This patient's stool studies with chronic liver disease and/or neurologic manifesta-
for enteric pathogens are negative, making Crohn colitis or tions. Wilson disease should be considered in all patients
ulcerative colitis the most likely cause of her symptoms. The younger than age 40 years who have unexplained liver dis-
mucosa! biopsy results showing crypt architectural distor- ease. When Wilson disease causes acute hepatitis, usually
tion confirm a chronic colitis such as ulcerative colitis or in young patients, the sudden release of copper from liver
Crohn disease. The endoscopic description of inflammation cells can also induce hemolytic anemia. In this patient
beglnning at the anorectal verge and extending proximally with evidence of hepatic encephalopathy, hemolytic ane-
in a continuous fashion with transition to normal mucosa at mia, low alkaline phosphatase level, and unconjugated
splenic flexure is consistent with left-sided ulcerative colitis. bilirubinemia, the diagnosis of Wilson disease should be
Crohn disease characteristically has a patchy progression considered. The serum ceruloplasmin level is used to test
pattern resulting in "skip lesions" and may spare the rectum, for Wilson disease. Slit-lamp examination of the cornea
making Crohn colitis less likely in this case. can be performed to evaluate for Kayser-Fleischer rings,
Entamoeba histolytica causes amoebic dysentery. In the which most commonly occur in patients with neurologic
United States, amebiasis is usually diagnosed in immigrants manifestations of Wilson disease. A 24-hour urine copper
or travelers returning from endemic areas, or in institutional assessment can be performed to confirm an excessive
settings. Although most Infected persons are asymptomatic, amount of copper excretion.
others may develop bloody or watery diarrhea, abdominal The diagnosis of primary biliary cholangitis is generally
pain, and fever. Rarely, a chronic syndrome of diarrhea, made on the basis of a cholestatic liver enzyme profile in the
weight loss, and abdominal pain can mimic inflammatory setting of a positive antimitochondrial antibody test. The
bowel disease. Stool microscopy for ova and parasites can aid predominant liver enzyme abnormality is an increase in
in the diagnosis by detecting the protozoa, but stool antigen serum alkaline phosphatase level. This patient's biochemical
testing is more sensitive. This patient's negatiVe travel history profile reflects hepatocellular injury, not cholestasis, and an
and findings on colonoscopy and biopsy are not consistent anti mitochondrial antibody test is not indicated.
with amebiasis. With a negative test for hepatitis B virus surface anti-
Cytomegalovirus infection is an uncommon infection gen, this patient is very unlikely to have a hepatitis B viral
in immunocompetent patients but can occur and cause infection and measurement of a hepatitis B virus DNA level
significant symptoms. The most common presenting is not indicated.

131
Answers and Critiques

Symptomatic hereditary hemochromatosis is usually such as esophageal perforation. Therefore, patients who are
diagnosed after age 40 years and is often diagnosed later in not surgical candidates should not undergo endoscopic dila-
women than ln men, due to iron losses associated with men- tion treatment of acbalasla.
struation, pregnancy, and lactation. The most sensitive initial
KEY POINT
diagnostic study is measurement of the serum transferrin
saturation. The patient's sex, young age, and presence of a • Patients with achalasia who are at high surgical risk
hemolytic anemia make the diagnosis of hereditary hemo- should be treated with endoscopic botulinum toxin
chromatosis unlikely and the measurement of a transferrin injection.
leve1 unnecessary.
Bibliography
KEY POINT
Vaezl MF, Pandolfino JE, Vela MF. ACG cl!nlcal guideline: diagnosis and
• Wilson disease should be considered in aJI patients management of achalasfa. Am J Gastroenterol. 2013;108:1238-49; quiz
1250. fPMID: 23877351] dol:l0.1038/ajg.2013.196
younger than age 40 years who have unexplained liver
disease.
Item 57 Answer: B
Bibliography Educational Objective: Recognize atypical chest
F.apen CE, Kumar S, FlemingJJ, Ramakrishna B. Abmham L, Ramachandran
J. Copper and liver disease. Gut. 2012;61:63. [PMID: 22139599]doi:I0.1136/ pain and evaluate before starting treatment for
gutjnl-2011-301743 gastroesophageal reflux disease.
The most appropriate next step ln management of this patient
is electrocardiography followed by evaluation for cardiac
Item 56 Answer: A
disease. Although gastroesophageal reflux disease (GERD)
Educational Objective: Treat acha1asia with botulinum can present with symptoms of chest pain, this patient's
toxin injection. pain is also exertional (occurs while walking up stairs),
Endoscopic botulinum toxin injection is the best treat- which is not typical in GERD. Failure to identify underlying
ment option for this patient because his cardiac status cardiac disease in the setting of atypical chest pain can be
places him at an unacceptab1e surgica1 risk. Achalasia ls catastrophic. Although this patient may have GERD, it is
a motility disorder of the esophagus that results in aperi- essential to evaluate her for heart disease before beginning
sta1sls and inadequate relaxation of the lower esophageal empiric treatment for GERO. GERD Is the most common
sphincter (LES). The cause of achalasia is unknown, but cause of noncardiac chest pain, and chest pain caused by
the pathophysiologic process involves ganglion cell and esophageal disorders can be difficult to distinguish from
myenteric plexus degeneration in the esophageal body cardiac chest pain because of the anatomic proximity and
and LES. This nerve imbalance leads to uncontested action common innervation of the esophagus and the heart.
by cholinergic nerves and incomplete LES relaxation. The Barium esophagography is recommended as the initial
clinical presentation of achalasia consists of dysphagia to test for evaluation of dysphagia but is not recommended for
both solids and liquids. Additional symptoms may include the eva1uation of typical GERD symptoms.
regurgitation, chest pain, and heartburn. Standard treat- In patients with typical GERD symptoms, no additional
ments for achalasia include surgical myotomy and endo- testing is required and an empiric trial of a proton pump
scopic pneumatic dilation. Botulinum toxin injection inhibitor (PPI) can be initiated. Failure to improve after
inhibits acetylcholine release, resulting in relaxation of 8 weeks of standard therapy should prompt further evalua-
the LES and relief of achalasia symptoms ln up to 85% tion. ln patients with atypical chest pain, treatment with a
of patients. However, approximately 50% of patients will PPI is appropriate after ruling out cardiac disease as a cause
experience recurrent symptoms within 6 to 24 months of the chest pain.
after injection. An upper endoscopy is indicated in patients with gas-
Medical therapy with calcium channel blockers trointestina1 alarm symptoms such as anemia, dysphagia, or
or long-acting nitrates is considered third-line therapy unintentional weight loss. Upper endoscopy ts the primary
because its effectiveness is limited. Medications may be tool to evaluate mucosal inflammation in the upper gastro-
used early in treatment, in patients who decline other intestinal tract.
interventions, before definitive treatment, and in patients K EY POINT
who have a recurrence in symptoms after botulinum toxin
• In patients with atypical chest pain, a cardiac cause
injection for acha1asia.
must be ruled out before starting treatment for gas-
Studies have shown that endoscopic pneumatic dilation
troesophageal reflux disease.
and surgical myotomy result In similar clinical outcomes in
patients with achalasla, with choice of therapy depending
on local expertise. Pneumatic dilation is the most effective Bibliography
Katz PO, Gerson LB, Vela MF. Guidelines for the diagnosis and management
nonsurgical treatment and is more cost-effective than surgi- of gastroesophageal reflux disease. Am J Gastroenterol. 2013;108:308-28;
ca1 myotomy, but it ls associated with serious complications, quiz 329. [PMID: 23419381] doi:l0.1038/ajg.2012.444

132
Answers and Critiques

Item 58 Answer: D In 2010, the FDA revised the prescription and nonprescrip-
Educational Objective: Diagnose the relapsing, tion labels for PPls to include possible increased risk for
remitting variant of hepatitis A viral infection. hip, wrist, and spine fractures. The FDA concluded that
fracture risk was greatest with higher doses of PP!s or PP!
Relapsing, remitting hepatitis A virus (HAV) infection is the exposure for I year or longer. The evidence associating PPI
most likely diagnosis. This patient returned from Mexico therapy with hip fracture was inconsistent, and guidelines
with an icteric illness and tested positive for HAV IgM, recommend not stopping PPI therapy when it is otherwise
consistent with acute HAV infection. Her HAV-infection indicated in patients at risk for osteoporosis. Other possi-
symptoms resolved and liver chemistry tests normalized, ble PPI-related adverse reactions include vitamin B12 and
but she developed an icteric illness 3 months later. This mineral (calcium and magnesium) malabsorptlon, as well
presentation is most consistent with an atypical course of as increased risk for community-acquired pneumonia, Clos-
HAV infection known as relapsing, remitting HAV infection. trtdium difficile infection, and cardiovascular events. Many
It is rare but was reported to occur in up to 10% of patients guidelines recommend that long-term PPI therapy be given
with HAY infection in one series. Patients may have multi- at the lowest effective dose, which may include as-needed
ple clinical or biochemical relapses but will spontaneously therapy. Other recommendations encourage an attempt to
improve within months to 1 year without intervention. The either reduce or stop chronic PP! therapy for uncomplicated
relapses tend to be milder, are more likely to be associated GERD at least once per year.
with cholestasis, and may be associated with extrahepatic Upper endoscopy Is not indicated in patients whose
manifestations including nephritis, arthralgia, vasculitis, GERD symptoms are controlled in the absence of alarm
and cryoglobulinemia. symptoms, such as dysphagia, weight loss, or anemia.
In rare cases, HAV infection has been observed to trig- Prokinetic agents such as metoclopramide should not
ger autoimmune hepatitis. However, testing does not show be used in the treatment of GERD unless gastroparesis is also
the typical antibodies (anti- smooth muscle or antinuclear present. Prokinetic drugs are no more effective than placebo,
antibodies) or elevated IgG levels commonly seen in patients and these drugs can be associated with acute dystonic reac-
with autoimmune hepatitis. tion and tardive dyskinesia. H2 blockers, such as ranitidine,
Acute leptospirosis is manifested by high fever, head- can be used alone in patients without erosive esophagitis
ache, severe myalgia, conjunctival injection, abdominal pain, whose symptoms respond to H2-blocker therapy.
diarrhea, pharyngitis, and occasionally a pretibial rash. The Sucralfate is sulfated sucrose complexed with alumi-
patient's clinical course is not compatible with leptospirosis. num hydroxide and can bind to damaged mucosa. It has
Malarial infection can result in jaundice due to hemoly- been shown to prevent acute mucosa! damage and heals
sis or, less commonly, cholestasis. However, Mexico is con- chronic ulcers without altering gastric acid concentration or
sidered to be a very low-risk travel destination for acquisi- pepsin secretion. Sucralfate has no role in the treatment of
tion of malaria. GERD because, as with metoclopramide, studies have shown
KEY POINT that it is no more effective than placebo.

• The relapsing, remitting variant of hepatitis A viral KEY POINT


infection is characterized by multiple clinical or bio- • Long-term proton pump inhibitor (PP!) therapy for
chemical relapses with spontaneous improvement uncomplicated gastroesophageal reflux disease should
within months to I year without intervention. be given at the lowest effective dose possible, and con-
sideration should be given to reducing or stopping PP!
Bibliography therapy at least once a year.
Matheny SC, Kingery JE. Hepatitis A. Am Fam Physician. 2012;86:1027-34;
quiz 1010-2. (PMID: 23198670)
Bibliography
Katz PO, Gerson LB. Vela MF. Guidelines for the diagnosis and management
ofgastroesophageal reflux disease. Am J Gastroenterol. 2013;108:308-28;
Item 59 Answer: A quiz 329. [PMID: 23419381] dol:I0.1038/ajg.2012.444

Educational Objective: Manage chronic proton pump


inhibitor therapy.
ltem60 Answer: D
The most appropriate next step In management is an attempt
Educational Objective: Treat intrahepatic cholestasis of
to discontinue or reduce pantoprazole. Guidelines recom-
pregnancy with ursodeoxychollc acid.
mend that patients with symptomatic gastroesophageal
reflux disease (GERD) syndromes without esophagitis be Ursodeoxycholic acid therapy is the most appropriate man-
treated with a short course of a proton pump inhibitor (PPI) agement for this patient. Intrahepatic cholestasis of preg-
to achieve symptom control. Maintenance PPI therapy Is rec- nancy Is a liver condition that affects pregnant women In
ommended for patients with GERO who continue to have the second or third trimester. This condition spontaneously
symptoms after the initial course of a PPI is discontinued, and improves within 48 hours of delivery. Typical symptoms
for those who have erosive esophagitis or Barrett esophagus. Include diffuse pruritus, and there is typically a mild increase

133
Answers and Critiques

in serum transaminase concentrations. Associated risks features are helpful in confirming this diagnosis. The most
include premature delivery as well as higher rates of fetal common cause of colon ischemia is a nonocclusive low-flow
death. Mutations in bile salt transporters have been impli- state in the colonic mtcrovasculature. Most cases of colonic
cated in the pathophysiology. The mainstay of therapy for ischemia involve the left colon, which is supplied by the
intrahepatic cholestasis of pregnancy is ursodeoxycholic acid, inferior mesenteric artery; as with ischemla involving the
which is associated with alleviated symptoms, improved fetal right colon, the diagnosis is clinical and supported by CT
outcomes, and improved liver test abnomialities. and colonoscopy. Patients with left-sided colonic ischemia
Cholestasis of pregnancy can be diagnosed in pregnant tend to heal well with conservative therapy alone. whereas
women with otherwise unexplained pruritus and abnormal isolated right-colon ischemia can be the harbinger of acute
liver chemistry tests. Elevated levels of total serum bile acid mesenteric ischemia caused by a focal thrombus or embolus
in a pregnant woman with pruritus are considered diagnos- of the superior mesenteric artery. This artery supplies both
tic for cholestasis of pregnancy in the absence of an alterna- the small intestine and right colon, and the consequences
tive diagnosis. In this patient with findings compatible with of acute mesenteric ischemia involving the small bowel ~e
intrahepatic cholestasis of pregnancy and a normal hepatic severe, with mortality rates that can approach 60%. For this
ultrasound, a liver biopsy is not needed. reason. patients with isolated right-colon ischemia require
HELLP (Hemolysis, Elevated Liver enzymes, and Low urgent. noninvasive imaging of the mesenteric vasculature to
Platelets) syndrome presents during the third trimester and assess the extent of ischemia and nature of the intervention.
is an advanced complication of preeclarnpsia. HELLP syn- CT angiography is the recommended method of imaging
drome typically presents with abdominal pain, new-onset for diagnosing acute mesenteric ischemia because it can be
nausea and vomiting, pruritus, and jaundice. A peripheral obtained rapidly. CT angiography visualizes the origins and
blood smear will reveal schistocytes. This patient is not ane- length of the vessels, characterizes the extent of occlusion.
mic and does not have thrombocytopenia or hypertension ; and aids in planning revascularization.
therefore, a peripheral blood smear to support the diagnosis Doppler ultrasonography of the mesenteric vessels is an
of HELLP syndrome is not needed. effective, low-cost tool that can assess the proximal visceral
Pruritic urticarial papules and plaques of preg- vessels but has limited ablllty to visualize distal vessels. It
nancy (PUPPP) is the most common specific derrnatosis is best reserved for the evaluation of patients with chronic
of pregnancy. PUPPP is a clinical diagnosis confirmed by mesenteric ischemia, which typically presents with post-
the appearance late in the third trimester of erythematous prandial abdom inal pain, sitophobia, and weight loss.
plaques in the distribution of striae. Persistent and bother- MR angiography provides information about mesen-
some pruritus is a symptomatic hallmark of the condition. It teric arterial flow and avoids the potential harms of radiation
is not associated with jaundice or elevated transaminase lev- and use of contrast that are associated with CT angiography;
els. The condition usually resolves shortly after delivery. The however, MR angiography takes longer to perform, lacks the
first-line option for therapy is usually topical glucocorticoids required resolution to identify arterial occlusion, and can
oflow to mid potency. Topical glucocorticoids are of no value overestimate the severity of stenosis.
In intrahepatic cholestasis of pregnancy. Selective catheter angiography was the standard method
for diagnosing mesenteric ischemia; however, it is now used
KEY POINT
after a revascularization plan has been chosen because CT
• The mainstay of therapy for intrahepatic cholestasis of angiography can be obtained rapidly and is noninvasive.
pregnancy is ursodeoxycholic acid, which is associ-
KEY POINT
ated with alleviated symptoms, improved fetal out-
comes, and improved liver test abnormalities. • Isolated right-colon ischemia may be a warning sign
of acute mesenteric ischemia caused by embolism or
Bibliography thrombosis of the superior mesenteric artery and
Bacq V. le Besco M, Lecuyer Al, Gendrot C, Polin J, Andres CR, et al. should be evaluated using CT angiography.
Ursodeoxycholic acid therapy in intrahepatlc cholestasls of pregnancy:
results In real-world conditions and factors predictive of response to
treatment. Dig Liver Dis. 2017:49:63-69. [PMID: 2782S922] doi:l0.1016/ Bibliography
j.dld.2016.10.006 Clair DG, Beach JM. Mesentertc ischemla. N Engl J Med. 2016;374:959-68.
[PMJO: 26962730] doi:10.1056/NEJMra1503884

c::J It em 61 Answer: A
Item 62 A nswer: B
Educational Objective: Evaluate Isolated right-colon
ischemia. Educational Objective: Diagnose medication-induced
CJ
enteropathy.
CT angiography is the best next test for this patient, whose clin-
ical presentation with the sudden onset of right-sided, cramp- Discontinuing olmesartan, an angiotensin II receptor
ing abdominal pain followed by a bloody bowel movement blocker, is the most appropriate next step in the manage-
is typical of isolated right-colon ischemia. A CT scan show- ment of this patient. The patient's presentation is most
ing thickening of the ascending colon and the colonoscopy consistent with drug-associated enteropathy related to

134
Answers and Critiques

olmesartan. In 2013, the FDA issued a warning that olme- visible on endoscopy. Barrett esophagus can present with no
CJ sartan medoxomil can cause intestinal symptoms known dysplasia, indefinite dysplasia, low-grade dysplasia, or high-
CONT. as sprue-like enteropathy and approved labeling changes to grade dysplasia, and in some patients dysplasia progresses
include this concern. The enteropathy may develop months to adenocarcinoma of the esophagus. Tn the past, guidelines
to years after starting olmesartan. Drug-associated enterop- recommended a surveillance endoscopy in 6 months for
athy can mimic refractory celiac disease with findings of patients with low-grade dysplasia. However, more recent
villous atrophy and increased intraepithelial lymphocytes guidelines recommend that patients with minimal comor-
in the first part of the duodenum. The clinical presenta- bidities undergo endoscopic ablation therapy for perma-
tion can include severe diarrhea, weight loss, and dehydra- nent eradication of Barrett esophagus. Endoscopic ablation
tion requiring hospitalization. Most of the reported cases should be considered after confirmation of dysplasia by a
of drug-associated enteropathy are caused by olmesartan, second expert pathologist. If a patient is ineligible for or
although other angiotensin-receptor blocking drugs have unwilling to undergo ablation therapy, annual surveillance
been implicated. In addition to adequate fluid resuscitation, endoscopy is recommended as an alternative. If two con-
the offending medication should be discontinued. In most secutive surveillance endoscopies show no dysplasia, the
cases of drug-associated enteropathy, symptoms and patho- surveillance interval is changed every 3 to 5 years, the same
logical changes resolve when the drug is stopped. interval used in patients with Barrett esophagus with no
Other medications, such as NSAlDs, proton pump inhib- dysplasia. Patients with Barrett esophagus with high-grade
itors, antibiotics, colchicine, metforrnin, and cholesterol- dysplasia are also treated with endoscopic ablation.
lowering drugs (such as statins), can cause diarrhea and Esophagectomy is reserved for patients in whom abla-
should be considered when evaluating a patient with symp- tion does not result in eradication of the dysplasia or who
toms of chronic diarrhea (>4 weeks' duration). However, have esophageal cancer.
because of the severity of the patient's presentation, the diar- Surgical treatments for GERO consist of laparoscopic
rhea is unlikely to be caused by atorvastatin. Additionally. fundoplication, an endoscopic procedure that does not cure
atorvastatin and other statins are not linked to villous atrophy. Barrett esophagus or reduce the risk for progression of dys-
A gluten-free diet is the mainstay of treatment for celiac plasia or cancer, and bariatric surgery (for obese patients).
disease. Given the patient's normal celiac serology testing, Indications for surgery include patient preference to stop
his symptoms are unlikely to be caused by celiac disease taking medication, medication side effects, large hiatal her-
and a gluten-free diet is not indicated. Villous atrophy and nia, and refractory symptoms despite maximal medical ther-
increased intraepithelial lymphocytes are not specific for apy (although patients with medically refractory symptoms
celiac disease. may be less likely to benefit from surgery). Approximately
Other causes of villous atrophy, such as refractory one third of patients require resumption of PPl therapy
celiac disease, Crohn disease, and autoimmune enteropathy, within 5 to 10 years after surgery. Postoperative complica-
are treated with glucocorticoids such as prednisone. This tions include dysphagia, diarrhea, and inability to belch
patient's presentation and his older age are not consistent because of a tight fundoplication.
with these causes of villous atrophy, so glucocorticoids are Guidelines recommend that patients with Barrett esoph-
not indicated. agus undergo surveillance only after adequate counseling
regarding the risks and benefits of surveillance. Patients with
KEY POINT
nondysplastic Barrett esophagus should undergo endoscopic
• Olmesartan causes medication-induced enteropathy surveillance no more frequently than every 3 to s years.
that can mimic refractory celiac disease.
KEY POINT

Bibliography • Barrett esophagus with low-grade dysplasia should be


Ianiro C, Bibi.JO s. Montalto M, Ri<.'Ci R, Casbarrinl A, Cammarota G. Systematic treated with endoscopic ablation therapy in patients
review: sprue-like enteropathy associated with olmesartan. Aliment without significant comorbidities.
Pharrnacol Ther. 2014;40:16-23. [PMID: 24805127) dol:l0.1111/apt.12780

Bibliography
Shaheen NJ, Falk GW, lyer PG. Gerson LB ; American College of
Item 63 Answer: A Gastroenterology. ACG Clinical guideline: diagnosis and management of
Barrett's esophagus. AmJ Gastroenterol. 2016;111:30-50; quiz 51. [PMID:
Educational Objective: Manage a patient with Barrett 26526079] dol:l0.1038/ajg.2015.322
esophagus with low-grade dysplasia.
Endoscopic ablation therapy is the most appropriate next Item 64 Answer: D
step in the management of this patient's Barrett esophagus
Educational Objective: Diagnose Gtardla lamblia
with low-grade dysplasia. Barrett esophagus is a conse- jnfection.
quence of chronic reflux, regardless of the presence of gas-
troesophageal reflux disease (GERD) symptoms. The damage Stool testing for Giardia infection is the most appropriate
from GERO causes a change in the norrnal squamous lining next step in the management of this patient. The patient
of the distal esophagus to a specialized columnar epithelium presents with persistent diarrhea, defined as lasting between

135
Answers and Critiques

2 and 4 weeks, and because of his occupational exposure, the postprandial fullness, nausea, vomiting, upper abdominal
most likely cause is Giardia lamblia (also known as Giardia pain, bloating, and weight loss, but these symptoms cor-
intestinalis). Transmission is waterborne or through the relate poorly with the findings on objective gastric emp-
fecal-oral route. It occurs most commonly among children tying tests. Various other upper gastrointestinal disorders
(especially in developing countries), child care workers, and can present with similar symptoms. Exclusion of other
backpackers or campers who drink untreated water from upper gastrointestinal disorders, objective documentation
lakes, rivers, or wells. Giard/a is a noninvasive organism of delayed gastric emptying, and an attempt to identify the
that infects the small intestine and causes diarrhea asso- cause of the gastroparesis are essential before treatment.
ciated with nausea, bloating, and foul -smelling gas due Retained food in the stomach during upper endoscopy is not
to malabsorption. Because Giardia cysts are intermittently objective evidence of delayed gastric emptying. The three
shed, stool microscopy is less sensitive than stool antigen tests to objectively demonstrate delayed gastric emptying
testing, and the recommended test is a stool enzyme-linked are gast1ic scintigraphy, wireless motility capsule, and the
immunosorbent assay. Treatment options include tinidazole, gastric emptying breath test. Jf scintigraphy is pursued, the
metronidazole, and nitawxanide. Treatment of giardiasis 4-hour study Is preferred over 90- or 120-minute studies due
with metronidazole is curative in more than 85% of patients. to increased diagnostic accuracy.
A colonoscopy is not indicated because this patient's A 24-hour pH probe may be considered when heart-
diarrhea is not associated with weight Joss, abdominal pain, burn symptoms do not respond to a higher dose of acid sup-
or blood in the stool, making inflammatory bowel disease an pression therapy, such as twice-daily proton pump inhibitor
unlikely diagnosis. Microscopic colitis would be an unlikely therapy or a proton pump inhibitor plus a histamine recep-
diagnosis in a 45-year-old man who Is not taking any med- tor antagorust. This patient's medical therapy for heartburn
ications known to be associated with microscopic colitis, symptoms should be optimized before further testing is pur-
such as NSAIDs. sued, and this test will not explain the patient's predominant
A CT scan of the abdomen and pelvis would be unlikely symptoms of nausea, bloating, and epigastric pain.
to suggest a cause for watery diarrhea of 4 weeks' duration. Both metoclopramide and domperidone are effective
The patient's symptoms and occupation suggest a parasitic in the treatment of gastroparesis. Metoclopramide is the
cause of diarrhea that would not be diagnosed with an only FDA-approved agent for the treatment of gastroparesis.
imaging study. Domperidone can be used under a special program adminis-
A 24-hour 5-hydroxyindoleacetic acid measurement tered by the FDA. The side effects of metoclopramide include
is used to evaluate for carclnoid tumors. Up to 85% of dystonia, Parkinson-type movements, and tardive dyski-
patients with gastrointestinal carcinoid syndrome experi- nesia. Domperidone can prolong the QT interval on elec-
ence intermittent flushing. Jn addition to flushing, diarrhea trocardiography, potentially leading to cardiac arrhythmia.
is prominent in most patients and is related to rapid intes- Before initiating treatment for gastroparesis, it is necessary
tinal transit time. Approximately 40% of patients also have to confirm the diagnosis.
right-sided valvular heart disease. This patient has no flush-
KEY POINT
ing or evidence of a heart murmur, making carcinoid syn-
drome unlikely and measurement of 5-hydroxyindoleacetic • The diagnosis of gastroparesis requires the presence of
unnecessary. specific symptoms, absence of mechanical outlet
obstruction, and objective evidence of delay in gastric
KEY POINT
emptying into the duodenum.
• Giard/a lamblia infection is a common parasitic
infection that occurs most often among children, Bibliography
child care workers. and backpackers or campers who Camilleri M. Parkman HP, Shafi MA, Abell TL. Gerson L; American College
drink untreated water from lakes, rivers, or wells. of Gastroenterology. Clinical guideline: management of gastmparesls.
Am J Gastroenterol. 2013;108:18-37: quiz 38. [PMID: 23147521) dol:to.
1038fajg.2012.373
Bibliography
DuPont HI. Persistent diarrhea: a clinical review. JAMA. 2016;315:2712- 23.
[PMID: 273572~1! doi:J0.1001/jama.2016. 7833
Item 66 A nswer: B
Educational O bjective: Treat bleeding esophageal
CJ
Item 65 Answer: A varlces.
Educational Objective: Diagnose gastroparesis.
Ciprofloxacin for 7 days is the most appropriate next treat-
Gastric emptying scintigraphy is the most appropriate ment for this patient with variceaJ bleeding. Variceal bleed-
next step in management. The diagnosis of gastroparesis ing is a life-threatening complication of portal hypertension.
requires: (1) the presence of specific symptoms; (2) the Risk factors forvariceal hemorrhage are Child-Turcotte-Pugh
absence of mechanical outlet obstruction; and (3) objec- class B and C cirrhosis, large varices (>5 mm). and the endo-
tive evidence of delay in gastric emptying into the duode- scopic finding of red markings on varices. Approximately
num. Commonly reported symptoms include early satiety, IS"lo to 20% of patients die within 6 weeks of hemorrhage.

136
Answers and Critiques

The mainstay of therapy for variceal hemorrhage is endo-


CJ scopic therapy. Antibiotic therapy is an important adjunctive
is a clinical diagnosis and should be suspected in patients
with cirrhosis who have changes in mental status, mood, or
CONT. therapy for variceal bleeding because bacterial infection behavior. Hepatic encephalopathy can be seen in the setting
occurs in 30% to 40% of patients within 1 week of variceal of acute liver failure as well as cirrhosis. The initial man-
bleeding. The most common infections seen are sponta- agement of hepatic encephalopathy centers on identifying
neous bacterial peritonitis as well as bacterernia, urinary and mitigating a precipitating factor. Up to 80% of patients
tract infections, and pneumonia. Antibiotic therapy after have a precipitating factor, most commonly infection or
variceal bleeding reduces rates of infection and rebleeding as gastrointestinal bleeding. Other precipitants include opi-
well as mortality after variceal bleeding. There is also benefit oids, benzodiazepines, electrolyte abnormalities, hypoglyce-
to administering antibiotics for patients with cirrhosis who mia, hypoxia, transjugular intrahepatic portosystemic shunt
present with nonvariceal upper gastrointestinal bleeding. placement, inappropriate lactulose dosing, and dehydra-
This patient does not require a blood transfusion. tion. In this patient who uses a benzodiazepine, alprazolam
Hemodynamically stable patients with acute upper gastro- therapy should be discontinued and alternative means of
intestinal bleeding, no acute coronary syndrome, and no his- managing anxiety must be sought. Tapering is likely unnec-
tory of peripheral vascular disease or stroke should be trans- essary in this patient because the medication was initiated
fused when the hemoglobin level is 7 g/dL (70 g/L) or less. recently. All patients with overt episodic hepatic encepha-
This transfusion threshold is associated with improved lopathy should undergo screening for infections, including
clinical outcomes compared to higher transfusion thresh- diagnostic paracentesis when ascites is present.
olds. The goal of transfusion is a hemoglobin level of7 g/dL CT of the head can be a useful study in patients with
(70 g/ L) to 9 g/dL (90 g/L) . In the setting ofhemodynamic altered mental status of unknown cause, but in patients with
stability, overtransfusion should be avoided because it can hepatic encephalopathy without a history of head trauma or
cause an increase of portal pressure and thereby precipi- a focal neurological examination, head CT is unnecessary.
tate rebleeding. Early concerns regarding dietary protein consumption
Platelet transfusion is not warranted because hemosta- as a precipitant of hepatic encephalopathy have been largely
sis has already been achieved. This patient has thrombocy- debunked, and outside of rare circumstances, dietary pro-
topenla, but a transfusion of platelets also has the potential tein restriction should not be undertaken, even in the setting
to cause volume overload leading to an increase in portal of acute hepatic encephalopathy. Furthermore, due to the
hypertension and rebleeding. Patients with thrombocyto- high risk for protein-calorie malnutrition in patients with
penla with clinically significant active bleeding, as well as cirrhosis, dietary protein restriction can result in worsened
patients without bleeding in whom nonneuraxial surgery clinical outcomes.
is planned, should be transfused to a target platelet count Lactulose is first-line treatment and should be titrated
of 50,000to100,000/µL (50-100x10 9 /L) depending on the to produce three stools per day. Rifaximin is added to Jac-
clinical circumstances. tulose for prevention of recurrent episodes after a second
Transjugular intrahepatic portosysternic shunt (TIPS) episode of hepatic encephalopathy. Due to its expense, it is
placement is useful in the 10% to 20% of patients with vari- not a first-line therapy for hepatic encephalopathy.
ceal bleeding in whom hemostasis is unable to be achieved
KEY POINT
by endoscopic therapy. In this patient in whom endoscopic
therapy was successful, TIPS placement is not indicated. • Up to 80% of patients with hepatic encephalopathy
have a precipitating factor, most commonly infection
KEY POINT
or gastrointestinal bleeding.
• The mainstay of therapy for variceal hemorrhage is
endoscopic therapy, and adjunctive therapies such as Bibliography
antibiotic therapy improve outcomes. Vllstrup H, Amodio P, Bajaj J, Cordoba J, Ferenci P, Mullen KO, et al. Hepatic
encephalopathy In chronic liver disease: 2014 practice guideline by the
American Association for the Study of Liver Diseases and the European
Bibliography Association for the Study of the Liver. Hepatology. 2014;60:715-35.
Satapathy SK, Sanyal AJ. Nonendoscopic management strategies tbr acute [PMID: 25042402] doi:l0.1002/hep.27210
esophagogastric variceal bleeding. Gastroenterol Clin North Am.
2014;43:819-33. [PMID: 25440928] dol:l0.1016/j.gtc.2014.08.011
ltem68 Answer: B
Educational Objective: Screen for colorectal cancer in a
CJ Item 67 Answer: D high-risk individual.
Educational Objective: Manage hepatic encephalopathy.
This patient should undergo his first screening colonos-
Discontinuing alprazolam is the most appropriate next step copy at age 40 years. Based on the patient's family history,
in the management of this patient. Hepatic encephalop- he is at increased risk for colon cancer. Individuals with a
athy is a significant, potentially reversible, complication first-degree relative with colon cancer or an advanced ade-
of cirrhosis, with cognitive Impairment ranging from mild noma diagnosed at an age younger than 60 years, or two or
personality changes to overt coma. Hepatic encephalopathy more first-degree relatives with colon cancer or advanced

137
Answers and Critiques

adenoma diagnosed at any age, should begin colon cancer Patients meeting the Milan criteria have excellent S-year
screening at age 40 years (or 10 years earlier than the young- survival rates after liver transplantation.
est age at which colon cancer was diagnosed in a first-degree Biopsy of the lesion ls not indicated In this patient. In
relative, whichever is first). Colonoscopy, rather than Imaging the context of cirrhosis, a lesion larger than I cm with con-
or stool-based testing, is the recommended screening modal- trast enhancement in the arterial phase and portal venous
ity. If colonoscopy is performed and findings are normal, the washout meets radiologic criteria for hepatocellular car-
recommended interval for repeat screening is S years if the cinoma and, therefore, does not require a lesion biopsy.
first-degree relative was younger than age 60 years at the time Additionally, there is potential for harm from a lesion biopsy
of diagnosis and 10 years if the first-degree relative was age due to coagulopathy or the very small risk for tumor seeding.
60 years or older at the time of diagnosis. Sorafenib is a multikinase inhibitor that is reserved for
Although age 42 years is 10 years earlier than the age at patients with advanced hepatocelJular carcinoma with vas-
which his father was diagnosed with colon cancer, guide- cular invasion or extrahepatic spread that is not amenable
lines recommend starting colon cancer screening at age to surgery, liver transplantation, or locoregional therapies.
40 years or 10 years earlier than the family member's diag- Treatment of these patients with sorafenib confers a survival
nosis, whichever comes first. Therefore, age 40 years is the benefit. This patient is not a candidate for resection and
appropriate age at which to begin screening in this patient. should be evaluated for liver transplantation; sorafenib is
In average-risk patients, SO years is the recommended not indicated.
age to begin screening for colon cancer. Because of this Surgical resection would be dangerous for this patient,
patient's family history, waiting until age SO years to begin given the evidence of portal hypertension, which confers
screening for colon cancer ls not appropriate. increased risk for intraoperative bleeding as well as risk for
postoperative liver failure.
KEY POINT
• Individuals with a first-degree relative with colon KEY POINT
cancer or an advanced adenoma diagnosed at an age • Patients with cirrhosis and who meet the Milan criteria
younger than 6 0 years, or two or more first-degree (up to three hepatocellular carcinoma tumors :S3 cm or
relatives with colon cancer or advanced adenoma one tumor :SS cm) are best treated with liver transplan-
diagnosed at any age, should begin colon cancer tation and have excellent 5-year survival rates.
screening at age 40 years (or 10 years earlier than the
youngest age at which colon cancer was diagnosed in Bibliography
a first-degree relative, whichever is first). Heimbach J, Kulik LM, finn R, Sirlin CB, Abecassls M, Roberts LR, et al.
AASLD guidelines for the treatment of hepatocellular carcinoma.
Hepatology. 2017. [PMID: 28130846] doi:l 0.1002/hep.29086
Bibliography
Lieberman DA, Rex DK, Winawer SJ, Giardiello FM, Johnson DA, Levin TR;
United States Multi-Society Task Force on Colorectal Cancer. Guidelines
for colonoscopy surveillance after screening and polypectomy: a t~msen­ Item 70 Answer: C
sus update by the US Multi-Society Task Rlrce on Colorectal Cancer.
Gastroenterology. 2012;143:844-57. [PMID: 22763141] doi:l0.1053/j.gas- Educational Objective: Diagnose eosinophilic
tro.2012.06.001 esophagitis.
Eosinophilic esophagitis is the most likely diagnosis in this
Item 69 Answer: B patient. Defined as esophageal squamous mucosal inflam-
mation caused by eosinophilic infiltration, eosinophilic
Educational Objective: Treat hepatocellular carcinoma
esophagitis is commonly seen in young men presenting
with liver transplantation.
with symptoms of dysphagia. It often co-occurs in patients
Referral for liver transplantation is the most appropriate with food allergies, asthma, and eczema. Rings and furrows
next step In management for this patient. A diagnosis of in the esophagus are common findings on upper endoscopy.
hepatocellular carcinoma can be made in a patient with The diagnosis is confirmed with biopsies of the esophagus
cirrhosis in the presence of lesions larger than 1 cm that showing more than lS eosinophils/hpf. Treatment begins
enhance in the arterial phase and have washout of contrast with a trial of a proton pump inhibitor (PPI) for a period of
in the venous phase. This patient meets the Milan criteria 2 months. Clinical improvement with a PPi indicates gas-
(up to three hepatocellular carcinoma tumors S3 cm or one troesophageal reflux disease- associated eosinophilia rather
tumor :SS cm) for liver transplantation. Patients who meet than eosinophilic esophagitis. In this case, repeat upper
the Milan criteria and have a tumor 2 cm or larger with endoscopy with biopsies should be considered; if esophageal
arterial enhancement and venous washout on CT or MRI eoslnophilia is still present, the diagnosis of eosinophilic
are eligible to receive Model for End-Stage Liver Disease esophagitis is confirmed. For patients with confirmed eosin-
exception points, placing them at a higher priority for liver ophilic esophagltis, treatment Includes restriction of dietary
transplantation. elements (elemental diet or targeted elimination diet) and
Liver transplantation is the only curative therapy for swallowed aerosolized topical glucocorticoids. Patients
hepatocellular carcinoma and for end-stage liver disease. should also be counseled that the condition may recur.

138
Answers and Critiques

This patient's endoscopic findings are not consistent with ous complications of gallstone disease are preceded by an
a diagnosis of achalasia. In patients with achalasia, endoscopy episode of biliary colic; therefore, cholecystectomy is not
often shows a dilated esophageal body with resistance at the generally advised in asymptomatic patients. Indications for
gastroesophageal junction. A manometry test confirms the cholecystectomy include symptomatic disease such as bil-
diagnosis of achalasia. 1he most effective treatment options iary colic or cholecystitis. Prophylactic cholecystectomy is
include pneumatic dilation and surgical myotomy. recommended for patients with an anomalous pancreatico-
Candida esophagitis rarely occurs in immunocompe- biliary duct junction, gallbladder polyps 1 cm or larger, gall-
tent patients but is seen more often in patients with HIV bladder polyp(s) with concomitant gallstones, or polyps of
and other immunocompromised patients. The most com- any size in the setting of primary sclerosing cholangitis. Pro-
mon symptom is odynophagia rather than dysphagia. Oral phylactic cholecystectomy can also be considered in patients
thrnsh may also be present, but its absence does not exclude with a porcelain gallbladder or with gallstones larger than
esophageal involvement. Endoscopic findings in patients 3 cm. These patients are at increased risk for gallbladder
with Candida esophagitis are small, raised, white plaques. cancer. In this patient with small, asymptomatic gallstones,
It is treated with oral fluconazole and management of the there is no indication for surgical intervention.
immunocompromised state. Referral for endoscopic retrograde cholangiopancrea-
Because this patient takes only a multivitamin and has tography (ERCP) is not warranted in this setting because the
no ulcers seen on endoscopy, pill-induced esophagitis is patient is asymptomatic. Patients with gallstones in the com-
unlikely. Pill-induced esophagitis has been observed with mon bile duct or patients who are not candidates for chole-
medications including alendronate, quinidine, tetracycline, cystectomy may benefit from ERCP with sphincterotomy to
doxycycline, potassium chloride, ferrous sulfate, and mex- reduce the risk for recurrent bouts of cholangitis.
iletine. Pills typically cause local injury at sites of anatomic Serial abdominal imaging, such as ultrasonography, to
narrowing of the esophagus. Clinical symptoms include monitor asymptomatic gallstones is not warranted. Patients
chest pain, dysphagia, and odynophagia. Symptoms may with gallbladder polyps may benefit from monitoring with
begin hours to days after starting therapy, and stopping the serial gallbladder ultrasound because gallbladder polyps are
medication often leads to symptom relief. a risk factor for gallbladder cancer. Diagnosis at a late stage
contributes to the poor prognosis of gallbladder cancer.
KEY POINT
• Eosinophilic esophagitis typically presents in young KEY POI NT
men with symptoms of dysphagia and in patients • Incidentally found gallstones with no associated
with a history of food allergies. eczema, and asthma. symptoms and no complications require no further
intervention.
Bibliography
Dellon ES, Gonsalves N, Hirano l, Furuta GT, Liacouras CA, Katzka DA; Bibliography
American College ofGastroenterology. ACG clinical guldellne: evidenced Warttig S, Ward S, Rogers G; Guideline Development Group. Diagnosis and
based approach to the diagnosis and management of esophageal eosino- management of gallstone disease: summary or NICE guidance. BMJ.
phllia and eoslnophilic esophagitis (EoE). Am J Gastroenterol. 2014;349:g6241. [PMID: 25360037] doi:10.1136/bmj.g6241
2013;108:679-92; quiz 693. [PMlD: 235673571 dol:l0.1038/ajg.2013.71

Item 71 Answer: D
Educational Objective: Manage asymptomatic
It em 72 Answer: A
Educational Objective: Diagnose aortoenteric fistuJa.
Cl
gallstones.
A CT scan with intravenous contrast is the most appropriate
This patienfs asymptomatic gallstone disease requires no next test for this patient. The patient presents with the classic
further intervention at this time but should be managed "herald bleed" of aortoenteric fistula: a brisk bleed associated
as symptoms arise. Gallstone disease is a common finding with hypotension that stops spontaneously and then is fol-
in up to 15% of patients in Western countries. Gallstones lowed later by massive gastrointestinal hemorrhage. An aor-
that are found incidentally during abdominal imaging per- toenteric fistula is a communication between the aorta and the
formed for another reason do not usually require interven- gastrointestinal tmct, most commonly located in the distal duo-
tion. Stones that produce symptoms typical of biliary colic denum, especially the third portion, because the duodenum is
or cause symptoms due to passage of the stone into the fixed and located just anterior to the aorta. The possibility of an
common bile duct require intervention. This patient has aortoenteric fistula must be considered in a patient with pre-
no symptoms potentially attributable to gallstones, and the vious aortic graft surgery who presents with gastrointestinal
ultrasonographic findings do not suggest any complications bleeding. It is a life-threatening condition, with a mortality
of gallstones. Because there are no complications, no inter- rate of 50% even with surgical intervention. In this setting,
vention is necessary at this time and clinical observation is the aortoenteric fistula is most commonly due to graft infec-
the appropriate management. tion, and associated fever and leukocytosis occurs. When
Eighty percent of patients with asymptomatic gallstones there is a high degree of suspicion for aortoenteric fistula, CT
remain asymptomatic over a 15-year period, and most seri- with intravenous contrast should be performed before other

139
Answers and Critiques

types of gastrointestinal evaluation because CT can be per-


CJ formed promptly and is noninvasive. CT can reveal evidence
thrombosis. This patient should be evaluated first with colo-
noscopy before using a more invasive treatment strategy.
CONT. of graft infection, such as perigraft soft-tissue thickening Radiographic techniques such as tagged red cell scin-
or loss of tissue planes, and its reported sensitivity for aor- tigraphy may be useful in evaluating overt gastrointestinal
toenteric fistula is 80% or greater. bleeding from an unknown source. Nuclear scans can iden-
Mesenteric angiogram can detect bleeding rates as slow tify only a general area where bleeding is occurring; they
as 1 mLJmin compared with 0.2 mUmin for tagged red blood cannot offer accuracy or intervention. Follow-up studies
cell scintigraphy; however, given the intermittent nature of after a positive scan can include repeat endoscopy or angiog-
bleeding from an aortoenteric fistula, mesenteric angiogram raphy; both can offer more accurate localization and therapy.
and tagged red blood cell scintigraphy are rarely helpful In Nuclear scans are often done before angiography.
the diagnosis of this condition. Transfusion strategies specifically for patients with LGIB
Upper endoscopy may be performed to rule out other have not been developed. Data extrapolated from patients
more common sources of bleeding, but in this patient, the with upper gastrointestinal bleeding found that a restric-
presence of fever, leukocytosis, and bleeding points to aor- tive transfusion strategy with a transfusion threshold of
toenteric fistula as the most likely diagnosis, and CT angio- hemoglobin less than 7 g/dl (70 g/L) improved survival and
gram is the most urgently needed test. decreased rebleeding when compared with a threshold of
9 g/dL (90 g/L) . Patients with massive bleeding, acute cor-
KEY POINT
onary syndrome, symptomatic peripheral vascular disease,
• Gastrointestinal bleeding occurring in patients follow- or a history of cerebrovascular disease were excluded from
ing aortic graft surgery should raise the possibility of these studies and may benefit from a more lenient transfu-
aortoenteric fistula; CT with contrast is the initial test sion strategy.
in appropriate patients.
KEY POINT

Bibliography • Colonoscopy is the recommended initial diagnostic


Singh M, Koyfman A. Martinez JP Abdominal vascularc.-ata~trophes. Emerg test after hemodynamic resuscitation in most patients
Med Cl!n North Am. 2016;34:327-39. [PMID: 27133247] dol:l0.1016/ with significant lower gastrointestinal bleeding.
j.emc.2015.12.014

Bibliography
c:J Item 73 Answer: C
Strate LL, Gralnek IM. ACG clinical guideline: management of patients with
acute lower gastrointestinal bleeding. Am J Gastroenterol. 2016;111:459-
74. [PMID: 26925883) doi:l0.1038/ajg.2016.41
Educational Objective: Manage lower gastrointestinal
bleeding.
Colonoscopy within 24 hours with adequate bowel prepa-
ration is the most appropriate next step in management.
Almost 80% of lower gastrointestinal bleeding (LGIB) is due
Item 74 Answer: C
Educational Objective: Treat uncomplicated
c:J
diverticulitis.
to diverticulosis, colitis, hemorrhoids, or postpolypectomy
bleeding. LGIB typically stops within 24 hours. Colonoscopy The patient has uncomplicated diverticulitis that can be
is the recommended initial diagnostic test after hemody- managed with oral antibiotics. Approximately 5"/o of patients
namic resuscitation in most patients with significant LGIB. with diverticula will experience an episode of diverticulitis.
LGIB typically occurs in older individuals and presents Symptoms include abdominal pain, fever, and altered bowel
as acute bright red blood per rectum or red- or maroon- habits. Physical examination findings include fever, left-
colored stool (hematochezia). Colonoscopy is able to identify lower-quadrant tenderness, and/or a lower abdominal or
the source of bleeding in two thirds of patients. The Amer- rectal mass. If clinical features are highly suggestive of diver-
ican College of Gastroenterology's 2016 guidelines for LGlB ticulitis, imaging studies are unnecessary. If the diagnosis
recommend oral bowel preparation to increase colonos- is not clear or if an abscess is suspected (severe pain, high
copy's diagnostic yield. Randomized controlled trials have fever, palpable mass), CT imaging is indicated. Although the
not shown a benefit in clinical outcomes or cost with rapid use of a 7-day course of oral antibiotics does not shorten the
bowel preparation and colonoscopy within 8 to 12 hours clinical course of diverticulitis, it may prevent complications
compared with a standard oral bowel preparation and colo- from diverticulitis, such as an abscess, and may help prevent
noscopy within 24 hours for patients with LGIB. a future episode of diverticulitis. The antimicrobial agents
For patients who continue to bleed and have failed used should cover colonic organisms and include anaer-
endoscopic hemostasis treatments (for example, electroco- obic coverage (such as ciprofloxacin and metronidazole) .
agulatlon, hemodips, submucosal epinephrine injection), Treatment can be done on an outpatient basis with close
the next therapeutic step is arterial embolization of the monitoring, and the patient should adhere to a liquid diet
bleeding source. Major complications include contrast dye for 2 to 3 days, then advance the diet as tolerated. The routine
reactions, acute kidney injury, transient ischemic attack, use of antibiotics In uncomplicated diverticulitis has been
bowel ischemia, hematoma formation, and femoral artery questioned because of overuse of antibiotics.

140
Answers and Critiques

Cl ulitisColonoscopy should not be done during acute divertic-


due to the risk for perforation. In patients who have not
acceptable for patients with low-grade dysplasia. Because
this patienfs Barrett esophagus is indefinite for dysplasia,
CONT. undergone screening colonoscopy within the past 3 years, endoscopic ablation therapy is inappropriate.
colonoscopy should be done 4 to 8 weeks after resolution of Esophagectomy is considered for patients with a diag-
symptoms to exclude colon malignancy. nosis of esophageal cancer.
Intravenous antibiotics are appropriate in patients A repeat endoscopy at 1 year without optimizing medi-
who cannot take oral medications or who have complicated cal therapy for GERD is not recommended by current guide-
diverticulitis, such as abscess or fistula formation. Fistu- lines. Patients with nondysplastic Barrett esophagus should
las may form between the colon and other adjacent struc- undergo a repeat surveillance endoscopy in 3 to 5 years.
tures including the bladder, vagina, skin, or peritoneum. Patients with Barrett esophagus with low-grade dysplasia
CT-guided percutaneous drainage of abscesses is preferred. should undergo surveillance endoscopy in 6 to 12 months
Surgery is required for diverticulitis complicated by to detect prevalent dysplasia, and dysplasia should be con-
peritonitis and may be lifesaving. Patients with recurrent firmed by an expert pathologist.
diverticulitis may also benefit from surgical resection of the
KEY POINT
affected area of the colon.
• Patients whose Barrett esophagus is indefinite for dys-
KEY POINT plasia should begin optimized antisecretory medical
• Patients with uncomplicated diverticulitis should be therapy and undergo a repeat endoscopy
treated conservatively with oral antibiotics.
Bibliography
Bibliography Shaheen NJ, Falk GW, Iyer PG, Gerson LB; American College of
Strate LL, Peery AF, Neumann r. American Gastroenterologlcal Association Gastroenterology. ACJ:, Clinical Guideline: Diagnosis and Management of
Institute technical review on the management of acute diverticulitis. Barrett's Esophagus. Am J Gastroenterol. 2016;111:30-50; quiz 51. [PMID:
Gastroenterology. 201S;l49:1950-1976.el2. [PMID: 26453776] doi: 26526079]
10.1053/j.gastro.2015.J0.001

Item 76 Answer: A
Item 75 Answer: C Educational Objective: Evaluate fecal incontinence
Educational Objective: Manage a patient with Barrett in an elderly patient.
esophagus that is indefinite for dysplasia.
An abdominal radiograph is the most appropriate next step
The most appropriate next step in managing this patient is to in management of this patient. Fecal incontinence is the
optimize medical therapy and then repeat upper endoscopy. involuntary loss ofstool. Prevalence increases with age and is
This patient's Barrett esophagus is indefinite for dysplasia, 16% in adults older than 70 years. Urge fecal incontinence is
which falls between nondysplastic Barrett esophagus and the inability to postpone defecation. This patient's explosive
low-grade dysplasia. In patients whose Barrett esophagus is diarrhea is the likely source of her urge fecal incontinence.
indefinite for dysplasia, medical therapy for gastroesophageal Before treating the diarrhea, it is essential to determine
reflux disease (GERD) should be optimized (In this patient, whether the diarrhea is due to overflow from fecal loading
proton pump inhibitor therapy would increase to twice (excess stool in the colon). An abdominal radiograph is a
daily) and then the patient should receive a repeat upper simple, safe, and inexpensive diagnostic test. Fecal loading
endoscopy. Guidelines do not specify a time for repeating the with resultant overflow diarrhea is a common cause of fecal
endoscopy; 6 months to 1 year after antisecretory therapy incontinence in elderly patients, particularly those who are
is optimized would be reasonable. If the repeat endoscopy hospitalized or have degenerative neurologic disorders. This
still shows Barrett esophagus that is indefinite for dysplasia, patient has several risk factors for developing fecal loading,
the patient should continue medical therapy and undergo including Alzheimer dementia, history of constipation, and
a repeat endoscopy in 1 year. If the endoscopy is normal at use of constipating medications (memantine and calcium).
1 year, a surveillance interval of 3 to 5 years can be resumed. Anorectal manometry is useful in the diagnostic evalu-
The patient has a history of GERO, which can lead to ation of fecal incontinence to help assess for anal sphincter
Barrett esophagus. Other risk factors for Barrett esophagus weakness, rectal hypersensitivity, and/or dyssynergic def-
include male gender, age, current or past smoking. central ecation. Due to its limited availability, cost, and logistical
obesity, and white race. Barrett esophagus is a precancerous considerations for the patient, its use is reserved for patients
condition that is classified with five levels of histologic find- with rectal findings suggestive of anal sphincter weakness
ings, from no dysplasia to esophageal cancer. It may progress or dyssynergia, or patients with a lack of response to initial
from intestinal metaplasia to low-grade dysplasia to high- treatment measures.
grade dysplasla to invasive adenocarclnoma. A trial of loperamide may alleviate this patient's diar-
Endoscopic ablation therapy is indicated for patients rhea in the short term; however, lopemmide use will add to
with Barrett esophagus with low-grade or high-grade dys- the underlying fecal loading, causing her diarrhea and fecal
plasia, although continued endoscopic surveillance is also incontinence to worsen over time.

141
Answers and Critiques

Use of psyllium is unlikely to alleviate the patient's development of gastrointestinal symptoms, 1 day after pre-
symptoms and would add stool bulk to an already over- sentation with hypotension secondary to urosepsis, makes
loaded colon, which would likely worsen her urgency, the diagnosis ofischemic colitis more likely.
explosive diarrhea, and fecal incontinence. Furthermore,
KEY POINT
the patient is likely to experience bloating and abdominal
distention caused by the fermentation of the psyllium in the • lschemic colitis is a low-flow state of the colon occur-
gastrointestinal tract. ring most frequently in the left colon and character-
ized by moderate, left-sided, cramping abdominal
KEY POINT
pain followed by bloody diarrhea.
• Fecal loading (excess stool in the colon) with resultant
overflow diarrhea is a common cause of fecal inconti-
Bibliography
nence in elderly patients, particularly those who are
Trotter JM. Hunt L, Peter MB. lschaemic colitis. BMJ. 2016;355:16600. IPMID:
hospitalized or have degenerative neurologic disorders. 28007701] doi:l0.1136/bmj.16600

Bibliography
Yu SW, Rao SS. Anorectal physiology and pathophysiology in the elderly. Clin
GeriatrMed.2014;30:95-106. [PMID: 24267605]doi:I0.10161j.cger.2013.10.003 Item 78 Answer: B
Educational Objective: Treat ileal Crohn disease.

c::J Item 77 Answer: D Infliximab is the most appropriate treatment for this
patient. This patient has moderate to severe ileal Crohn
Educational Objective: Diagnose ischemic colitis.
disease that has required multiple courses of tapering
lschemic colitis is the most likely diagnosis in this patient. prednisone for flares of disease over the last year, despite
The patient initially presented in septic shock from a u ri- treatment with the immunomodulator azathioprine.
nary tract infection. He was hypotensive and required lnfliximab is an anti- tumor necrosis factor (TNF) -a
intravenous fluids and vasopressor support, which are risk antagonist effective in inducing and maintaining remis-
factors for ischemic colitis in elderly patients. Ischemic sion in moderate to severe Crohn disease. Other FDA-
colitis is a low-flow state of the colon occurring most fre- approved anti-TNF agents include adalimumab and cer-
quently in the left colon. Conditions that can alter circula- tolizumab pegol. Evidence indicates that efficacy is better
tion include hypotension, dehydration, strenuous physical when an anti-TNF agent Is used together with an immu-
activity, medications and illicit drugs, thrombophilia, aor- nomodulator. In addition, the risk for developing anti-
tic or cardiac bypass, vasculitis, or an obstructing colon bodies against the anti-TNF agent is lower with combi-
lesion. Diarrhea with rectal bleeding is common. Colo- nation therapy. Patients whose disease does not respond
noscopy may show sharply demarcated pale mucosa with to one anti-TNF agent are often switched to a second or
petechiaJ bleeding, as it does in this patient. Left colon third anti-TNF agent. Fibrostenosing Crohn disease in the
inflammatory changes may be detected by colonoscopy or absence of ongoing mucosa! inflammation is unlikely to
abdominal CT. Treatment is supportive care with normal- respond to any anti-TNF agent. Patients with no response
ization of blood pressure. to or intolerance of anti-TNF agents should be treated
Acute mesenteric ischemia (AMI) is an uncommon with either surgery or a leukocyte trafficking blocker
vascular emergency. Embolism to the mesenteric arteries (natalizumab or vedolizumab) .
causes 50% of cases of AMI. Most emboli are from cardiac Budesonide is a potent glucocorticoid with high first-
sources. Patients typically present in the seventh decade of pass metabolism in the liver, which limits systemic side
life and often have associated cardiovascular comorbidities. effects related to conventional glucocorticoids. Budesonide
The classic presentation of early AMI is central abdominal can be an effective therapy for treating mild flares of ileal
pain out of proportion to the physical examination findings. Crohn disease, but it is unlikely to induce remission in
This patient's gradual onset ofleft-sided, moderate-intensity more severe Crohn disease and cannot be used to maintain
abdominal pain is not typical of AMI. remission.
Dlverticula represent herniation of mucosa or sub- Mesalamine agents are mainly used to treat ulcerative
mucosa through the muscular layers of the colon, typi- colitis of mild to moderate severity. Mesalamine may have
cally at the entry site ofvasa recta (small arteries), which efficacy in treating mild to moderate Crohn colitis, but it is
are a source for bleeding. Diverticular bleeding is usually not efficacious in treating small-bowel Crohn disease.
painless with passage of large-volume red- to maroon- Reinitiating prednisone may induce remission of the
colored blood per rectum. rt occurs spontaneously without patient's current disease flare, but it would not be helpful
associated infection or other illness, making it unlikely in for maintenance of remission. Because this patient has
this patient. required three separate tapering doses of glucocorticoids
The most common initial symptoms of enterohemor- over the last year, she requires a medication such as inflix-
rhagic Escherichia coli are bloody diarrhea and abdomi- imab that can both induce remission and maintain Crohn
nal tenderness, without fever. The timing of this patient's disease in remission.

142
Answers and Critiques

KEY POINT Screening starting at age 40 years or 10 years earlier


• Anti - tumor necrosis factor agents such as infliximab than the youngest age of colon cancer diagnosis in the family
are effective in inducing and maintaining remission in is appropriate for individuals with a family history of colon
cancer that does not meet crite1ia for Lynch syndrome.
moderate to severe Crohn disease.
KEY POINT
Bibliography • Patients with Lynch syndrome should begin screening
Lichtenstein GR. Hanauer SR, Sandborn WJ; Practice Parameters Committee colonoscopy between ages 20 and 2S years or 2 to
of American College ofGastroentemlogy. Management of Crohn's disease
in adults. Am J Gastroenterol. 2009;104:465-83; quiz ~64. 484. [PMID: S years before the earliest age of colorectal cancer
19174807] dol:I0.1038/ajg.2008.168 diagnosis in the family, whichever comes first, and
colonoscopy should be repeated every 1 to 2 years if
Item 79 Answer: D the baseline examination is normal.
Educational Objective: Manage Lynch syndrome.
Bibliography
This patient should undergo a screening colonoscopy now, Rubenstein JH, Enns R, Heidelbaugh J, Barkun A; Clinical Guidelines
and if it is normal, have a repeat colonoscopy every 1 to Committee. American Gastroenterological Association Institute guide-
line on the diagnosis and management of lynch syndrome.
2 years until age 40 years, and then yearly thereafter. The Gastroenterology. 2015;149:777-82; quiz e16-7. (PMID: 26226577]
patient has Lynch syndrome based on her family history doi:l0.1053/j.gastro.2015.07.036
and the identification of a deleterious mutation in the MSH2
gene. The Amsterdam II criteria (known as the "3-2-1-1-0
Item 80 Answer: D
rule") can be used to screen for Lynch syndrome; a diagnosis
is warranted if the following criteria are met: Educational Objective: Diagnose hereditary diffuse
gastric cancer.
• Three family members are affected with a Lynch
syndrome- associated cancer Upper endoscopy is the most appropriate next test for this
patient. The patient's presentation is suggestive of gastric
• At least two successive generations are affected
cancer, specifically diffuse gastric cancer. Proposed crite-
• One affected family member is a first-degree relative of ria for selection of patients for genetic testing for hered-
the other two affected family members itary diffuse gastric cancer include the following: family
• One of the cancers was diagnosed before age SO years members with two or more documented cases of gastric
cancer in first- or second-degree relatives, with at least
• Familial adenomatous polyposis has been excluded
one diffuse gastric cancer diagnosed before age so years;
• Tumors have been verified histologically family members with multiple lobular breast cancers with
The Amsterdam criteria are specific for Lynch syndrome or without diffuse gastric cancer in first- or second-degree
but lack sensitivity. Additional clinical tools, such as the relatives; and, a personal diagnosis of diffuse gastric cancer
Bethesda criteria or clinical models such as the PREdiction before age 35 years from a low-incidence population such
Model for gene Mutations S (PREMM5) (premm.dfci.harvard. as in Canada and the United States. Based on the patient's
edu), may be used to screen for Lynch syndrome. young age and history of multiple family members with
Lynch syndrome is caused by germline mutation in one of gastric and lobular breast cancer, hereditary diffuse gastric
the DNA mismatch repair genes (MLHl, MSH2, MSH6, PMS2) cancer is a likely diagnosis. The best diagnostic test for gas-
or the epithelial cell adhesion molecule (EPCAM) gene. The tric cancer is an upper endoscopy with multiple biopsies
appropriate age to begin screening colonoscopy in patients of the stomach. The syndrome of hereditary diffuse gastric
with Lynch syndrome is between ages 20 and 2S years or 2 to cancer is associated with mutations in the CDHl gene. The
5 years before the earliest age of colorectal cancer diagnosis risk for diffuse gastric cancer approaches 80% in carriers
in the family, whichever comes first; thus, this patient should of the mutation, and prophylactic gastrectomy is recom-
undergo a colonoscopy now. The risk for colorectal cancer in mended in mutation carriers who have not developed
patients with Lynch syndrome is elevated significantly com- gastric cancer.
pared with the general population and is as high as 80% A colonoscopy is not indicated as the initial test
for some individuals. Women with Lynch syndrome are at because the patient has primarily upper gastrointestinal
increased risk for endometrial cancer. Additional cancers, such symptoms. Colon polyps and cancer have been associated
as gastric, small intestinal, urothelial, ovarian, and pancreati- with hereditary diffuse gastric cancer, and more frequent
cobiliary cancers, are also associated with this syndrome. screening colonoscopy for colon polyps is indicated in
Colectomy and continued annual endoscopic surveil- mutation carriers.
lance of the remaining rectum is the recommended man- Delayed gastric emptying can present with abdominal
agement for patients with Lynch syndrome who are found to pain and early satiety but is also often accompanied by
have colorectal cancer on screening colonoscopy. Colectomy nausea and vomiting. The patient has no risk factors for gas-
is not a recommended option for managing patients with troparesis, such as diabetes mellitus, so a gastric emptying
Lynch syndrome in the absence of a documented cancer. study is not indicated.

143
Answers and Critiques
- -

While Helicobacter pylori can present with dyspeptic include hepatitis, cholestasis, steatosis, or granulomas. This
symptoms, serological testing is not the most appropriate patient has a classic presentation, and a cholestatic bio-
next step for this patient because his weight loss and family chemical profile and negative antibody tests make autoim-
history raise concern for gastric cancer. mune hepatitis unlikely.
An upper gastrointestinal radiograph series might show No specific antidotes are available for idiosyncratic
thickened gastric folds related to diffuse gastric cancer, but it drug-induced liver injury. The administration of glucocor-
cannot be used to obtain biopsies to diagnose cancer. ticoids like prednisone is not indicated unless there is sus-
picion for drug hypersensitivity reaction. The absence of a
KEY POINT
morbilliform rash, fever, facial swelling, lymphadenopathy,
• A history of multiple family members with gastric and substantially elevated aminotransferase levels make a
cancer, particularly before age SO years, or multiple hypersensitivity reaction unlikely and prednisone unneces-
family members with lobular breast cancer with or sary in this patient.
without gastric cancer, suggest the possibility of
KEY POINT
hereditary diffuse gastric cancer and the need for
upper endoscopy and testing for mutations of the • In patients with well-preserved liver function, drug-
CDH1gene. induced liver injury should be managed with discon-
tinuation of the offending medication and observation
Bibliography until resolution of symptoms occurs.
Syngal S, Brand RE. Church JM, Giardiello FM, Hampel HL. Burt RW;
American College of Gastroenterology. ACG clinical guideline: genetic Bibliography
testing and management of hereditary gastrointestinal cancer syn-
dromes. Am J Gastroenterol. 2015:110:223-62; quli.263. [PMID: 25645574] Chalasani NP, Hayashi PH, Bonkovsky HL, Navarro VJ, Lee WM, Fontana RJ;
dol: 10.1 038/ajg.2014.435 Practice Parameters Committee of the American College of
Gastroenterology. ACG clinical guideline: the diagnosis and management
of Idiosyncratic drug-Induced liver injury. Am J Gastroenterol.
2014;109:950-66; quiz 967. [PMID: 24935270] dol:10.1038/ajg.2014.131

Item 81 Answer: D
Educational Objective: Manage drug-induced liver
injury. Item 82 Answer: C
Educational Objective: Treat centrally mediated
Continued observation is the most appropriate management
abdominal pain syndrome.
of this patient. Drug-induced liver injury is a rare adverse
reaction to medication that can result in jaundice, liver fail- Cognitive-behavioral therapy is the most appropriate treat-
ure, and, potentially; death. lhis patient meets the criteria ment for this patient. She meets the diagnostic criteria
for diagnosis, which requires a history of drug or supple- for centrally mediated abdominal pain syndrome (CAPS):
ment exposure within 6 to 12 months, a biochemical pattern near-constant abdominal pain lasting longer than 6 months,
that fits the hepatotoxicity profile of the causative agent (in involving a large anatomic distribution, and without initi-
thls case, cholestatic for amoxiclllin-clavulanate), improve- ating triggers or alarm features. Treatment includes a com-
ment after drug removal (dechallenge), and the absence bination of pharmacologic and/or psychological therapies.
of underlying liver or biliary diseases. The most common She already takes a selective serotonin reuptake inhibitor
medications associated with drug-induced liver injury are for depression, but it has not improved her abdominal pain
antibacterial agents (especially amoxicillin-clavulanate) as syndrome. Four classes of psychotherapy have shown ben-
well as herbal and dietary supplements. In this patient with efit in patients with CAPS when combined with medical
drug-induced liver injury and well-preserved liver function, therapy: cognitive-behavioral therapy, psychodynamic-in-
the offending medication has already been discontinued, terpersonal therapy, mindfulness- and acceptance-based
and observation until resolution of symptoms occurs is the therapy, and hypnotherapy.
best course of action. The prognosis of drug-induced liver Irritable bowel syndrome (lBS) is a symptom complex
injury is generally good, with 70% of patients recovering characterized by abdominal pain and altered bowel habits.
without needing hospitalization and 90'Yo recovering with- The diagnosis is made by the fulfillment of symptom-based
out developing acute liver failure. criteria, including the presence of recurrent abdominal pain
Endoscopic retrograde cholangiography (ERCP) is used or discomfort at least 3 days per month in the last 3 months
to treat biliary obstruction. In this setting, ultrasound has that is associated with two or more of the following: relief
confirmed the absence of biliary dilation, and therefore, with defecation, onset associated with a change in frequency
ERCP would not be expected to provide benefit. of stool, or onset associated with a change in form of stool.
A liver biopsy is rarely necessary in patients with JBS is further subtyped based on the predominant stool pat-
drug-Induced liver injury but is helpful in cases of uncer- tern as JBS with predominant constlpatlon (IBS-C), IBS with
tainty or suspected drug-induced autoimmune hepatitis. predominant diarrhea (IBS-D), mixed IBS, or IBS unclassi-
Zone 3 necrosis and eosinophilia are classic histologic find- fied. CAPS differs from IBS in that there are no consistent
ings in drug-induced liver injury, but other findings may initiating triggers and symptoms are not alleviated with

144
Answers and Critiques

bowel movements. The patient reports no constipation, and patients with cardiovascular disease. It may be reasonable
her pain is not relieved with bowel movements, so IBS is an to give transfusions to patients who are hemodynamically
unlikely diagnosis. unstable before a decline in hemoglobin level to less than 7
Alosetron is a 5-HTi antagonist available for women g/dL (70 g/L) to prevent complications of tissue underper-
with IBS-D whose symptoms have not responded to con- fusion. There is uncertainty regarding the need for a higher
ventional therapy; however, owing to an increased risk for transfusion threshold in patients with cardiovascular disease,
severe constipation and ischemic colitis with its use, it is but current guidelines recommend considering transfusion
restricted by a mandatory prescribing program. Because this when hemoglobin levels decrease below 8 g/dL (80 g/ L) or
patient does not have TBS, alosetron is not indicated. when cardiovascular symptoms develop (for example, chest
Budesonide is indicated in patients with inflammatory pain, dyspnea) in patients who are otherwise hemodynam-
bowel disease. This patient reports no chronic diarrhea, ically stable.
a hallmark symptom of inflammatory bowel disease, and
KEY POINT
a colonoscopy earlier this year was normal, showing no
inflammation. • In patients with upper gastrointestinal bleeding, a
Linaclotide is a guanylate cyclase-C activator used in the restrictive transfusion strategy (transfusion threshold
treatment of JBS-C after fiber supplementation or osmotic of less than 7 g/dL [70 g/ L] with a target hemoglobin
and stimulatory laxative therapy fails. It has no role in the level of7-9 g/dL [70-90 g /L)) is associated with
treatment of CAPS. decreased mortality, length of hospital stay, and trans-
fusion -related adverse events compared to a liberal
KEY POINT
transfusion strategy.
• Centrally mediated abdominal pain syndrome is
characterized by near-constant abdominal pain last- Bibliography
ing longer than 6 months, involving a large anatomic Fortin sky KJ, Bardou M, Barkun AN. Role of medical therapy for nonvariceal
distribution, and without initiating triggers or alarm upper gastrointestinal hleeding. Gastrointest Endosc Clin N Am.
2015;25:463-78. [PMID: 26142032] dol:l0.1016/j.giec.2015.02.003
features.

Bibliography Item 84 Answer: C


Keefer L, Drossman DA. Guthrie E, Slmren M, Tillisch K. Olden K, et al.
Centrally mediated disorders of gastrointestinal pain. Gastroenterology. Educational Objedive: Screen for duodenal cancer in a
2016. [PMIO: 27M4628J doi:l0.1053/J.gastro.2016.02.034 patient with familial adenomatous polyposis syndrome.
Upper endoscopy with duodenoscopy is the most appro-
l:J Item 83 Answ er: D priate next test for this patient The adenomatous polyposis
syndromes include familial adenomatous polyposis (FAP)
Educational Objective: Treat upper gastrointestinal
and MutYH-associated polyposis (MAP). Patients with FAP
bleeding.
and MAP should receive genetic counseling and should be
This patient does not reqllire transfusion. For patients with offered genetic testing. Although an autosomal dominant
upper gastrointestinal bleeding, initial resuscitation is the disorder, a positive family history is found in only 54% to
first priority and should include stabilization of blood pres- 78% of patients diagnosed with FAP. Upper endoscopy with
sure with infusion of sufficient volumes of crystalloid fluid duodenoscopy, using both a standard upper scope and a
and/or red blood cells. The decision to transfuse red blood side viewing scope, is indicated to screen for periarnpullary
cells is based mainly on the presenting hemoglobin level. In and duodenal adenomas and adenocarcinoma. At least 50%
hemodynamically stable patients, a restrictive transfusion of patients with FAP develop adenomatous changes of the
strategy (transfusion threshold of less than 7 g/dL [70 g/L] periampullary region of the duodenum. Upper-endoscopy
with a target hemoglobin level of 7-9 g/dL [70-90 g/L]) is screening in patients with FAP should begin at the time of
associated with decreased mortality, length of hospital stay, onset of colonic polyps or at age 25 to 30 years, whichever
and transfusion-related adverse events compared to a lib- comes first. Upper endoscopy with duodenoscopy for sur-
eral transfusion strategy (transfusion threshold of less than veillance of duodenal cancer is indicated every 1 to 5 years
9 g/dL [90 g/L] with a target hemoglobin level of 9-10 g/dL at an interval based on the stage of the duodenal polyposis.
[90-100 g/L]). This patient is hemodynamically and physio- A barium upper gastrointestinal series has a low sen-
logically stable with no evidence of ongoing overt gastroin- sitivity for periampullary and duodenal adenornas, which
testinal blood loss or symptoms of tissue ischemia; therefore, may be flat or sessile lesions, and it does not allow for tissue
it is appropriate to continue maintenance intravenous fluids biopsy iflesions are found.
because he is at an appropriate target hemoglobin level of Double-balloon enteroscopy allows endoscopy to reach
7 to 9 g/dL (70-90 g/L). the entire small-bowel lumen and is used in patients with
A modification of the restrictive transfusion thresh- occult gastrointestinal blood loss to look for a source ofbleed-
old may be considered in specific subpopulations, such ing and provide therapeutic options. It may also be used
as patients with hypotension due to severe bleeding and to remove small-bowel mucosa! lesions such as adenomas

145
Answers and Critiques

if they are located beyond the reach of a standard esoph- Guidelines recommend screening average-risk individ-
agogastroduodenoscopy (EGD). Double-balloon enteroscopy uals beginning at age 50 years. Repeat colonoscopy is based
requires longer anesthesia time and is higher-risk than stan- on initial findings. Individuals with no adenomas should
dard EGD, and its availability is limited. Therefore, it is not the have a repeat colonoscopy in 10 years. Individuals who have
standard recommended procedure for initial screening for two or fewer tubular adenomas are also considered low-risk
duodenal and periampul!ary adenomas in patients with FAP. for colon cancer and should repeat screening in 5 to 10 years.
Patients with FAP should undergo upper endoscopy Appropriate follow-up recommendations after screening
because there is more than a 50% chance of discovering colonoscopy, including avoidance of inappropriate use of
duodenal adenomas at the first upper endoscopy. Conduct- colonoscopy after removal of adenomatous polyps, are core
ing no further tests in this patient would be an inappropriate gastroenterology quality measures.
approach, given the high probability of finding one or more High-risk findings include more than 10 polyps on
potentially premalignant lesions. baseline colonoscopic examination, which should prompt
consideration for a polyposis syndrome. These individuals
KEY POINT
should have a follow-up colonoscopy in less than 3 years,
• Upper-endoscopy screening for duodenal cancer in depending on polyp burden and polyposis syndrome. Any
patients with familial adenomatous polyposis should patient who has a polyp removed piecemeal should return
begin at onset of colonic polyposis or at age 25 to within 3 to 6 months for repeat colonoscopy, especially if
30 years, whichever comes first. there is any concern for incomplete polyp removal.
KEY POINT
Bibliography
Syngal S, Brand RE, Church JM, Giardlello FM, Hampel HL, Burt RW; • Patients with three or more adenomas on screening
American Coll~e of Gastroenterology. ACG clinical guideline: genetic colonoscopy should undergo surveillance colonoscopy
testing and management of hereditary gastrointestinal cancer syn-
dromes. Am J Gastroenterol. 2015;110:223- 62; quiz 263. [PMID: 25645574] in 3 years.
dol:l0.1038/ajg.2014.435
Bibliography
Lieberman DA, Rex DK, Wtnawer SJ, Giardiello FM, Johnson DA, Levin TR:
Item 85 Answer: B United States Multi-Society Tusk Force on Colorectal Cancer. Guidelines
for colonoscopy surveillance after screening and polypectomy: a consen -
Educational Objective: Manage colonoscopy sus update by the US Multi-Society Task Force on Colorectal Cancer.
surveillance following polypectomy. Gastroenterology. 2012;143:844 -57. ll'MID: 22763141) dol:l0.1053/j.gas-
tro.2012.06.001
This patient should undergo surveillance colonoscopy in
3 years. Current recommendations emphasize that an ade-
quate bowel preparation and a high-quality colonoscopy Item 86 Answer: C
that reaches and examines the cecum are required for an
Educational Objective: Treat functional dyspepsia.
adequate screening examination. Screening colonoscopy
studies demonstrate that polyps are detected in approxi- Starting a trial of a tricyclic antidepressant is the most appro-
mately 60% of average-risk individuals. The prevalence of priate next step in the management of this patient. The
polyps on average-risk screening colonoscopy is 22% to 25% patient has functional dyspepsia meeting the diagnostic cri-
for adenomas, 12'X. for hyperplastic polyps, and 0.6o/o for teria for epigastric pain syndrome: (1) bothersome postpran-
sessile serrated polyps. The degree of dysplasia in a polyp is dial fullness; (2) early satiety; (3) epigastric pain; and/or (4)
reported as high or low. Adenomatous polyps are neoplas- epigastric burning for at least 3 days per week. These criteria
tic lesions and, therefore, have malignant potential; most should be met for the 3 months leading up to diagnosis,
colorectal cancers arise from adenomatous polyps. Adeno- with symptoms starting at least 6 months before diagnosis
matous polyps are further defined by their glandular archi- and with no evidence of structural disease to explain the
tecture: tubular, villous, or a combination of both. The most symptoms. The absence of an underlying organic disease
common pattern is tubular, followed by tubulovillous, with is demonstrated by this patient's normal upper endoscopy,
the least common pattern being villous. After neoplastic including gastric and small-bowel biopsies, as well as nor-
polyps (adenomas, sessile serrated polyps, or traditional ser- mal laboratory testing and the lack of alarm features such as
rated adenomas) are completely resected, postpolypectomy vomiting, weight loss, or family history of gastrointestinal
colonoscopy should be performed; the surveillance interval malignancy. Because she tested negative for Helicobacter
depends on the size, number, and pathology of the polyp. pylori infection and her symptoms did not respond to a
This patient was found to have three tubular adenomas trial of once-daily omeprazole for a minimum of 4 weeks,
smaller than 1 cm in size. The presence of three or more ade- the recommended next step in the treatment of functional
nomas, any adenoma greater than or equal to 1 cm in sire, or dyspepsia is a trial of a tricyclic antidepressant. In the treat-
any adenoma with villous features or high-grade dysplasfa ment of functional dyspepsia, this class of antidepressants
has been associated with increased risk for metachronous was found to be more effective than other classes, including
neoplasia (multiple primary tumors developing at different selective serotonin reuptake Inhibitors or serotonin-norepi-
time intervals), warranting a 3-year surveillance interval. nephrine reuptake Inhibitors.

146
Answers and Critiques

Given this patient's absence of alarm symptoms and virologic response {cure) in more than 90% of patients, even
normal laboratory test results, further structural testing with in those who were previously and unsuccessfully treated
CT imaging or abdominal ultrasonography is likely to be of with pegylated interferon and ribavirin. In approximately
low yield; therefore, such testing is neither clinically indi- 90% of cases, eradication of HCV leads to resolution of the
cated nor cost effective. mixed cryoglobulinemia.
A gastric emptying test Is used to evaluate suspected Most of the data pertaining to the treatment of HCV
gastroparesis. Gastroparesis commonly presents with infection in patients with mixed cryoglobulinernia involve
symptoms of early satiety, postprandial fullness, nausea, treatment with pegylated interferon and ribavirin. However,
vomiting, upper abdominal pain, bloating, and weight loss; pegylated interferon and ribavirin are no longer recom-
this patient's lack of compatible symptoms makes the diag- mended for treatment of HCV infection due to adverse effects
nosis unlikely. and lower efficacy compared to interferon-free, direct-acting
There is no evidence that a higher dose of a proton antiviral therapy.
pump inhibitor performs better in the treatment of func- For severe end-organ damage from mixed cryoglobu-
tional dyspepsia than once-daily omeprazole, which did not linemia, including kidney failure, gastrointestinal vasculitis,
alleviate the patient's symptoms. rapidly progressive neuropathy, pulmonary or central ner-
vous system vasculitis, or heart failure, rituximab is a first-
KEY POINT
line agent and may be used with pulse-dose glucocorticoids.
• First-line treatment for functional dyspepsia is once- However, this patient has mild manifestations of mixed
daily omeprazole for at least 4 weeks; if symptoms do cryoglobulinemia and, therefore, does not require immuno-
not respond, a tricyclic antidepressant is the next rec- suppressive therapy with rituximab, cyclophospharnide, or
ommended treatment. glucocorticoids.
KEV POINT
Bibliography
Moayyedi PM, Lacy BE, Andrews CN. Enns RA, Howden CW, VakO N. ACG • Mixed cryoglobulinemia arising from chronic hepati-
and CAG clinical guideline: management of dyspepsia. i\m J tis C viral infection resolves after treatment and eradi-
Gastroenterol. 2017;112:988-1013. [PMID: 28631728] doi:l0.1038/
ajg.2017.154 cation of the virus.

Bibliography
Dammacco F, Sansonno D. Therapy for hepatitis C virus-related cryoglobu-
Item 87 Answer: C linemic vasculitis. N Engl J Med. 2013;369:1035-45. [PMID: 24024840]
doi:I0.1056/NEJMral208642
Educational Objective: Treat cryoglobulinemia in a
patient with chronic hepatitis C viral infection.
A combination of ledipasvir and sofosbuvir is the most Item 88 Answer: B
appropriate treatment for this patient. The patient has II,
Educational Objective: Treat microscopic colitis.
Meltzer triad-consisting of asthenia, arthralgia, and pal- 'I
pable purpura- which is the classic presentation of type II Budesonide is the most appropriate treatment for this
mixed cryoglobullnemia, a vasculitis that most often arises patient. Her primary symptom of chronic watery diarrhea,
in the context of chronic hepatitis C virus (HCV) infec- colonoscopy results showing normal-appearing mucosa,
tion. Meltzer triad is seen in less than 30% of patients, but and biopsy results revealing lymphocytic infiltration and a
nearly all patients with type II cryoglobulinemia develop subepithelial collagen band are diagnostic for collagenous
cutaneous findings, as seen in this patient. Other findings colitis. Collagenous colitis is a subtype of microscopic coli-
may include peripheral neuropathy; membranoproliferative tis. It is a clinicopathologic diagnosis made based on clinical
glomerulonephritis; and pulmonary, central nervous system, presentation, endoscopy features, and histopathology. The
or gastrointestinal vasculitis. Urinalysis may show dysmor- condition occurs more commonly in women than in men
phic erythrocytes and proteinuria, which are features of and typically presents with abrupt or gradual onset of watery
glomerulonephritis, but this patient shows no evidence of diarrhea that has a relapsing and remitting course over
kidney involvement or other end-organ damage. The best months to years. Mild weight loss may occur. The colonic
initial treatment for a mild presentation of mixed cryoglob- mucosa is macroscopically normal, and inflammatory
ulinernia arising from chronic HCV infection is to treat and changes are only appreciated on histopathologic review of
eradicate HCV with sofosbuvir-ledipasvir. Other direct-acting colon biopsy specimens. Several medication classes, includ-
antiviral agents that could be used interchangeably to treat ing NSAIDs, selective serotonin reuptake inhibitors, and pro-
genotype 1 HCV include grazoprevir-elbasvir; paritaprevir- ton pump inhibitors, have been associated with the develop-
ritonavir, ombitasvir, and dasabuvir; glecaprevir-pibrent- ment of microscopic colitis. The first step in management is
asvir; sofosbuvir-daclatasvir; and sofosbuvir-velpatasvir. It to discontinue a potentially causative medication. Support-
is expected that other combinations of direct-acting anti- ive treatment with antidiarrheal agents such loperamide can
virals will be developed. Treatment of HCV infection with be tried as initial treatment. For patients like this one; whose
direct-acting antiviral combinations results in a sustained symptoms do not respond to antidiarrheal medication, the

147
Answers and Critiques

American Gastroenterological Association (AGA) strongly right-upper-quadrant abdominal pain . The diagnosis is often
recommends, based on moderate-quality evidence, the use based on the characteristic imaging findings in a woman of
of budesonide for induction of clinical remission of micro- reproductive age. Histology can confirm the diagnosis, but
scopic colitis because of its favorable harm-benefit profile biopsy is not always needed in cases where imaging findings
and convenience of once-daily dosing. The rate of relapse are characteristic; needle biopsy is avoided because it is asso-
after discontinuation of budesonide is high, and mainte- ciated with bleeding.
nance therapy with the lowest possible dose to maintain Hepatic adenomas may increase in size during preg-
remission may be required. Patients treated with budesonide nancy. Women with adenomas greater than 2 cm who are
for longer than 6 months should be monitored for cortico- contemplating pregnancy, or who are pregnant, can be
steroid-related adverse effects. treated with radiofrequency ablation but the decision to
Because budesonide is expensive, alternative treat- treat and the treatment modality must be individualized.
ments such as bismuth salicylate may be considered if cost The natural history and prognosis of hepatic adeno-
is a determining factor. The AGA conditionally recommends, mas Is ill-defined. The risk for malignant transformation of
based on low-quality evidence, bismuth subsalicylate for hepatic adenomas is approximately 10%. Adenomas larger
induction of remission when budesonide therapy is not fea- than S cm in size have an increased risk for bleeding that can
sible. Bismuth subsalicylate therapy consists of two to three occasionally cause hemodynamic compromise. In general
262-mg tablets taken orally three to four times daily. terms, surgical resection is typically recommended for any
The benefit of mesalamine in achieving clinical remis- symptomatic hepatic adenoma, adenomas 5 cm or larger, or
sion in microscopic colitis Is uncertain, and it is recom- an adenoma found in a male.
mended as a potential second-line therapy. Asymptomatic women who are taking estrogen-
Prednisone should not be used as first-line treatment containing oral contraceptive agents and who have adenomas
of microscopic colitis because of its unfavorable side effects, smaller than 5 cm in si7£ can be managed conservatively, as
but it may be considered in patients who have microscopic the risk for hemorrhage or malignant transformation is rela-
colitis refractory to budesonide. tively low. However, liver imaging should be conducted every
The AGA conditionally recommends, based on low-quality 6 months for at least 2 years to ensure that the lesion is not
evidence, against the use ofprobiotics over no treatment for growing. Most adenomas tend to regress or even disappear
induction of clinical remission. Various probiotic strains, when oral contraceptive agents are discontinued. Failure to
dosages, and formulations are available, but most have not regress, continued growth, or an elevation of the o.-fetopro-
been evaluated in the treatment of microscopic colitis. tein level may be an indication for surgical resection.
KEY POINT KEY POINT
• The first step in the management of microscopic coli- • For women with asymptomatic hepatic adenomas
tis is to discontinue a potentially causative medica- smaller than s cm in size, estrogen-containing oral
tion, after which supportive treatment with antidiar- contraceptive agents should be discontinued, and
rheal agents such loperamide can be tried, with follow-up liver imaging is recommended every 6
budesonide recommended for patients whose symp- months for at least 2 years.
toms do not respond.
Bibliography
Bibliography Marrero JA, Ahn J, Rajender Reddy K; America] College of Gastroenterolo&Y.
ACG clinical guideline: the diagnosis and management of focal liver
Pardi DS, Tremaine WJ, Carrasco-Labra A. American Gastroenterologlcal
lesions. Am J Gastroenterol. 2014:109:1328-47; quiz 1348. [PMJD:
Association Institute technical review on the medical management of
25135008) doi:l0.1038/ajg.2014.213
microscopic colitis. Gastroenterology. 2016;150:247-274.ell. [PMID:
26584602] doi:l0.1053/j.gastro.2015.11.006

Item 90 Answer: A
Item 89 Answer: A Educatlonal Objective: Evaluate the small bowel for a
Educational Objective: Manage hepatic adenoma. source of gastrointestinal bleeding.
The most appropriate management is to discontinue the The most appropriate next step in management of this
oral contraceptive agent. Hepatocellular adenomas are gen- patient is capsule endoscopy. The patient has occult gastro-
erally seen in women because they can be dependent on intestinal bleeding, indicated by unexplained iron deficiency
estrogen for growth. They may also occur In men, and these anemia with stool testing positive for occult blood and no
adenomas carry a higher risk for malignant transformation. source of bleeding identified on repeated endoscopy and
Hepatic adenomas may also be associated with anabolic- colonoscopy. The next step is evaluation for potential small-
steroid use, obesity, and the metabollc syndrome. They are bowel bleeding. Causes of small-bowel bleeding depend on
often solitary, ranging in size from 1 to 30 cm. Most patients the patient's age. In older patients, vascular lesions such
are asymptomatic, and the adenoma is discovered inciden- as angiodysplasia account for up to 40% of cases. Capsule
tally during imaging for unrelated problems or because of endoscopy is noninvasive and is generally able to visuali7£ the

148
Answers and Critiques

entire small bowel. 1he main disadvantages are its inability upper limit of normal and HBY DNA exceeding 20,000 IU/
to sample tissue or treat findings. Tts overall diagnostic yield mL. Treatment is necessary to decrease hepatic inflamma-
ranges from 30% to 70%. Capsule endoscopy is contraindi- tion, risk for progression to fibrosis, and eventual cirrhosis
cated in patients with gastroparesis, swallowing difficulty, that will occur without treatment. In approximately 50%
and partial bowel obstructions, and in any patient who of untreated patients with chronic HBV infection in the
cam10t undergo follow-up surgery. Capsule endoscopy has United States, HBV infection will contribute to the cause of
become the first-line test in evaluating the small bowel after death (from hepatocellular carcinoma or other complica-
a negative endoscopy and colonoscopy in patients with gas- tions of end-stage liver disease, such as cirrhosis). Patients
trointestinal bleeding of an unknown cause. who respond to treatment have a decreased risk of liver-
CT enterography should be considered for patients pre- related complications of HBV infection. Tenofovir or enteca-
senting with or having a history of small-bowel obstruction vir is first-line treatment for immune-active HBV infection
where a lesion or mass is suspected. Patients aged 40 years or because of low rates of resistance. Treatment goals for HBV
younger who present with occult small-bowel bleeding are infection in the immune-active, HBeAg-positive phase are
more likely to have tumors (leiomyoma, carcinoid, adeno- loss of HBeAg and anti-hepatitis B e antibody serocon-
carcinoma, or lymphoma) as the cause of bleeding. version. For most patients, chronic treatment is necessary
Push enteroscopy is performed with advancement of because seroconversion of HBeAg or hepatitis B surface
the endoscope beyond the ligament of Treitz into the jeju- antigen is not commonly achieved.
num. The depth of insertion is operator dependent but is Adefovir and lamivudine are not preferred antiviral
also limited because oflooping of the scope in the stomach. agents given the risk for HBV resistance. Resistance rates
The diagnostic yield is between 24% and 56%. The primary at 5 years are approximately 30% for adefovir and 70% for
disadvantage of using push enteroscopy is that the amount lamivudine.
of additional small bowel that can be examined is limited In this patient, oral antiviral regimens are preferred over
compared to capsule endoscopy; however, it does allow for pegylated interferon because pegylated interferon may exac-
tissue sampling and therapeutic inteivention. Studies have erbate psoriasis. Pegylated interferon is most appropriate for
shown that sequencing capsule endoscopy followed by push patients who have higher ALT levels, low HBV DNA levels,
enteroscopy is an effective approach for management of and no cirrhosis.
occult small-bowel bleeding.
KEY POINT
The diagnostic yield of radiographic imaging with
small-bowel follow-through is very low (0%-5%) and, there- • Chronic hepatitis B viral infection in the immune-
fore, should not be used in the evaluation of suspected small- active, hepatitis Be antigen- positive phase should be
bowel bleeding. treated with tenofovir or entecavir to decrease hepatic
inflammation and the risk for progression to fibrosis.
KEY POINT
• Capsule endoscopy is the most appropriate test to Bibliography
evaluate patients for causes of small- bowel bleeding Terrault NA, Bzowej NH, Chang KM, Hwang JP, Jonas MM, Murad MH;
after negative upper endoscopy and colonoscopy. American Association for the Study of Liver Diseases. AASLD guidelines
for treatment of chronic hepatitis B. Hepatology. 2016:63:261-83. [PM!D:
26566064] doi:l0.1002/hep.28156
Bibliography
Gerson LB, Fidler JL, Cave DR, Leighton JA. ACG cllnlcal guideline: diagnosis
and management of small bowel bleeding. Am J Gastroenterol.
2015;110:1265- 87: quiz 1288. [PM!D: 26303132] dol:l0.1038/ajg.2015.246 Item 92 Answer: C
Educational Objective: Treat irritable bowel syndrome
with predominant constipation.
Item 91 Answer: D
A trial of polyethylene glycol (PEG) 3350 therapy is the most
Educational Objective: Treat chronic hepatitis B viral
appropriate next step for this patient. She has abdominal
infection In Its immune-active, hepatitis B e antlgen-
discomfort and a change in bowel habits predominated by
positive phase.
constipation; these symptoms fulfill the diagnostic criteria
Tenofovir is the most appropriate treatment for this patient. for irritable bowel syndrome with predominant constipation
Entecavir would be an equally suitable choice. Chronic hepatitis (IBS-C). Given her young age and lack of any alarm features,
B virus (HBV) infection is characteristically divided into phases such as blood in the stool, anemia, or a first-degree family
of disease. Not all patients go through each phase. Patients in member with colon cancer, a diagnosis of IBS-C can be made
the hepatitis Be antigen (HBeAg)-positive ("immune-active") and treatment can be initiated. The soluble fiber supplement
or HBeAg-negative ("reactivation") phases have an elevated psyllium has demonstrated limited efficacy in JBS, primar-
alanine aminotransferase (ALT) level and an HBV DNA level ily addressing stool frequency and consistency. Insoluble
above 10,000 IU/mL. The patient has chronic HBV infection fiber supplements such as bran appear to worsen IBS-C
in the immune-active, HBeAg-positive phase, with ALT and symptoms. Although various surfactant, osmotic, and stim-
aspartate aminotransferase levels exceeding two times the ulant laxatives can be used for the constipation symptoms

149
Answers and Critiques

associated with IBS-C, only the osmotic laxative PEG has been whose history includes quinolone antibiotic use for any
tested in TBS. PEG 3350 has demonstrated efficacy in IBS, and reason, salvage therapies could include bismuth quadruple
a trial of It is an appropriate first step in management. therapy, rlfabutin triple therapy (rifabutin, penicillin, and a
A colonoscopy is not warranted, given this patient's age proton pump inhibitor [PP!) such as omeprazole) or high-
and the nature of her symptoms. 'Ihe remote family history dose dual therapy (amoxicillin and a PPI). Dismption of
of colorectal cancer does not increase her risk enough to therapy can cause treatment failure, and the likelihood of
require a colonoscopy at this time. treatment success diminishes with each successive treat-
This patient has no risk factors for small intestinal bac- ment attempt. It is essential to counsel the patient carefully
terial overgrowth, such as small-bowel disease, previous on the importance of treatment adherence and potential side
bowel resection, or a primary/secondary gastrointestinal effects of the therapy.
motility disorder; therefore, a glucose breath test is not Treatment with the same regimen (amoxicillin,
indicated. Bloating is commonly reported in patients with clarithromycin, and omeprazole) for a longer period of
JBS. If bloating persists after correction of constipation, then time will be ineffective because the patient's Helicobacter
testing for small intestinal bacterial overgrowth could be pylori infection is likely resistant to clarithromycin and/or
considered. penicillin.
Although symptoms of bloating, abdominal pain, and The combination of amoxicillin, metronidazole, and
constipation can occur in celiac disease, this patient's symp- omeprazole is not a recognized treatment regimen for
toms do not warrant serum anti-tissue transglutaminase H. pylori in any setting. Metronidazole is typically used in
antibody testing for celiac disease. Testing would be indi- patients with penicillin allergy.
cated if she had a first-degree relative with celiac disease Using metronidawle rather than amoxicillin is unlikely
or if she had symptoms of IBS with predominant diarrhea to result in H. pylori eradication because resistance to clar-
(IBS-D) or TBS with mixed diarrhea and constipation. Fur- ithromycin is also a likely cause of the initial treatment's
thermore, results of testing for celiac antibodies such as the failure. Furthermore, the regimen of clarithromycin, met-
anti-tissue transglutaminase may be normal in patients on a ronidazole, and omeprawle could lead to the development
gluten-free diet. The patient's response to gluten elimination of metronidazole resistance in the likely event of treatment
does not necessarily indicate celiac disease because gluten is failure with this regimen.
a FODMAP (Fermentable Oligosaccharides, Disaccharides,
KEY POINT
Monosaccharides, And Polyols) carbohydrate, which can
cause bloating due to FODMAP intolerance. • For Helicobacter pylori infection that persists after
eradication therapy, the salvage therapy regimen
KEY POINT
should consist of different antibiotics from those used
• Polyethylene glycol 3350 is a first-line treatment for in the initial, unsuccessful regimen.
patients whose symptoms meet the criteria for irrita-
ble bowel syndrome with predominant constipation. Bibliography
Chey WO, Leontiadis GI, Howden CW, Moss SF. Affi clinical guideline:
Bibliography treatment of Helicobacter pylort infection. Am J Gastroenterol.
2017;112:212-239. [PMTD: 28071659) doi:l0.1038/ajg,2016.563
Menrln F, Lacy BE, Chang L, Chey WD, Lembo /\J, Simren M, et al. Bowel
disorders. Gastroenterology. 2016. [PMID: 27144627] doi:l0.1053/
J.gastro.2016.02. 031
Item 94 Answer: C CJ
Educational Objective: Diagnose hepatorenal syndrome.
Item 93 Answer: C
Hepatorenal syndrome is the most likely diagnosis in this
Educational Objective: Treat Helicobacter pylori
patient. In patients with end-stage liver disease and portal
infection persisting after treatment.
hypertension, hepatorenaJ syndrome is characterized by the
Bismuth, metronidazole, omeprazole, and tetracycline for development of oliguric kidney failure, bland urine sediment,
14 days is the most appropriate treatment regimen for this and marked sodium retention (edema, ascites, low urinary
patient. The salvage therapy regimen should consist of dit:. sodium). Two types ofhepatorenal syndrome have been rec-
ferent antibiotics from those used in the initial, unsuccessful ogniz.ed. Type 1 is characteri1.ed by acute kidney dysfunction
regimen. This strategy reduces the likelihood of antibiotic and is usually triggered by a precipitating event such as spon-
resistance, the major reason for treatment failure. taneous bacterial peritonitis, other infections, gastrointestinal
Additional factors in choosing salvage therapy after ini- hemorrhage, or a major surgical procedure. Type 2 is more
tial treatment failure include a history of quinolone anti- common and is characteri1.ed by more slowly progressive kid-
biotic use and penicillin allergy. If the initial unsuccessful ney failure in patients with refractory ascites. Type l hepato-
treatment was clarithromycin triple therapy and the patient renal syndrome is characterized by a rise in serum creatinine
is allergic to penicillin, the best salvage therapy is a bismuth of at least 0.3 mg/dL (26 µmol/L) and/or ~50% from baseline
quadruple therapy (bismuth, metronidazole, omepra:wle, within 48 hours with a bland urinalysis and normal findings
and tetracycline). In patients with no penicillin allergy, but on renal ultrasonography. It is also supported by a lack of

150
Answers and Critiques

improvement in kidney function after withdrawal of diuretics


CJ and two days of volume expansion with intravenous albu-
with uninfected pancreatic necrosis do not benefit from
antibiotic use during the acute phase ofpancreatitis or later,
CONT. min. Often patients with hepatorenal syndrome also have low in the resolving stage. Patients whose condition does not
urine sodium, low fractional excretion of sodium, and oligu- improve or deteriorates 7 to 10 days after presentation of
ria. In addition, patients should have no evidence of shock, no acute pancreatitis may have infected necrosis. CT-guided
current or recent use of nephrotoxic drugs, and no evidence fine-needle aspiration may help guide treatment decisions
of renal parenchymal disease (proteinuria less than 0.5 g/day, regarding antibiotic use, drainage, and continued supportive
no microhematuria, and normal renal ultrasound). The main care. Neither CT-guided fine-needle aspiration nor antibiotic
treatment of hepatorenal syndrome is the removal of drugs therapy is needed In this asymptomatic patient with walled-
that may reduce kidney perfusion and volume expansion. off necrosis of the pancreas that Is likely to resolve sponta-
Ultimately, hepatorenal syndrome is a condition for which the neously. The patient's left flank pain is due to passage of a
only cure is liver transplantation. kidney stone, not the pancreatic fluid collection.
Acute interstitial nephritis may be associated with Many walled-off necroses of the pancreas resolve spon-
drugs, infection, autoimmune diseases, and malignancy. It taneously, but some persist, enlarge, have a mass effect, and/
should be suspected in a patient who presents with an ele- or cause symptoms. Symptomatic collections may require
vated serum creatinine level and urinalysis showing leuko- decompression or debridement. However, this patient is
cytes, leukocyte casts, and possibly eosinophiluria; urinary asymptomatic and no intervention Is required at this time.
sodium level is typically elevated.
KEY POINT
Acute tubular necrosis is most commonly caused by
ischemia or toxins (including drugs) . Urinalysis typically • Asymptomatic patients with walled-off necrosis of the
shows pigmented granular (muddy brown) casts and tubular pancreas require no intervention.
epithelial cells; urinary sodium is typically elevated.
Membranous glomerulonephritis is associated with Bibliography
the nephrotic syndrome (proteinuria, edema, hypertension, Tyberg A, Karla K, Gabr M, Desai A. Doshi R, Galdhane M, et al. Management
of pancreatic fluid collections: a comprehensive review of the literature.
microhematuria) and erythrocytes or erythrocyte casts; uri- World J Gastroenterol. 2016;22:2256-70. [PMIJ): 26900288] doi:J0.3748/
nary sodium can be low. wjg.v22.17.2256

KEY POINT
• In patients with end-stage liver disease and portal
Item 96 Answer: C
hypertension, hepatorenal syndrome is characterized
by the development of oliguric kidney failure. bland Educational Objective: Screen for hepatocellular
urine sediment. and marked sodium retention carcinoma in a patient with cirrhosis.
(edema, ascites, low urinary sodium). The most appropriate management for this patient is ultra-
sonography screening for hepatocellular carcinoma every
Bibliography 6 months. Liver diseases associated with the highest risk for
Glass L. Sharma P. Evidence-based therapeutic options for hepatorenal hepatocellular carcinoma are hepatitis B virus (HBV) and
syndrome. Gastroenterology. 2016;1.50:1031-3. [PMTD: 26922867]
dol:l0.1053/j.gll~Tro.2016.02.0SO
hepatitis C virus (HCV) infections and hemochromatosis.
Approximately 80% of hepatocellular carcinoma occurs in
patients with cirrhosis, but it can develop in the absence
c:::J Item 95 Answer: D of cirrhosis in patients with HBV infection. All patients
with cirrhosis from any cause should undergo liver ultra-
Educational Objective: Manage walled-off necrosis
sonography every 6 months with or without a-fetoprotein
after acute pancreatitis.
measurement. Patients with HCV Infection who achieve
The most appropriate management of the fluid collection is sustained virologic response (which is synonymous with
observation. Walled-off necrosis of the pancreas is the most vlrologic cure) have a reduced risk for hepatocellular car-
likely diagnosis. The pancreatic fluid collection incidentally cinoma. Regardless of virologic response, surveillance is
found during kidney stone- protocol CT corresponds to the recommended for patients with stage 3 or stage 4 fibrosis
location of pancreatic necrosis during his acute gallstone (stage 4 fibrosis signifies cirrhosis, as found in this patient).
pancreatitis 6 months earlier. By definition, acute necrotic Liver transplantation evaluation is not indicated for
collections are classified as walled-off necrosis after 4 weeks. this patient with Child-Turcotte-Pugh Class A cirrhosis and
The necrotic tissue liquifies with time and develops a mature a very low Model for End-Stage Liver Disease (MELD) score.
wall as part of the healing process, as seen on imaging. These Indications for liver transplantation are a MELD score of
fluid collections do not require therapy if they are asymp- at least 15 or decompensated cirrhosis. Virologic cure in
tomatic, and as many as 60% may resolve spontaneously patients with compensated cirrhosis prevents decompen-
within 1 year; fluid-related complications are rare. sation. However, development ofhepatocelluJar carcinoma
Areas of pancreatic necrosis are frequently identified would be a reason for referral for liver transplantation
on imaging during an episode of acute pancreatitis. Patients evaluation.

151
Answers and Critiques

Measuring HCV RNA again in 12 weeks is not indicated KEY POINT


because this patient has achieved virologic cure with unde-
• Patients with hepatitis C viral infection who achieve
tectable HCV RNA at week 12 after completing treatment
sustained virologic response have a reduced risk for
for HCV infection. Large studies have demonstrated 98% to
99% concordance between sustained virologic response at 12 hepatocellular carcinoma; regardless ofvirologic
and at 24 weeks. Therefore, sustained virologic response at response, ultrasonographic surveillance is recom-
12 weeks is considered to be consistent with virologic cure, mended for patients with stage 3 or stage 4 fibrosis.
and additional testing is unnecessary.
Upper endoscopy is not necessary because the patient Bibliography
had an upper endoscopy 1 year ago. The standard follow-up Morgan RL, Baack B, Smith BD, Yartel A, Pitasi M, Falck-Ytter Y. Eradication
of hepatitis C virus infection and the development of hepatocellular
interval for small varices in a patient who is not taking a carcinoma: a meta-analysis of' observational ''tudies. Ann Intern Med.
nonselective ~-blocker is 2 years. 2013;158:329-37. [PMID: 23460056)

1 52
MKSAP 18 Made Easy
r

Gastroenterology and
Hepatology

0 i <9
* ♦
r\
+ MEDICAL
f
Gastroenterology and Hepatology

High Value Care Recommendation: A recommendation to choose diagnostic and management strategies for patients in
specific clinical situations that balance clinical benefit with cost and harms with the goal of improving patient outcomes.
Below are the High Value Care Recommendations for the Gastroenterology and Hepatology section ofMKSAP 18.

• In patients with gastroesophageal reflux symptoms without alarm features, empiric proton
pump inhibitor therapy is the first therapeutic and diagnostic step.

• Attempts to stop or reduce long-term proton pump inhibitor therapy for uncomplicated
gastroesophageal reflux disease should be considered at least annually (see Item 59).

• Women with gastroesophageal reflux disease do not require screening for Barrett esophagus.

• Patients 60 years and older with dyspepsia should undergo upper endoscopy.
• Patients younger than age 60 years with dyspepsia should be tested and treated for
Helicobucter pylori infection.
• Enteral nutrition is preferred in patients with acute pancreatitis (see Item 26).
• Same-admission cholecystectomy reduces rates of gallstone-related complications for patients
with mild gallstone pancreatitis (see Item 20).
• Asymptomatic patients with walled-off necrosis of the pancreas require no intervention (see
Item 95).
• Biopsy and endoscopic retrograde cholangiopancreatography are not indicated in the diagnosis
of chronic pancreatitis.
• Evaluation with imaging, endoscopy, and other testing is not indicated for irritable bowel
syndrome or functional diarrhea.
• Irritable bowel syndrome can often be managed with reassurance, lifestyle modifications, and
dietary modifications.
• The evaluation for centrally mediated pain syndrome does not require extensive laboratory
testing or imaging.
• Acute diverticulitis usually does not require abdominal imaging.
• Patients with small hyperplastic polyps should undergo repeat colonoscopy no sooner than 10
years (see Item 11).
• Patients with uncomplicated diverticulitis should be treated with oral antibiotics (see Item 74).
• Ultrasonography is the most effective means of diagnosing ascites.
• Asymptomatic hepatic cysts require no follow-up.
• Focal nodular hyperplasia does not require follow-up.
• Incidentally diagnosed gallstones do not require cholecystectomy.
• The diagnosis of acute cholecystitis can be made by ultrasonography.
• In patients with upper gastrointestinal bleeding, a transfusion threshold of less than 7 g/dL (70
g/L) with a target hemoglobin level of7-9 g/dL (70-90 g/L) is associated with decreased mortality
and length of hospital stay (see Item 83).

MKSAP 18 Made Easy Yasino.M.Habayeb


Gastroenterology and Hepatology Disorders of the Esophagus

Causes of Dysphagia
Condition Diagnostic Clues
Oropharyngeal Dysphagia

Structural disorders
Cervical osteophytes High dysphagia, degenerative joint disease
Cricoid webs High dysphagia, iron deficiency
Pharyngoesophageal (Zenker) diverticulum Aspiration, neck mass, and regurgitation of foul-smelling food
Goiter Neck mass
Neurologic/myogenic disorders
Amyotrophic lateral sclerosis Upper and lower motoneuron signs, fasciculations
Central nervous system tumor Headache, vision changes, nausea, seizures, balance problem
Stroke Focal neurologic deficits
Muscular dystrophy Slow progression of muscular weakness over years
Myasthenia gravis Weakness with repetitive activity
Multiple sclerosis Episodes of neurologic dysfunction with variable degrees of recovery
Parkinson disease Bradykmesia. rigidity, tremor
Dementia Altered cognition
Sjogren syndrome Dry mouth, dry eyes
Esophageal Dysphagia

Structural disorders
Dysphagia lusoria (vascular dysphagia) Vascular extrinsic compression on the esophagus on imaging
Epiphrenic/traction diverticulum Outpouching of the esophagus at any level on imaging
Esophageal strictures Intermittent dysphagia, especially for solid food; history of reflux
Eosinophilic esophagitis Food impactions, atopic history, rings or strictures on endoscopy
Esophageal webs or rings Upper esophageal webs may be associated with iron deficiency anemia
Neoplasms Rapidly progressive dysphagia for solids, then liquids, anorexia, weight loss
Motility disorders
Achalasia Concomitant liquid and solid dysphagia
Diffuse esophageal spasm Chest pain
Systemic sclerosis Tight skin, telangiecrtasias, scJerodactyly. gastroesophageal reflux disease, Raynaud
phenomenon

US

i
3
FIGURE 1. Barium esophaqram showing a Schatzki ring, a subtype of esophageal
ring located at the squamocoiumnar junction and a common cause of dysphagia

MKSAP 18 Made Easy Yasino.M.Habayeb


Gastroenterology and Hepatology Disorders of the Esophagus

KEY POINTS
• Oropharyngeal dysphagia occurs when the patient is
unable to transfer the food bolus from the mouth into
the upper esophagus by swallowing and should be eval
uated with a modified barium swallow.
* Ksophageal dysphagia occurring with solids alone sug¬
gests a mechanical obstruction, whereas dysphagia with
either liquids alone or the combination of liquids and
solids favors a motility disorder.

* Upper endoscopy is diagnostic and may be therapeutic


for esophageal dysphagia
* Chest pain is common in patients with gastroesopha¬
geal reflux disease, but a cardiac cause of chest pain
must be ruled out first,

Po-ctoe-s Associated wirth Reflux


Category Factor
i-
LiFestyle Cigarene smoking
Ofc>e-3ftyr
bating H-ifeiitS Pÿtfng fcercje meals
Parting lata ax nig.hu
Lying supine- shortly after eating
foods -an-ci Aicohol
ti everages
Chocol ate
Citrus f ruits. and juices

Coffee
F-atry and fri-eci foods,
Onÿoris

Feppermi nt
3vle*die:.at io ns agents
Aspirin and N SAlOs
Calcium- dirinnel blotlscrs
NHEMS-S
Pragestfi-rone
Opiosds(due to decayed gastric emptying)
Body position Bending over., exercising (both result ir>
:ncreasÿd in t r-gi- o tied om in ad pressure)

O tfie : Pregrtanicy
T'ighit-fitting clotKirsg
Mi I Hernia
E*

MKSAP 18 Made Easy Yasino.M.Habayeb


Gastroenterology and Hepatology Disorders of the Esophagus

Esophageal syndromes Extr oesophageal syndromes

Symptomatic Syndromes with Established Proposed


syndromes esophageal injury associations associations

i
1. Typical reflux 1. Reflux esophagitis V Reflux cough 1- Pharyngitis
syndrome 2. Reflux stricture syndrome 2. Sinusitis
2. Reflux chest 2. Reflux laryngitis 3. Idiopathic
3. Barrett esophagus pulmonary
pain syndrome syndrome
4. Esophageal fibrosis
adenocarcinoma 3. Reflux asthma 4. Recurrent otitis
syndrome
media
4. Reflux dental
erosion syndrome

FIGURE 2 . Classification of gastroesophageal reflux disease and its subsets.


feprixled ait* pmiw IrcnVaMN, on hum ST. lUhnl* r, 0«rt J. Jones ». GhW Csnseraas Group 1h« MonirtW deln.ton dnsif alion of guaraeuphspul reCw disuse « gbW eaidonse tased ronseisjus
XnsJ Gnltceemral 20C6;»01 10002», quit 1M1 lrU.O ia«l»S»i Copyri jlx 2004 Sprnger ilieuw

GFPD symptoms

Esophageal symptoms Exlraesophageal xy ■ optom;


(hearltjuen, add regurgitation] (cough, laryngitis. asthma)

▼ V
Alarm symptoms-1 Alarm symptoms' Esophageal features Esophageal symptoms/
absent present present complications absent

'' + T
Therapeutic trial Refer to a Therapeutic Irial Explore ullernativc
of a PP4 gas tree nte ro I o gi st of a PPI causes
for endoscopy end
other evaluation

V
J L
+ +
if clinical response, If clinical response,
No or No or
continue with PPI continue with PPI
incomplete mcoiripfele
regimen at lowest regimen at lowest
response response
effective dose effective dose

.
FIGURE 3 Management of gastroesophageal reflux disease.
(itED® gasIrwSpphaÿfciJ rafliu disease; PPI * proto* pwnp inhibitor.

■MMW symptoms rvtodo djnpivpj. un otentiixul Height loo, hemitrmtu'J, *r»d melena

MKSAP 18 Made Easy Yasino.M.Habayeb


Gastroenterology and Hepatology Disorders of the Esophagus

TABLE 3- Adverse EffecrTs of Proton Purtip I ntiifeito rs


1
Cpmman Unusual
i

i

Keadacho Vrta-min B1 5 -de-fioieTi cy i-Tijur’y

Didrih4?a Hyponn agn eÿmid Oerne rstta


Dyspepiia Community-arcjuired
pneumonb
d<Dstrjcf\ijm diffic/Je infection
Hip fractu re

FIGURE 4. Upper ersdascupy shewing white adherent plaques suggestive of


Candida esopfiag itis.

F pfiaÿram showing Uhe typical ap p-e-a rÿrtte of a d ilac&tJ

ten t
and nrilrA
wiifi achalas-ia
fc' n arrow i ng at Ihe gast34>ewpti jy ea I junclion in a

MKSAP 18 Made Easy Yasino.M.Habayeb


Gastroenterology and Hepatology Disorders of the Esophagus

m -

FIGURE
*. Findi ngs
on cisopKa goprapÿiy
ophacp u.-s* f caused by multiple srmultaneous
are typical ol d i ffu 5e esopha;

KEY POINT;

* In patients with symptoms of gastroesophageal reflux


disease but without alarm features, an empiric trial of a
proton pump Inhibitor can relieve symptoms and con
firm the diagnosis.
* Most patients with gastroesophageal reflux disease do
not require surgery, indications include failure of opti¬
mal proton pump inhibitor therapy, wanting to stop
and intolerable medication side effects.
medication,

- The diagnostic criteria for eosinophilic esophagitis are


esophageal symptoms (most commonly dysphagia),
esophageal biopsies showing persistent counts of IS
eosinophils/hpf or greater despite a trial of proton
pump inhibitor therapy, and exclusion of other causes
of eosinophilia.
* Odynophagia and dysphagia are the most common pre
sent mg symptoms of infectious esophagitis.
• To prevent pill-induced esophagitis, patients should
drink water with medication and remain uprighl for at
[east 30 minutes a fteT ingestion.

MKSAP 18 Made Easy Yasino.M.Habayeb


Gastroenterology and Hepatology Disorders of the Esophagus

FIGURE T .
Upper endoscopic view of Barrett mucosa, with salmon-Eolored
mucosa representing Barrett mucesa comfiared with the normal peart-colorerf
squam&iii mutiosa-

TABLE 4. Practice Guidelines for Endoscopic Surveillance of Barrett Esophagus


Dysplasia Grade Recommendation

Mode ]f no dysplasia is present,repeat upper endoscopy every 3 to 5 years

Indefinite Start or adjust proton pump inhibitor therapy, then repeat endoscopy in 3 to 6 months
If still present, then repeat endoscopy in 1 year
Low-grade Confirmation by expert pathologist, then proceed to endoscopic eradiation therapy (surveillance
endoscopy in 12 months is an alternative)
High-grade Confirmation by expert pathologist
Endoscopic eradication therapy is preferred
Golfp o' Gmmxmbgy, ACG dirwctl gu del™; diagnosis ard marwgmwt of 0*rvetts
Adaotcd wih oemmion from Snjbtc'i NJ, fffl GW, tp PC. Gmon LB; Amt ricm : 1C. 1 03S/ajg 201 5.322
(JOphagus AmJG»tfwi«*rol.2016;111:30'W; qvir St . (PMID: 2ÿ6260791 do

MKSAP 18 Made Easy Yasino.M.Habayeb


Gastroenterology and Hepatology Disorders of the Esophagus

KEY POINTS
* Barrett esophagus is a premalignant condition caused
by longstanding gastroesophageal reflux disease.
• Women with gastroesophageal reflux disease do not
require routine screening for Barrett esophagus.

- Patients with Barrett esophagus should receive proton


pump inhibitor therapy with dosing based on symptom
relief and healing of erosive esophagitis.
* Dysphagia with solid foods is the most common pre¬
senting symptom of esophageal cancer

MKSAP 18 Made Easy Yasino.M.Habayeb


Gastroenterology and Hepatology Disorders of the Stomach and Duodenum

TAÿLE 5. Organic Causes of tyspepsiA

Gastroesop ha g«ai G aslnoesophagea I r&JIujt di;easer peptic


ulc(?rdisease, H&iiaotyacrter pylori
gastntis, gastric eancef/lynnphDma,
gaetric amyloidosis, Menetrier disEHs«,.
gastnopa resis.
Small bowel Celiac disease. O-rd'i.n disease
[ Pancreatic Pa ncreatitis, pan erratic cancer
Infectious GiarO1ÿ /.3mt>}iar StrongyfokJes
stortzarafis, tuberculosi S. syphilis
Common NSAIE>sr aspinn, ■ron, antibiotics
medications terythromycin, ampicllin), narcotics,
estrogens, arsd oral toittraceptives,
theophylline,. levodopa. d-ngitaNs
Other systemic Coronary artery disease, kidney disease,
conditions thyroid dysfunction,, adrenal i nsuffroi fidcy,
hyperparathyroidism, pregnancy

TABLE 6- Rome 4 LJiflgnostic Criteria For


Functional Dyspepsia

One or more of the following r"


Bothartcnne postprandial fullness
Bothersome early satiation
Bothersome epigastric pain
Bothersome epigastric burning
Nd evidence of structural disease (including upper endoscopy}
that is likely to explain the symptoms

fulfilled to-' ti--.* last 3 nvhntha «irth syTnfKtMTi OTLM( ar l-ea-st 6 months Ewforp-
di-agnosn
I
Rep flBTfrd- JjeJm.iE5loni v, Chsn FK, VJL Malaÿgo-lada Jit,
VrfTjkj K J. ot al. GjstrtcJu-udEi 'hÿl dionrdfers. Gastfs&itai akig/.
2Dt AlSOiiaatS-Oÿ. LPMID. Z7T4ft?Z] doJ;10 1053ÿ.g»sHÿ 2U16.-03 - Gapyrighc
1

2316r T*.e AGA I -ÿstlt dts.

MKSAP 18 Made Easy Yasino.M.Habayeb


Gastroenterology and Hepatology Disorders of the Stomach and Duodenum

TABLE 7. Rome 4 Diagnostic Criteria for Postprandial Distress Syndrome


Criteria1 Supportive Remarks

Must include one or both of the following at least 3 days Postprandial epigastric pain or burning, epigastric bloating, excessive
perweek: belching, end nausea can also be present
Bothersome postprandial fullness (i.c., severe enough Vomiting warrants consideration of another disorder
to affect usual activities)
Heartburn is not a dyspeptic symptom but may often coexist
Botÿome estly Mlatioo severe enough to
Symptomsll,itJrere|ievedbyevacuato„0|fece50Igasshould8e„efa||y
prevent finishing a regUar-siied meal) rÿbecons.d.iedaspttUdyspepsia
No evidence of organic, systemic, or metabolic disease Other individual digestive symptoms or groups of symptoms, such as from
that is likely to explain the symptoms on routine gastroesophageal reflux disease and irritable bowel syndrome, may coexist
investigationsfincluding upper endoscopy)
•Olrene UFIltd for the list 3 month) with symptom onset it foist 4 months before (tignoss.
Reprinted with pentwsion from StanghiSlint V.Chen f*,HaslrW., MalageUda JR, Suufo H, Tack J, «til Gutrcduodenil disorder* GaWoenwofogy. 2014:1WtlMOM.
|FMiD: 2714?U2|dai:lC.I0S3rj.g«WO.20l6.02iQ1 1 Copyright 2014, the AGAInstUito

TABLE 8. Rome 4 Diagnostic Criteria for Epigastric Pain Syndrome


Criteria* Supportive Remarks
Must includethe following symptoms at least Pain may be induced by ingestion of a meal, relieved by ingestion of a meal, or may
1 day per week: occur while fasting
Bothersome epigastric pain and/or burning Postprandial epigastric bloating, belching, and nausea can also be present
[Le., severe enough to affect usual activities)
Persistent vomiting suggests another disorder
No evidence of organic, systemic, or metabolic
diseasethat is likely to explain the symptoms on Heartburn is not a dyspeptic symptom but may often coexist
routine investigations {including upper The pain does not fulfill biliary pain criteria
endoscopy)
Symptoms that a re relieved by evacuation of feces or gas should generally not be
considered as part of dyspepsia
Other individual digestive symptoms or groups of symptoms, such as from
gastroesophageal reflux disease and imtable bowel syndrome, may coexist
| “Cnteri fulfilled for the last 3 months with symptom onset it leest t months before diignosis.

Reprinted with permission from 5tinghe!lini V, Chen RC toiler Wl, MafogeUde JR, Suzuki H, Tetk J, at el. Gistroduodenil dworders. OesVMitterofogy. 2016, 15ft 1360-92.
|N®:27l47132|doi:lft1Q534gMWmO2.0r Copyright 2014. ftie AGA Institute

MKSAP 18 Made Easy Yasino.M.Habayeb


Gastroenterology and Hepatology Disorders of the Stomach and Duodenum

KEY POINTS
* For patients with dyspepsia, routine upper endoscopy
to exclude malignancy is reserved for patients older
than age 60 years.
* Upper endoscopy should be considered fur patients
with alarm features such as a family history of gastric
cancer, immigration from a region with increased risk
for gastric cancer, or severe symptoms, regardless of age.
* Patients younger than age 60 years with dyspepsia
symptoms should be tested and treated for Helicobacter
pyiari infection.

Oigtary/Liltstyl* Intervention*
Avoid anco of food trigger
Smaller, morli Sequent meals
Low (at diet
El i m i n atron diets (dairy, gluten, FODMAP)
Avoidance of NSAIDs, coffee, alcohol,
and smoking

No rel ef of
Epigastric Pain Syndrome symptoms Postprandial Distress Syndrome

Ant isec rutory Ag 0(1 ts Pro trinetic Agents


I if not already tried ) Meloclopramide (use for shortterm and with
Proton pump inhibitor caution}
Histamine receptor antagonist Domperidone (where available}
Buspirone

r*o relief of
symptoms

An tide pressa nts


Tricyclic antidepressants
Mirtajtapirtc

No relief of
_ symptoms

Referral for additional functional testing and


a Its motive th etapi os

FIGURE 8, Treatment ol fundi Ihnal dyspepsia.


fODMA* •firmenpble Cttgauctarldti. Diuahirfdei. VMOMtchanies Arif

MKSAP 18 Made Easy Yasino.M.Habayeb


Gastroenterology and Hepatology Disorders of the Stomach and Duodenum

KEY POINTS
* NSAJDs and Helicobacter pylori infection are the two
most common causes of peptic ulcer dlsease.
* Upper endoscopy is the diagnostic test of choice for
peptic ulcer disease.
* Repeat upper endoscopy is reserved For patients with
persistent symptoms after 8 to 12 weeks of therapy,
a!cers of unknown cause, or If gastric ulcer biopsy was
not performed during the initial upper endoscopy

KEY POINTS
• Patients with active peptic ulcer disease, history of
peptic ulcer disease without documented cure of
Helicobacter pylori infection, gastric mucosa-associated
lymphoid tissue lymphoma, or history of endoscopic
resection of early gastric cancer must be tested For
H. pylori and treated if positive.
• Gastric biopsies during upper endoscopy or nordnvasive
testing methods, including JJOunea breath and stool
antigen testing, can confirm the presence of H. pylori
infection; negative serologic testing can exclude infec¬
tion. but a positive serologic rest*)! requires confirmation
* Treatment regimens for H. pylori consist of a minimum
of three agents, including two antimicrobial agents and
one antisecretory agent; treatment duration Is 14 days in
most cases.
* Testing for eradication of H. pylori is performed using
a E3C-urea breath test, fecal antigen test, or biopsies
obtained during upper endoscopy at least 4 weeks after
completion of eradication therapy

MKSAP 18 Made Easy Yasino.M.Habayeb


Gastroenterology and Hepatology Disorders of the Stomach and Duodenum

TABLE 9. First- Line Treatment Options for Helicobacter pylori Infection


Treatment Regimen Duration of Therapy Clinical Indicator!

PPl, standard' or double dose twice daily 1 4 days H pylon clarithromycin resistance is known to be <15%
(esomeprazole, once daily only)
No previous history of macrolide exposure for any reason
Clanthromycin, 500 mg twice daily
Amoxicillin, I g twice daily
PPl, standard or double dose twice daily 1 4 days H. pylori clanthromycin resistance is known to be <15%
Clanthromycin, 500 mg twice daily No previous history of macrolide exposure for any reason
Metronidazole, 500 mg three times daily Penicillin allergy
PPl, standard dose twice daily 10-14 days Previous macrolide exposure
Bismuth subcitrate, 120-300 mg or Penicillin allergy
subsalicylate, 300 mg four times daily
Tetracydme, 500 mg three times daily
Metronidazole, 250 mg four times daily or
500 mg three times daily
PPl, standard dose twice daily 10-14 days May be an alternative to standard clarithromycin triple therapy
Clarithromycin, 500 mg twice daily Not validated in North America
Amoxicillin, 1 g twice daily
Nitroimidazole, 500 mg twice daily
PPl, standard dose twice daily 10-14 days May be an alternative to standard clarithromycin triple therapy
Levofloxacin, 500 mg twice daily Not validated in North America
Amoxicillin, 1 g twice daily
PPl •proton pump inhibitor.

'Standard doia PPl. aaomap-aioia, *0 mg; lantooraroto. 30 mg, omapraiola. 20 mg. oantoprsiola, «0 mg. rabapratole 20 mg.
Adapted with p»»mi»ion from Chey WO. leorwad. J Gl. Mowden CW hfott Sf. ACG d.rjtal guidelma treatment of Heteobarter p/fori infect.on Am J Gaitmeoterol.
70 1 7; 1 12:212-239 [PMID 28071459| doi: 1 0.1038/a,g-2014 S&3

TABLE 10. Second-Line Treatment for Helicobacter pylori Infection after Failure of Initial Treatment
Treatment Regimen Duration of Therapy Clinical Indicators

PPl, standard dose twice daily 14 days Failure of c'arithromycin-based or


levofloxacin-based therapy
Bismuth subcitrate, 1 20-300 mg. or subsalicylate, 300 mg four times daily
Tetracycline, 500 mg three times daily
Metronidazole, 250 mg fourtimes daily, or 500 mg three times daily
PPl, standard dose twice daily 14 days Failure of bismuth-based or
darithromycin-bascd therapy
Levofloxacin, 500 mg twice daily
Amoxicillin, 1 g twice daily
PPl, standard or double dose twice daily 1 4 days Failure of bismuth-based therapy
Clarithromycin. 500 mg twice daily
Amoxicillin, 1 g twice daily
PPl, standard or double dose twice daily 14 days Failure of bismuth based therapy
Clarithromycin, 500 mg twice daily Penicillin allergy
Metronidazole, 500 mg three times daily
PP» - proton pump inhibitor.

Adapted wmh permission from Chey WO. lecntiadis Gl. Ho*den CW. Moss $F. ACG dir cal goceii-i* treatment of HcJicobiKtof pylon infection. Am J Cestioenierol.
201 7.1'2.212-239.|PMIO 28371659)00« 10.1038/ajg.2016 563.

MKSAP 18 Made Easy Yasino.M.Habayeb


Gastroenterology and Hepatology Disorders of the Stomach and Duodenum

KEY POINTS
* Autoimmune atrophic gastritis is associated with perni¬
cious anemia, iron deficiency, small intestinal bacterial
overgrow th, and gastric cancer.
* 7F<eficobncter pytart is associated with upper gastroin¬
testinal conditions other than peptic ulcer disease
including environmental metapJastic atrophic gastritis,
gastric intestinal metaplasia, and, rarely, lymphocytic
gastritis,

TABLE 1 1 . Risk Factors for NS AID -Related Upper


tjaitro intestinal Complications

History of upper gastrointestinal bleeding


History of peptic u lew disease

. Helicobacter pylori infectfon


Age older than 65 years
Hemodialysis or peritoneal dialysis
Use of high-dose or multiple NSAIDs
Concomitant use of aspirin (even low- dose), nonaspi rin
antJpiatelet agents, anticoagulants, or&r glucoccrtKroHds.
selective serotonin rauptake inhibitors

MKSAP 18 Made Easy Yasino.M.Habayeb


Gastroenterology and Hepatology Disorders of the Stomach and Duodenum

KEY POI NTS

* Upper gastrointestinal complications, such as bleeding,


are common with the use of NSAIDs (both short- and
long-term) and low-dose aspirin.
* Proton pump inhibitors are the preferred agent for the
prevention and treatment of NSAlD-related (including
aspirin -related) upper gastrointestinal compticarions.
* In high risk individuals without cardiovascular
disease,
including those with previous NSAID induced gastroin
testinal bleeding, the combination of a selective
cyclooxygenase -2 Inhibitor plus a proton pump inhlbi
tor provides the best gastrointestinal protection if avoid¬
ance of NSAIDs is not possible.

TABLE 1 2. Strategies lor Prevention of NSAlD’Related Ifc&r Complications


No Gl Risk Factors' One to TWo Gl Risk Factors'ÿ Mow thenTWo Gl Risk Factors1

No aspirin NSAID NSAID plus PPi Ideally, avoidance of NSAIDs


If no alternative to NSAIDs. then a
selective COX-2 inhibitor plu&a PPl :

Naproxen plus PPI Naproxen plus PPl Avoid selective COX-2 inhibitors and
Low-dose aspirin (81 mg/d)fot use alternative analgesic
cardiovascular prophylaxis
- - -
COX-2 qelOttyjwewi; 0 pfawferiNt NSW rvjRtftfOKW anri-iÿimramÿ
drug: PPI pm> pump inhibitor. -
oINSNOf ««d pplrfaflneluciislowdi* upMiik MinÿMn iiiliiMMl, OflleomulUie.
•Ql rUchdors: hiAny of « Nwÿ «Tpÿucdwdiwir
oldpr than 65 yean; any cJvonk, debitiwiÿ g Hlwsi
Or glifioti'tieoids; a<j*

"With no prior N SAlD-reWd upper Gl blending

lgl«edul«rcfrM<*iiflM.AnOGtfrti"fllMl20l)9;10*:?t838iiFMIO. W2tft69B] dot. 0. 038ftj8 _

MKSAP 18 Made Easy Yasino.M.Habayeb


Gastroenterology and Hepatology Disorders of the Stomach and Duodenum

TABLE Causes of Gaitfapdresis

Gomrnon causes LUja fciette-s melFrlus (ÿ10% in long-standing


type T diabetes m &l litt>3, 20%. in tyPe 2
diabfltM ■me(titus)
Ppstsuÿhoal fe.g.j Nissen fundoplicaticrn,
banatnc suÿgey pancreatic surgery}
JcJiopath-it {e.g., pos-tutral1}
Infrequent causes Connective tissue disease (e g., systenni'C
sclerosis)
Meunjlogjcdisease [-e g -F Parkinson
disease)
Eatirig disorde=rs
Hypothyroidism
! Afnyroidosis
Paraneoplastic syndromes {e.g., smalt
eeJI Fi_rng cancer)
Mesenteric ischemia
Medications , opiates,
arrticholrnergic: agents}

KEY POINTS

* Gastroparesis is a heterogeneous cJinteaI syndrome with


Three components to the diagnosis: the presence of spe¬
cific symptoms, absence of mechanical outlet obstruc¬
tion. and objective evidence of delay in gastric emptying
into the duodenum

* Initial management of gastroparesis includes correction


of dehydration and electrolyte abnonnaiities; nutri¬
tional support; smallT frequent meals that are low in fat
and soluble fiber; and improved glycemic control m
patients with diabetes mellitus.
* Metoclopramide is a prokinetie drug that improves gas
trie emptying but is associated with dystonia, tardive
dyskinesia, and Parkinsonism.

MKSAP 18 Made Easy Yasino.M.Habayeb


Gastroenterology and Hepatology Disorders of the Stomach and Duodenum

TABLE 1 4. Diagnostic Tests Assessing Gastric Emptying


Test Modality Advantages Disadvantages Clinical Pearls

Gastric scintigraphy Co nsidered the gold Radiation exposure (technetium 4-Hour study is most accurate
standard radiolabeled meal)
Assesses solid emptying (liquid emptying
Requires specially trained personnel is less accurate)
Cost Blood sugar should be lessthan 275 mg/
dL(15,3mmol/U
Wireless motility Can also assess small Cost Consider in a patient with suspected
capsule bowel, colon, and global motility problem
Can't be used with pacemaker or
whole gut transit defibrillator Stop antisecrctory agents as study relies
No radiation on measurement of pH
Risk for capsule retention
Ambulatory study
I Gastric emptying Low cost Only recently commercially available
breath test Efficacy and accuracy limited to
clinical trials

KEY POINTS
- Polyps in the? stomach include fundic gland polyps,
which haw no malignant potential; hyperplastic pol¬
yps; and, less commonly, adenomas, which should be
resected.
* Gastrointestinal stromal tumors should be evaluated
with endoscopic ultrasonography and excised if symp
toms or high-risk features are present.

* Carcinoid tumors are well-differentiated


neuroendo
erine tumors that can occur throughout the gastrointes
tinal tract, including the stomach-

MKSAP 18 Made Easy Yasino.M.Habayeb


Gastroenterology and Hepatology Disorders of the Stomach and Duodenum

KEY POINTS
* \n courts He* with a low incidence of gasl ric placer,
screening for gastric cancer should be reserved for
patients with genetic cancer syndrome 5 .

* The primary nortgenetfc risk factor for gastric cancer is


Heiicohocter pylori infection
* Upper endoscopy with biopsy is the diagnostic test of
choice for gastric cancer

KEY POINTS
• Patients who undergo partial gastrectomy for malig
nancy require lifelong surveillance for recurrence of
cancer.
* Dumping syndrome results from rapid gastric emptying
after gastric surgery; first-line treatment is smaller, more
frequent meals with liquids taken following meals,

10

MKSAP 18 Made Easy Yasino.M.Habayeb


Gastroenterology and Hepatology Disorders of the Pancreas

TABLE 1 S. Causes of Aou t L- Pa-ncro-at i*L5


Comm on
'
fc3 ii i rj ry tf isease
Ga IIst-cm 65
iVitcro I ith i jsii ( 1 - to 2-mm stores tJn-at aie n ot detected by
irn-agintj studios)
Alcohol use
Post-endoscopic retrograde theIan g iops ncreatoy na p hy
!-
Occasional
FVT e-d i cation s
Fu noseir.i de
Did a nos-i n td
A-s pa-racj i n ase
tvlcsa! arti i ne5
Hyd rochI O'trthiazi do
<£>- IV3 e rce pto pu rine/a zathropr i no
5 i mva statr n
H ypedTriglyc arido nr ;H
HY pe tea l-cem i □■
Chic-le-dot ho
Rare

Auto it.TiurKj

Infectious
Vir31 {mumps, coKsadcic EE virus, Cytomcgalcur ■"usj i

Fa-ras-itJc { 7o*op/asn J species, Ascans iu TI JbrrcordiK )


Ischemia
Trauma
heo pr.ysis
Celiec dTseÿse
Genetic {only if attacks ara recurrent)

MKSAP 18 Made Easy Yasino.M.Habayeb


Gastroenterology and Hepatology Disorders of the Pancreas

Stic
Ed ma

3
■k
J

F IGU C T snan showing acu le pa ncreaUCis with peripa ncreatic fat sfranrl ing
a rwf i nfia raimati-on,The Kajy appears nuc a-f Ifte m eniteric fat the
p anc r c In thi5 image is call-e' cl fat strandi ntj,- and Che blurring of the- margins c-"f
the pH. na:rea.s ts consEstent with pen'pancreatic edema, featu res seen iltr
i nflam m ntory changes -o f 3 cu te pan creatitis.

N crr>si-s
-
Si - ,-vS

- :

Norm
.
Tati

FIGUR ■® O „ CT scan sh-owing acute pancreatitrs w ith tiypa perfusion of the


body of the pancreas as indicated by 1»tk &f enhancemeni f oil ow iimg intravenous
contrast infusion (near-os is) and nor mal pexfuslon of Che p>a ncreatic ta il

«
j

FIGURE 1 - CT scar* showing maturation and liquefaction of pancreas necrosis


of nearly the entire pancreas over 4 weeks in duration with a well-defined nm or
wall (arrows), known as walled-off necrosis.

MKSAP 18 Made Easy Yasino.M.Habayeb


Gastroenterology and Hepatology Disorders of the Pancreas

Kir POINTS
* Biliary disease, such as gallstone o r
microlithiasis, is the
most common cause of acute pancreatitis.

- Diagnosis of acute pancreatitis requires two of three cri¬


teria: <1> acute onset upper abdominal pain, (2) serum
lipase or amylase levels elevated at least three times
greater than the upper limit of normal, and £3} charac
ter i Stic findings on imaging.
• Patients with acute pancreatitis should undergo
transabdominal ultrasonography to evaluate for biliary
disease
* Patients with acute pancreatitis should be hospitalised
for fluid administration and close monitoring of cardio¬
vascular. respiratory, and kidney status.

TABLE 16. Ca use s. of Chronic Pancreatitis


Toiricor metabolic
Alcohol, tobacco, hypercalcemia, hypertriglyceridemia,
ch ron i c k id n ay d isea se
Genetic
Mutations or p olym Q rp h ism 5 of the CFTR, SPlNKi.
CTRCr CASR, CLON2 gen es I
Recurrent and severe acute pancreatitis
Vascular disease/:schemi a
Obstructive
Pancreatic tumor, intraductal papillary mucinous neoplasm
Postlraumatic ( pancreatic duct stricture}
Autoimmune (type 1 and type 2)
JcfropathFc

MKSAP 18 Made Easy Yasino.M.Habayeb


Gastroenterology and Hepatology Disorders of the Pancreas

Ourt

FIGURE 12 - CT ican sticzwfng cfi r»n ic ca iciTic pane rc-atitis- wwiih multtpÿe ilone?
FFI tin e nwi n dv cl a net j de hr& n-chues at th.e pya mreas.

K£V POINTS
* Abdominal pain, which may occur as intermittent
attacks or as ongoing, dally pain, is th|j most common
symptom of chronic pancreatitis,
* Pancreatic btopsy and endoscopic retrograde cholangio
pancreatography are not indicated in the diagnosis of
chronic pancreatitis,
* Symptomatic management is the cornerstone of treat
ment for chronic pancreatitis, including patients with
refractory pain
• Patients with chronic pancteatitis should l>e counseled
to avoid alcohol and tobacco use

MKSAP 18 Made Easy Yasino.M.Habayeb


Gastroenterology and Hepatology Disorders of the Pancreas

V" V
ifgMllgNs £ÿ
S5|

SLveStjnÿ
V
/
FIGURE 13. CTT Stan showing ttve- hnmagsn#oui, h
CaxlnftwiS Mferr in autoifnrame pancreQtrtfS.
'iSysage sh-ÿp«d“

* Diagnosis of autoimmune pancreatitis requires the


presence of a narrowed main pancreatic duct and
parenchymal swelling ("sausage shaped'’ pancreas) on
imaging and disease response to glucocorticoids.
• Type J autoimmune pancreatitis is characterized by ele
vated numbers of JgG4 jxjsitive cells in pancreatic tis¬
sue; most patients also have a significant elevation of
IgG4 in serum,
* Patients with type 2 autoimmune pancreatitis have nor
maJ IgG4 -positive cell counts.
* Types l and 2 autoimmune pancreatitis are treated with
glucocorticoids, with a relapse rate of up to 60%.

MKSAP 18 Made Easy Yasino.M.Habayeb


Gastroenterology and Hepatology Disorders of the Pancreas

KEY POINTS
• Diagrrcusfls of pancreatic cantor is tjy wolglit
loss, atÿct-r>m[r?a.t pairt, iau nd i and new onset di-abttes
mel I i (i_Lfi; pancreas protectÿ CT or ME! help support the
Cl (.3 £yj osls and. d-el i rT.ea.tc the extent Of <JlS>e*iSC.
* RntEofsctjpic ultrasound jÿuided fi rtc- needle aspiration of
the pancreas is recommended id provide 1 L LSTOL QgjcaJ
confirmation o f cancer

XEV POINTS
• Most pancreatic cysts never become malignant, with the
exception of Intraductal papillary mucinous neoplasms
involving the main pancreatic duet,
* Cystic lesions are managed with surveillance and surgi¬
cal resection of high -risk cysts.

KEY POINTS
* Ten to twenty-five percent of pancreatic neuroendo
crine tumors are functional and hyperseerete hor¬
mones, most commonly gastrin or insulin.
• Genetic testing for inherited tumor syndromes should
be considered based on patient age and history.

MKSAP 18 Made Easy Yasino.M.Habayeb


Gastroenterology and Hepatology Disorders of the Small and Large Bowel

TABLE 1 7. Types of Chronic fiarrhea


Diarrhoa Type Causes (examples)

Osmotic VI e dica cions { laxatives)


Undigested sugars (tac-Lose, fructose.
sorbitoE, mannitol)
Secretory Medications (nonosmotie laxatives,
antibiotics)

Endocrine (carcinoid, gastrinoma, VI Poma,


adrenal insufficiency, hyperthyroidism}
Bile salt malabsortition (ileal resection.
cholecystectomy)
Noninvisive infections (giand-asis,
cryptiSpor'dioS’S)
Smail intestinal bacteria I overgrowth
Steotorr h ea Maldigestion (decreased brie salts,
pancreatic dysfunction)
Malabsorption (celiac disease, tropical
sp-ue, giardiasis. Whipple disea se, chronic
mesenteric ischemia, short bowel syndrome,
bacteria.I overgrowth, lymphatic obstruction)
inflammatory bowel disea se ( uIterative
' -lflam matory colitis, Crohn disease, microscopic colitis)
Malignancy (colorectal cancer, lymphoma}
invasive infections (C’/ostrid/ucn ciifficriie,
cytomegalovirus, Entamoeba (ri.seotytic-3,
tuberculosis)
Ischemia
Radiation colitiE/enteritii
Motility Post5urgjcal (vagotomy, dumping)
Endocrtie (diabetes mellitus,
hypertbyroidism '•
Scleroderma
Miscellaneous Irr table bowel syndrome
Functional diarrhea*
Factrti ou 5
Overflow
:
Fecal incontinence
"‘Fnuqtfsnn or bose or ry stools- rti ithoÿrl afedominsni di strewn fort the jbEcncc
of (Jthcr d*iif.i*idble Cdust=3.

Adÿpfed with pnr rrri'action fscMnn ScJhilir;.- LR, Pi*rai £35, SgrilTitb- % 5 £nnradi CE_ viu —it ClML
&i£H4TiT«fllai RA, e-t a- ?Q1 3 AP CfyVA./7OQ«3 ihairigihsi wonting pdfiy repoft:
cH.-oni-c di*:-rhfr*: cieP nllfeon. cFÿsÿrfi,ire scion dCia igir J CLIL s.1roe -rvt-e rcJ hterwEx? -
Z 01 A .Z4? S 25 |PMID: 241 T 7999] etc: 1 O ' I I T/jgn t Z39Z.

MKSAP 18 Made Easy Yasino.M.Habayeb


Gastroenterology and Hepatology Disorders of the Small and Large Bowel

TABLE 1 B. ■M e cJ i cations th at Cau se Lj-f a rrh e a

Common

proton pump i ni'ihfci.itd rs


Chemotherapy
AreTifciiatie;?
Colchicine
etfOrdlrtFrt
MSAIDs, mesaiamine
Cholesterol-toÿerir.ÿ drtrgs
Rarer
ACE rn-hibrnars
AngÿoteFhSJit receptor blockers
fl- ad reneng nz rece ptor anrta gon ists
rbamaiepi ne
Li pa se inhibitors
Lithium
frostangfandin
! Vitrimt n./-rr>jo.tf!-ra.l supplements
Adapter' *with pBfinilJum .III*. LR. Pardi t>5, splller Ft, SflTinJ Cfc. 5-ÿr*™«
13 APOW/WCOG ShailflPHH party
rppmt chronic dranH«a: d«Tiairiiati LlAaiifKdLlDn aiagncs-s J" GDiÿ-ÿcnrs rti"
Hepatol. 201 A. ?9"*-.25. (PMJO: 2.AA 1 7W?i &c*.TC 1 1 :/jgh.t33W.

K € V POINTS

• Acute diarrhea (of less than 2 weeks' duration) is usually


caused by infectious agents; chronic diarrhea (of more
than 4 weeks' duration) is mosi commonly nonin fectious,
* Maintaining hydration is the primary goal of treatment
for acute diarrhea.
* The primary causes of chronic diarrhea are irritable
bowel syndrome with predominant diarrhea, functional
causes, and medic at lons.
* Evaluation with imaging, endoscopy, and additional
testing should be reserved for patients whose symptoms
do not suggest irritable bowel syndrome or a functional
cause of chronic diarrhea

MKSAP 18 Made Easy Yasino.M.Habayeb


Gastroenterology and Hepatology Disorders of the Small and Large Bowel

TABLE 1 ?, Sources of FOCMAF Carbohydrates

Fructose: honey, apples, p&ars, peaches, mangos, fruit juice.


dried fruit
\ Lactose: milk, custard, tee cream
. yogurt, soft cheeses
! Fructans: wheat, rye, onions, leeks, 2ucchiri
Galectansr legumes
Sugar alcohols: x.ylit.ol', sorbitol, maltitol, mannitol
FODMAP - Fermentable Cligmacchÿridgs. Ois»ccK*nde*r ifl-snoiaeertarides. And
Prtlyo *.

it

FIGURE If. Dermatitis herpetilbfmis. A ro an i lestalio n of celiac: disease. Is


characterfczpd by pruritic papules and transient atmost immediately excoriated
blisters. on the elbows, knees, and buttocks..

MKSAP 18 Made Easy Yasino.M.Habayeb


Gastroenterology and Hepatology Disorders of the Small and Large Bowel

KEY POINTS
* The best initial test for celiac disease is testmg for IgA
tissue transglutaminase antibodies.
m Up to 40% of the general population carries the HLA-DQ2
or HLA-OQ8 mutation* so genetic testing alone cannot
be used to diagnose celiac disease
* The treatment of celiac disease Es lifelong avoidance
of gluren
- Tire most common cause of refractory celiac disease
symptoms is gluten exposure.

KEY POINTS
* Small intestinal bacterial overgrow ih is caused by vai i
ous conditions, including impaired motility, strictures,
or blind loops, and is treated with antibiotic therapy,
* Short bowel syndrome is defined by a small -intestine
length of less than 200 cm. resulting in maldigestion,
malabsorption, and malnutrition.
* Lactose malabsorption is common, with symptoms
occurring when lactose is ingested and subsiding with
exclusion of lactose from the diet

MKSAP 18 Made Easy Yasino.M.Habayeb


Gastroenterology and Hepatology Disorders of the Small and Large Bowel

TABLE 20. Features of Ulcerative Colitis and Crohn Disease


Feature Ulcerative Colitis Crohn Disease
Depth of inflammation Mucosal Transmurai
Pattern of disease Contiguous and symmetric Skips areas and asymmetric
Location Colorectum Mouth to anus
Rectal involvement Nearly 100% Less common
Ileal disease Backwash ileitis (15%) Common
Fistulas, abscess, and strictures Rare Common
Perianal disease Rare Common
Granulomas Unlikely In approximately 30%
Ovel rectal bleeding Common Less common
Tobacco use Protective Exacerbates

FIGURE 16. Erytmerma nodosum, a mantleslatian af inflammatory bowel


disease, typically appeals as iN-deFmed erythema overlying su.t>tLiianeoLrs, tender
nodules most commonly symmetrically located cm the anterior shins.

MKSAP 18 Made Easy Yasino.M.Habayeb


Gastroenterology and Hepatology Disorders of the Small and Large Bowel

* .

1
jrT5
c

-= Z.-*
/
■■Mi| MM
.
fiGtfitE. 17 Pyio-de rÿa g-j os.um, =t maiiife-static ™ of in ria*r«mÿ:o ry
diseÿaseÿ typically t-ngini af 3 s'n-iJ: puÿtuJe nr red nodule- th-at raprdfy tHfM-ncEs n
-an erJemsiousj ktfrterated. acUveay irlinfid harder and a p-air-fu!. -oÿt U5CHC
"
he- -hordflT is chafaaerisrica 1 1> violarÿau; wMh- an adaethaloverhanaÿ :he ultje-T-

Jb

*
rW> ■■'

r
FIGURE "IS. CT scan of ifre a bda mien and pelvis, in a pati-ent w if h Crchn
disease* showing smalt-bowEl obstruct ic-n i«ith drlaced loops o-f smalf intestine
Om?w) *nd mailed' loops of bowel Caÿre-w/nead'} in the pelvisÿ

MKSAP 18 Made Easy Yasino.M.Habayeb


Gastroenterology and Hepatology Disorders of the Small and Large Bowel

Disease Severity: Prior Therapy;


Mild Medicationÿ R"e &po isi?
Mocfftrat* -> Choice J off*ci5
Severe CompFhanc«

Dritribution of
Disease:
Pnodilis
Leflÿided
Ejcte-rvaive

FIGURE 19. Faciors DO consider when cheesing medical therapy For ulcerative
co U Lie.

IrfipHplSK "iir:
— ■"
- '.' " ~'
— - -- “
- _ ~“

FIGURE 2-0. Callageno-us cjo-MUs-: colon tnucau:l biopsy showing a pink,


abnormal siÿbeplthielial cnllagen band (drfOirfteorfl and lam in a propria ejepandad
by Intlirnmatoiyrells (afiow].

KEY POINTS
* The major symptoms of ulcerative colitis include diar
rhear abdominal discomfort, rectal bleeding, and tenes
mus, with a slow onset of symptoms.
* Fistula, abscess, and stricture are characteristic compli¬
cations of Crohn disease.
* Endoscopy with biopsy is needed to help make the
diagnosis of inflammatory bowel disease.
* The goals of therapy are to induce and maintain remis¬
sion of inflammatory bowel disease, and to prevent dis¬
ease- and treatment -related complications.
* Unlike patients with Crohn disease or ulcerative colitis,
patients with microscopic colitis are not at increased
risk for colorectal cancer.

MKSAP 18 Made Easy Yasino.M.Habayeb


Gastroenterology and Hepatology Disorders of the Small and Large Bowel

MKSAP 18 Made Easy Yasino.M.Habayeb


Gastroenterology and Hepatology Disorders of the Small and Large Bowel

TA a i_ E 21, SecQn<Jaj-y of Conslipÿticn


Mudicstions

Cp;o i nils
Anti -d : 21 1 rPi e y I &
I Ant i-choJimerg iCS {dnltspaÿmCH.1iC', ri parki n son ian d rugs,
t icyc I i-c jnti cd-e pore s s.a rits. a nti osych ok rzs J
'
| A.ntihi&ca'ninea
MSA IDs
Iron supplements
■C I loum sypplemerns
(ikSiTiij-Th

Antshype rter-5- ves foal r i urn channei O lockers, diurerics.


rd of id; n &]ÿ
S-s ro tors E> rgir F>ri (sgofi -&ts { o nd assetronJ i

Mechanical Causes

Colorectal
Rectncfila
ff-ecta I intussusception
Recta I prolapse
Sigmoidocele
■ Entarocele
:
Anjstorn o-titr strictu re

Ansi ■aien os is/st ri ct»jr&

£s trin sic com press io n trorr' pÿ-Lv ic/ÿfeidomir'icd I orucess

Systemic Alnesses

E ldocn n-o: og-io


Dia to-etos melfitus
Hyp-ot Fiyro id' Eim
F-? nlnypopi tu -tÿ ri snr>
pjtiesoch rom o crto m a

CS I u-crÿg onom a
Meciropath y/myopathy
'
Ahm fed Physiologic 5tat«

My per cini I n--e mÿa


H:ypjoScoEÿ£ m ci
Pregnancy
Po rps hyn ei
Heavy-metal poisoning iarsenlc, feS-cd, rr.errun,-)
Psychosocial

Depress ton
Ccg-nitive i mpÿ r ~n p? rr

Im m oto : 'icy
9

MKSAP 18 Made Easy Yasino.M.Habayeb


Gastroenterology and Hepatology Disorders of the Small and Large Bowel

TABLE 22. Treatments for Constipation


Intervention Mechanism of Action Considerations

Bulk Laxative Increases ability of stool to retain water Start with a low dose and increase slowly
Soluble fiber (psyllium, methylcellulose, Bulks stool Soluble fiber works better than insoluble fiber j
calcium polycarbophil, wheat dextrin) Speeds movement of stool through colon Bloating, distension, flatulence, and cramping
Insoluble fiber (bran, rye, flax seed) may be limiting
Stool Softener A detergent that allows water to penetrate Minimal effectiveness in clinical trials
the stool Only role is in mild constipation symptoms
Docusate sodium, docusate calcium
Well tolerated
Few side effects
Osmotic Laxative F*oorly absorbed compounds Polyethylene glycol 3350 and lactulose
improve stool frequency and consistency;
Polyethylene glycol 3350 Creates an osmotic gradient others have not been tested in dm ca! trials
Lactulose Water moves into the bowel lumen
Bloating and gas can be limiting
Magnesium hydroxide/magnesium citrate Caution with use of magnesium in renal
Sorbitol insufficiency
Stimulant Laxative Irritates the colon wall, increasing Quickest acting (works within 8-12 hours of
contractions ingestion)
Anthraquinones (senna, cascara)
Stimulates sensory nerves lining the colon Senna can cause melanosis coli (benign
Diphenyl methanes (bisacodyl, sodium pigmentation of the colon)
picosulfate) May inhibit water absorption in the colon
Diarrhea, cramping, bloating, and nausea can
be limiting
Secretagogue Lubiprostone activates type-2 chloride Lubiprostone improves stool form and
channels on enterocytes lining the gut frequency, straining, and abdominal pain;
Lubiprostone lumen, causing chloride ions to move into nausea is a common side effect
Linadotide the colonic lumen with sodium and water Linadotide and plecanatide improve stool
Plecanatide following the ionic gradient form and frequency, straining, bloating, and
Linadotide and plecanatide activate abdominal pam; diarrhea Is a common side
guanylate cyclase C receptors on enterocytes, effect
leading to chionde channel activation through
a series of processes within the cell
Serotonergic Agent Increases intestinal contractions Not available In the United States
Prucalopride Increases intestinal secretion Available in Canada and Europe
Neuromuscular Reeducation Retrains the skeletal muscle involved in Effective for dyssynergic defecation
(biofeedback) defecation (abdominal wall, pelvic floor, Requires a specially trained physical therapist
anorectal)

Corrects altered rectal sensation

KEY POINTS
* The most common cause of secondary constipation is
medication.
• Treatment strategies include dietary and lifestyle modi
Fication. addressing causes of secondary constipation.
arid various pharmacologic agents

10

MKSAP 18 Made Easy Yasino.M.Habayeb


Gastroenterology and Hepatology Disorders of the Small and Large Bowel

KEY POINTS
* Irritable bowel syndrome is diagnosed based on clinical
criteria and is no longer a diagnosis of exclusion.
* Many cases of irritable bowel syndrome can be effec¬
tively managed with reassurance, lifestyle modifica
tions, and dietary modifications.
Pharmacotherapy should target predominant symp
toms and be used if conservative treatment is not
effective.

KEY POINTS
* The evaluation for centrally mediated pain syndrome in
the setting of chronic abdominal pain requires a
detailed medical and psychosocial history and physical
examination with limited laboratory testing
♦ Narcotic bowel syndrome, also known as opiate
induced gastrointestinal hyperalgesia, is characterized
by the paradoxical increase in abdominal pain with
increasing doses of narcotics.

KEY POINTS
* Diverticulitis is the consequence of a diverticulum
becoming blocked, trapping bacteria, and subsequently
developing inflam motion

- The main presenting symptom of divert icLiiit is is col -


icky abdominal pain that is relieved with flatus or n
bowel movement,
* Acute diverticulitis is often a clinical diagnosis, usually
not requiring abdominal imaging.

11

MKSAP 18 Made Easy Yasino.M.Habayeb


Gastroenterology and Hepatology Disorders of the Small and Large Bowel

KEY POINTS
* Acute abdominal pain out of proportion to physical
examination Findings should immediateiy raise suspi¬
cion for early acute mesenteric ischemia.
* CT angiography is the recommended imaging method
for the diagnosis of acute mesenteric ischemia.

- The classic symptom triad of chronic mesenteric


ischemia is postprandial abdominal painT sitophobia
(fear of eating), and weight loss.
* Colonic ischemia is the most com anon form of ischemic
bovycl diÿasc, and most cases resolve spontaneously
and rapidly,

/V B

VI

- ■_

L'j

jf D

-
V

-J

F 4 E* LI ft £ ? -
Wemorrhoids ate vascular cu shions thÿt
al sphincter by sr*v e I I i JTI g up as neede-d to mafntam stoa)
r? the- musdes of ihe
-3rr Prolapsed
conrinÿrKe.
h-emorrhoi ifs inle-rnal hemorrhoids that protru de out of 1 he rectviim. The
hallmark of rectal prol apse (fi) is Ibe idefilifTcÿtion of to-hcenlfk rings of tint rectunn
protruding tfir-ough theanui. Prolapsed polyps (C, i5) pteiant as; mucosÿ troveFifd
globular masses prot-rud isn g froe*i the- SCLLS.

12

MKSAP 18 Made Easy Yasino.M.Habayeb


Gastroenterology and Hepatology Disorders of the Small and Large Bowel

.rS* -

FIGURE 2 3- . Anal fissures drepainfu1 fongitudmal mucosal tears in theanal cartel


IrugF ccv!t«ÿ cJ* RkhaiÿL. AtoflMtf, Fr-o!&« I sfl SÿrgBlY- fcBihigjn Unnÿs»it|f of Wfchtgart

* m~

F E 4
_ 1L.

yvÿ-EsI cancer presenting as- a n o Iceraied mass protruding fra m the


anal canai. The presentation of anal cancer can va ry fro- m an abstru-cBin g an a mass
to a nortpxolruding polyp or flat growl?' in cfae anal canal.
1a urlffÿ ot Richartf E. Burn pÿ. .
P— *-1 Frofersior D Mhfelgan McdicIrM ai U*» U n >w* rr ■ of P-* kc*i .g»n

KEV PO r MTS
» First iine therapy for hemorrhoids includes increased
fiber intake, adequate liquid in Lake, and avoidance of
straining,
* Fecal incontinence is grossly underreported due to
embarrassment, and health care providers should
inquire about it during clinic visits, particularly in
patients at risk,
*ÿ
Presenting symptoms of anal cancer may include bleed
ing, pain, or pruritus; however, 25% of cases present
without any anorectal symptoms.
* Patients with Crohn disease with longstanding perianal
disease are particularly at risk for anal cancer.

13

MKSAP 18 Made Easy Yasino.M.Habayeb


Gastroenterology and Hepatology Coloredal Neoplasia

KEY POINT

* Colorectal cancer is the thin! most common cancer and


second most common cause oF cancer -related mortality
among men and women in the United States.

KEY POINT
* Chromosomal instability is the most common mecha¬
nism responsible for the development of colorectal can¬
cer, accounting tor 85% of cases.

r-.-

Bja

FIGURE 25, A serrated polyp in the ascending coEÿsn, showing flat moipbology
that can make 1 hese padyps difficult to identify on cole nasenpy.

MKSAP 18 Made Easy Yasino.M.Habayeb


Gastroenterology and Hepatology Coloredal Neoplasia

KEY POINTS

- Non modifiable risk factors for cxilorectal cancer include


age (50 years and olderK male sex, black race, and a
persona* or family history of colorectal cancer.
• Modifiable risk factors for colorectal cancer Include diets
high in red and processed meat' Low intake of fruits,
vegetables, fiber, and dairy: use of alcohol and tobacco;
type 2 diabetes melJitus: sedentary lifestyle: and obesity.
* Longstanding inflammatory bowel disease (both ulcera
five colitis and Crohn disease) is associated with
increased risk for colorectal cancer.

KEY POINT
• The US. Preventive Services Tÿsk Force recommends the
use of low-dose aspirin for preventing colorectal cancer
and cardiovascular disease in individuals aged 50 and 59
years who are at increased risk for cardiovascular disease

KEY POINTS
* Screening for colon cancer should be individualized in
patients at increased risk.
• The risk for colon cancer depends on the size of the
adenoma, degree of villous histology, and presence of
dysplasia.

MKSAP 18 Made Easy Yasino.M.Habayeb


Gastroenterology and Hepatology Coloredal Neoplasia

TABLE 23. Screening for Colorectal Cancer in Individuals at Increased Risk


Risk Category Criteria Screening Recommendations (age; modality; interval)

Increased Family history of CRC;


CRC diagnosed in FDR <60 years old or Begin at age 40 years or 10 years earlier than age of youngest
two or more FDRs at any age FDR at diagnosis, whichever comes first, colonoscopy; repeat
every 5 years*
CRC diagnosed in FDR >60 years old Begin at age 50 years; any modality, repeat every 10 years*
Personal history of CRC Perform at time of diagnosis; colonoscopy; repeat at 1 year, 3
years, and, if normal, every 5 years thereafter until the benefit of
continued screening is outweighed by risks
High Familial adenomatous polyposis Begin stage 10- 1 2 years; flexible sigmoidoscopy or
colonoscopy; repeat every 1-2 years until colectomy
Lynch syndrome Begin at age 20-25 years or 10 years earlier than youngest
cancer in family; colonoscopy; repeat evey 1-2 years
Inflammatory bowel disease (Crohn disease Begin after 8 years of chronic colitis; colonoscopy with
or ulcerative colitis) biopsies; repeat every 1-2 years
! - -
CRC colorectal anew; FOR first degree relative (parent sibling, or child).
*lf baseline examination to normal
|
Data from Rax DK, Johr ton DA, Anderton JC ScKoaofaW PS, Swrlca CA» Inado'niJM: American College of Gastroenterology Ainarlcan College of Gastroenterology guidelines
for colorectal cancer screening 2009 'corrected) Am J Gastroenterol. 2009,104:739 50 JPMID: 19240699) do«:10.1Q38/ajg 2009.104 Rubinstein JH, Fnn« R. Heidalhaugh J.
Baricun A, Oirural Guidelines Committee Amen car Gast'oenteroiogical Association institute Guideline on the Diagnosis and Management of Lynch Syndrome Gastroenterology
2015,149 777-82, guv *14-7 IPM®. 24226577] do.:10.10537).g»nro.2015.07 036. Syngal S, Brand RF. Oa«h JM. Giaadwlb FM. Hampel HI. Bert RW; American College at
Oeit-oentero ogy. ACC clinical guideline: Genetic testing and management of heredtery gastrointesltnel cancer syndromes Am J Gastroenterol 2015;' 10:223-42- qua 263
[PMID: 254X55741 do :10.1038/ai9.201M35.

KfV POINTS

• The diagnosis of colored cancer is usual ]y made by


colonoscopy with biopsy
* Staging of colon cancer is based on tumor size and
extent of invasion into local tissues, lymph node
involvement, and evidence of metastasis.

MKSAP 18 Made Easy Yasino.M.Habayeb


Gastroenterology and Hepatology Coloredal Neoplasia

TABLE 24, Classification of Cofarectal Polyps


Adenomatous Polyps"

TubuFsf aoeroma
Tubulovrlbus adenoma
Villous adenoma
Serrated Polyps
Hyperplastic polyp
Sessile serrated polyp with or witJiout cytologic dysplasia
Traditional serrated adenoma
Other

Hamartomstous polyp
Inflammatory polyp
■W'th OfwWxiul bagfl cyspbsia.

TABLE 35. Surveillance for Colorectal Cancer After Scrgening or Polypectomy


Adeno IT atom Polyps Interval to NextColonoseopy

1-2 tubular adenomas <10 mm in size 5-10 years


3-1Q adenomas 510 mm in site, villous histology, or higÿ grade dysplasia 3 yeah
£1Q adenomas on single examination <3 years; a genetic cause of disease should be
investigated
Serrated Polyps Interval to Next Colonoscopy

Rectosigmoid hyperplasticpolyps <10 mm in size 1 0 yean

SSP <10 mm in size 5 years


SSP >10 mm in size or SSP with dysplasia or TSA 3 years

Serrated polyposis syndrome lyear

| S5f «:«lils polypi; TSA * Irad'li inal wfrrdted aden0W.

MKSAP 18 Made Easy Yasino.M.Habayeb


Gastroenterology and Hepatology Coloredal Neoplasia

*ÿ’
r 5 s*.
>
; i '*• / v
'i
* u
t
V f-
.01:

-
r
t<M! fflP
t. h '
>:0fl
/
f

V V
t
■■' V
r
‘ t

?v!

*
1

f
* *' r -*i *+'

Ml Mg i
FIGURE 26 . Intestinal features of familial adenomatous polyposls(FAP). Patients with FAP can develop polyps throughout the gastrointestinal tract. Adenomatous polyps
always develop in the colon. Duodenal adenomas can also develop in patients with FAP, especially Involving the ampulla of Vater. Regular upper endoscopic surveillance is
indicated to remove polyps larger than 10 mm (left). If a patient with FAP has had surgery and has an ileorectal anastomosis, that patient must continue to be surveyed
regularly because rectal polyps can develop in the remaining rectum (center). Numerous fundk gland polyps of the stomach can develop in FAP (ngfrf).

m \

J
FIGU1E 27. Juvenile pcfyp5 in [Ire talon ot A patient with juven-ile polyposis
syai d rorne, on endnsoopy. JcnreniJe p-olypÿ tan have on erythecrjiOLisarHd
A
appearance,

MKSAP 18 Made Easy Yasino.M.Habayeb


Gastroenterology and Hepatology Coloredal Neoplasia

*
c 4
••
%

A
%•• i
0
'
#• *•
.
iP

L
FIGURE 28. Peutz Jeghers syndrome (PJS) polyps. Capsule endoscopy showing a PJ5 hamartomatous polyp in the small intestine (left); polyps in the small intestine can
cause bleeding and/or obstructions Colonoscopy showing a hamartomatous polyp in the colon of a patient with PJS (right)

m ■

.
FIGURE 2 9 Peutz-Jÿghers syndrome is- watts truLLiple
ha martomstous polype >r» the gÿstroincesiiRal trad and distinctive mucotutaneoui
pigmentations. The pigmented Insians occur moss commorrly on the irps andl
pynordl region but can a Ysa occur an the nose, perianal area, and- g ertil-a Is.

KEY POINTS
* Lynch syndrome is causeni by germline muiations in rhe
mismatch repair geires and carries a lifetime risk, [or
colorectal cancer of 50% to 80%.
• Syrndromes that predispose persons to multiple adeno
matous polyps in tine colon include familial adenoma
tous polyposis, MtttYH associated, polyposis, and poly-
merase pmotYeading associated polyposis.
* Hamartomatous polyposis syndromes are associated
with increased risk for multiple cancers, including
colon cancer.

MKSAP 18 Made Easy Yasino.M.Habayeb


Gastroenterology and Hepatology Disorders of the Liver

KEY POINTS

* Elevations of aspartate ami rsotransfemse and alitnine


amiaotxansl'eEase levels represent laepatic parenchymal
inflammation; alanine a rnitrotrarts fera se levels are more
specific for hepatic inflammation.
* Elevations in alkaline phosphatase and btTinrbin levels
result from infla mnaat ion of the biliary tree or bile flow
ahnormali t: i es.
* The pint hroml nn time and serum albumin levels reflect
the synthetic function of the liver.

TABLE 26. Typical Liver Chemistry Studies in Common Hepatobiliary Disorders


Disease AST ALT AW Bilirubin Other Features

Acute viral hepatitis TTT TTT Normal to t Normal to TTT Exposure history, fatigue,
nausea
Chronic viral hepatitis T n Normal to T T if advanced History of exposure to infected
blood or body fluids
Nonalcoholic steatobepatitis T T Normal to t Normal Metabolic syndrome
Alcoholic hepatitis TT Normal or T T Normal to TTT Excess alcohol intake
Acute autoimmune hepatitis rn TTT Normal to T Normal to tt Autoantibodies
Chronic autoimmune hepatitis T TT Normal to T Normal Autoantibodies
Wilson disease T T Low T and often Hemolysis if acute, neurologic
unconjugated symptoms if chronic
a,-Antitrypsin deficiency T t Normal I if advanced May have pulmonary disease
i Hemochromatosis Normal Normal Normal Normal Joint symptoms, family history, j
other organ involvement
Primary biliary cholangitis T T TTT T if advanced Female, sicca symptoms,
antimitochondrial antibody
Primary sclerosing cholangitis T T TTT T if advanced or Ulcerative colitis, abnormal
dominantstricture cholangiogram
is present

Large duct obstruction TfTtif T(TT if TT Tt Pain if acute, dilated ducts on


acute) acute) imaging
Infiltrative liver disease T T ttt Normal Features of malignancy, sarcoid,
amyloid, or mycobacterial or
fungal infection
Hepatic ischemia TTT TTT Normal Normal AST >5000 U/L. history of
hypotension
Celiac disease Normal or T Normal or T Normal or T Normal Usually other features of celiac
disease

-
ALP alkaline phosphatase, ALr - alan ne jrmogi'arsferjsc AST
- aspartate aminotransferase.

MKSAP 18 Made Easy Yasino.M.Habayeb


Gastroenterology and Hepatology Disorders of the Liver

TABLE 27. Risk Factors Requiring Testing for Hepatitis B Virus

Individuals bom or raised in regions with high rates of hepatitis B virus infection, including Asia, Africa, the South Pacific, European
Mediterranean countries. Eastern Europe, most of South America, Honduras, Guatemala, and the Middle East (except Israel and Cyprus)
U.S.-bom persons not vaccinated as infants whose parents were born in endemic areas
Household or sexual contact with hepatitis B surface antigen-positive persons
Intravenous drug use
Multiple sex partners or history of sexually transmitted infection
Men who have sex with men
History of incarceration
History of hepatitis C vims or HIV infection
Hemodialysis
Pregnancy
Elevated aminotransferase levels of unknown cause

TABLE 28. Interpretation of Hepatitis B Virus Test Results


Clinical Scenario HBsAg Anti-HBs IgM IgG HBeAg Anti-HBe HBVDNA
anti-HBc anti-HBc [lUMLj

Acute hepatitis B, occasionally t + >20,000


reactivation of chronic hepatitis B
Resolved previous infection t M- Undetected
Immunity due to previous vaccination + Undetected
False positive anti-HBc or resolved + Undetected
previous infection
Immune-tolerant chronic hepatitis B + + t >1 million
(perinatally acquired, age <30 years)
Inactive chronic hepatitis B 1 + <10,000
HBeAg-positive immune-active + + + >10,000
chronic hepatitis B
HBeAg-negative immune-reactive + + r >10,000
chrome hepatitis B
Anf HBc * h«)Jtiti[ 9 COT intibody; anti'HBe “ hepjtti? 3 » jrtkxjdy; untiHfij » hopcitis 6 induce antibody; HftoAg > hipililH 3 1antigen, HBsAg •hepilrtis 6 surldCK
inligin, HBV- l*|Miiin Bwui

MKSAP 18 Made Easy Yasino.M.Habayeb


Gastroenterology and Hepatology Disorders of the Liver

Vertically Horizontally
acquired HBV acquired HBV

'’ ▼
Immune tolerant Immune active
(HBeAg-positive chronic HBV) (HBsAg-positive chronic HBV)
Age <30 years Abnormal ALT
Normal ALT HBeAg positive, anti-HBe negative
HBeAg positive, ant HBc negative ALT abnormal
HBV DNA >1 million HBV DNA >10.000 lU/ml
No inflammation or fibrosis Inflammation and fibrosis

Immune control
t
(chronic HBV inactive carrier)
Normal ALT
HBeAg negative, anti HBe positive | Liver injury
HBV DNA <10,000 lU/mL
No inflammation, variable fibrosis

Reactivation
(HBeAg negative chronic HBV)
Abnormal ALT
HBeAg negative, anti-HBe positive
HBV DNA >10,000 lU/mL
Inflammation and fibrosis

FIGURE 30. Phases of chronic hepatitis B infection ll is assumed that patients progress through the phases in sequence, although not all patients develop HBeAg
negative chronic hepatitis B, and only patients with vertical transmission of hepatitis B have a clinically recognized immune-tolerant phase All phases have positive HBsAg.
negative ant»-H8s, and positive igG anti HBc.
-
- -
AU iliniM immrMnrftnu, me Hk hepiiitn B core
HBV •HpaUIhIvtnis; HBV DM htyMlin Brims DHL
jnh H3(
- - - -
hrpjttii 9 r irttoCy ami Hfc hepatitis IluiicmntboOf HBrAj tepjtiM 9 1rrOgt- HlbAg hrMtikeiuCK* antigen

TABLE 2ÿ. Risk FJ ct u rs far EJ-e veloping Cir-rhasis or


Ca r c tnom d fn Patients with Chronic Hepatitis
B Vi r uS I n f E ert i o n

Age o!dei th-rfm 40 yes rs


■l&patilis. 0 virus DMA fevcl :U/niL
FEcjateef BTiiroTran sferase ievel
Genorype C infection
Heqvyÿloohot use
Ot-vtebpm E rrf of hepatitis B e antig-en.-oeQ'ativie reactivation
phase of chronic R vlnjÿ infection
HIV fnÿecthon
Hep arms C virus or hep-atitfs D virus infection

MKSAP 18 Made Easy Yasino.M.Habayeb


Gastroenterology and Hepatology Disorders of the Liver

TABLE 30. Co n cf jt i on s Requiring Testing for H e pa irtjs C


Vjj-ius

ysÿr 1 945- 1
i
JnjectFo n-d rug use orintranasaf illicit-drug use foiyer)
Lo-ng-terrrs heiTiOdralysis f ever)
P&rcutarteeus/ pa re ntpra I exposures in an unregulated setting
(nonsterile technique)

Needlest cks, sharps, or n-ucosa! exijosure to nepatitrs C v; rus


irs fe trt «=5H b.loo-iH

Child re n born to- wo m en infracted with be-p-a-titis C virus


Receipt of b ood or bbod -compone nts trangfus'on of organ
trÿrtspJantdlFun before 1992
fiece. ot of clotting- Fa-Ctor cancentrates produced before ?
History of incarceration
HIV infection
Sexually active persons a bou t to stÿrt prefinpusirre prophyi axis
for HIV
Undiagnosed chronic liver disease
Elevated alanine a minotra nsferase levei
Living organ donors, before demotion

TieatmcnE Rcgifn-ens tor H cp fttit i s (Z Vims


I rtfe ctF-o n
Drug Troatnent R«glmens
Crazoprevir' elbasuir*1
-+ÿ

Paritnpn&vir11 -+ ombilaswi* + dssabuvir11


Sim&previr1 4- sofosbiivuÿ
Daclatasv i r=* -+- sofosb uvird
|.ecJ[pasvirh + sofosbuvir'1
Vo Ip-atas-v i rs + SO to sbuvi H3

dan: NS3/4A prolojt* ir'J’ri fciiI;Of f-pro-vlfl


HÿTVR daos: N"Si5-A irthlbi(&rC-95vÿI

TMSEiH ndn-ntJCl«OGMHe polymerale mhiDitor l-buvjf)

nkjcf v> -ci11 cJ C" iptÿymeÿ-asÿ irt hit-uEo- [*btnrifl _I

MKSAP 18 Made Easy Yasino.M.Habayeb


Gastroenterology and Hepatology Disorders of the Liver

KEY POINTS

* Treatment of hepatitis A virus infection is supportive*


and 90% of patients or more recover fully within 3 to 6
months of infection.
* Treatment of hepatitis B virus infection is advised for
patients with acute tiver failure, infection in the
immune-active phase or the reactivation phase, cirrho
sis, and in immunosuppressed patients.

* Individuals horn between 1945 and 1965 require one


time testing for hepatitis C virus,
* Up to 95% of patients with hepatitis C virus infection
can be cured with direct-acting antiviral agents.

KEY POINTS
• Diagnosis of autoimmune hepatitis is made based on
the presence of elevated aminotransferase levels, posi
rive antinuclear antibody and smooth muscle antibody,
elevated (evels of fgG, and compatible li ridings on liver
biopsy
* Wilson disease, viral hepatitis, and drug- induced liver
injury need to be excluded before making the diagnosis
of autoimmune hepa t itis .

MKSAP 18 Made Easy Yasino.M.Habayeb


Gastroenterology and Hepatology Disorders of the Liver

KEY POINTS
* The Alcoholic Liver Disease /Nonalcoholic Fatly Liver
Disease Index score can be helpful in distinguishing
alcoholic liver disease from nonalcoholic fatty Hver
disease.
• M a n i fe stati ons of a I cohol ic h epa ti ti s include fever.
jaundice, tender hepatomegaly, and leukocytosis
• Severity of alcoholic hepatitis is determined by the
Maddrey discriminant function score; patients with a
score of 32 or greater or with hepatic encephalopathy
may be considered for prednisone therapy,

KEY POINTS
• tadaminophen, antibiotics (particularly amoxicillin-
davulanate}* and a|}tiepileptic agents (phenytoin and
valproate) are the most common causes or drug -induced
liver injury.
* L iver injury ly pscally resolves after discontinuation of
the offending drug.

MKSAP 18 Made Easy Yasino.M.Habayeb


Gastroenterology and Hepatology Disorders of the Liver

TABLE 32 . Causes of Acute Liver Failure


Ac-et a mi rtoph-e n
Hepatitis A Virus
Herpes simplex virus
Aut&frnrrturie
Other medications (anti tuberculosis drugs. sulfa-containi-ng
an timicrobi Ft! agents, ar.trfi_ingat agents, and herbal
supplements}
Hepatitis B virus
Amanita phalfotd&s mushroomi
Acute -fatty liver of pregnancy

KEY POINTS
* The most common cause of acute liver failure in the
United States is acetaminophen overdose.
* The diagnosis of acute liver failure requires immediate
referral to a liver transplant center,

KEY POINTS
• Nonalcoholic fatty liver disease is the commonest cause
of liver disease in the world .
- Managemeut of nonalcoholic liver disease is focused on
weight loss through dietary and lifestyle modification.
• oti - Antitrypsin deficiency is art autosomal recessive
genetic disorder that results in accumulation of a vari¬
ant protein, in the liver; homozygosity can result in liver
injury and cirrhosis.
* Hereditary hemochromatosis is a condition character
i7ed by excessive iron absorption that results in accu¬
mulation of iron in the liver and the development of
cirrhosis.
* Wilson disease is a rare autosomal recessive disorder
that causes accumuiatiOT) of copper in the liver.

MKSAP 18 Made Easy Yasino.M.Habayeb


Gastroenterology and Hepatology Disorders of the Liver

K £ Y POINTS

* Jn patients with primary biliary cholangitis, ursodeoxy


cholic acid treatment results in histologic improvement,
better survival rates, and diminished need to for liver
transplantation
* Primary sclerosing cholangitis is associated with inflam¬
matory bowel disease in about 85% of cases: these
patients have an increased risk for colorectal cancer and
require surveillance colonoscopy every 1 Lo 2 years-

* Primary sclerosing cholangitis often requires liver


transplantation and has the highest case-based mortality
rate among the autoimmune liver diseases

TABLE 33. Child-Turcotte-Pugh Score1


1 Point 2 Points 3 Points

Encephalopathy None Grade HI Grade IH-IV


Ascites None Mild/moderate Severe
Bilirubin <2mg/dL(34.2pmol/l) 2-3 mg/dl (34.2-51.3 pmol/l) >3mg/dl(51-3pmol/L)
Albumin >3,5g/dL(35g/L) 2.8-3.5g/dl(28-35g/l) <2.Bg/dL(20g/l)
Prothrombin time/lNR <4s/<1 7 4-6 s/1.7'2.3 >6s/>2,3

-
■5-S points •Chld Tu'cottt Pjgh dass A; 7 9 points ChlldTufcotte-Pugh class 8; 10 lb points » Child Turcot®- Pixjh class C

X E Y POI NT

- The Model for Hnd Stage Liver Disease- sodium formula


accurately predicts 3-month mortality and is used for
liver transplant allocation.

MKSAP 18 Made Easy Yasino.M.Habayeb


Gastroenterology and Hepatology Disorders of the Liver

TABLE 3*. Management of Esophageal Varices in Patients with Cirrhosis


Findings Management

No varices Repeat upper endoscopy in 3 years (unless decompensation occurs)


Small varices (<5 mm) Repeat upper endoscopy in 2 years (unless decompensation occurs)
Small vances withred wale marks (erythematous raised areas) Initiate nonselective p- blocker therapy (propranolol nadolol, or caivedilol)
Large vances (>5 mm) Initiate nonselective therapy or endoscopic variceal ligation
Varices in patients unable to tolerate p-Wockers Endoscopic variceal ligation

TABLE 35 „ Hepatic Encephalopathy Classification


Severity of Description Findings
Encephalopathy
Minimal Detected only on neuropsychiatry testing Few or absent clinical manifestations
Grade 1 Mild lack of awareness, sleep disturbances Oriented to time and place, butwith psychomoto'slowing
Grade 2 Lethargic, disoriented, and with personalitychange Disoriented to time, with asterixis
Grade 3 Somnolence, confusion, and significant disorientation Disoriented to time, place, and situation, with asterixis
Grade 4 Coma Lack of response even to painful stimuli

Adapted with perff ijjton Iron* Vibfrup H, Arrad io P, Baja) J, Cwdaba J, Fermo P, Mul#n to, al Hepatic ernephalcpatty i -t coronic live- 4 wise- 2C14 practice guideline by
the Anwcjn Anociitkm tor the Study o( Liver Diseases arid the Furapem Aaocatwi fa the Study of the Lrw Hepatology. 2C 1 4:60:71 5-35. \m’l 2M«402| dodO. 1002/
hep.27210,

MKSAP 18 Made Easy Yasino.M.Habayeb


Gastroenterology and Hepatology Disorders of the Liver

TABLE 36. Characteristics of Ascites


5AAG Ascitic Fluid Total Other Characteristics
Protein Level

Cirrhosis High* LCHAÿ Clear, straw-colored


i
Heart failure High* High4
Chylous ascites Lowb or high" High*1 High triglycerides
SMG can be high if caused by portal hypertension
SAAG is low if caused by lymphatic disruption
Tuberculosis Low1* High** Peritoneal biopsy may be required to confirm diagnosis of
tuberculosis
Malignancy Low* High0 Positive cytology
Nephrotic syndrome Low1 Low* Proteinuria and edema

-
SAAG serum-ascites albumin gradient.

1 g/dl(119/.)

»<1.1 g'dUHg/U
'<2.Sgrdll2Sg/l)
-M.5gMl(K gil)

KEY POINTS

• Upper endoscopy should be performed on ail patients


with cirrhosis to assess for the presence of varices: pri
mary prophylaxis to prevent bleeding includes nonse
lective (5 blocker therapy (propranolol, nadolol, or
carvedilol) or endoscopic variceal ligation.

• Precipitating causes of hepatic encephalopathy may


include infection, sedating medications, volume deple¬
tion or gastrointestinal bleeding; treatment involves
addressing the precipitating cause and lactulose.
• Ultrasound is the most effective means of ascertaining
the presence of ascites: the serum-ascites albumin gra¬
dient and total protein levels in ascitic fluid assist in
determining the cause of ascites.
• The management of ascites involves a sodium - restricted
diet and diuretic therapy with spironolactone.
• An ascitic fluid neutrophil count of 2SO/pLor greater
confirms the diagnosis of spontaneous bacterial perito¬
nitis; patients who develop this condition have a high
mortality rate and should be considered for liver
transplantation.
• Mepatopulmonary syndrome- should be suspected in
patients with portal hypertension who have symptoms
of dyspnea and evidence of hypoxia; the diagnosis is
confirmed by demonstrating intrapulmonary shunting
with agitated saline administration during
echocardiography.

10

MKSAP 18 Made Easy Yasino.M.Habayeb


Gastroenterology and Hepatology Disorders of the Liver

KEY POINTS

• Patients with cirrhosis should avoid alcohol, sedating


drugs including opioids, and raw shellfish,
* Patients with cirrhosis should be screened for osteopo¬
rosis; confirmed osteoporosis should be treated with
either oral or imervenous bispfrosp ho nates.
* Patients with chronic liver d isease should be vaccinated
against hepatitis A and R viruses and should receive
annual influenza vaccination and recommended pne\i
mococcal vaccinations.

KEY POINTS
• Asymptomatic hepatic cysts arc benign and require no
follow-up.
* Focal nodular hyperplasia, a common incidental find¬
ing. does not have malignant potential or a risk for
bleeding, and does not require follow up.
* Hepatic adenomas 5 cm in size or smaller can be man
aged with serial imaging; hepatic adenomas larger than
5 cm m size should be considered for surgical resection
* Patients with cirrhosis and high-risk patients with hep¬
atitis B virus infection require hepatocellular carcinoma
surveillance using liver ultrasound with or without
cx- fetoprotein measurement every 6 month s.

KEY POINT
* Referral to a transplant center is indicated for patients
with acute liver failure or for patients with cirrhosis
with a Model for End-Stage Liver Disease score of 15 or
greater or decompensated cirrhosis.

11

MKSAP 18 Made Easy Yasino.M.Habayeb


Gastroenterology and Hepatology Disorders of the Liver

KEY POINTS
* Measures to reduce venical transmission of hepatitis B
virus include adm inis I ration of hepatitis B virus
immune globulin to newborns and immediate vaccina
tion of newborns

- The most serious liver com plications of pregnancy


occur in the third trimester and include HELLP
(Hemolysis, Elevated Liver enzymes, and Low Platelets)
syndrome and acute fauy liver of pregnancy; both are
managed in high risJk obstetrical units and with early
delivery,

• Portal vein thrombosis is Common in patients with


decompensated cirrhosis and is a consequence of poor
flow through the portal veins
* The classic presentation of Budd -Chian syndrome
includes hepatomegaly, ascites, and right upper quad¬
rant abdominal pain.

12

MKSAP 18 Made Easy Yasino.M.Habayeb


Gastroenterology and Hepatology Disorders of the Gallbladder and Bile Ducts

KEY POINT
* Gallstones that are found incidentally do not typically
cause any symptoms* and cholecystectomy is generally
not recommended because most stones remain
asymptomatic.

KEY POINT
• Patients with typical biliary colic symptoms and gall
stones on imaging should undergo cholecystectomy

KEY POINTS
* The diagnosis of acute cÿolecystit is can be made by
ultrasonography showing gallbladder wall thickening
and or edema and sonographic Murphy sis11
* Treatment of acute cholecystitis includes pain control,
intravenous antibiotics with gram -negative and anaero
bie coverage, and cholecystectomy during the initial
hospitalization in good candidates for surgery.

1
MKSAP 18 Made Easy Yasino.M.Habayeb
Gastroenterology and Hepatology Disorders of the Gallbladder and Bile Ducts

KEY POINTS
* Acalculous cholecystitis typically occurs in critically ill
patients and Is a result of gallbladder ischemia that can
be complicated by enteric bacterial infection.
* Left untreated, the mortality rate for acalculous chole¬
cystitis is as high as 75%.

KEY POINTS
* Endoscopic retrograde cholangiopancreatography Is the
preferred therapeutic method for relieving obstruction
due to A common bile duct stone
* Cholangitis (heralded by the onset of fever; jaundice,
and right upper-quadrant abdominal pain) is poten¬
tially life threatening' antibiotic therapy targeting
gram negative £n re ra bacte riaceae should bead minis
icred, and identified common btie duct stones should
be removed urgently.

KEY POINT
• Gallbladder polyps greater than 1 cm in size and those
associated with gallbladder stones or primary sclerosing
cholangitis are more likely to he neoplastic

2
MKSAP 18 Made Easy Yasino.M.Habayeb
Gastroenterology and Hepatology Disorders of the Gallbladder and Bile Ducts

Gallbladder polyp $5 fnm Gallbl»dd*r polyp A-9mm Gallbladder polypiI cm Gallbladder polyp [any sue) +
gallstones, biliary colic
,r or primary sclerosing cholangitis
Repeat ultrasound in 1 2 months Repeat ultrasound In 6 months Cholecystectomy * gallbladder polyp >8 mm
Cholecystectomy it t size then yearly
Cholecystectomy it t size
Cholecystectomy

KEY POINTS
• Resection is first-line treatment forcholangiocarcinonia
* Selected patients with unresectable hilar cholangio
carcinoma smaller than 3 cm in size and without
extrahepatlc spread may be candidates for liver
transplantation.

3
MKSAP 18 Made Easy Yasino.M.Habayeb
Gastroenterology and Hepatology Gastrointestinal Bleeding

KEY POINT

• upper gastrointestinal bleeding is mure common, more


severe, and has a higher mortality rate than lower gas
trointestinal bleeding

TABLE 37 , Less Common Causes of Upper Gastrointestinal Bleeding


Lesion Pathogenesis Presentation Treatment
Cameron erosion Mechanical trauma to mucosal Typically chronic Gl bleeding Includes medical therapy with PPI
folds of hiatal herma presenting as iron deficiency and iron, and surgical repair of
anemia hiatal hernia
Dieulafoy lesion Dilated, aberra nt submucosal Included in differential diagnosis of Endoscopic
vessel recurrent, often massive bleeding
without dear source
Gastric antral Most idiopathic; associated with Acute bleeding or iron deficiency Endoscopic
vascular ectasia cirrhosis and systemic sderosis anemia
Aortoentcric fistula Direct communication between "Herald' bleed followed by Surgical
aorta and Gl tract massive exsanguination
J pancreaticus
Hemosuccus Erosion of pancreatic pseudocyst
or tumor into a vessel with
Upper Gl bleeding in setting of Mesenteric angiography with coil
pancreatic disease embolization
bleeding into pancreatic duct
Hemobilia Bleeding from the hepatobiliary Triad of jaundice, biliary colic, and Angiography or surgical
tract often caused by a rteriobiliary Gl bleeding
bstula from trauma or liver biopsy
UpperGl tumors Benign or malignant neoplasms Slow or massive hemorrhage Palliative radiographic and
endoscopy for malignant tumors,
surgical resection for benign
tumors
- -
G gastrointestinal, PPI proton pump inhibitor.

MKSAP 18 Made Easy Yasino.M.Habayeb


Gastroenterology and Hepatology Gastrointestinal Bleeding

K A

FIGURE 33. left Duodenal ulcer with nonbleeding visible vessel {arrow) that Is at high risk for rebleeding and must be treated endoscopically. Right: Active arterial
spurting {dotted arrow) from a duodenal ulcer [solid arrow). This lesion is at the highest risk for reWeeding and must be treated endoscopically
Caui*t> oC louts M Won} IMSOH* MD, Miyo Clhvc.

> -

FIGURE 34. D uod i?nal u\ce r ivil h idhecent clot w.v} th at ts ar ri sk for
re-bleedin g. Th is un be treated tried its My at by clol ramoual and endoscopic
-herapy in addition to .standard ■traedi-cal therapy
HuLkJleSh- oh LDU<5 W wtjr jHtc Son j,ME. Hays CSnic

MKSAP 18 Made Easy Yasino.M.Habayeb


Gastroenterology and Hepatology Gastrointestinal Bleeding

r
■7*
9

w
ft
m

-
FIGURE 35. Ulcers at low risk for rebleeding, lor which endoscopic therapy is not indicated. Left: Clean based gastric ulcer with no blood vessels, pigmented spots/
protuberances, oi dots noted in the base, Right Nonprotuberani pigmented spot (arrow) in a duodenal ulcer bed
UunarUUus M. Wooj[»Swg,MfcMipOhit

¥ ' %

FIGURE it. Ma !Eo ry Weiss tear. fc tu pe rfi cÿsl 1 1 near m ucosai tea rÿrrow'! seen
on enctascopic nerroftescion in the proximal ■stcwnatfi.
c*a J c* ■ s M. Wang Kee Snog. Wo. r*avn dime.

MKSAP 18 Made Easy Yasino.M.Habayeb


Gastroenterology and Hepatology Gastrointestinal Bleeding

l-Gtl ft £ 37, Arude vÿÿkeaf iVernc ri+ia ge. in the distal


ciophag us is s-een ipuitiftg bright j&d blood.

KEY POINTS
The most common causes of upper gastrointestinal
bleeding include peptic ulcer disease, gastroesophageal
varices, and Malory Weiss tear.
Tachycardia (pulse rate >IOO/min), hypotension (sys
tolic blood pressure clGO mm Hg), age older than 60
years, and major comorbid medical conditions are at J
associated with increased risk for rebleeding and death
in patients with upper gastrointestinal bleeding.
Upper endoscopy is I he primary diagnostic modality for
evaluating upper gastrointestinal bleeding
* Management of an ulcer depends on the endoscopic
appearance and risk for rebleeding.

MKSAP 18 Made Easy Yasino.M.Habayeb


Gastroenterology and Hepatology Gastrointestinal Bleeding

TABLE 3S, Cause? of Severe Lower Gastrointestinal


Breeding
Drve rticu!o5is
Aortoe nterc fi sruta
Colonic or rectal varices
□Heul fcy lÿicr>£

Neoplasm
Colitis
Ischemic
In’1Ui mm bowel disease
Infections
Intussusception
Meckel dtverticLlum
Angiodysp l-asia

K £ V POINTS

* Patients with lower gastrointestinal bleeding usually


present with sudden onset of herfiatocheidat (maroon or
red blood per rectum).
* Most eases of lower gastrointestfea! bleeding stop spon
taifeousJy and have good outcomes- however, higher
rates of morbidity' and mortality are seer in older
patients and in those with comorhid conditions
* Colonoscopy identifies a source of lower gastrointestinal
bleeding in two thirds of patients.

MKSAP 18 Made Easy Yasino.M.Habayeb


Gastroenterology and Hepatology Gastrointestinal Bleeding

TABLE 39. Causes of Small-Bowe! Gastrointestinal


Bleeding
Differential Patient Clinical Clues
Diagnosis Age (Years}

Angiody&pla&ia >60 -usually -occult


:n|yrmittBnt.
bleeding, may a Iso occur in
the colon
Peutz-Jeghers <r3G Perioral pigmentation,
syndrone obstru-CtFve symptoms
M ecke I 20-60 RassFbl-e abdominal pain
diverticu-ium
Hema ngiomÿ <2Q Possible cutaneous
I hemangiomas
Malignancy >50 Weight Joss, abdominal pain
Hereditary >50 Mucocutaneous
hemorrhagic tela ngfe ctas-ies
telangiectasia
-

~"-v

FIGUkE 3 9. Hecedita*y he mg rrfi agic (elangfetf esia (Osÿf-Weber-flendu


syndiDnie] is a disorder of develc.pr-.eric erf the- vasculature characterised by
t-el angtecfiSM and a-rteriov-enous maHbnTvariorvs *n specific locations. IE is one of
the mosl common monaÿemc disced1ers, but affetlecf Frulividej-als STC- frequently
itDHJjagFiDwd. The mÿ.t common features gf The disorder— nosebleeds and
■efangiectase* on the lips, hands, and oral mucosa— are often quite subtle.

MKSAP 18 Made Easy Yasino.M.Habayeb


Gastroenterology and Hepatology Gastrointestinal Bleeding

•m
r: r
ill! Iy
mm
‘'Wppmimi I! MS! SIS
i IT
i -ÿ
MEL

o cm 3 A
| I
F({i U R[ 40, Endoscopy capful Pg-_
of Biiabei' hL3| jrn. *.1D. Mdry.- IT I : n t_

KEY POINTS
■ Capsule endoscopy is the preferred test for evaluating
stable patients for small bowel bleeding after normal
result on endoscopy and colonoscopy.

After achieving hemodynamic stabilization, therapy for
small- bowel bleeding is guided by the underlying
source of bleeding.

MKSAP 18 Made Easy Yasino.M.Habayeb


Gastroenterology and Hepatology MCQ Answers and Critiques

Item 1

Educational Objective: Treat opioid-induced constipation.

KEY POINT
• Oral naloxegol is a peripherally acting p -opioid recep¬
tor antagonist that is FDA-approved for the treatment
of opioid -induced constipation in adults with chronic
noncancer pain.

ltem2

Educational Objective: Manage the immune-tolerant phase of hepatitis B viral infection.

KEY POINT
♦ Patients with hepatitis B infection in the immune-
tolerant phase require serial monitoring of
aminotransferase levels.

Item 3

Educational Objective: Manage aspirin use before and after polypectomy.

KEY POINT
• Aspirin for secondary prophylaxis in patients with
established cardiovascular disease should be contin¬
ued after colonoscopy with polypectomy.

1
MKSAP 18 Made Easy Yasino.M.Habayeb
Gastroenterology and Hepatology MCQ Answers and Critiques

ltem4
Educational Objective: Diagnose nonalcoholic fatty liver disease.
KEY POINT
• Nonalcoholic fatty liver disease is the most common
cause of abnormal liver test results in the United
States.

Items5
Educational Objective: Diagnose pseudoachaJasia.
KEY POINT
• Pseudoachalasia is caused by a tumor at the gastroe
sophagea! Junction infiltrating the myenteric plexus
causing esophageal motor abnormalities, symptoms,
barium imaging and manometric findings, and endo
scopic appearance are similar to achalasia.

ltem6
Educational Objective: Treat diarrhea-predominant irritable bowel syndrome.
KEY POINT
• A low FODMAP (Fermentable Oligosaccharides,
Disaccharides, Monosaccharides, And Polyols) diet
can reduce abdominal pain and bloating and improve
stool consistency, frequency, and urgency in patients
with diarrhea predominant irritable bowel syndrome

ltem7
Educational Objective: Diagnose a Zenker diverticulum.
KEY POINT
• Patients vvitli dysphagia associated with regurgitation
of undigested food should be evaluated with a barium
esophagrarn. for the presence of a Zenker diverticulum.

2
MKSAP 18 Made Easy Yasino.M.Habayeb
Gastroenterology and Hepatology MCQ Answers and Critiques

Items 8
Educational Objective: Diagnose hepatopulrnonary syndrome.

KEY POINT
• The diagnosis of hepatopulmonary syndrome is made
by demonstrating an arterial oxygen tension less than
80 mm Hg (10.7 kPa) breathing ambient air. or an
alveolar arterial gradient of IS mm Hg (2 k Pa) or
greater, along with evidence of intrapul monary
shunting on echocardiography with agitated saline or
macroaggregated albumin study.

Item 9
Educational Objective: Treat acalculous cholecystitis.

• Acalculous cholecystitis can present with biliary colic


symptoms in the alert patient or with unexplained
leukocytosis, sepsis, and jaundice in the critically ill
patient.

Item 10
Educational Objective: Prevent recurrent NSAIDrelated peptic ulcer disease.

KEY POINT
• In patients requiring NSAIDs, an evidence- based
treatment strategy to prevent recurrent NSAID-
induced peptic ulcers is the use of a cyclooxygenase- 2
selective NSAID plus a proton pump inhibitor.

3
MKSAP 18 Made Easy Yasino.M.Habayeb
Gastroenterology and Hepatology MCQ Answers and Critiques
Item 11
Educational Objective: Manage follow-up colonoscopy for hyperplastic polyps.
KEY POINT
• Patients wit h smal1 (<tO mm) hyperplastic polyps on
baseline colonoscopic examination should undergo
surveillance colonoscopy in 10 years.

Item 12
Educational Objective: Treat acute cholangitis with biliary obstruction.
KEY POINT
• Patients who have cholangitis with evidence of biliary
obstruction should be treated with antibiotic therapy
and biliary decompression with endoscopic retro¬
grade chola ngiopa ncreatography.

Item 13
Educational Objective: Treat an esophageal stricture.
KEY POINT
• Esophageal stricture in patients with eosinophilic
esophagitis requires treatment with endoscopic dilation
when symptoms do not respond to medical therapy.

Item 14

Educational Objective: Treat toxic megacolon.

KEY POINT
• Toxic megacolon is defined by the presence of toxicity
and evidence of colonic dilation; It requires prompt
surgical treatment.

4
MKSAP 18 Made Easy Yasino.M.Habayeb
Gastroenterology and Hepatology MCQ Answers and Critiques

Item 15

Educational Objective: Diagnose microscopic colitis.

KEY POINT
• Microscopic colitis is a cause of
nonbloody. watery
diarrhea In older adults and is diagnosed by colonos
copy with random biopsies from multiple colonic
segments.

Item 16

Educational Objective: Treat autoimmune pancreatitls.

KEY POINT
• Almost all patients (>90%) with autoimmune pancre
atitis enter clinical remission in response to glucocor
ticoids.

Item 17

Educational Objective: Treat acute fatty liver of pregnancy with immediate delivery of
the fetus.

KEY POINT
• The fetus should be delivered immediately upon rec¬
ognition of acute fatty Liver of pregnancy.

5
MKSAP 18 Made Easy Yasino.M.Habayeb
Gastroenterology and Hepatology MCQ Answers and Critiques

Item 18

Educational Objective: Treat a gallbladder polyp.

KEY POINT
• The finding of a gallbladder polyp larger than I cm in
si/e. or a polyp of any size associated with gallstones,
is an indication for cholecystectomy even if the
patient is asymptomatic.

Item 19

Educational Objective: Evaluate for Heltcobacter pylori eradication following treatment.

KEY POINT
• After eradication therapy for Helicobacter pylon
infection, eradication should be confirmed using the
urea breath test or fecal antigen test.

Item 20

Educational Objective: Treat gallstone pancreatitis with prompt cholecystectomy.

KEY POINT
• Same admission cholecystectomy reduces rates of
gallstone related complications compared with chole
cystectomy after hospital discharge for patients with
mild gallstone pancreatitis.

6
MKSAP 18 Made Easy Yasino.M.Habayeb
Gastroenterology and Hepatology MCQ Answers and Critiques

Item 21

Educational Objective: Treat cholangitis using endoscopic retrograde


cholangiopancreatography.

KEY POINT
• Indications for endoscopic retrograde cholangiopan
creatography in patients with primary sclerosing
cholangitis are bacterial cholangitis, increasing ja cm
dice, increasing pruritus, or a dominant stricture on
imaging.

Item 22

Educational Objective: Evaluate rectal bleeding in a young patient.

K E V POINT
• Red flag symptoms such as rectal bleeding with iron
deficiency anemia, abdominal pain and weight loss
should prompt evaluation by colonoscopy for colorcc
tal cancer regardless of the patient’s age or the pres¬
ence or bleeding hemorrhoids

Item 23

Educational Objective: Diagnose narcotic bowel syndrome.

KEY POINT
• Narcotic bowel syndrome, also known as opiate
induced gastrointestinal hyperalgesia, is a centrally
mediated disorder of gastrointestinal pain character¬
ized by a paradoxical increase in abdominal pain with
increasing doses of opioids.

7
MKSAP 18 Made Easy Yasino.M.Habayeb
Gastroenterology and Hepatology MCQ Answers and Critiques

Item 24

Educational Objective: Treat pain in chronic pancreatitis.

KEY POINT
• Mild hepatitis B virus-related polya rten t is nodosa is
treated with a r111vi ra 1 agents.

Item 25

Educational Objective: Treat hepatitis B virus-related polyarteritis nodosa,

KEY POINT
• Mild hepatitis B virus-related polyarteritis nodosa is
treated with antiviral agents.

Item 26

Educational Objective: Treat acute necrotic pancreatitis with enteral feeding.

KEY POINT
• Enteral nutrition is preferred in patients with acute
pancreatitis because of the benefit of maintaining a
healthy gut mucosal barrier to prevent translocation
of bacteria .

8
MKSAP 18 Made Easy Yasino.M.Habayeb
Gastroenterology and Hepatology MCQ Answers and Critiques

Item 27

Educational Objective: Prevent spontaneous bacterial peritonitis.

K E V POI N T
• Primary prophylactic: antibiotic therapy is indicated
for patients at high risk for the development of spun
taneous bacterial peritonitis, including patients with
very low ascitic fluid protein levels and those with
advanced liver failure.

Item 28

Educational Objective: Diagnose dumping syndrome after gastrojejunostomy.

KEY POINT
• Rapid gastric emptying of hyperosmolar chyme into
the small intestine after partial gastric resection can
lead to postprandial vasomotor symptoms, abdominal
pain, and diarrhea, collectively known as dumping
syndrome

Item 29

Educational Objective: Treat chronic idiopathic constipation unresponsive to first-line


treatment.

KEY POINT
• Linadotide is a peripherally acting guanylate cyclase C
receptor agonist that is FDA approved for the treat
merit of chronic idiopathic constipation ir» adults with
symptoms refractory to first- line therapies.

9
MKSAP 18 Made Easy Yasino.M.Habayeb
Gastroenterology and Hepatology MCQ Answers and Critiques

Item 30

Educational Objective: Prevent vertical transmission of hepatitis B viral infection.


KEY POINT
• Pregnant women who have hepatitis B varus DNA
levels greater than 200.000 lU/mL at 24 to 28 weeks'
gestation should he treated with tenofovlr to prevent
vertical transmission during delivery

Item 31

Educational Objective: Diagnose pancreatic cancer.

KEY POINT
• Pancreatic neoplasm must be considered as a cause of
acute pancreatitis in patients older than age -TO years
when no other cause has been identified and/or when
worrisome features, such as weight loss or new onset
of diabetes me) litus. are present.

Item 32

Educational Objective: Manage antithrombotic therapy after gastrointestinal bleeding.

KEY POINT
• Once endoscopic hemostasis has been achieved in a
patient with gastrointestinal bleeding, anticoagulation
should be reinitiated, and in most cases, this can be
done on the same day as the procedure.

10
MKSAP 18 Made Easy Yasino.M.Habayeb
Gastroenterology and Hepatology MCQ Answers and Critiques

Item 33

Educational Objective: Diagnose celiac disease.

KEY POINT

• Anti-tissue transglutaminase IgA .an ti hod y testing is


the best screening rest for celiac disease.

Item 34

Educational Objective: Monitor for colorectal neoplasia after colon cancer resection.
KEY POINT
♦ After treatment of colon cancer, patients should
undergo surveillance colonoscopy 1 year after diagnosis

Item 35

Educational Objective: Treat main-duct intraductal papillary mucinous neoplasm of the


pancreas.

KEY POINT
• Surgical resection is the best management option for
high risk cystic lesions of the pancreas, such as infra
ductal papillary mucinous neoplasms that involve the
main duct-

11
MKSAP 18 Made Easy Yasino.M.Habayeb
Gastroenterology and Hepatology MCQ Answers and Critiques

Item 36

Educational Objective: Diagnose the cause of hematochezia.

KEY POINT
• Herrutochezia associated with hemodynamic Instabil
ity in a young patient is likely due to an upper gastro
intestinal source

Item 37

Educational Objective: Diagnose secretory diarrhea.

KEY POINT
• Patients with secretory diarrhea may pass liters of
stool daily, causing severe dehydration and electrolyte
disturbances, with persistenr stool ing despite lasting.

Item 38

Educational Objective: Diagnose Crohn disease.

KEY POINT
• Colonoscopy results in Crohn disease show patchy
distribution of mucosal inflammatory changes
with “skip areas” of normal intervening mucosa,
and biopsy results for involved mucosa show
features of chroniclty (distorted and branching
colonic crypts).

12
MKSAP 18 Made Easy Yasino.M.Habayeb
Gastroenterology and Hepatology MCQ Answers and Critiques

Item 39

Educational Objective: Manage follow-up colonoscopy after uncomplicated diverticulitis.

KEY POINT
• Patients with uncomplicated diverticulitis should undergo
colonoscopy 1 to 2 months after the episode of acute
diverticulitis, when colonic inflammation has resolved.

ltem40

Educational Objective: Diagnose atrophic gastritis.

KEY POINT
• Patients wdth newly diagnosed pernicious anemia
should be evaluated for gastric adenocarcinoma and
gastric carcinoid with upper endoscopy and gastric
biopsy.

Item 41

Educational Objective: Treat acute anal fissure.

KEY POINT
• The most effective treatment approach for anal fissure
is daily warm-water sic* haths and the use of the bulk
laxative psyllium.

13
MKSAP 18 Made Easy Yasino.M.Habayeb
Gastroenterology and Hepatology MCQ Answers and Critiques

ltem42

Educational Objective: Diagnose familial adenomatous polyposis

KEY POINT
• Patients with multiple fundic gland polyps found at a
young age should be evaluated for familial adenoma¬
tous polyposis

ltem43

Educational Objective: Diagnose primary sclerosing cholangitis.

KEY POINT

• Approximately 5'5£» of patients with inflammatory


bowel disease will develop primary sclerosing cholan
gitis during the course of their disease, typically pre¬
senting as cholestatic liver injury with a characteristic
imaging study showing bile* duct strictures and dila¬
tions (“string of beads”).

ltem44

Educational Objective: Treat gastroesopbageal reflux disease

KfY POINT
• Patients with a clinical diagnosis of gastroesophageal
reflux disease should start an empiric trial of a proton
pump inhibitor in conjunction with lifestyle and die¬
tary changes, with no further testing.

14
MKSAP 18 Made Easy Yasino.M.Habayeb
Gastroenterology and Hepatology MCQ Answers and Critiques

ltem45

Educational Objective: Screen for osteoporosis in a patient with cirrhosis.


KEY POINT
• Patients with cirrhosis are at increased risk for the
development of osteoporosis and should be screened
using bone densitometry.

Item 46

Educational Objective: Prevent venous thromboembolism in a hospitallzed patient with inflammatory bowel disease.

KEY 1» O I N T
• All hospitalized patients with inflammatory bowel
disease should be given pharmacologic venous throm-
lioeniboJism prophylaxis with subcutaneous heparin

ltem47

Educational Objective: Diagnose Lynch syndrome.

KEY POINT
• Hatients with a family history suggesting Lynch syn
drome {three family members are affected with a
Lynch syndrome associated cancer, at least two sue
cess t ve generations are affected, one of the affected
family members is a first -degree relative of the other
two affected family members, and at least one caneer
was diagnosed in a family member younger than age
SO years) should be referred for genetic counseling.

15
MKSAP 18 Made Easy Yasino.M.Habayeb
Gastroenterology and Hepatology MCQ Answers and Critiques

Item 48

Educational Objective: Evaluate persistent gastroesophageal reflux disease.

KEY POINT
• Medications that decrease kidney perfusion, includ
ing \SAIDs. ACE inhibitors, and angiotensin receptor
blockers, should be discontinued in patients with
ascites.

ltem49

Educational Objective: Treat ascites caused by portal hypertension.

KEY POINT

• Medications that decrease kidney perfusion, includ


ing NSAIDs. ACE Inhibitors, and angiotensin receptor
blockers, should he discontinued in patients with
ascites.

Item 50

Educational Objective: Treat acute liver failure.

K E Y POINT
• Acute liver failure is aa indication for immediate
referral to a Irver transplantation center

16
MKSAP 18 Made Easy Yasino.M.Habayeb
Gastroenterology and Hepatology MCQ Answers and Critiques
Item 51

Educational Objective: Treat left-sided ulcerative colitis.

KEY POINT
• Combined mesalamlne therapy ( oral and topical) is
superior for induction of remission in mild to mcxier
ately active ulcerative colitis compared with oral or
topical therapies alone.

Item 52

Educational Objective: Evaluate dyspepsia

KEY POINT
• Patients younger than age 60 years presenting wit h
dyspepsia should first undergo a non invasive test for
Hÿltcokxicter pylori followed hy eradication therapy if
testing is positive.

Item 53

Educational Objective: Treat amebic liver abscess.

K E V POINT
• The mainstay of therapy for amebic liver abscesses is
antibiotic therapy, such as metronidazole, plus a
luminal agent, such as paromomycin, to eradicate the
coexisting intestinal infection.

17
MKSAP 18 Made Easy Yasino.M.Habayeb
Gastroenterology and Hepatology MCQ Answers and Critiques

Item 54

Educational Objective: Diagnose ulcerative colitis

KEY POINT
• Chronic bloody diarrhea arid abdominal discomfort
are rypical presenting symptoms of inflammatory
bowel disease; endoscopic findings help distinguish
ulcerative eolitis from Crohn disease.

Item 55

Educational Objective: Test for Wilson disease.

KEY POINT
• Wilson disease should be considered in all patients
younger than age 40 years who have unexplained liver
disease.

Item 56

Educational Objective: Treat achalasia with botulinum toxin injection.

KEY POINT ■■■■■


• Patients with achalasia who are at high surgical risk
should be treated w ith endoscopic borulinum toxin
injection.

18
MKSAP 18 Made Easy Yasino.M.Habayeb
Gastroenterology and Hepatology MCQ Answers and Critiques

Item 57

Educational Objective: Recognize atypical chest pain and evaluate before starting treatment for gastroesophageal reflux
disease.

KEY POINT
• In patients with atypical chest pain, a cardiac cause
must be ruled out before starting treatment for gas
troesophageal reflux disease.

Item 58

Educational Objective: Diagnose the relapsing, remitting variant of hepatitis A viral


infection.

KEY POINT
• The relapsing, remitting variant of hepatitis A viral
infection is characterized by multiple clinical or bio
chemical relapses with spontaneous improvement
within months to 1 year without intervention.

Item 59

Educational Objective: Manage chronic proton pump inhibitor therapy.


KEY POI NT
• Long-term proton pump inhibitor (PP1) therapy for
uncomplicated gastroesophageal reflux disease should
be given at the lowest effective dose possible, and con¬
sideration should be given to reducing or stopping PP1
therapy at least once a year.

19
MKSAP 18 Made Easy Yasino.M.Habayeb
Gastroenterology and Hepatology MCQ Answers and Critiques

Item 60

Educational Objective: Treat intrahepatic cholestasis of pregnancy with ursodeoxychollc


acid

KEY POINT
• The mainstay of therapy for intrahepatic cholestasis of
pregnancy is ursodeoxycholic acid, which is associ¬
ated with alleviated symptoms, improved fetal out¬
comes, and improved liver test abnormalities.

Item 61

Educational Objective: Evaluate isolated right-colon ischemia.

KEY POINT
• Isolated right-colon ischemia may be a warning sign
of acute mesenteric ischemia caused by embolism or
thrombosis of the superior mesenteric artery and
should be evaluated using CT angiography.

Item 62

Educational Objective: Diagnose medication-induced enteropathy.

KEY POINT
• Olmesartan causes medication- induced enteropathy
that can mimic refractory celiac disease.

20
MKSAP 18 Made Easy Yasino.M.Habayeb
Gastroenterology and Hepatology MCQ Answers and Critiques

Item 63

Educational Objective: Manage a patient with Barrett esophagus with low-grade


dysplasia
KEY POINT
• Barrett esophagus with low-grade dysplasia should be
treated with endoscopic ablat itm therapy in patients
without significant comorhid ities.

Item 64

Educational Objective: Diagnose Gtardla lamblia infection.


KEY POINT
■ Giarctia larriblicj infection is a common parasitic
infection that occurs most often among children.
child care workers, and backpackers or campers who
drink untreated water from lakes, rivers, or wells.

Item 65

Educational Objective: Diagnose gastroparesls,

KEY POINT
• The diagnosis of gastroparesis requires the presence of
specific symptoms, absence of mechanical outlet
obstruction, and objective evidence of delay in gastric
emptying into the duodenum

21
MKSAP 18 Made Easy Yasino.M.Habayeb
Gastroenterology and Hepatology MCQ Answers and Critiques

Item 66

Educational Objective: Treat bleeding esophageal varices.

KEY POINT
• The mainstay of therapy for variceal hemorrhage Is
endoscopic therapy, and adjunctive therapies such as
antibiotic therapy improve outcomes.

Item 67

Educational Objective: Manage hepatic encephalopathy.

KEY POINT
• Up to flO'X. of patients with hepatic encephalopathy-
have a precipitating factor, most commonly infection
or gastrointestinal bleeding

ltem68

Educational Objective: Screen for colorectal cancer in a high-risk individual.

KEY POINT
• Individuals with a first-degree relative with colon
cancer or an advanced adenoma diagnosed at an age
younger than €»0 years, or two or more first degree
relatives with colon cancer or advanced adenoma
diagnosed at any age, should begin colon cancer
screening at age 40 years (or 10 years earlier than the
youngest age at which colon cancer was diagnosed in
a first degree relative, whichever is first),

22
MKSAP 18 Made Easy Yasino.M.Habayeb
Gastroenterology and Hepatology MCQ Answers and Critiques

Item 69

Educational Objective: Treat hepatocellular carcinoma with liver transplantation.

KEY POINT
• Patients with cirrhosis and who meet the Milan criteria
(tap to three hepatocellular carcinoma rumors <:i cm or
one tumor ≤5 cm) are best treated with liver transplan¬
tation and have excellent 5-year survival rates

Item 70

Educational Objective: Diagnose eosinophilic esophagitis.

KEY POINT

• Eosinophilic esophagitis typically presents in young


men with symptoms of dysphagia and in patients
with a history of food allergies, eczema, and asthma

Item 71

Educational Objective: Manage asymptomatic gallstones.


KEY POINT
• Incidentally found gallstones with no associated
symptoms and no complications require no further
i ntervention

23
MKSAP 18 Made Easy Yasino.M.Habayeb
Gastroenterology and Hepatology MCQ Answers and Critiques

Item 72

Educational Objective: Diagnose aortoenteric fistuJa.

KEY POINT
• Gastrointestinal bleeding occurring in patients follow¬
ing aortic graft surgery should raise the possibility of
aortoenteric fistula; CT with contrast Is the initial test
in appropriate patients.

Item 73

Educational Objective: Manage lower gastrointestinal bleeding.

KEY POINT
• Colonoscopy is the recommended initial diagnostic
test after hemodynamic resuscitation in most patients
with significant lower gastrointestinal bleeding

Item 74

Educational Objective: Treat uncomplicated diverticulitis.


KEY POINT
• Patients with uncomplicated diverticulitis should be
treated conservatively with oral antibiotics.

24
MKSAP 18 Made Easy Yasino.M.Habayeb
Gastroenterology and Hepatology MCQ Answers and Critiques

Item 75

Educational Objective: Manage a patient with Barrett esophagus that is indefinite for
dysplasia.
KEY POINT
• Fatienls whose Barrett esophagus is indefinite for dys
plasla should begin optimized antisee re to ry medical
therapy and undergo a repeat endoscopy

Item 76

Educational Objective: Evaluate fecal incontinence in an elderly patient.

KiliSlOlNT ■■
• Fecal loading (excess stool in the colon) with resultant
overflow diarrhea is a common cause of fecal inconti
ne nee in elderly patients, particularly those who are
hospitalized or have degenerative neurologic disorders.

Item 77

Educational Objective: Diagnose ischemic colitis.

• Ischemic colitis is a low flow state of the colon occur


ring most frequently in the left colon and character¬
ized by moderate, left-sided, cramping abdominal
pain followed by bloody diarrhea.

25
MKSAP 18 Made Easy Yasino.M.Habayeb
Gastroenterology and Hepatology MCQ Answers and Critiques

Item 78

Educational Objective: Treat ileal Crohn disease.

KEY POINT
• Anti tumor necrosis factor agents such as infliximab
are effective in inducing and maintaining remission in
moderate to severe Crohn disease

Item 79

Educational Objective: Manage Lynch syndrome.

• Patients with Lynch syndrome should begin screening


colonoscopy between ages 20 and 25 years or 2 to
5 years before the earliest age of colorectal cancer
diagnosis tn the family, whichever comes first, and
colonoscopy should be repeated every 1 to 2 years If
the baseline examination is normal.

Item 80

Educational Objective: Diagnose hereditary diffuse gastric cancer.

KEY POINT
•A history' of multiple family members with gastric
cancer, particularly before age 50 years, or multiple
family members with lobular breast cancer with or
without gastric cancer, suggest the possibility of
hereditary diffuse gastric cancer and the need for
upper endoscopy and testing for mutations of the
CDHi gene.

26
MKSAP 18 Made Easy Yasino.M.Habayeb
Gastroenterology and Hepatology MCQ Answers and Critiques

Item 81

Educational Objective: Manage drug-induced liver injury.

KEY POINT
• In patients wit hi well preserved liver function, drug-
induced liver injury' should be managed with discon¬
tinuation of the offending medication and observation
until resolution of symptoms occurs.

Item 82

Educational Objective: Treat centrally mediated abdominal pain syndrome.

KEY POINT
• Centrally mediated abdominal pain syndrome is
characterized by near— constant abdominal pain last¬
ing longer than 6» months, involving a large anatomic
distribution, and without initiating triggers or alarm
features.

Item 83

Educational Objective: Treat upper gastrointestinal bleeding.

KEY POINT
• In patients with upper gastrointestinal bleeding, a
restrictive transfusion strategy (transfusion threshold
of less than 7 g 'dL [70 g/L] with a target hemoglobin
level of 7-9 g/dl. [70 90 g;L]) is associated with
decreased mortality, length of hospital stay, and trans¬
fusion related adverse events compared to a liberal
transfusion strategy.

27
MKSAP 18 Made Easy Yasino.M.Habayeb
Gastroenterology and Hepatology MCQ Answers and Critiques

Item 84

Educational Objective: Screen for duodenal cancer in a patient with familial


adenomatous polyposis syndrome.

KEY POINT
• Upper endoscopy screening for duodenal cancer in
patients with familial adenomatous polyposis should
begin at onset of colonic polyposis or at age 25 to
30 years, whichever comes first.

Item 85

Educational Objective: Manage colonoscopy surveillance following polypectomy.

KEY POINT
• Patients with three or more adenomas on screening
colonoscopy should undergo surveillance colonoscopy
in 3 years.

Item 86

Educational Objective: Treat functional dyspepsia.

KEY POINT
• First line treatment for functional dyspepsia is once
daily omeprazole for at least 4 weeks, if symptoms do
not respond, a tricyclic antidepressant is the next ret
om mended treatment.

28
MKSAP 18 Made Easy Yasino.M.Habayeb
Gastroenterology and Hepatology MCQ Answers and Critiques

Item 87

Educational Objective: Treat cryoglobulinemia in a patient with chronic hepatitis C viral


infection.

KEY POINT
• Mixed cryoglobulinemia arising from chronic hepati
tis C viral infection resolves after treatment and cradi
cation of the vims.

Item 88

Educational Objective: Treat microscopic colitis.

KEY POI NT
• The first step in the management of microscopic coli¬
tis is to discontinue a potentially causative medica
tion, after which supportive treatment with antidiar
rheal agents such loperamide can he tried, with
budesonide recommended for patients whose symp¬
toms do not respond.

Item 89

Educational Objective: Manage hepatic adenoma.

KEY POINT
• For women with asymptomatic hepatic adenomas
smaller than 5 cm in size, estrogen-containing oral
contraceptive agents should t>e discontinued, and
follow up liver imaging is recommended every 6
months for at least 2 years.

29
MKSAP 18 Made Easy Yasino.M.Habayeb
Gastroenterology and Hepatology MCQ Answers and Critiques

ltem90

Educational Objective: Evaluate the small bowel for a source of gastrointestinal bleeding

KEY POINT
• Capsule endoscopy is the most appropriate test to
evaluate patients for causes of small bowel bleeding
after negative upper endoscopy and colonoscopy.

Item 91

Educational Objective: Treat chronic hepatitis B viral infection In Its immune-active,


hepatitis B e antigen positive phase.

KEY POINT
• Chronic hepatitis 3 viral infection in the immune-
active, hepatitis B e antigen positive phase should be
treated wi rh tenofovir or entecavir to decrease hepatic
inflammation and the risk for progression to fibrosis

Item 92

Educational Objective: Treat irritable bowel syndrome with predominant constipation.

• Polyethylene glycol 3350 is a first line treatment for


patients whose symptoms meet the criteria for irrita
ble bowel syndrome with predominant constipation

30
MKSAP 18 Made Easy Yasino.M.Habayeb
Gastroenterology and Hepatology MCQ Answers and Critiques

Item 93
Educational Objective: Treat Helicobacter pylori infection persisting after treatment.
KEY P O I NT
• For /-/eficobacter pyiorf infection that persists after
eradication therapy, the salvage therapy regimen
should consist of different antibiotics from those used
in the initial, unsuccessful regimen

Item 94
KEY POINT
• In patients with end-stage liver disease and portal
hypertension, hepatorenal syndrome is characterized
by the development of oliguric Kidney failure, bland
urine sediment, and marked sodium retention
(edema, ascites, low urinary sodium).

Item 95
Educational Objective: Manage walled-off necrosis after acute pancreatitis.
KEY POINT
• Asymptomatic patients with walled off necrosis of the
pancreas require no intervention.

Item 96

Educational Objective: Screen for hepatocellular carcinoma in a patient with cirrhosis.

KEY POINT
• Patients with hepatitis C viral infection who achieve
sustained virologic response have a reduced risk for
hepatocellular carcinoma . regardless of virologic
response, ultrasonographic surveillance is recom
mended for patients with stage 3 or stage 4 fibrosis.

31
MKSAP 18 Made Easy Yasino.M.Habayeb

You might also like